soluções elon grande

Post on 12-Feb-2016

106 Views

Category:

Documents

21 Downloads

Preview:

Click to see full reader

DESCRIPTION

Solução IMPA Elon

TRANSCRIPT

Solucoes de Exercıcios do Livro

“Curso de Analise”, Volume I,

de Elon Lages Lima

Cleber Fernando Colle,Edson Jose Teixeira,

Julio C. C. da Silva (jcconegundes@gmail.com) eRodrigo Carlos Silva de Lima (rodrigo.uff.math@gmail.com)

12 de janeiro de 2014

Sumario

1 Conjuntos e Funcoes 7Exercıcio 1.1 . . . . . . . . . . . . . . . . . . . . . . . . . . . . . . . . . . . . . . . . . . . . . . . . . . . . 8Exercıcio 1.2 . . . . . . . . . . . . . . . . . . . . . . . . . . . . . . . . . . . . . . . . . . . . . . . . . . . . 9Exercıcio 1.3 . . . . . . . . . . . . . . . . . . . . . . . . . . . . . . . . . . . . . . . . . . . . . . . . . . . . 10Exercıcio 1.4 . . . . . . . . . . . . . . . . . . . . . . . . . . . . . . . . . . . . . . . . . . . . . . . . . . . . 11Exercıcio 1.5 . . . . . . . . . . . . . . . . . . . . . . . . . . . . . . . . . . . . . . . . . . . . . . . . . . . . 12Exercıcio 1.6 . . . . . . . . . . . . . . . . . . . . . . . . . . . . . . . . . . . . . . . . . . . . . . . . . . . . 13Exercıcio 1.7 . . . . . . . . . . . . . . . . . . . . . . . . . . . . . . . . . . . . . . . . . . . . . . . . . . . . 14Exercıcio 1.8 . . . . . . . . . . . . . . . . . . . . . . . . . . . . . . . . . . . . . . . . . . . . . . . . . . . . 15Exercıcio 1.9 . . . . . . . . . . . . . . . . . . . . . . . . . . . . . . . . . . . . . . . . . . . . . . . . . . . . 16Exercıcio 1.10 . . . . . . . . . . . . . . . . . . . . . . . . . . . . . . . . . . . . . . . . . . . . . . . . . . . . 17Exercıcio 1.11 . . . . . . . . . . . . . . . . . . . . . . . . . . . . . . . . . . . . . . . . . . . . . . . . . . . . 18Exercıcio 1.12 . . . . . . . . . . . . . . . . . . . . . . . . . . . . . . . . . . . . . . . . . . . . . . . . . . . . 19Exercıcio 1.13 . . . . . . . . . . . . . . . . . . . . . . . . . . . . . . . . . . . . . . . . . . . . . . . . . . . . 20Exercıcio 1.14 . . . . . . . . . . . . . . . . . . . . . . . . . . . . . . . . . . . . . . . . . . . . . . . . . . . . 21Exercıcio 1.15 . . . . . . . . . . . . . . . . . . . . . . . . . . . . . . . . . . . . . . . . . . . . . . . . . . . . 22Exercıcio 1.16 . . . . . . . . . . . . . . . . . . . . . . . . . . . . . . . . . . . . . . . . . . . . . . . . . . . . 23Exercıcio 1.17 . . . . . . . . . . . . . . . . . . . . . . . . . . . . . . . . . . . . . . . . . . . . . . . . . . . . 24Exercıcio 1.18 . . . . . . . . . . . . . . . . . . . . . . . . . . . . . . . . . . . . . . . . . . . . . . . . . . . . 25Exercıcio 1.19 . . . . . . . . . . . . . . . . . . . . . . . . . . . . . . . . . . . . . . . . . . . . . . . . . . . . 26Exercıcio 1.20 . . . . . . . . . . . . . . . . . . . . . . . . . . . . . . . . . . . . . . . . . . . . . . . . . . . . 28Exercıcio 1.21 . . . . . . . . . . . . . . . . . . . . . . . . . . . . . . . . . . . . . . . . . . . . . . . . . . . . 29

2 Conjuntos Finitos, Enumeraveis e Nao-Enumeraveis 30Exercıcio 2.1 . . . . . . . . . . . . . . . . . . . . . . . . . . . . . . . . . . . . . . . . . . . . . . . . . . . . 31Exercıcio 2.2 . . . . . . . . . . . . . . . . . . . . . . . . . . . . . . . . . . . . . . . . . . . . . . . . . . . . 32Exercıcio 2.3 . . . . . . . . . . . . . . . . . . . . . . . . . . . . . . . . . . . . . . . . . . . . . . . . . . . . 33Exercıcio 2.4 . . . . . . . . . . . . . . . . . . . . . . . . . . . . . . . . . . . . . . . . . . . . . . . . . . . . 34Exercıcio 2.5 . . . . . . . . . . . . . . . . . . . . . . . . . . . . . . . . . . . . . . . . . . . . . . . . . . . . 38Exercıcio 2.6 . . . . . . . . . . . . . . . . . . . . . . . . . . . . . . . . . . . . . . . . . . . . . . . . . . . . 39Exercıcio 2.7 . . . . . . . . . . . . . . . . . . . . . . . . . . . . . . . . . . . . . . . . . . . . . . . . . . . . 40Exercıcio 2.8 . . . . . . . . . . . . . . . . . . . . . . . . . . . . . . . . . . . . . . . . . . . . . . . . . . . . 42Exercıcio 2.9 . . . . . . . . . . . . . . . . . . . . . . . . . . . . . . . . . . . . . . . . . . . . . . . . . . . . 44Exercıcio 2.10 . . . . . . . . . . . . . . . . . . . . . . . . . . . . . . . . . . . . . . . . . . . . . . . . . . . . 45Exercıcio 2.11 . . . . . . . . . . . . . . . . . . . . . . . . . . . . . . . . . . . . . . . . . . . . . . . . . . . . 46Exercıcio 2.12 . . . . . . . . . . . . . . . . . . . . . . . . . . . . . . . . . . . . . . . . . . . . . . . . . . . . 47Exercıcio 2.13 . . . . . . . . . . . . . . . . . . . . . . . . . . . . . . . . . . . . . . . . . . . . . . . . . . . . 48Exercıcio 2.14 . . . . . . . . . . . . . . . . . . . . . . . . . . . . . . . . . . . . . . . . . . . . . . . . . . . . 49Exercıcio 2.15 . . . . . . . . . . . . . . . . . . . . . . . . . . . . . . . . . . . . . . . . . . . . . . . . . . . . 50Exercıcio 2.16 . . . . . . . . . . . . . . . . . . . . . . . . . . . . . . . . . . . . . . . . . . . . . . . . . . . . 51Exercıcio 2.17 . . . . . . . . . . . . . . . . . . . . . . . . . . . . . . . . . . . . . . . . . . . . . . . . . . . . 52Exercıcio 2.18 . . . . . . . . . . . . . . . . . . . . . . . . . . . . . . . . . . . . . . . . . . . . . . . . . . . . 53Exercıcio 2.10 . . . . . . . . . . . . . . . . . . . . . . . . . . . . . . . . . . . . . . . . . . . . . . . . . . . . 54Exercıcio 2.20 . . . . . . . . . . . . . . . . . . . . . . . . . . . . . . . . . . . . . . . . . . . . . . . . . . . . 55Exercıcio 2.21 . . . . . . . . . . . . . . . . . . . . . . . . . . . . . . . . . . . . . . . . . . . . . . . . . . . . 56

1

Exercıcio 2.22 . . . . . . . . . . . . . . . . . . . . . . . . . . . . . . . . . . . . . . . . . . . . . . . . . . . . 57Exercıcio 2.23 . . . . . . . . . . . . . . . . . . . . . . . . . . . . . . . . . . . . . . . . . . . . . . . . . . . . 58Exercıcio 2.24 . . . . . . . . . . . . . . . . . . . . . . . . . . . . . . . . . . . . . . . . . . . . . . . . . . . . 60Exercıcio 2.25 . . . . . . . . . . . . . . . . . . . . . . . . . . . . . . . . . . . . . . . . . . . . . . . . . . . . 61Exercıcio 2.26 . . . . . . . . . . . . . . . . . . . . . . . . . . . . . . . . . . . . . . . . . . . . . . . . . . . . 62Exercıcio 2.27 . . . . . . . . . . . . . . . . . . . . . . . . . . . . . . . . . . . . . . . . . . . . . . . . . . . . 63Exercıcio 2.28 . . . . . . . . . . . . . . . . . . . . . . . . . . . . . . . . . . . . . . . . . . . . . . . . . . . . 66Exercıcio 2.29 . . . . . . . . . . . . . . . . . . . . . . . . . . . . . . . . . . . . . . . . . . . . . . . . . . . . 69

3 Numeros Reais 71Exercıcio 3.01 . . . . . . . . . . . . . . . . . . . . . . . . . . . . . . . . . . . . . . . . . . . . . . . . . . . . 72Exercıcio 3.02 . . . . . . . . . . . . . . . . . . . . . . . . . . . . . . . . . . . . . . . . . . . . . . . . . . . . 73Exercıcio 3.03 . . . . . . . . . . . . . . . . . . . . . . . . . . . . . . . . . . . . . . . . . . . . . . . . . . . . 75Exercıcio 3.04 . . . . . . . . . . . . . . . . . . . . . . . . . . . . . . . . . . . . . . . . . . . . . . . . . . . . 76Exercıcio 3.05 . . . . . . . . . . . . . . . . . . . . . . . . . . . . . . . . . . . . . . . . . . . . . . . . . . . . 77Exercıcio 3.08 . . . . . . . . . . . . . . . . . . . . . . . . . . . . . . . . . . . . . . . . . . . . . . . . . . . . 78Exercıcio 3.09 . . . . . . . . . . . . . . . . . . . . . . . . . . . . . . . . . . . . . . . . . . . . . . . . . . . . 79Exercıcio 3.10 . . . . . . . . . . . . . . . . . . . . . . . . . . . . . . . . . . . . . . . . . . . . . . . . . . . . 80Exercıcio 3.11 . . . . . . . . . . . . . . . . . . . . . . . . . . . . . . . . . . . . . . . . . . . . . . . . . . . . 81Exercıcio 3.12 . . . . . . . . . . . . . . . . . . . . . . . . . . . . . . . . . . . . . . . . . . . . . . . . . . . . 82Exercıcio 3.13 . . . . . . . . . . . . . . . . . . . . . . . . . . . . . . . . . . . . . . . . . . . . . . . . . . . . 83Exercıcio 3.14 . . . . . . . . . . . . . . . . . . . . . . . . . . . . . . . . . . . . . . . . . . . . . . . . . . . . 84Exercıcio 3.15 . . . . . . . . . . . . . . . . . . . . . . . . . . . . . . . . . . . . . . . . . . . . . . . . . . . . 85Exercıcio 3.16 . . . . . . . . . . . . . . . . . . . . . . . . . . . . . . . . . . . . . . . . . . . . . . . . . . . . 86Exercıcio 3.17 . . . . . . . . . . . . . . . . . . . . . . . . . . . . . . . . . . . . . . . . . . . . . . . . . . . . 87Exercıcio 3.10 . . . . . . . . . . . . . . . . . . . . . . . . . . . . . . . . . . . . . . . . . . . . . . . . . . . . 88Exercıcio 3.19 . . . . . . . . . . . . . . . . . . . . . . . . . . . . . . . . . . . . . . . . . . . . . . . . . . . . 89Exercıcio 3.20 . . . . . . . . . . . . . . . . . . . . . . . . . . . . . . . . . . . . . . . . . . . . . . . . . . . . 90Exercıcio 3.22 . . . . . . . . . . . . . . . . . . . . . . . . . . . . . . . . . . . . . . . . . . . . . . . . . . . . 92Exercıcio 3.23 . . . . . . . . . . . . . . . . . . . . . . . . . . . . . . . . . . . . . . . . . . . . . . . . . . . . 94Exercıcio 3.24 . . . . . . . . . . . . . . . . . . . . . . . . . . . . . . . . . . . . . . . . . . . . . . . . . . . . 95Exercıcio 3.25 . . . . . . . . . . . . . . . . . . . . . . . . . . . . . . . . . . . . . . . . . . . . . . . . . . . . 96Exercıcio 3.26 . . . . . . . . . . . . . . . . . . . . . . . . . . . . . . . . . . . . . . . . . . . . . . . . . . . . 97Exercıcio 3.27 . . . . . . . . . . . . . . . . . . . . . . . . . . . . . . . . . . . . . . . . . . . . . . . . . . . . 98Exercıcio 3.28 . . . . . . . . . . . . . . . . . . . . . . . . . . . . . . . . . . . . . . . . . . . . . . . . . . . . 99Exercıcio 3.29 . . . . . . . . . . . . . . . . . . . . . . . . . . . . . . . . . . . . . . . . . . . . . . . . . . . . 100Exercıcio 3.30 . . . . . . . . . . . . . . . . . . . . . . . . . . . . . . . . . . . . . . . . . . . . . . . . . . . . 101Exercıcio 3.31 . . . . . . . . . . . . . . . . . . . . . . . . . . . . . . . . . . . . . . . . . . . . . . . . . . . . 102Exercıcio 3.32 . . . . . . . . . . . . . . . . . . . . . . . . . . . . . . . . . . . . . . . . . . . . . . . . . . . . 103Exercıcio 3.33 . . . . . . . . . . . . . . . . . . . . . . . . . . . . . . . . . . . . . . . . . . . . . . . . . . . . 104Exercıcio 3.31 . . . . . . . . . . . . . . . . . . . . . . . . . . . . . . . . . . . . . . . . . . . . . . . . . . . . 105Exercıcio 3.32 . . . . . . . . . . . . . . . . . . . . . . . . . . . . . . . . . . . . . . . . . . . . . . . . . . . . 106Exercıcio 3.33 . . . . . . . . . . . . . . . . . . . . . . . . . . . . . . . . . . . . . . . . . . . . . . . . . . . . 107Exercıcio 3.34 . . . . . . . . . . . . . . . . . . . . . . . . . . . . . . . . . . . . . . . . . . . . . . . . . . . . 108Exercıcio 3.35 . . . . . . . . . . . . . . . . . . . . . . . . . . . . . . . . . . . . . . . . . . . . . . . . . . . . 109Exercıcio 3.37 . . . . . . . . . . . . . . . . . . . . . . . . . . . . . . . . . . . . . . . . . . . . . . . . . . . . 110Exercıcio 3.38 . . . . . . . . . . . . . . . . . . . . . . . . . . . . . . . . . . . . . . . . . . . . . . . . . . . . 111Exercıcio 3.39 . . . . . . . . . . . . . . . . . . . . . . . . . . . . . . . . . . . . . . . . . . . . . . . . . . . . 112Exercıcio 3.40 . . . . . . . . . . . . . . . . . . . . . . . . . . . . . . . . . . . . . . . . . . . . . . . . . . . . 113Exercıcio 3.42 . . . . . . . . . . . . . . . . . . . . . . . . . . . . . . . . . . . . . . . . . . . . . . . . . . . . 114Exercıcio 3.43 . . . . . . . . . . . . . . . . . . . . . . . . . . . . . . . . . . . . . . . . . . . . . . . . . . . . 115Exercıcio 3.44 . . . . . . . . . . . . . . . . . . . . . . . . . . . . . . . . . . . . . . . . . . . . . . . . . . . . 116Exercıcio 3.45 . . . . . . . . . . . . . . . . . . . . . . . . . . . . . . . . . . . . . . . . . . . . . . . . . . . . 117Exercıcio 3.46 . . . . . . . . . . . . . . . . . . . . . . . . . . . . . . . . . . . . . . . . . . . . . . . . . . . . 118Exercıcio 3.47 . . . . . . . . . . . . . . . . . . . . . . . . . . . . . . . . . . . . . . . . . . . . . . . . . . . . 119Exercıcio 3.48 . . . . . . . . . . . . . . . . . . . . . . . . . . . . . . . . . . . . . . . . . . . . . . . . . . . . 120Exercıcio 3.49 . . . . . . . . . . . . . . . . . . . . . . . . . . . . . . . . . . . . . . . . . . . . . . . . . . . . 121

2

Exercıcio 3.50 . . . . . . . . . . . . . . . . . . . . . . . . . . . . . . . . . . . . . . . . . . . . . . . . . . . . 122Exercıcio 3.51 . . . . . . . . . . . . . . . . . . . . . . . . . . . . . . . . . . . . . . . . . . . . . . . . . . . . 123Exercıcio 3.52 . . . . . . . . . . . . . . . . . . . . . . . . . . . . . . . . . . . . . . . . . . . . . . . . . . . . 124Exercıcio 3.53 . . . . . . . . . . . . . . . . . . . . . . . . . . . . . . . . . . . . . . . . . . . . . . . . . . . . 126Exercıcio 3.54 . . . . . . . . . . . . . . . . . . . . . . . . . . . . . . . . . . . . . . . . . . . . . . . . . . . . 127Exercıcio 3.55 . . . . . . . . . . . . . . . . . . . . . . . . . . . . . . . . . . . . . . . . . . . . . . . . . . . . 128Exercıcio 3.56 . . . . . . . . . . . . . . . . . . . . . . . . . . . . . . . . . . . . . . . . . . . . . . . . . . . . 130Exercıcio 3.57 . . . . . . . . . . . . . . . . . . . . . . . . . . . . . . . . . . . . . . . . . . . . . . . . . . . . 131Exercıcio 3.58 . . . . . . . . . . . . . . . . . . . . . . . . . . . . . . . . . . . . . . . . . . . . . . . . . . . . 132Exercıcio 3.59 . . . . . . . . . . . . . . . . . . . . . . . . . . . . . . . . . . . . . . . . . . . . . . . . . . . . 133Exercıcio 3.60 . . . . . . . . . . . . . . . . . . . . . . . . . . . . . . . . . . . . . . . . . . . . . . . . . . . . 134

4 Sequencias e Series de Numeros Reais 136Exercıcio 4.1 . . . . . . . . . . . . . . . . . . . . . . . . . . . . . . . . . . . . . . . . . . . . . . . . . . . . 137Exercıcio 4.2 . . . . . . . . . . . . . . . . . . . . . . . . . . . . . . . . . . . . . . . . . . . . . . . . . . . . 138Exercıcio 4.3 . . . . . . . . . . . . . . . . . . . . . . . . . . . . . . . . . . . . . . . . . . . . . . . . . . . . 139Exercıcio 4.4 . . . . . . . . . . . . . . . . . . . . . . . . . . . . . . . . . . . . . . . . . . . . . . . . . . . . 140Exercıcio 4.5 . . . . . . . . . . . . . . . . . . . . . . . . . . . . . . . . . . . . . . . . . . . . . . . . . . . . 141Exercıcio 4.6 . . . . . . . . . . . . . . . . . . . . . . . . . . . . . . . . . . . . . . . . . . . . . . . . . . . . 142Exercıcio 4.7 . . . . . . . . . . . . . . . . . . . . . . . . . . . . . . . . . . . . . . . . . . . . . . . . . . . . 143Exercıcio 4.8 . . . . . . . . . . . . . . . . . . . . . . . . . . . . . . . . . . . . . . . . . . . . . . . . . . . . 144Exercıcio 4.9 . . . . . . . . . . . . . . . . . . . . . . . . . . . . . . . . . . . . . . . . . . . . . . . . . . . . 145Exercıcio 4.10 . . . . . . . . . . . . . . . . . . . . . . . . . . . . . . . . . . . . . . . . . . . . . . . . . . . . 146Exercıcio 4.10 . . . . . . . . . . . . . . . . . . . . . . . . . . . . . . . . . . . . . . . . . . . . . . . . . . . . 147Exercıcio 4.11 . . . . . . . . . . . . . . . . . . . . . . . . . . . . . . . . . . . . . . . . . . . . . . . . . . . . 148Exercıcio 4.11a . . . . . . . . . . . . . . . . . . . . . . . . . . . . . . . . . . . . . . . . . . . . . . . . . . . 149Exercıcio 4.12 . . . . . . . . . . . . . . . . . . . . . . . . . . . . . . . . . . . . . . . . . . . . . . . . . . . . 150Exercıcio 4.14 . . . . . . . . . . . . . . . . . . . . . . . . . . . . . . . . . . . . . . . . . . . . . . . . . . . . 151Exercıcio 4.15 . . . . . . . . . . . . . . . . . . . . . . . . . . . . . . . . . . . . . . . . . . . . . . . . . . . . 152Exercıcio 4.18 . . . . . . . . . . . . . . . . . . . . . . . . . . . . . . . . . . . . . . . . . . . . . . . . . . . . 153Exercıcio 4.19 . . . . . . . . . . . . . . . . . . . . . . . . . . . . . . . . . . . . . . . . . . . . . . . . . . . . 154Exercıcio 4.20 . . . . . . . . . . . . . . . . . . . . . . . . . . . . . . . . . . . . . . . . . . . . . . . . . . . . 156Exercıcio 4.21 . . . . . . . . . . . . . . . . . . . . . . . . . . . . . . . . . . . . . . . . . . . . . . . . . . . . 157Exercıcio 4.22 . . . . . . . . . . . . . . . . . . . . . . . . . . . . . . . . . . . . . . . . . . . . . . . . . . . . 158Exercıcio 4.25 . . . . . . . . . . . . . . . . . . . . . . . . . . . . . . . . . . . . . . . . . . . . . . . . . . . . 159Exercıcio 4.31 . . . . . . . . . . . . . . . . . . . . . . . . . . . . . . . . . . . . . . . . . . . . . . . . . . . . 161Exercıcio 4.33 . . . . . . . . . . . . . . . . . . . . . . . . . . . . . . . . . . . . . . . . . . . . . . . . . . . . 162Exercıcio 4.35 . . . . . . . . . . . . . . . . . . . . . . . . . . . . . . . . . . . . . . . . . . . . . . . . . . . . 163Exercıcio 4.36 . . . . . . . . . . . . . . . . . . . . . . . . . . . . . . . . . . . . . . . . . . . . . . . . . . . . 164Exercıcio 4.40 . . . . . . . . . . . . . . . . . . . . . . . . . . . . . . . . . . . . . . . . . . . . . . . . . . . . 165Exercıcio 4.41 . . . . . . . . . . . . . . . . . . . . . . . . . . . . . . . . . . . . . . . . . . . . . . . . . . . . 166Exercıcio 4.42 . . . . . . . . . . . . . . . . . . . . . . . . . . . . . . . . . . . . . . . . . . . . . . . . . . . . 167Exercıcio 4.43 . . . . . . . . . . . . . . . . . . . . . . . . . . . . . . . . . . . . . . . . . . . . . . . . . . . . 168Exercıcio 4.44 . . . . . . . . . . . . . . . . . . . . . . . . . . . . . . . . . . . . . . . . . . . . . . . . . . . . 169Exercıcio 4.45 . . . . . . . . . . . . . . . . . . . . . . . . . . . . . . . . . . . . . . . . . . . . . . . . . . . . 170Exercıcio 4.46 . . . . . . . . . . . . . . . . . . . . . . . . . . . . . . . . . . . . . . . . . . . . . . . . . . . . 171Exercıcio 4.47 . . . . . . . . . . . . . . . . . . . . . . . . . . . . . . . . . . . . . . . . . . . . . . . . . . . . 172Exercıcio 4.48 . . . . . . . . . . . . . . . . . . . . . . . . . . . . . . . . . . . . . . . . . . . . . . . . . . . . 173Exercıcio 4.49 . . . . . . . . . . . . . . . . . . . . . . . . . . . . . . . . . . . . . . . . . . . . . . . . . . . . 175

5 Topologia da Reta 177Exercıcio 5.01 . . . . . . . . . . . . . . . . . . . . . . . . . . . . . . . . . . . . . . . . . . . . . . . . . . . . 178Exercıcio 5.02 . . . . . . . . . . . . . . . . . . . . . . . . . . . . . . . . . . . . . . . . . . . . . . . . . . . . 179Exercıcio 5.03 . . . . . . . . . . . . . . . . . . . . . . . . . . . . . . . . . . . . . . . . . . . . . . . . . . . . 180Exercıcio 5.04 . . . . . . . . . . . . . . . . . . . . . . . . . . . . . . . . . . . . . . . . . . . . . . . . . . . . 181Exercıcio 5.05 . . . . . . . . . . . . . . . . . . . . . . . . . . . . . . . . . . . . . . . . . . . . . . . . . . . . 182Exercıcio 5.06 . . . . . . . . . . . . . . . . . . . . . . . . . . . . . . . . . . . . . . . . . . . . . . . . . . . . 183

3

Exercıcio 5.07 . . . . . . . . . . . . . . . . . . . . . . . . . . . . . . . . . . . . . . . . . . . . . . . . . . . . 184Exercıcio 5.08 . . . . . . . . . . . . . . . . . . . . . . . . . . . . . . . . . . . . . . . . . . . . . . . . . . . . 185Exercıcio 5.09 . . . . . . . . . . . . . . . . . . . . . . . . . . . . . . . . . . . . . . . . . . . . . . . . . . . . 186Exercıcio 5.10 . . . . . . . . . . . . . . . . . . . . . . . . . . . . . . . . . . . . . . . . . . . . . . . . . . . . 187Exercıcio 5.11 . . . . . . . . . . . . . . . . . . . . . . . . . . . . . . . . . . . . . . . . . . . . . . . . . . . . 188Exercıcio 5.12 . . . . . . . . . . . . . . . . . . . . . . . . . . . . . . . . . . . . . . . . . . . . . . . . . . . . 189Exercıcio 5.13 . . . . . . . . . . . . . . . . . . . . . . . . . . . . . . . . . . . . . . . . . . . . . . . . . . . . 190Exercıcio 5.14 . . . . . . . . . . . . . . . . . . . . . . . . . . . . . . . . . . . . . . . . . . . . . . . . . . . . 191Exercıcio 5.15 . . . . . . . . . . . . . . . . . . . . . . . . . . . . . . . . . . . . . . . . . . . . . . . . . . . . 192Exercıcio 5.16 . . . . . . . . . . . . . . . . . . . . . . . . . . . . . . . . . . . . . . . . . . . . . . . . . . . . 193Exercıcio 5.17 . . . . . . . . . . . . . . . . . . . . . . . . . . . . . . . . . . . . . . . . . . . . . . . . . . . . 194Exercıcio 5.18 . . . . . . . . . . . . . . . . . . . . . . . . . . . . . . . . . . . . . . . . . . . . . . . . . . . . 196Exercıcio 5.19 . . . . . . . . . . . . . . . . . . . . . . . . . . . . . . . . . . . . . . . . . . . . . . . . . . . . 197Exercıcio 5.20 . . . . . . . . . . . . . . . . . . . . . . . . . . . . . . . . . . . . . . . . . . . . . . . . . . . . 198Exercıcio 5.21 . . . . . . . . . . . . . . . . . . . . . . . . . . . . . . . . . . . . . . . . . . . . . . . . . . . . 199Exercıcio 5.22 . . . . . . . . . . . . . . . . . . . . . . . . . . . . . . . . . . . . . . . . . . . . . . . . . . . . 200Exercıcio 5.23 . . . . . . . . . . . . . . . . . . . . . . . . . . . . . . . . . . . . . . . . . . . . . . . . . . . . 201Exercıcio 5.24 . . . . . . . . . . . . . . . . . . . . . . . . . . . . . . . . . . . . . . . . . . . . . . . . . . . . 202Exercıcio 5.25 . . . . . . . . . . . . . . . . . . . . . . . . . . . . . . . . . . . . . . . . . . . . . . . . . . . . 203Exercıcio 5.26 . . . . . . . . . . . . . . . . . . . . . . . . . . . . . . . . . . . . . . . . . . . . . . . . . . . . 204Exercıcio 5.27 . . . . . . . . . . . . . . . . . . . . . . . . . . . . . . . . . . . . . . . . . . . . . . . . . . . . 205Exercıcio 5.28 . . . . . . . . . . . . . . . . . . . . . . . . . . . . . . . . . . . . . . . . . . . . . . . . . . . . 206Exercıcio 5.29 . . . . . . . . . . . . . . . . . . . . . . . . . . . . . . . . . . . . . . . . . . . . . . . . . . . . 207Exercıcio 5.30 . . . . . . . . . . . . . . . . . . . . . . . . . . . . . . . . . . . . . . . . . . . . . . . . . . . . 208Exercıcio 5.31 . . . . . . . . . . . . . . . . . . . . . . . . . . . . . . . . . . . . . . . . . . . . . . . . . . . . 209Exercıcio 5.32 . . . . . . . . . . . . . . . . . . . . . . . . . . . . . . . . . . . . . . . . . . . . . . . . . . . . 210Exercıcio 5.33 . . . . . . . . . . . . . . . . . . . . . . . . . . . . . . . . . . . . . . . . . . . . . . . . . . . . 211Exercıcio 5.34 . . . . . . . . . . . . . . . . . . . . . . . . . . . . . . . . . . . . . . . . . . . . . . . . . . . . 212Exercıcio 5.35 . . . . . . . . . . . . . . . . . . . . . . . . . . . . . . . . . . . . . . . . . . . . . . . . . . . . 213Exercıcio 5.36 . . . . . . . . . . . . . . . . . . . . . . . . . . . . . . . . . . . . . . . . . . . . . . . . . . . . 214Exercıcio 5.37 . . . . . . . . . . . . . . . . . . . . . . . . . . . . . . . . . . . . . . . . . . . . . . . . . . . . 215Exercıcio 5.38 . . . . . . . . . . . . . . . . . . . . . . . . . . . . . . . . . . . . . . . . . . . . . . . . . . . . 216Exercıcio 5.39 . . . . . . . . . . . . . . . . . . . . . . . . . . . . . . . . . . . . . . . . . . . . . . . . . . . . 217Exercıcio 5.40 . . . . . . . . . . . . . . . . . . . . . . . . . . . . . . . . . . . . . . . . . . . . . . . . . . . . 218Exercıcio 5.41 . . . . . . . . . . . . . . . . . . . . . . . . . . . . . . . . . . . . . . . . . . . . . . . . . . . . 219Exercıcio 5.42 . . . . . . . . . . . . . . . . . . . . . . . . . . . . . . . . . . . . . . . . . . . . . . . . . . . . 220Exercıcio 5.43 . . . . . . . . . . . . . . . . . . . . . . . . . . . . . . . . . . . . . . . . . . . . . . . . . . . . 221Exercıcio 5.44 . . . . . . . . . . . . . . . . . . . . . . . . . . . . . . . . . . . . . . . . . . . . . . . . . . . . 222Exercıcio 5.45 . . . . . . . . . . . . . . . . . . . . . . . . . . . . . . . . . . . . . . . . . . . . . . . . . . . . 223Exercıcio 5.46 . . . . . . . . . . . . . . . . . . . . . . . . . . . . . . . . . . . . . . . . . . . . . . . . . . . . 224Exercıcio 5.47 . . . . . . . . . . . . . . . . . . . . . . . . . . . . . . . . . . . . . . . . . . . . . . . . . . . . 225Exercıcio 5.48 . . . . . . . . . . . . . . . . . . . . . . . . . . . . . . . . . . . . . . . . . . . . . . . . . . . . 226Exercıcio 5.49 . . . . . . . . . . . . . . . . . . . . . . . . . . . . . . . . . . . . . . . . . . . . . . . . . . . . 227Exercıcio 5.50 . . . . . . . . . . . . . . . . . . . . . . . . . . . . . . . . . . . . . . . . . . . . . . . . . . . . 228Exercıcio 5.51 . . . . . . . . . . . . . . . . . . . . . . . . . . . . . . . . . . . . . . . . . . . . . . . . . . . . 229Exercıcio 5.52 . . . . . . . . . . . . . . . . . . . . . . . . . . . . . . . . . . . . . . . . . . . . . . . . . . . . 230Exercıcio 5.53 . . . . . . . . . . . . . . . . . . . . . . . . . . . . . . . . . . . . . . . . . . . . . . . . . . . . 231Exercıcio 5.54 . . . . . . . . . . . . . . . . . . . . . . . . . . . . . . . . . . . . . . . . . . . . . . . . . . . . 232Exercıcio 5.55 . . . . . . . . . . . . . . . . . . . . . . . . . . . . . . . . . . . . . . . . . . . . . . . . . . . . 233Exercıcio 5.56 . . . . . . . . . . . . . . . . . . . . . . . . . . . . . . . . . . . . . . . . . . . . . . . . . . . . 234Exercıcio 5.57 . . . . . . . . . . . . . . . . . . . . . . . . . . . . . . . . . . . . . . . . . . . . . . . . . . . . 236Exercıcio 5.58 . . . . . . . . . . . . . . . . . . . . . . . . . . . . . . . . . . . . . . . . . . . . . . . . . . . . 237Exercıcio 5.59 . . . . . . . . . . . . . . . . . . . . . . . . . . . . . . . . . . . . . . . . . . . . . . . . . . . . 238Exercıcio 5.60 . . . . . . . . . . . . . . . . . . . . . . . . . . . . . . . . . . . . . . . . . . . . . . . . . . . . 239Exercıcio 5.61 . . . . . . . . . . . . . . . . . . . . . . . . . . . . . . . . . . . . . . . . . . . . . . . . . . . . 240Exercıcio 5.62 . . . . . . . . . . . . . . . . . . . . . . . . . . . . . . . . . . . . . . . . . . . . . . . . . . . . 241Exercıcio 5.63 . . . . . . . . . . . . . . . . . . . . . . . . . . . . . . . . . . . . . . . . . . . . . . . . . . . . 242

4

Exercıcio 5.64 . . . . . . . . . . . . . . . . . . . . . . . . . . . . . . . . . . . . . . . . . . . . . . . . . . . . 243

6 Limites de Funcoes 246Exercıcio 6.01 . . . . . . . . . . . . . . . . . . . . . . . . . . . . . . . . . . . . . . . . . . . . . . . . . . . . 247Exercıcio 6.02 . . . . . . . . . . . . . . . . . . . . . . . . . . . . . . . . . . . . . . . . . . . . . . . . . . . . 248Exercıcio 6.03 . . . . . . . . . . . . . . . . . . . . . . . . . . . . . . . . . . . . . . . . . . . . . . . . . . . . 249Exercıcio 6.04 . . . . . . . . . . . . . . . . . . . . . . . . . . . . . . . . . . . . . . . . . . . . . . . . . . . . 250Exercıcio 6.05 . . . . . . . . . . . . . . . . . . . . . . . . . . . . . . . . . . . . . . . . . . . . . . . . . . . . 251Exercıcio 6.06 . . . . . . . . . . . . . . . . . . . . . . . . . . . . . . . . . . . . . . . . . . . . . . . . . . . . 252Exercıcio 6.07 . . . . . . . . . . . . . . . . . . . . . . . . . . . . . . . . . . . . . . . . . . . . . . . . . . . . 253Exercıcio 6.08 . . . . . . . . . . . . . . . . . . . . . . . . . . . . . . . . . . . . . . . . . . . . . . . . . . . . 254Exercıcio 6.09 . . . . . . . . . . . . . . . . . . . . . . . . . . . . . . . . . . . . . . . . . . . . . . . . . . . . 255Exercıcio 6.10 . . . . . . . . . . . . . . . . . . . . . . . . . . . . . . . . . . . . . . . . . . . . . . . . . . . . 256Exercıcio 6.11 . . . . . . . . . . . . . . . . . . . . . . . . . . . . . . . . . . . . . . . . . . . . . . . . . . . . 258Exercıcio 6.12 . . . . . . . . . . . . . . . . . . . . . . . . . . . . . . . . . . . . . . . . . . . . . . . . . . . . 260Exercıcio 6.13 . . . . . . . . . . . . . . . . . . . . . . . . . . . . . . . . . . . . . . . . . . . . . . . . . . . . 261Exercıcio 6.14 . . . . . . . . . . . . . . . . . . . . . . . . . . . . . . . . . . . . . . . . . . . . . . . . . . . . 262Exercıcio 6.15 . . . . . . . . . . . . . . . . . . . . . . . . . . . . . . . . . . . . . . . . . . . . . . . . . . . . 263Exercıcio 6.16 . . . . . . . . . . . . . . . . . . . . . . . . . . . . . . . . . . . . . . . . . . . . . . . . . . . . 264Exercıcio 6.17 . . . . . . . . . . . . . . . . . . . . . . . . . . . . . . . . . . . . . . . . . . . . . . . . . . . . 265Exercıcio 6.18 . . . . . . . . . . . . . . . . . . . . . . . . . . . . . . . . . . . . . . . . . . . . . . . . . . . . 266Exercıcio 6.19 . . . . . . . . . . . . . . . . . . . . . . . . . . . . . . . . . . . . . . . . . . . . . . . . . . . . 267Exercıcio 6.20 . . . . . . . . . . . . . . . . . . . . . . . . . . . . . . . . . . . . . . . . . . . . . . . . . . . . 269Exercıcio 6.21 . . . . . . . . . . . . . . . . . . . . . . . . . . . . . . . . . . . . . . . . . . . . . . . . . . . . 270Exercıcio 6.22 . . . . . . . . . . . . . . . . . . . . . . . . . . . . . . . . . . . . . . . . . . . . . . . . . . . . 273Exercıcio 6.23 . . . . . . . . . . . . . . . . . . . . . . . . . . . . . . . . . . . . . . . . . . . . . . . . . . . . 275Exercıcio 6.24 . . . . . . . . . . . . . . . . . . . . . . . . . . . . . . . . . . . . . . . . . . . . . . . . . . . . 276

7 Funcoes Contınuas 277Exercıcio 7.38 . . . . . . . . . . . . . . . . . . . . . . . . . . . . . . . . . . . . . . . . . . . . . . . . . . . . 278Exercıcio 7.39 . . . . . . . . . . . . . . . . . . . . . . . . . . . . . . . . . . . . . . . . . . . . . . . . . . . . 280Exercıcio 7.40 . . . . . . . . . . . . . . . . . . . . . . . . . . . . . . . . . . . . . . . . . . . . . . . . . . . . 281Exercıcio 7.41 . . . . . . . . . . . . . . . . . . . . . . . . . . . . . . . . . . . . . . . . . . . . . . . . . . . . 282Exercıcio 7.42 . . . . . . . . . . . . . . . . . . . . . . . . . . . . . . . . . . . . . . . . . . . . . . . . . . . . 284Exercıcio 7.43 . . . . . . . . . . . . . . . . . . . . . . . . . . . . . . . . . . . . . . . . . . . . . . . . . . . . 285Exercıcio 7.44 . . . . . . . . . . . . . . . . . . . . . . . . . . . . . . . . . . . . . . . . . . . . . . . . . . . . 286Exercıcio 7.45 . . . . . . . . . . . . . . . . . . . . . . . . . . . . . . . . . . . . . . . . . . . . . . . . . . . . 287Exercıcio 7.46 . . . . . . . . . . . . . . . . . . . . . . . . . . . . . . . . . . . . . . . . . . . . . . . . . . . . 288Exercıcio 7.47 . . . . . . . . . . . . . . . . . . . . . . . . . . . . . . . . . . . . . . . . . . . . . . . . . . . . 289

8 Derivadas 297Exercıcio 8.46 . . . . . . . . . . . . . . . . . . . . . . . . . . . . . . . . . . . . . . . . . . . . . . . . . . . . 298Exercıcio 8.47 . . . . . . . . . . . . . . . . . . . . . . . . . . . . . . . . . . . . . . . . . . . . . . . . . . . . 299Exercıcio 8.48 . . . . . . . . . . . . . . . . . . . . . . . . . . . . . . . . . . . . . . . . . . . . . . . . . . . . 300Exercıcio 8.49 . . . . . . . . . . . . . . . . . . . . . . . . . . . . . . . . . . . . . . . . . . . . . . . . . . . . 301Exercıcio 8.50 . . . . . . . . . . . . . . . . . . . . . . . . . . . . . . . . . . . . . . . . . . . . . . . . . . . . 302Exercıcio 8.51 . . . . . . . . . . . . . . . . . . . . . . . . . . . . . . . . . . . . . . . . . . . . . . . . . . . . 303Exercıcio 8.52 . . . . . . . . . . . . . . . . . . . . . . . . . . . . . . . . . . . . . . . . . . . . . . . . . . . . 305Exercıcio 8.53 . . . . . . . . . . . . . . . . . . . . . . . . . . . . . . . . . . . . . . . . . . . . . . . . . . . . 306Exercıcio 8.54 . . . . . . . . . . . . . . . . . . . . . . . . . . . . . . . . . . . . . . . . . . . . . . . . . . . . 307Exercıcio 8.55 . . . . . . . . . . . . . . . . . . . . . . . . . . . . . . . . . . . . . . . . . . . . . . . . . . . . 308

9 Integral de Riemann 309Exercıcio 9.36 . . . . . . . . . . . . . . . . . . . . . . . . . . . . . . . . . . . . . . . . . . . . . . . . . . . . 310Exercıcio 9.37 . . . . . . . . . . . . . . . . . . . . . . . . . . . . . . . . . . . . . . . . . . . . . . . . . . . . 311Exercıcio 9.38 . . . . . . . . . . . . . . . . . . . . . . . . . . . . . . . . . . . . . . . . . . . . . . . . . . . . 312

5

10 Sequencias e Series de Funcoes 315Exercıcio 10.44 . . . . . . . . . . . . . . . . . . . . . . . . . . . . . . . . . . . . . . . . . . . . . . . . . . . 316Exercıcio 10.45 . . . . . . . . . . . . . . . . . . . . . . . . . . . . . . . . . . . . . . . . . . . . . . . . . . . 317Exercıcio 10.46 . . . . . . . . . . . . . . . . . . . . . . . . . . . . . . . . . . . . . . . . . . . . . . . . . . . 318Exercıcio 10.47 . . . . . . . . . . . . . . . . . . . . . . . . . . . . . . . . . . . . . . . . . . . . . . . . . . . 320Exercıcio 10.48 . . . . . . . . . . . . . . . . . . . . . . . . . . . . . . . . . . . . . . . . . . . . . . . . . . . 321Exercıcio 10.49 . . . . . . . . . . . . . . . . . . . . . . . . . . . . . . . . . . . . . . . . . . . . . . . . . . . 323Exercıcio 10.50 . . . . . . . . . . . . . . . . . . . . . . . . . . . . . . . . . . . . . . . . . . . . . . . . . . . 324Exercıcio 10.51 . . . . . . . . . . . . . . . . . . . . . . . . . . . . . . . . . . . . . . . . . . . . . . . . . . . 325Exercıcio 10.52 . . . . . . . . . . . . . . . . . . . . . . . . . . . . . . . . . . . . . . . . . . . . . . . . . . . 326Exercıcio 10.53 . . . . . . . . . . . . . . . . . . . . . . . . . . . . . . . . . . . . . . . . . . . . . . . . . . . 327

6

Capıtulo 1

Conjuntos e Funcoes

7

Exercıcio 1.1:

Dados os conjuntos A e B, seja X um conjunto com as seguintes propriedades:

(1a) X ⊃ A e X ⊃ B,

(2a) Se Y ⊃ A e Y ⊃ B entao Y ⊃ X.

Prove que X = A ∪B.

A inclusao A ∪ B ⊂ X e fornecida pela primeira hipotese. De fato, se x ∈ A ⊂ X ou x ∈ B ⊂ X (isto e, sex ∈ A ∪B) entao x ∈ X.

E a segunda hipotese fornece a inclusao A ∪B ⊂ X pois A ∪B ⊃ A e A ∪B ⊃ B.Portanto, X = A ∪B.

8

Exercıcio 1.2:

Enuncie e prove um resultado, analogo ao anterior, caracterizando A ∩B.

Enunciado:Dados os conjuntos A e B, seja X um conjunto com as seguintes propriedades:

1a X ⊂ A e X ⊂ B,

2a Se Y ⊂ A e Y ⊂ B entao Y ⊂ X.

Prove que X = A ∩B.Prova:A inclusao A∩B ⊃ X e fornecida pela primeira hipotese. De fato, se x ∈ X temos que A ⊃ X 3 x e B ⊃ X 3 x.

Consequentemente, se x ∈ X entao x ∈ A ∩B.E a segunda hipotese fornece a inclusao A ∩B ⊂ X pois A ∩B ⊂ A e A ∩B ⊂ B.Portanto, X = A ∩B.

9

Exercıcio 1.3:

Sejam A,B ⊂ E. Prove que A ∩ B = ∅ se, e somente se, A ⊂ E\B. Prove tambem que A ∪ B = E se, e somentese, E\A ⊂ B.

• A ∩B = ∅ se e somente se A ⊂ E\B:

Suponhamos que A ∩ B = ∅. Se x ∈ A devemos ter que x pertence a E\B. De fato, como x pertence a A eA esta contido em E, segue que x pertence a B ou E\B. Como A ∩ B = ∅, temos que x /∈ B. Logo, x ∈ E\B.Assim, A ⊂ E\B.

Consideremos o caso em que A ⊂ E\B. Se existisse x ∈ A∩B terıamos que x ∈ A e x ∈ B. Mas, como A e umsubconjunto de E\B, terıamos tambem que x ∈ E\B. Um absurdo, pois se x ∈ E\B entao x /∈ B. Desta forma,concluimos que A ∩B = ∅.

• A ∪B = E se e somente se E\A ⊂ B:

Suponhamos que A ∪ B = E. Se x ∈ E\A devemos ter que x pertence a B. De fato, como x pertence a E eE = A ∪ B, devemos ter que x ∈ A ou x ∈ B. Alem disso, como x ∈ E\A, temos tambem que x /∈ A. O que nosgarante que x ∈ B. Logo, E\A ⊂ B.

Consideremos o caso em que E\A ⊂ B. Seja x ∈ E. Segue que, x ∈ A ou x ∈ E\A. Se x ∈ E\A entao xpertence a B pois E\A esta contido em B. Logo, x ∈ A ou x ∈ B. Ou seja, x ∈ A ∪ B. Assim, devemos ter queE ⊂ A ∪B. E, como A e B estao contidos em E, segue (veja o exercicio 1.1) que E = A ∪B.

10

Exercıcio 1.4:

Dados A,B ⊂ E, prove que A ⊂ B se, e somente se, A ∩ (E\B) = ∅.

Suponhamos que A ⊂ B. Se existisse x ∈ A∩ (E\B) terıamos que x ∈ A e x ∈ E\B. Isto e, existiria x ∈ E talque x ∈ A e x /∈ B. Mas, isto e um absurdo, pois, como A ⊂ B, se x ∈ A entao x ∈ B. Portanto, A ∩ (E\B) = ∅.

Consideremos, agora, o caso em que A ∩ (E\B) = ∅. Seja x ∈ A. Como A ⊂ E, temos que x ∈ E. Assim,x ∈ B ou x ∈ E\B. Logo, x ∈ B pois se x ∈ E\B terıamos que x ∈ A ∩ (E\B) = ∅.

11

Exercıcio 1.5:

De exemplo de conjuntos A,B,C tais que (A ∪B) ∩ C 6= A ∪ (B ∩ C).

Tome A = {1, 2, 3}, B = {1, 3} e C = {1, 2}. Desta forma, temos

(A ∪B) ∩ C = {1, 2} 6= {1, 2, 3} = A ∪ (B ∩ C).

12

Exercıcio 1.6:

Se A,X ⊂ E sao tais que A ∩X = ∅ e A ∪X = E, prove que X = E\A.

Seja x ∈ X. Uma vez que x /∈ ∅ = A ∩X, temos que x /∈ A. E, como x ∈ X ⊂ E, devemos ter, tambem, quex ∈ E. Logo, x ∈ E\A. Portanto, como x ∈ X e arbitraro, devemos ter que X ⊂ E\A.

Considere, agora, x ∈ E\A. Segue que x ∈ E e x /∈ A. Como x ∈ E = A ∪ X e x /∈ A, temos que x ∈ X.Portanto, como x ∈ E\A e arbitraro, devemos ter que X ⊂ E\A.

13

Exercıcio 1.7:

Se A ⊂ B, entaoB ∩ (A ∪ C) = (B ∩ C) ∪A,

para todo conjunto C. Por outro lado, se existir C de modo que a igualdade acima seja satisfeita, entao A ⊂ B.

Primeiramente, mostremos que se A ⊂ B entao, para qualquer conjunto C, temos

B ∩ (A ∪ C) = (B ∩ C) ∪A.

Seja x ∈ B ∩ (A ∪ C). Assim, x ∈ B e (x ∈ C ou x ∈ A).

• Se x ∈ C temos que x ∈ B ∩ C. Logo, x ∈ (B ∩ C) ∪A.

• Se x ∈ A temos imediatamente que x ∈ (B ∩ C) ∪A.

Segue, em todo caso, que x ∈ (B ∩ C) ∪A. Logo, concluimos que B ∩ (A ∪ C) ⊂ (B ∩ C) ∪A.Considere, agora, que x ∈ (B ∩ C) ∪A. Assim, x ∈ B ∩ C ou x ∈ A.

• Se x ∈ B ∩ C entao x ∈ B e x ∈ C. Logo, x ∈ B, x ∈ A ∪ C e, consequentemente, x ∈ B ∩ (A ∪ C).

• Se x ∈ A temos que x ∈ B, ja que A ⊂ B. Assim, x ∈ B e x ∈ A ⊂ A ∪ C. Logo, x ∈ B ∩ (A ∪ C).

Em ambos os casos, x ∈ B ∩ (A ∪B). Desta forma, tem-se que B ∩ (A ∪ C) ⊃ (B ∩ C) ∪A.Portanto, se A ⊂ B entao B ∩ (A ∪ C) = (B ∩ C) ∪A, para qualquer conjunto C.Reciprocamente, suponhamos que exista um conjunto C tal que x ∈ (B ∩ C) ∪A = B ∩ (A ∪ C).Se x ∈ A temos que x ∈ (B ∩ C) ∪ A. Mas, como (B ∩ C) ∪ A = B ∩ (A ∪ C), devemos ter que x ∈ B. Logo,

conclui-se que A ⊂ B.

14

Exercıcio 1.8:

Suponhamos que A e B sejam subconjuntos de E. Prove que A = B se, e somente se,(A ∩ (E\B)

)∪((E\A) ∩B

)= ∅.

Suponhamos que A = B. Neste caso, temos que

E\A = E\B.

Logo,A ∩ (E\B) = A ∩ (E\A) = ∅

eB ∩ (E\A) = B ∩ (E\B) = ∅.

Portanto, (A ∩ (E\B)

)∪(B ∩ (E\A)

)= ∅ ∪ ∅ = ∅.

Reciprocamente, consideremos o caso em que(A ∩ (E\B)

)∪(B ∩ (E\A)

)= ∅.

Seja x ∈ A. Se supusermos, por absurdo, que x /∈ B teremos que x ∈ A ∩ (E\B) e, consequentemente,

x ∈(A ∩ (E\B)

)∪(B ∩ (E\A)

)= ∅.

Uma contradicao. De modo inteiramente analogo e impossıvel que x ∈ B e x /∈ A. Portanto, A = B.

15

Exercıcio 1.9:

Prove que(A\B) ∪ (B\A) = (A ∪B)\(A ∩B).

• (A\B) ∪ (B\A) ⊂ (A ∪B)\(A ∩B)

Seja x ∈ (A\B) ∪ (B\A). Neste caso, x ∈ A\B ou x ∈ B\A. Se x ∈ B\A entao temos que x ∈ A e x /∈ B.Logo, x ∈ A∪B e x /∈ A∩B, ou seja, x ∈ (A∪B)\(A∩B). Analogamente, x ∈ B\A implica x ∈ (A∪B)\(A∩B).

• (A\B) ∪ (B\A) ⊃ (A ∪B)\(A ∩B)

Seja x ∈ (A∪B)\(A∩B). Neste caso, x ∈ A∪B e x /∈ A∩B. Se x ∈ A entao x /∈ B, uma vez que x /∈ A∩B.Isto e, se x ∈ A entao x ∈ A\B. Analogamente, se x ∈ B, temos que x ∈ B\A. Portanto, x ∈ (A\B) ∪ (B\A).

16

Exercıcio 1.10:

Para conjuntos A e B, definimos o conjunto

A∆B := (A\B) ∪ (B\A).

Prove que A∆B = A∆C implica que B = C. Examine a validade um resultado analogo com ∩, ∪ ou × em vez de∆.

Suponhamos que A∆B = A∆C.Mostraremos que os conjuntos B ∩ A e B\A estao contidos em C. Desta forma, como B = (B ∩ A) ∪ (B\A),

concluiremos que B ⊂ C.Seja x ∈ B∩A. Temos que x /∈ A∆B = (A\B)∪ (B\A), pois x /∈ A\B e x /∈ B\A. Assim, como A∆B = A∆C,

temos que x /∈ A∆C = (A\C) ∪ (C\A) e, consequentemente, x /∈ A\C. Logo, x ∈ C pois x ∈ A e x /∈ A\C. Comox ∈ B ∩A e arbitrario, concluimos que B ∩A ⊂ C.

Seja x ∈ B\A. Logo, x ∈ (A\B) ∪ (B\A) = A∆B. E, como A∆B = A∆C, temos que x ∈ A∆C. Sendox ∈ A∆C = (A\C) ∪ (C\A), segue que x ∈ A\C ou x /∈ C\A. Assim, ja que x /∈ A, devemos ter que x ∈ C\A e,consequentemente, x ∈ C. Como x ∈ B\A e arbitrario, concluimos que B\A ⊂ C.

Por fim, como B ∩ A e B\A estao contidos em C, devemos ter que B ⊂ C. E, de forma analoga, prova-se queC ∩A e C\A estao contidos em B. Logo, C ⊂ B. Portanto, supondo que A∆B = A∆C, temos que B = C.

Consideremos agora a validade dos casos analogos para ∩, ∪ e × ao inves de ∆.Existem A, B e C tais que

• A ∩B = A ∩ C e B 6= C. Por exemplo: A = {1}, B = {1, 2} e C = {1, 2, 3};

• A ∪B = A ∪ C e B 6= C. Por exemplo: A = {1}, B = {2} e C = {1, 2};

• A×B = A× C e B 6= C. Por exemplo: A = ∅, B = {1} e C = {2}.

17

Exercıcio 1.11:

Prove as seguintes afirmacoes:

(a) (A ∪B)× C = (A× C) ∪ (B × C);

(b) (A ∩B)× C = (A× C) ∩ (B × C);

(c) (A−B)× C = (A× C)− (B × C);

(d) A ⊂ A′, B ⊂ B′ =⇒ A×B ⊂ A′ ×B′.

(a) Temos que a igualdade (A ∪B)× C = (A× C) ∪ (B × C) e valida pois

(x, c) ∈ (A ∪B)× C ⇐⇒ x ∈ A ∪B e c ∈ C⇐⇒ (x ∈ A e c ∈ C) ou (x ∈ B e c ∈ C)⇐⇒ (x, c) ∈ A× C ou (x, c) ∈ B × C⇐⇒ (x, c) ∈ (A× C) ∪ (B × C).

(b) Temos que a igualdade (A ∩B)× C = (A× C) ∩ (B × C) e valida pois

(x, c) ∈ (A ∩B)× C ⇐⇒ x ∈ (A ∩B) e c ∈ C⇐⇒ (x ∈ A e c ∈ C) e (x ∈ B e c ∈ C)⇐⇒ (x, c) ∈ A× C e (x, c) ∈ B × C⇐⇒ (x, c) ∈ (A× C) ∩ (B × C).

(c) Temos que a igualdade (A−B)× C = (A× C)− (B × C) e valida pois

(x, c) ∈ (A−B)× C ⇐⇒ x ∈ A−B e c ∈ C⇐⇒ (x ∈ A e c ∈ C) e (x /∈ B e c ∈ C)⇐⇒ (x, c) ∈ A× C e (x, c) /∈ B × C⇐⇒ (x, c) ∈ (A× C)− (B × C).

(d) Seja (a, b) ∈ A × B. Entao, a ∈ A′ e b ∈ B′ pois A ⊂ A′ e B ⊂ B′. Logo, (a, b) ∈ A′ × B′. Portanto,concluimos que A×B ⊂ A′ ×B′.

18

Exercıcio 1.12:

Dada uma funcao f : A→ B:

(a) Prove que se tem f(X\Y ) ⊃ f(X)\f(Y ), sejam quais forem os subconjuntos X e Y de A;

(b) Mostre que se f for injetora entao f(X\Y ) = f(X)\f(Y ) para quaisquer X e Y contidos em A.

(a)

Suponhamos que z ∈ f(X)\f(Y ). Desta forma, temos que z ∈ f(X) e, consequentemente, existe x ∈ X talque f(x) = z. Como z /∈ f(Y ) e z = f(x), devemos ter que x /∈ Y . Logo, x ∈ X\Y . Assim, concluimos quez = f(x) ∈ f(X\Y ).

Portanto, devemos ter que f(X\Y ) ⊂ f(X)\f(Y ).

(b)

Pelo item (a), temos que f(X\Y ) ⊂ f(X)\f(Y ). Logo, basta verificarmos que f(X\Y ) ⊃ f(X)\f(Y ).Seja z ∈ f(X\Y ). Entao, podemos escolher x ∈ X\Y tal que f(x) = z. Assim, z = f(x) ∈ f(X) pois x ∈ X.

Por outro lado, como f e injetivo, f(x) = z e x /∈ Y , nenhum y ∈ Y e tal que f(y) = z. Logo, z /∈ f(Y ). Portanto,z ∈ f(X)\f(Y ).

Com isso, concluimos que f(X\Y ) = f(X)\f(Y ).

19

Exercıcio 1.13:

Mostre que a funcao f : A→ B e injetora se, e somente se, f(A\X) = f(A)\f(X) para todo X ⊂ A.

Se f : A→ B e injetiva, pelo item (b) do exercıcio 1.12, a igualdade f(A\X) = f(A)\f(X) e valida para todoX ⊂ A.

Suponhamos que a igualdade f(A\X) = f(A)\f(X) seja valida para todo X ⊂ A. Seja a ∈ A e denotemos porb o elemento f(a) ∈ B. Assim,

b /∈ f(A\{a}) = f(A)\f({a}).

Logo, nao existe a′ ∈ A\{a} tal que f(a′) = b = f(a). Desta forma, como a ∈ A e arbitrario, concluimos que f einjetivo.

20

Exercıcio 1.14:

Dada a funcao f : A→ B, prove que:

(a) f−1(f(X)) ⊃ X para todo X ⊂ A;

(b) f e injetora se, e somente se, f−1(f(X)) = X para todo X ⊂ A.

(a)

Se x ∈ X entao x ∈ f−1(f(X)) pois f(x) ∈ f(X). Assim, devemos ter que f−1(f(X)) ⊃ X.

(b)

Suponhamos que f e injetora e fixemos X ⊂ A. Provaremos que f−1(f(X)) ⊂ X e concluiremos, pelo item(a), que f−1(f(X)) = X. Desta forma, podemos concluir que se f e injetora entao f−1(f(X)) = X, para qualquerX ⊂ A.

Seja y ∈ f−1(f(X)). Segue que f(y) ∈ f(X). Assim, existe x ∈ X tal que f(x) = f(y). Sendo f injetiva,conclui-se que y = x ∈ X. Portanto, como y ∈ f−1(f(X)) e arbitrario, temos que f−1(f(X)) ⊂ X.

Suponhamos, por outro lado, que f seja tal que f−1(f(X)) = X, para qualquer X ⊂ A. Sejam x e y ∈ Atais que f(x) = f(y). Neste caso, temos que f({x}) = f({x, y}). Assim, f−1(f({x})) = f−1(f({x, y})) e, pelahipotese,

{x} = f−1(f({x})) = f−1(f({x, y})) = {x, y}.

Desta forma, y ∈ {x} e, consequentemente, x = y. Com isso, concluimos que se x e y ∈ A sao tais que f(x) = f(y)entao x = y. Portanto, f e injetiva.

21

Exercıcio 1.15:

Dada f : A→ B, prove:

(a) Para todo Z ⊂ B, tem-se que f(f−1(Z)) ⊂ Z;

(b) f e sobrejetora se, e somente se, f(f−1(Z)) = Z para todo Z ⊂ B.

(a)

Seja z ∈ f(f−1(Z)). Existe x ∈ f−1(Z) tal que f(x) = z. Assim, como x ∈ f−1(Z), z = f(x) ∈ Z.Portanto, podemos concluir que f(f−1(Z)) ⊂ Z.

(b)

Suponhamos que f seja sobrejetora. Provaremos, para um Z ⊂ B arbitrario, que f(f−1(Z)) = Z.Pelo item (a), temos que f(f−1(Z)) ⊂ Z.Seja z ∈ Z. Como f e sobrejetiva, existe x ∈ A tal que z = f(x). Desta forma, como f(x) = z ∈ Z, segue que

x ∈ f−1(Z). Logo, z = f(x) ∈ f(f−1(Z)).Desta forma, concluimos que f(f−1(Z)) ⊃ Z.Portanto, devemos ter que f(f−1(Z)) = Z.Suponhamos, por outro lado, que f(f−1(Z)) = Z, para todo Z ⊂ B.Seja z ∈ B. Definindo Z = {z}, temos que

f(f−1(Z)) = Z = {z}.

Desta forma, temos que z ∈ f(f−1(Z)). Assim, existe x ∈ f−1(Z) ⊂ A tal que f(x) = z.Portanto, neste caso, f e sobrejetiva.

22

Exercıcio 1.16:

Dada uma famılia de conjuntos (Aλ)λ∈L, seja X um conjunto com as seguintes propriedades:

(1a) Para todo λ ∈ L, tem-se X ⊃ Aλ;

(2a) Se Y ⊃ Aλ, para todo λ ∈ L, entao Y ⊃ X.

Prove que, nestas condicoes, tem-se X =⋃λ∈L

Aλ.

Pela primeira condicao, temos que X ⊃ Aλ para cada λ ∈ L. Assim,⋃λ∈L

Aλ ⊂ X pois cada x ∈⋃λ∈L

Aλ pertence

a Aλ ⊂ X, para algum λ ∈ L.

O conjunto⋃λ∈L

Aλ e tal que⋃λ∈L

Aλ ⊃ Aλ, para todo λ ∈ L. Logo, pela segunda condicao,⋃λ∈L

Aλ ⊃ X.

Portanto, X =⋃λ∈L

Aλ.

23

Exercıcio 1.17:

Enuncie e demonstre um resultado analogo ao anterior, caracterizando⋂λ∈L

Aλ.

Enunciado: Dada uma famılia de conjuntos (Aλ)λ∈L, seja X um conjunto com as seguintes propriedades:

(1a) Para todo λ ∈ L, tem-se X ⊂ Aλ;

(2a) Se Y ⊂ Aλ para todo λ ∈ L, entao Y ⊂ X.

Nestas condicoes, tem-se X =⋂λ∈L

Aλ.

Demonstracao:

Todo elemento x de X pertence a⋂λ∈L

Aλ pois x ∈ X ⊂ Aλ, pela primeira hipotese sobre X. Logo,⋂λ∈L

Aλ ⊃ X.

O conjunto⋂λ∈L

Aλ e tal que⋂λ∈L

Aλ ⊂ Aλ, para todo λ ∈ L. Assim, pela segunda hipotese sobre X,⋂λ∈L

Aλ ⊂ X.

Portanto, X =⋂λ∈L

Aλ.

24

Exercıcio 1.18:

Seja f : P(A) −→ P(A) uma funcao tal que X ⊂ Y =⇒ f(Y ) ⊂ f(X) e f(f(X)) = X. Prove que f(∪Xλ) = ∩f(Xλ)e f(∩Xλ) = ∪f(Xλ).[Aqui X,Y e cada Xλ sao subconjuntos de A].

Facamos cada inclusao separadamente.

(i) f (⋃Xλ) ⊂

⋂f (Xλ)

Como ∪Xλ ⊃ Xλ, para todo λ, temos por hipotese que f(∪Xλ) ⊂ f(Xλ), para todo λ. Daı, f(∪Xλ) ⊂ ∩f(Xλ).

(ii) f (⋃Xλ) ⊃

⋂f(Xλ)

Por (ii), temos que f(∩f(Xλ)) ⊃ ∪f(f(Xλ)) = ∪Xλ. Daı, f(f(∩f(Xλ))) ⊂ f(∪Xλ). Logo, ∩f(Xλ) ⊂ f(∪Xλ).

(iii) f (⋂Xλ) ⊃

⋃f (Xλ)

Como ∩Xλ ⊂ Xλ, para todo λ, temos por hipotese que f(∩Xλ) ⊃ f(Xλ), para todo λ. Daı, f(∩Xλ) ⊃ ∪f(Xλ).

(iv) f (⋂Xλ) ⊂

⋃f (Xλ)

Por (i), temos que f(∪f(Xλ)) ⊂ ∩f(f(Xλ)) = ∩Xλ. Daı, f(f(∪f(Xλ))) ⊃ f(∩Xλ). Logo, ∪f(Xλ) ⊃ f(∩Xλ).

De (i) e (ii), temos que f(∪Xλ) = ∩f(Xλ) e de (iii) e (iv), temos f(∩Xλ) = ∪f(Xλ).

25

Exercıcio 1.19:

Dadas as famılias (Aλ)λ∈L e (Bµ)µ∈M , forme duas famılias com ındices em L×M considerando os conjuntos

(Aλ ∪Bµ)(λ,µ)∈L×M e (Aλ ∩Bmu)(λ,µ)∈L×M .

Prove que se tem (⋃λ∈L

)∩

⋃µ∈M

=⋃

(λ,µ)∈L×M

(Aλ ∩Bµ),

(⋂λ∈L

)∪

⋂µ∈M

=⋂

(λ,µ)∈L×M

(Aλ ∪Bµ).

Primeiramente provemos que(⋃λ∈L

)∩

⋃µ∈M

=⋃

(λ,µ)∈L×M

(Aλ ∩Bµ).

Como ⋃λ∈L

Aλ ⊃ Aλ ⊃ Aλ ∩Bµ,

para todo (λ, µ) ∈ L×M, temos que ⋃λ∈L

Aλ ⊃⋃

(λ,µ)∈L×M

(Aλ ∩Bµ).

Analogamente, mostra-se que ⋃µ∈M

Bµ ⊃⋃

(λ,µ)∈L×M

(Aλ ∩Bµ).

Assim, segue que (⋃λ∈L

)∩

⋃µ∈M

⊃ ⋃(λ,µ)∈L×M

(Aλ ∩Bµ) .

Seja x ∈ (∪λ∈LAλ) ∩ (∪µ∈MBµ). Desta forma, x ∈ ∪λ∈LAλ e x ∈ ∪µ∈MBµ. Assim, existem λ ∈ L e µ ∈ Mtais que x ∈ Aλ e x ∈ Bµ. Logo,

x ∈ Aλ ∩Bµ ⊂⋃

(λ,µ)∈L×M

(Aλ ∩Bµ) .

Com isso, podemos concluir que(⋃λ∈L

)∩

⋃µ∈M

⊂ ⋃(λ,µ)∈L×M

(Aλ ∩Bµ) .

Mostremos agora que (⋂λ∈L

)∪

⋂µ∈M

=⋂

(λ,µ)∈L×M

(Aλ ∪Bµ).

Como(Aλ ∪Bµ) ⊃ Aλ ⊃

⋂λ∈L

Aλ,

para todo (λ, µ) ∈ L×M , temos que ⋂(λ,µ)∈L×M

(Aλ ∪Bµ) ⊃⋂λ∈L

Aλ.

26

Analogamente, mostra-se que ⋂(λ,µ)∈L×M

(Aλ ∪Bµ) ⊃⋂µ∈M

Bµ.

Assim, segue que ⋂(λ,µ)∈L×M

(Aλ ∪Bµ) ⊃

(⋂λ∈L

)∪

⋂µ∈M

.

Seja x ∈ ∩(λ,µ)∈L×M (Aλ∪Bµ). Suponhamos, por absurdo, que x /∈ (∩λ∈LAλ)∪(∩µ∈MBµ). Entao, x /∈ ∩λ∈LAλe x /∈ ∩µ∈MBµ. Assim, existem λ ∈ L e µ ∈M tais que x /∈ Aλ e x /∈ Bµ. Com igual razao, existe (λ, µ) ∈ L×M talque x /∈ Aλ∪Bµ. Um absurdo, pois como Aλ∪Bµ ⊂ ∩(λ,µ)∈L×M (Aλ∪Bµ), terıamos que x /∈ ∩(λ,µ)∈L×M (Aλ∪Bµ).Logo, devemos ter que x ∈ (∩λ∈LAλ) ∪ (∩µ∈MBµ). Com isso, concluimos que

⋂(λ,µ)∈L×M

(Aλ ∪Bµ) ⊂

(⋂λ∈L

)∪

⋂µ∈M

.

27

Exercıcio 1.20:

Seja (Aij)(i,j)∈N×N uma famılia de subconjuntos com ındices em N× N. Prove, ou disprove por contra-exemplo, aigualdade

∞⋃j=1

( ∞⋂i=1

Aij

)=

∞⋂i=1

∞⋃j=1

Aij

.

A igualdade e falsa em geral. De fato, tomando-se

Aij :=

{{1}, se i = j,∅, se i 6= j,

temos que∞⋃j=1

( ∞⋂i=1

Aij

)=

∞⋃j=1

(∅) = ∅

e∞⋂i=1

∞⋃j=1

Aij

=

∞⋂i=1

({1}) = {1}.

28

Exercıcio 1.21:

Dados os conjuntos A,B,C, estabeleca uma bijecao entre F(A×B;C) e F(A;F(B;C)).

Seja f : A × B → C. Podemos definir uma funcao ϕf : A → F(B;C) definindo ϕf (a) : B → C como sendo afuncao dada por (

ϕf (a))(b) := f(a, b),

para todo b ∈ B. Verificaremos que a funcao ϕ : F(A×B;C)→ F(A;F(B;C)), dada por

ϕ(f) := ϕf ,

para cada f ∈ F(A×B;C), e uma bijecao.Suponhamos que f e g ∈ F(A×B;C) sejam tais que ϕ(f) = ϕ(g). Assim, ϕf = ϕg. Logo, dado (a, b) ∈ A×B,

temos queϕf (a) = ϕg(a)

e, consequentemente,f(a, b) =

(ϕf (a)

)(b) =

(ϕg(b)

)(b) = g(a, b).

Portanto, f = g. Com isso, concluimos que ϕ e injetiva.Seja ψ : A→ F(B;C). Podemos definir uma funcao f : A×B → C por

f(a, b) :=(ψ(a)

)(b),

para todo (a, b) ∈ A×B. Seja a ∈ A. Temos que(ϕf (a)

)(b) = f(a, b) =

(ψ(a)

)(b),

para todo b ∈ B. Desta forma ϕf (a) = ψ(a). Portanto, como a e arbitrario, concluımos que ϕf = ψ. Com isso,concluimos que ϕ e sobrejetiva.

Portanto, ϕ : F(A×B;C)→ F(A;F(B;C)) e uma bijecao como querıamos demonstrar.

29

Capıtulo 2

Conjuntos Finitos, Enumeraveis eNao-Enumeraveis

30

Exercıcio 2.1:

Prove que, na presenca dos axiomas P1 e P2, o axioma A abaixo e equivalente a P3:

A : Para todo subconjunto nao-vazio X ⊂ N, tem-se X\s(X) 6= ∅.

Relembremos as propriedades:

P1 : s : N→ N e injetora;

P2 : N\s(N) = {1};

P3 : Se X ⊂ N e tal que 1 ∈ X e, para todo n ∈ X, s(n) ∈ X, entao X = N.

Suponhamos que as afirmacoes P1, P2 e P3 sejam validos. Concluiremos que o axioma A e valido mostrando quese X ⊂ N e tal que X\s(X) = ∅ entao X = ∅. Equivalentemente, se X ⊂ s(X) entao N\X = N. Primeiramente,temos que 1 ∈ N\X, pois, caso contrario, 1 ∈ s(N) ja que

X ⊂ s(X) ⊂ s(N),

contradizendo P2. Por P1,s(N\X) = s(N)\s(X) ⊃ s(N)\X.

Desta forma, se n ∈ N\X entao s(n) /∈ X e, consequentemente, s(n) ∈ N\X. Assim, por P3, concluimos queN\X = N.

Reciprocamente, suponhamos que os axiomas P1, P2 e A sejam validos. Seja X ⊂ N tal que 1 ∈ X e, paratodo n ∈ X, s(n) ∈ X. Provaremos que X = N e concluiremos daı que P3 e valido. Suponhamos por absurdo queN\X 6= ∅. Por A, segue que existe

n ∈ (N\X)\s(N\X).

Como 1 /∈ N\X, devemos ter que n 6= 1 e, por P2, existe m ∈ N tal que

s(m) = n.

Por P1, m /∈ N\X ja que s(m) = n /∈ s(N\X). Assim, m ∈ X e s(m) = n /∈ X, contradizendo a hipotese sobre X.

31

Exercıcio 2.2:

Dados os numeros naturais a e b, prove que existe um numero natural m tal que m · a > b.

Se a = 1, basta tomar m = b+ 1, pois1(b+ 1) = b+ 1 > b.

Se a 6= 1 entao a > 1 ja que a ∈ Z+. Assim, pela monoticidade da multiplicacao em Z+,

ba > b.

Logo, para m := b, temos que ma > b.

32

Exercıcio 2.3:

Seja a um numero natural. Se um conjunto X e tal que a ∈ X e, alem disso, n ∈ X ⇒ n+ 1 ∈ X, entao X contemtodos os numeros naturais ≥ a.

SejaA := {k ∈ Z+ : a+ k ∈ X}.

Pela definicao da relacao 6 em Z+, b > a se e somente se b = a+k para algum k ∈ Z>0. Desta forma, provandoque A = Z+ podemos concluir que X contem todos os numeros naturais > a.

Como a ∈ X, temos, pela propriedade de X, que a+ 1 ∈ X. Logo, 1 ∈ A.Suponhamos que k ∈ A. Pela definicao de A, isto implica que a+ k ∈ X. Assim pela propriedade de X, temos

que a+ k + 1 ∈ X. Logo, k + 1 ∈ A.Portanto, pelo PIF, segue que A = Z+.

33

Exercıcio 2.4:

Tente descobrir, independentemente, algumas das demonstracoes omitidas no texto.

Associatividade: m+ (n+ p) = (m+ n) + p.

Provada no livro.

Comutatividade: m+ n = n+m.

Primeiramente mostraremos quem+ 1 = 1 +m,

para todo m ∈ Z+. O caso em que m = 1 e tautologico. Supondo, como hipotese de inducao, que

m+ 1 = 1 +m

para algum m ∈ Z+, segue que

s(m) + 1 = s(s(m)) = s(m+ 1) = s(1 +m) = 1 + s(m).

Assim, pelo PIF, temos que m+ 1 = 1 +m, para todo m ∈ Z+.Por fim, provaremos, para m ∈ Z+ arbitrario e por inducao em n ∈ Z+, que

m+ n = n+m.

O caso n = 1 foi provado no paragrafo anterior. Supondo, como hipotese de inducao, que

m+ n = n+m

para algum n ∈ Z+, segue que

m+ s(n) = s(m+ n) = s(n+m)= n+ s(m) = n+ (m+ 1)= n+ (1 +m) = (n+ 1) +m= s(n) +m.

E o resultado segue pelo PIF.

Lei do Corte: m+ n = m+ p⇒ n = p.

Sejam n e p ∈ Z+. Provaremos, por inducao em m ∈ Z+, que se m+ n = m+ p entao n = p.O caso em que m = 1 resume-se a injetividade da funcao s : Z+ → Z+. Isto e, como

s(n) = n+ 1 = 1 + n = 1 + p = p+ 1 = s(p),

temos quen = p.

Suponhamos, como hipotese de inducao, que m+n = m+p implique que n = p. Assim, se s(m)+n = s(m)+pentao

s(m+ n) = s(n+m) = n+ s(m)= s(m) + n = s(m) + p= p+ s(m) = s(p+m)

= s(m+ p).

Assim, se s(m) + n = s(m) + p temos, novamente pela injetividade de s : Z+ → Z+, que m + n = m + p e, pelahipotese de inducao, n = p.

E o resultado segue pelo PIF.

Tricotomia: Dados m e n ∈ Z+, exatamente uma das tres alternativas seguintes podem ocorrer: ou m = n,ou existe p ∈ Z+ tal que m = n+ p, ou, entao, existe q ∈ Z+ com n = m+ q.

Dizemos que (m,n) ∈ Z+×Z+ satisfaz a condicao C se exatamente uma das exatamente uma das tres alterna-tivas ocorre:

34

• m = n;

• m = n+ p, para algum p ∈ Z+;

• n = m+ q, para algum q ∈ Z+.

Seja X o subconjunto de Z+ × Z+ definido por

T := {(m,n) ∈ Z+ × Z+ : (m,n) satisfaz C}.

Observemos que, como

T =⋃

m∈Z+

{m} × Tm,

ondeTm := {n ∈ Z+ : (m,n) satisfaz C},

mostrando queTm = Z+,

para cada m ∈ Z+, podemos concluir que

T =⋃

m∈Z+

{m} × Tm =⋃

m∈Z+

{m} × Z+ = Z+ × Z+.

Portanto, concluimos a Lei da Tricotomia.Procederemos com a demonstracao de que Tm = Z+ por inducao em m ∈ Z+.Consideremos o caso em que m = 1. Se n = 1 temos que n = m. Alem disso, como 1 /∈ s(Z), segue que

m = 1 6= sp(n) = n+ p

en = 1 6= sq(m) = m+ q,

para todos p e q ∈ Z+. Logo, (1, 1) satisfaz a condicao C e, consequentemente, 1 ∈ T1. Supondo que n ∈ T1, comonao se pode ter que 1 = m+ q = sq(m) ja que 1 /∈ s(Z+), temos que exatamente uma das duas alternativas ocorre:

• n = 1 e, equivalentemente, s(n) = 1 + 1;

• n = 1 + q e, equivalentemente, s(n) = s(1 + q) = 1 + s(q).

Logo, se n ∈ T1 entao s(n) ∈ T1. Com isso, concluimos, pelo PIF, que T1 = Z+.Suponhamos, como hipotese de inducao, que Tm = Z+. Provaremos que Ts(m) = Z+.Como X1 = Z+, temos imediatamente que (1, s(m)) satisfaz a condicao C e, consequentemente, (s(m), 1)

satisfaz a condicao C. Logo, 1 ∈ Ts(m). Supondo que n ∈ Ts(m), temos que exatamente uma das tres alternativasocorrem:

• n = s(m): Neste caso, s(n) = s(s(m)) = s(m) + 1;

• n = s(m) + q, para algum q ∈ Z+: Neste caso, s(n) = s(s(m) + q) = s(m) + s(q);

• s(m) = n+ p, para algum p ∈ Z+: Neste caso, se p = 1 entao s(m) = s(n). E, se p ∈ Z+\{1} = s(Z+), existepZ+ tal que p = s(p), e assim

s(m) = n+ p = n+ s(p) = n+ (1 + p) = (n+ 1) + p = s(n) + p.

Assim, se n ∈ Ts(m), temos que exatamente uma das tres alternativas ocorrem:

• s(n) = s(m) (no caso em que n = s(m));

• s(n) = s(m) + q (no caso em que n = s(m) ou n = s(m) + q, onde q = s(q));

• s(m) = s(n) + p (no caso em que s(m) = n+ p, onde p = s(p)).

35

Logo, se n ∈ Ts(m) entao s(n) ∈ Ts(m). Com isso, concluimos, pelo PIF, que Ts(m) = Z+.Portanto, Xm = Z+, para todo m ∈ Z+.

Transitividade: se m < n e n < p entao m < p.

Se, para m, n e p ∈ Z+, tivermos que m < n e n < p entao existem r e s ∈ Z+ tais que

m+ r = n

en+ s = p.

Desta forma,p = n+ s = (m+ r) + s = m+ (r + s).

Logo,m < p.

Tricotomia: dados m e n ∈ Z+ exatamente uma das alternativas seguintes pode ocorrer: ou m = n, ou m < nou n < m.

Sejam m e n ∈ Z+. Segundo a tricotomia da adicao (provada acima), exatamente uma das tres condicoes evalida: ou m = n; ou existe p ∈ Z+ tal que m = n+ p e, portanto, m > n; ou existe q ∈ Z+ tal que n = m+ q e,portanto, m < n.

Monoticidade da adicao: se m < n entao, para todo p ∈ Z+, tem-se m+ p < n+ p.

Provada no livro.

Associatividade: m · (n · p) = (m · n) · p.

Provada no livro.

Comutatividade: m · n = n ·m.

Primeiramente, provaremos que m · 1 = 1 ·m, para todo m ∈ Z+. Depois, supondo, como hipotese de inducao,que n ∈ Z+ e tal que m · n = n ·m, para todo m ∈ Z+, provaremos que n+ 1 e tal que m · (n+ 1) = (n+ 1) ·m.Como isso, o resultado segue pelo Princıpio da Inducao Finita.

Provaremos a igualdade m · 1 = 1 ·m por inducao em m ∈ Z+. Para m = 1 a igualdade e trivial. Suponhamos,como hipotese de inducao, que m · 1 = 1 ·m, para algum m ∈ Z+. Desta forma, temos que

(m+ 1) · 1 = m+ 1 = m · 1 + 1 = 1 ·m+ 1 = 1 · (m+ 1).

Logo, pelo PIF, a igualdade e valida.Suponhamos que n ∈ Z+ seja tal que m · n = n ·m, para todo m ∈ Z+. Mostraremos, por inducao em m, que

m · (n+ 1) = (n+ 1) ·m, para todo m ∈ Z+. Para m = 1, o resultado segue do paragrafo anterior. E, supondo quem · (n+ 1) = (n+ 1) ·m, temos que

(m+ 1) · (n+ 1) = (m+ 1) · n+ (m+ 1)= n · (m+ 1) + (m+ 1)= n ·m+ n+m+ 1= m · n+m+ n+ 1= m · (n+ 1) + (n+ 1)= (n+ 1) ·m+ (n+ 1)= (n+ 1) · (m+ 1).

E temos o resultado.

Distributividade: m(n+ p) = m · n+m · p.

36

Provada no livro.

Lei do Corte: m · p = n · p⇒ m = n.

Suponhamos que m, n e p ∈ Z+ sao tais que

m · p = n · p.

Pela tricotomia, exatamente uma das tres condicoes e satisfeita: ou m = n+ q, para algum q ∈ Z+; ou m = n+ q,m = n + q, para algum q ∈ Z+; ou m = n. Provaremos que as duas primeiras condicoes nao sao possıveis e, comisso, teremos o resultado.

Suponhamos que m = n+ q, para algum q ∈ Z+. Segue que

n · p = m · p = (n+ q) · p = p · (n+ q) = p · n+ p · q = n · p+ p · q.

Contradizendo a tricotomia.De forma analoga, nao podemos ter n = m+ q, para algum q ∈ Z+.

Monoticidade: m < n⇒ m · p < n · p.

Sejam n e m ∈ Z+ tais quem < n.

Provaremos quem · p < n · p,

para todo p ∈ Z+, por inducao em p.Para p = 1, a desigualdade e imediata.Suponhamos, como hipotese de inducao, que m · p < n · p, para um certo p ∈ Z+. Como m < n, existe q ∈ Z+

tal quen = m+ q.

Assim,

n · (p+ 1) = (m+ q) · (p+ 1) = (p+ 1) · (m+ q) = (p+ 1) ·m+ (p+ 1) · q = m · (p+ 1) + (p+ 1) · q.

e, consequentemente,n · (p+ 1) < m · (p+ 1).

E o resultado segue, como querıamos, pelo PIF.

37

Exercıcio 2.5:

Um elemento a ∈ Z+ chama-se antecessor de b ∈ Z quando se tem a < b mas nao existe c ∈ Z+ tal que a < c < b.Prove que, exceto 1, todo numero natural possui um antecessor.

Seja x ∈ Z+\{1}. Mostraremos que x possui um antecesor.Pelo axioma de Peano P2, x = s(y) = y + 1 para algum y ∈ Z+. Logo, y < x.Suponhamos que z ∈ Z+ e tal que z < x. Mostraremos que z 6 y. Temos que

x = z + n,

para algum n ∈ Z+. Se n = 1 temos quey + 1 = x = z + 1

e, consequentemente, pela Lei do Corte, y = z. Se n ∈ Z+\{1} entao, novamente pelo axioma de Peano P2, existem ∈ Z+ tal que n = s(m). Assim,

s(y) = x = z + n = z + s(m) = s(z +m)

e, pela injetividade da funcao s (axioma de Peano P1),

y = z +m.

Logo, z < y.Portanto, y e um antecessor de x.

38

Exercıcio 2.6:

Use inducao para demonstrar os seguintes fatos:

(a) 2(1 + 2 + 3 + · · ·+ n) = n(n+ 1);

(b) 1 + 3 + 5 + · · ·+ (2n+ 1) = (n+ 1)2;

(c) (a− 1)(1 + a+ a2 + · · ·+ an) = an+1 − 1, seja quais forem a, n ∈ Z+;

(d) n ≥ 4⇒ n! > 2n.

(a)

Para n = 1, temos a igualdade ja que2(1) = 2 = 1(1 + 1).

Supondo que a igualdade seja verdadeira para n = k, segue que

2(1 + 2 + 3 + · · ·+ k + (k + 1)) = 2(1 + 2 + 3 + · · ·+ k) + 2(k + 1)= k(k + 1) + 2(k + 1)= (k + 2)(k + 1)= (k + 1)((k + 1) + 1).

Portanto, pelo PIF, temos o resultado.

(b)

Para n = 1, temos a igualdade ja que1 + 3 = 4 = (1 + 1)2.

Supondo que a igualdade seja verificada para n = k, segue que

1 + 3 + 5 + · · ·+ (2k + 1) + (2(k + 1) + 1) = (k + 1)2 + (2(k + 1) + 1)= (k + 1)2 + 2(k + 1)1 + 12

= ((k + 1) + 1)2.

Portanto, pelo PIF, temos o resultado.

(c)

Para n = 1, temos a igualdade ja que

(a− 1)(1 + a) = a2 − 1.

Supondo que a igualdade seja verdadeira para n = k, segue que

(a− 1)(1 + a+ a2 + · · ·+ ak + ak+1) = (a− 1)(1 + a+ a2 + · · ·+ ak) + (a− 1)(ak+1)= (ak+1 − 1) + (ak+2 − ak+1)= ak+2 − 1.

Portanto, pelo PIF, temos o resultado.

(d)

Para n = 4, temos a desigualdade ja que

4! = 24 > 16 = 24.

Supondo que a desigualdade seja verdadeira para n = k > 4, segue que

(k + 1)! > (k!)(k + 1) > 2k > 2k2 = 2k+1.

Portanto, pelo PIF, temos o resultado.

39

Exercıcio 2.7:

Use o Segundo Princıpio de Inducao para demonstrar a unicidade de decomposicao de um numero natural emfatores primos.

O resultado do enunciado e comumente demonstrado nos livros sobre Teoria do Numeros utilizando-se resultadosprovados com o uso do conceito de maximo divisor comum como, por exemplo, a implicacao: Se p ∈ Z+ e primoe p divide o produto mn dos elementos m e n ∈ Z+entao p divide m ou n. Para evitarmos a utilizacao deferramentas de fora do texto, faremos uma demonstracao mais longa, mas que usa somente as propriedades dasoma, multiplicacao (apresentadas neste capıtulo) e da subtracao (que sera muito brevemente tratada no proximocapıtulo). Esta demonstracao e uma adapatacao de uma demonstracao encontrada em:

• http://en.wikipedia.org/wiki/Fundamental theorem of arithmetic

Seja n ∈ Z+ tal que todo m < n em Z+ possui uma unica decomposicao em fatores primos. Provaremos quen possui uma unica decomposicao em fatores primos e concluiremos, do Segundo Princıpio da Inducao, que todon ∈ Z+ possui uma unica fatoracao em fatores primos.

Suponhamos que α1α2 . . . αp e β1β2 . . . βq sejam duas decomposicoes de n em fatore primos αi e βj . Devemosmostrar que a sequencia α1, α2, . . . , αp e uma permutacao da sequencia β1, β2, . . . , βq.

Podemos supor, sem perda de generalidade, que α1 6 β1. Tambem podemos supor que m nao e primo, poispela propria definicao de numero primo, m teria imediatamente uma unica fatoraca em fatores primos. Ou seja, pe q sao maiores que 1.

Se α1 = βi para algum i ∈ {1, . . . , q}, temos, pela Lei do Corte, que

α2α3 . . . αp = β1 . . . βi−1βi+1 . . . βq.

Assim, como α2α3 . . . αp < n, devemos ter, pela hipotese de inducao que β2, α3, . . . , αp e uma permutacao dasequencia β1, . . . , βi−1, βi+1, . . . , βq. Portanto, α1, α2, α3, . . . , αp e uma permutacao da sequencia β1, . . . ,βi−1, βi, βi+1, . . . , βq.

No restante desta demonstracao, encontraremos uma contradicao supondo que α1 ∈ {β1, . . . , βq}. Assim, comomostrado acima, teremos o resultado.

Suponhamos que α1 /∈ {β1, . . . , βq}. Segue que α1 < β1.O inteiro

m := (β1 − α1)(β2 . . . βq)

e positivo, pois β1 − α1 > 0 e β2 . . . βq > 0. Alem disso,

m = (β1 − α1)(β2 . . . βq) = n− α1β2 . . . βq < n.

Devemos ter que β1 − α1 = 1 em = β2 . . . βq;

ou β1 − α1 = γ1 . . . γs em = γ1 . . . γsβ2 . . . βq,

para numeros primos γ1, . . . , γs ∈ Z+.Tambem temos que

m = (β1 − α1)(β2 . . . βq)= n− α1β2 . . . βq= α1α2 . . . αp − α1β2 . . . βq= α1(α2 . . . αp − β2 . . . βq)

Como m e α1 sao positivos, devemos ter que α2 . . . αp−β2 . . . βq tambem e positivo. Logo, α2 . . . αp−β2 . . . βq = 1e

m = α1;

ou α2 . . . αp − β2 . . . βq = θ1 . . . θr em = α1θ1 . . . θr,

para numeros primos θ1, . . . , θr ∈ Z+.

40

Com isso, concluimos que{β2, . . . , βq} ou {γ1, . . . , γs, β2, . . . , βq}

e uma permutacao de{α1} ou {α1, θ1, . . . , θr},

ja que m < n possui uma unica fatoracao em fatores primos. Em especial, devemos ter que

α1 ∈ {β2, . . . , βq} ou {γ1, . . . , γs, β2, . . . , βq}.

Logo, como α1 /∈ {β1, . . . , βq}, devemos ter que

α1 ∈ {γ1, . . . , γs}.

Por fim, para algum k ∈ Z+,α1k = γ1 . . . γs = β1 − α1

e, consequentemente,α1(k + 1) = β1.

Contradizendo o fato de β1 ser primo.

41

Exercıcio 2.8:

Seja X um conjunto com n elementos. Use inducao para provar que o conjunto das bijecoes (ou permutacoes)f : X → X tem n! elementos.

Seja X o conjunto formado pelos elementos (distintos) x1, x2, . . . , xn. Provaremos, por inducao em n ∈ Z+,que o conjunto SX , das bijecoes f : X → X, tem n! elementos.

O resultado e valido para n = 1 uma vez que, neste caso, so existe uma funcao f : X → X e esta e bijetiva.Suponhamos que n > 1 e, como hipotese de inducao, que o conjunto Y = X\{xn} seja tal que o conjunto SY ,

das funcoes bijetivas g : Y → Y , tenha (n− 1)! elementos.Sejam

SX,k := {f ∈ SX : f(xn) = xk},

para todo k = 1, . . . , n. Segue desta definicao que

SX,i ∩ SX,j = ∅

quando i 6= j e queSX = SX,1 ∪ SX,2 ∪ · · · ∪ SX,n.

Assim, pelo Corolario 1 do Teorema 6, temos que

card(SX) = card(SX,1) + card(SX,2) + · · ·+ card(SX,n).

Desta forma, mostrando que

card(SX,1) = card(SX,2) = · · · = card(SX,n) = SY = (n− 1)!

concluiremos quecard(SX) = n · (n− 1)! = n!,

como querıamos demonstrar.Dado f ∈ SX,n, temos que f(xn) = xn e, como f e uma bijecao, f(Y ) = Y . Assim, cada f ∈ SX,n define uma

bijecao φ(f) : Y → Y dada por (φ(f)

)(y) = f(y),

em cada y ∈ Y . Com isso, temos uma funcao φ : SX,n → SY . Se f1 e f2 ∈ SX,n sao tais que φ(f1) = φ(f2) entao

f1(y) =(φ(f1)

)(y) =

(φ(f2)

)(y) = f2(y),

f1(xn) = xn = f2(xn)

e, consequentemente,f1 = f2.

Dado g ∈ SY , podemos definir uma funcao f ∈ SX por

f(xi) =

{g(xi), se i = 1, . . . , n− 1,xn, se i = n.

Desta forma,φ(f) = g.

Portanto, concluimos que φ : SX,n → SY e uma bijecao e, consequentemente, que

card(SX,n) = card(SY ).

Provaremos, para k = 1, . . . , n− 1, que

card(SX,k) = card(SX,n).

42

Considermos a bijecao σ ∈ SX dada por

σ(xi) =

xi, se i 6= k, n,xn, se i = k,xk, se i = n.

Segue desta definicao que σ ◦ σ = IX (a funcao identidade em X).Dado h ∈ SX,k, temos que σ ◦h : X → X e uma composicao de bijecoes e, logo, uma bijecao. Alem disso, como

σ ◦ h(xn) = σ(xk) = xn,

temos que σ ◦ h ∈ SX,n. Assim, podemos definir uma funcao ψ : SX,k → SX,n por

ψ(h) = σ ◦ h,

para cada h ∈ SX,k.De forma analoga, verifica-se que uma funcao ρ : SX,n → SX,k fica bem definida pela igualdade

ρ(f) = σ ◦ f,

para cada f ∈ SX,n.Por fim, para cada f ∈ SX,n,

ψ ◦ ρ(f) = ψ(σ ◦ f) = σ ◦ (σ ◦ f) = (σ ◦ σ) ◦ f = f

e, para cada h ∈ SX,k,ρ ◦ ψ(h) = ρ(σ ◦ h) = σ ◦ (σ ◦ h) = (σ ◦ σ) ◦ h = h.

Logo, ρ e uma inversa para ψ : SX,k → SX,n. E, portanto,

card(SX,k) = card(SX,n).

Como querıamos demonstrar.

43

Exercıcio 2.9:

Sejam X e Y conjuntos finitos.

a) Prove que card(X ∪ Y ) + card(X ∩ Y ) = card(X) + card(Y ).

b) Qual seria a formula correspondente para tres conjuntos?

c) Generalize.

a) Sejam A = {(1, x);x ∈ X} ∪ {(2, y); y ∈ Y } e B = {(3, z); z ∈ X ∪ Y } ∪ {(4, w);w ∈ X ∩ Y }. Definamosf : A→ B como sendo

f(1, x) = (3, x)

f(2, y) =

{(3, y); se y ∈ Y \X(4, y); se y ∈ X ∩ Y

.

Temos trivialmente que f e uma bijecao entre A e B. Alem disso, card(A) = card(X ∪ Y ) + card(X ∩ Y ) ecard(B) = card(X) + card(Y ). Daı segue o resultado.

b) card(X ∪ Y ∪ Z) + card((X ∩ Y ) ∪ (X ∩ Z) ∪ (Y ∩ Z)) = card(X) + card(Y ) + card(Z).

c) card

(n⋃i=1

Xi

)+ card

(⋃i 6=j

(Xi ∩Xj)

)= card(X1) + card(X2) + ...+ card(Xn).

44

Exercıcio 2.10:

Dado um conjunto finito X, prove que uma funcao f : X → X e injetora se, e somente se, e sobrejetora.

(⇒) Temos que g : X → f(X) dada por g(x) = f(x) e uma bijecao. Se f(X) 6= X terıamos um absurdo poisnao pode haver bijecao entre um conjunto finito e um subconjunto proprio deste conjunto.

(⇐) Seja X = {x1, x2, ..., xn}. Suponha que f nao seja injetora, ou seja, existem xi 6= xj em X tais quef(x1) = f(x2). Assim, f(X) = {f(x1), f(x2), ..., f(xn)} teria no maximo n− 1 elementos e desta forma f(X) 6= X,o que e um absurdo. Logo, f e injetora.

45

Exercıcio 2.11:

Formule matematicamente e demonstre o seguinte fato(conhecido como princıpio das gavetas). Se m < n, entaode qualquer modo como se guardem n objetos em m gavetas, havera sempre uma gaveta, pelo menos, que conteramais de um objeto.

f : In → Im com n > m nao e injetiva.

Se f nao e sobrejetora, f |In tambem nao sera. Logo, f |In tambem nao sera injetiva pelo exercıcio anterior. Econsequentemente f tambem nao seria injetiva.

Por outro lado, mesmo que f fosse sobrejetiva, se fosse tambem injetiva, f seria uma bijecao entre um conjuntofinito e um subconjunto proprio dele, que e um absurdo.

46

Exercıcio 2.12:

Seja X um conjunto com n elementos. Determine o numero de funcoes injetivas f : Ip → X.

Princıpio da contagem. Escolhamos um dos n elementos de X para ser f(1). Daı escolhamos 1 dos n − 1elementos restantes para ser f(2). E assim sucessivamente temos que o numero de funcoes injetivas possıveis e

n(n− 1)(n− 2)...(n− p+ 1).

47

Exercıcio 2.13:

Quantos subconjuntos com p elementos possui um subconjunto X, sabendo-se que X tem n elementos?

Se n < p, vem de P1 que nao existe subconjunto de X com p elementos. Caso contrario podemos definir umafuncao f : [1, p] ∩ N→ X(pelo axioma da escolha). Pelo princıpio da contagem, temos que f pode ser definida de

n!

p!(n− p)!modos distintos. Porem, para cada imagem de f, f pode ter sido definida de p! formas. Sendo assim,

existemn!

p!(n− p)!imagens de f.

48

Exercıcio 2.14:

Prove que se A tem n elementos, entao P (A) tem 2n elementos.

Associemos a cada X ∈ P (A) uma funcao fX : A → {0, 1} dada por f(x) = 1 se x ∈ X e f(x) = 0 se x /∈ X.Temos entao que a aplicacao X → fX e uma bijecao. E como, pelo princıpio da contagem, e possıvel se fazer 2funcoes f : A→ {0, 1} diferentes, temos que a ordem de P (A) e exatamente 2.

49

Exercıcio 2.15:

Defina um funcao sobrejetiva f : N→ N tal que, para todo n ∈ N, o conjunto f−1(n) seja infinito.

Seja f : N→ N tal que f(2n3m) = n e f(x) = 1 para x divisıvel por qualquer primo diferente de 2 e 3. Portanto,

f−1(N) ⊃ {2n3, 2n32, ..., 2n3m, ...}.

50

Exercıcio 2.16:

Prove que se X e infinito enumeravel, o conjunto das partes finitas de X tambem e (infinito) enumeravel.

Seja X = {x1, x2, ...}. Temos que

P =

∞⋃i=1

{A ⊂ {x1, x2, ..., xi}} =

∞⋃i=1

Fi.

Temos que cardFi = 2i. Como P e uma reuniao enumeravel de conjuntos enumeraveis, P e enumeravel.

51

Exercıcio 2.17:

Seja f : X → X uma funcao. Um subconjunto Y ⊂ X chama-se estavel relativamente a f quando f(Y ) ⊂ Y. Proveque um subconjunto X e finito se, e somente existe um funcao f : X → X que so admite os subconjuntos estaveis∅ e X.

(⇒) Seja X = {x1, x2, ..., xn} e f : X → X dado por f(xi) = xi+1 se 1 ≤ i < n e f(xn) = x1. Se f e estavel emA e xp ∈ A, temos que xq = fq−p(modn)(xp) ∈ A. Logo, A = X.

(⇐) Dado x0 ∈ X (se X 6= ∅, X e finito) consideremos o conjunto A = {x0, f(x0), f2(x0), ..., fn(x0), ...}. DaıX = A, pois f e estavel em A e A 6= ∅.

Se nao existir k ∈ N tal que fk(x0) = x0, A − {x0} e estavel por f e logo A − {x0} = X − {x0} = ∅, ou seja,X = {x0}, ou A = X = A− {x0}, absurdo.

Por outro lado, se existir k ∈ N tal que fk(x0) = x0 o conjunto {x0, f(x0), f2(x0), ..., fk−1(x0)} e estavel por Ae nao vazio, logo e igual a X.

52

Exercıcio 2.18:

Seja f : X → X uma funcao injetiva tal que f(X) 6= X. Tomando x ∈ X − f(X), prove que os elementosx, f(x), f(f(x)), ... sao dois a dois distintos.

Provaremos por inducao em n que para todo p ∈ N, temos que fn(x) 6= fn+p(x) e a proposic ao estar´ademonstrada. Com x /∈ f(X), temos que x 6= fp(x), para todo p ∈ N. Suponhamos que fn(x) 6= fn+p(x). Entaofn+1(x) 6= fn+1+p(x) pois f e injetora. Pelo PIF o resultado segue.

53

Exercıcio 2.19:

Dado um conjunto finito X, prove que uma funcao f : X → X e injetora se, e somente se, e sobrejetora.

(⇒) Temos que g : X → f(X) dada por g(x) = f(x) e uma bijecao. Se f(X) 6= X terıamos um absurdo poisnao pode haver bijecao entre um conjunto finito e um subconjunto proprio deste conjunto.

(⇐) Seja X = {x1, x2, ..., xn}. Suponha que f nao seja injetora, ou seja, existem xi 6= xj em X tais quef(x1) = f(x2). Assim, f(X) = {f(x1), f(x2), ..., f(xn)} teria no maximo n− 1 elementos e desta forma f(X) 6= X,o que e um absurdo. Logo, f e injetora.

54

Exercıcio 2.20:

(a) Se X e finito e Y e enumeravel, entao F(X,Y ) e enumeravel.

(b) Para cada funcao f : N → N seja Af = {n ∈ N; f(n) 6= 1}. Prove que o conjunto X das funcoes f : N → Ntais que Af e finito e um conjunto enumeravel.

Item (a)Seja X = {x1, ..., xn}. Definimos

φ : F(X,Y ) → Y n

f → (f(x1), ..., f(xn)).

Temos que φ e claramente injetiva. Logo, F(X,Y ) esta em bijecao com o conjunto φ(F(X,Y )) ⊂ Y n. ComoY e enumeravel, Y n e enumeravel (pois e produto finito de conjuntos enumeraveis). Assim, φ(F(X,Y )) ⊂ Y n eanumeravel e, consequentemente, F(X,Y ) e enumeravel.

Item (b)Seja

Fn := {Y ⊂ N; cardY = n}.

Definimosφ : Fn → Y n

Y = {y1, ..., yn} → (y1, ..., yn).

Claramente, φ e injetiva. Como Y n e enumeravel, segue que Fn e enumeravel. Portanto,

F :=

∞⋃n=1

Fn

e enumeravel.Seja

ψ : X →⋃Y ∈FF(Y,N)

f → f |Af.

Temos que ψ e injetiva. De fato, se f, g ∈ X sao tais que ψ(f) = ψ(g) temos que

f |Af= g|Ag

implicando que Af = Ag,f |Af

= g|Ag,

ef = g

ja quef |N\Af

= 1 = g|N\Af.

Pelo item anterior,⋃Y ∈F F(Y,N) e uma uniao enumeravel de conjuntos enumeraveis. Logo,

⋃Y ∈F F(Y,N) e

enumeravel. Assim, como ψ e injetiva, segue que X e enumeravel.

55

Exercıcio 2.21:

Obtenha uma decomposicao N = ∪∞i=1Xi tal que os conjuntos Xi sao infinitos e dois a dois disjuntos.

Para todo n ∈ N, existe um unico k ∈ Z>0 tal que

2k 6 n < 2k+1.

Por isso, fica bem definida a funcao f : N→ Z>0 dada por

f(n) = n− 2k,

onde 2k 6 n < 2k+1. Desta forma, temos, para

Xi := f−1(i− 1),

que

N =

∞⋃i=1

Xi

com os conjuntos Xi sendo dois a dois disjuntos. Adiante, como

Xi = {2k + i− 1 | k ∈ Z>0, i− 1 < 2k},

temos que cada Xi e infinito.

56

Exercıcio 2.22:

Defina f : N× N→ N, pondo f(1, n) = 2n− 1 e f(m+ 1, n) = 2m(2n− 1). Prove que f e uma bijecao.

Para cada numero natural p, temos, pela unicidade da decomposicao de numeros naturais em numeros primos,que existem unicos m e q ∈ Z+ tais que p = 2m−1q e q e ımpar. Sendo q ımpar, existe um unico n ∈ Z+ tal queq = 2n− 1. Assim, existem unicos m e n ∈ Z>0 tais que p = 2m−1(2n− 1). Portanto, e bem definida a funcao

g : Z+ → Z+ × Z+

p = 2m−1(2n− 1) → (m,n).

Como g e uma inversa para f , temos que f e bijetiva.

57

Exercıcio 2.23:

Seja X ⊂ N um subconjunto infinito. Prove que existe uma unica bijecao crescente f : N→ X.

Definimos, indutivamente f : N→ X porf(1) = min(X)

e

f(n) = min

(X −

n−1⋃i=1

{f(i)}

),

para n > 1. Temos, pelo PIF e pelo fato de X ⊂ N ser bem ordenado, que f esta bem definida.Dados m < n ∈ N, temos que

f(m) < min

(X −

n−1⋃i=1

{f(i)}

)= f(n)

pois f(m) 6 x, para todo x ∈ X −⋃m−1i=1 ⊃ X −

⋃n−1i=1 , e f(m) /∈ X −

⋃n−1i=1 . Com isso, concluimos que f e

estritamente crescente e, consequentemente que f e injetiva.Provaremos, agora que f e sobrejetiva. Comecaremos mostrando, por inducao que

n 6 f(n).

Para n = 1, temos de X ⊂ N, que1 = min(N) 6 min(X) = f(1).

Usando o passo indutivo, temos quen 6 f(n) < f(n+ 1)

implicando quen+ 1 6 f(n+ 1).

Logo, vale a desigualdade acima. Adiante, dado x ∈ XN, provaremos que x ∈ f(N). Suponhamos por absurdo queexista x ∈ X − f(N). Existe, pela arquimedianidade de N, n ∈ N tal que

x < n 6 f(n).

Mas, como

x ∈ X −n−1⋃i=1

{f(i)},

terıamos uma contradicao com o fato de que

x < min

(X −

n−1⋃i=1

{f(i)}

).

Portanto, f e sobrejetiva.Provaremos, agora, que se g : N→ X e uma bijecao crescente entao f = g. Devemos ter que

g(1) = min(X) = f(1)

pois, caso contrario, existiria n ∈ N, com n > 1, tal que

g(n) = min(X) < g(1).

Contradizendo o fato de g ser crescente. Usando o passo indutivo,

g(k) = f(k),

para todo k < n+ 1, devemos que ter

g(n+ 1) = min

(X −

n⋃i=1

{g(i)}

)= min

(X −

n⋃i=1

{f(i)}

)= f(n+ 1)

58

pois, caso contrario, existiria p > n+ 1 tal que

g(p) = min

(X −

n⋃i=1

{g(i)}

)< g(n+ 1).

Contradizendo o fato de g ser crescente. E o resultado segue.

59

Exercıcio 2.24:

Prove que todo conjunto infinito se decompoe como reuniao de uma infinidade enumeravel de conjuntos infinitos,dois a dois disjuntos.

Seja C um conjunto infinito.Pelo exercıcio 2.21, existe uma decomposicao

N =⋃i∈N

Xi

na qual os conjuntos Xi sao infitos e dois a dois disjuntos.Se C e enumeravel, existe uma bijecao f : N→ C. Logo,

C =⋃i∈N

f(Xi)

e uma decomposicao na qual os conjuntos f(Xi) sao infitos e dois a dois disjuntos.Se C e nao-enumeravel, existe uma aplicacao injetiva f : N→ C tal que C − f(N) e infinito. Assim,

C = (C − f(N)) ∪⋃i∈N

f(Xi)

e uma decomposicao na qual os conjuntos C − f(N) e f(Xi) sao infitos e dois a dois disjuntos.

60

Exercıcio 2.25:

Seja A um conjunto. Dadas funcoes f, g : A → N, defina a soma f + g : A → N, o produto f · g : A → N, e de osignificado da afirmacao f 6 g. Indicando com ξX a funcao caracterıstica de um subconjunto X ⊂ A, prove:

a) ξX∩Y = ξX · ξY ;

b) ξX∪Y = ξX + ξY − ξX∩Y . Em particular, ξX∪Y = ξX + ξY ⇔ X ∩ Y = ∅;

c) X ⊂ Y ⇔ ξX 6 ξY ;

d) ξA−X = 1− ξX .

Definimosf + g : A → N

a → f(a) + g(a)e

f · g : A → Na → f(a)g(a).

E dizemos que f 6 g se e somente sef(a) 6 g(a)

para todo a ∈ A.a)Seja a ∈ A.Se ξX∩Y (a) = 0, entao a /∈ X∩Y e, consequentemente, a /∈ X ou a /∈ Y . Assim, ξX∩Y (a) = 0 implica ξX(a) = 0

ou ξY (a) = 0. Logo, ξX∩Y (a) = 0 implica ξX(a)ξY (a) = 0 = ξX∩Y (a).Se ξX∩Y (a) = 1, entao a ∈ X ∩ Y e, consequentemente, a ∈ X e a ∈ Y . Assim, ξX∩Y (a) = 1 implica ξX(a) = 1

e ξY (a) = 1. Logo, ξX∩Y (a) = 1 implica ξX(a)ξY (a) = 1 = ξX∩Y (a).Como ξX∩Y (a) = 0 ou 1, temos que ξX(a)ξY (a) = ξX∩Y (a) em todos os casos. E, como a ∈ A e arbitrario,

temos que ξX · ξY = ξX∩Y .b)Seja a ∈ A.Se ξX∪Y (a) = 0, entao a /∈ X ∪Y e, consequentemente, a /∈ X e a /∈ Y . Assim, ξX∪Y (a) = 0 implica ξX(a) = 0,

ξY (a) = 0 e ξX∩Y (a) = 0. Logo, ξX∪Y (a) = 0 implica ξX(a) + ξY (a)− ξX∩Y (a) = 0 = ξX∪Y (a).Se ξX∪Y (a) = 1 e ξX∩Y (a) = 0, entao a ∈ X∪Y −X∩Y = (X−Y )∪(Y −X) e, consequentemente, a ∈ X−Y ou

a ∈ Y −X. Se a ∈ X−Y entao ξX(a) = 1, ξY (a) = 0 e, consequentemente, ξX(a)+ξY (a) = 1. Se a ∈ Y −X entaoξX(a) = 0, ξY (a) = 1 e, consequentemente, ξX(a) + ξY (a) = 1. Logo, ξX∪Y (a) = 1 e ξX∩Y (a) = 0 implicam queξX(a) + ξY (a) = 1. Portanto, ξX∪Y (a) = 1 e ξX∩Y (a) = 0 implicam que ξX(a) + ξY (a)− ξX∩Y (a) = 1 = ξX∪Y (a).

Se ξX∪Y (a) = 1 e ξX∩Y (a) = 1, entao a ∈ X ∩ Y e, consequentemente, a ∈ X e a ∈ Y . Assim, ξX∪Y (a) = 1 eξX∩Y (a) = 1 implicam que ξX(a) = ξY (a) = 1. Logo, ξX∪Y (a) = 1 e ξX∩Y (a) = 1 implicam que ξX(a)+ξY (a) = 2.Portanto, ξX∪Y (a) = 1 e ξX∩Y (a) = 1 implicam que ξX(a) + ξY (a)− ξX∩Y (a) = 1 = ξX∪Y (a).

Como ξX∪Y (a) = 0 ou 1 e ξX∩Y (a) = 0 ou 1, temos que ξX(a) + ξY (a) − ξX∩Y (a) = ξX∪Y (a) em todos oscasos. E, como a ∈ A e arbitrario, temos que ξX + ξY − ξX∩Y = ξX∪Y .

Em particular, temos que X ∩ Y = ∅ e equivalente a ξX∩Y = 0 (i.e. ξX∩Y e a funcao nula) que e equivalente(pelo que foi demonstrado acima) a ξX + ξY = ξX∪Y .

c)Suponhamos que X ⊂ Y . Dado a ∈ A, temos que ξX(a) = 0 ou 1. No primeiro caso, temos imediatamente que

ξX(a) 6 ξY (a) = 0 ou 1. No segundo, temos que a ∈ X ⊂ Y e, consequentemente, ξX(a) = 1 = ξY (a). Em todocaso, ξX(a) 6 ξY (a). Como a ∈ A e arbitrario, concluimos que ξX 6 ξY .

Suponhamos que ξX 6 ξY . Dado x ∈ X, temos que ξX(x) 6 ξY (x) e, consequentemente, ξY (x) = 1. Logo,x ∈ X implica que x ∈ Y . E temos que X ⊂ Y .

d)Seja a ∈ A.Se ξA−X(a) = 0 temos que a /∈ A−X e, consequentemente, a ∈ X. Logo, ξA−X(a) = 0 implica que ξX(a) = 0.

Assim, ξA−X(a) = 0 implica que 1(a)− ξX(a) = 0 = ξA−X(a).Se ξA−X(a) = 1 temos que a ∈ A−X e, consequentemente, a /∈ X. Logo, ξA−X(a) = 1 implica que ξX(a) = 0.

Assim, ξA−X(a) = 1 implica que 1(a)− ξX(a) = 1 = ξA−X(a).Em todos casos, temos que 1(a)− ξX(a) = ξA−X(a). E, como a ∈ A e arbitrario, temos que 1− ξX = ξA−X .

61

Exercıcio 2.26:

Prove que o conjunta das sequencias crescentes (n1 < n2 < n3 < ...) de numeros naturais nao e enumeravel.

Usaremos o argumento da diagonal de Cantor.Suponhamos, por absurdo, que exista um enumeracao a1, a2, a3, ... das sequencias crescentes de numeros naturais

ai, i ∈ N, dadas porai1 < ai2 < ai3 < ...

Temos que a sequencia b, definida indutivamente por

b1 = a1 + 1

ebn+1 = max(bn, an+1) + 1

nao pertence a enumeracao acima. De fato, temos, pela definicao de b, que

aii < bi

e,consequentemente,b 6= ai

para todo i ∈ N.

62

Exercıcio 2.27:

Sejan (N, s) e (N′, s′) dois pares formados, cada um, por um conjunto e uma funcao. Suponhamos que amboscumpram os axiomas de Peano. Prove que existe uma unica bijecao f : N→ N′ tal que f(1) = 1′, f(s(n)) = s′(f(n)).Conclua que:

a) m < n⇔ f(m) < f(n);

b) f(m+ n) = f(m) + f(n);

c) f(m · n) = f(m) · f(n).

Como (N, s) e (N′, s′) satisfazem os axiomas de Peano, devemos ter que a funcao f : N → N fica bem definidapor

f(1) = 1′

ef(s(n)) = s′(f(n))

para todo n ∈ N. De fato, f esta definida em todo N = {1} ∪ s(N) pois esta definida em {1} e s(N). Como se injetiva, segue que f esta bem definida. Em particular, segue que f e a unica funcao N → N′ satisfazendo ascondicoes do enunciado. Temos, tambem, que f e injetiva. De fato, seja

X = {n ∈ N; f(n) /∈ f(N− {n})}.

Temos que 1 ∈ X pois, pela definicao de f ,

f(N− {1}) = f(s(N)) ⊂ s′(N′) = N′ − {1′} = N′ − {f(1)}.

E, se n ∈ X, temos que s(n) ∈ X. De fato, temos que f(s(n)) 6= f(1) = 1′ pois

f(s(n)) = s′(f(n)) ∈ s′(N′) = N′ − {1′}

e, se m 6= n e f(m) 6= f(s(m)), entao temos que

f(n) 6= f(m),

pois n ∈ X, e, consequentemente,

f(s(n)) = s′(f(n)) 6= s′(f(m)) = f(s(m)).

Implicando, pelo PIF, que f(s(n)) /∈ f(N − {s(n)} ou, equivalentemente, s(n) ∈ X. Portanto, pelo PIF, temosque X = N e, da definicao de X, concluımos que f e injetiva. Temos, tambem, que f e sobrejetiva. De fato,1′ = f(1) ∈ f(N) e, se n = f(k) ∈ f(N) temos que

s′(n) = s′(f(k)) = f(s(k)) ∈ f(N).

Assim, pelo PIF, temos que f(N) = N′. Portanto, f e uma bijecao.a)Provaremos primeiro que m < n implica f(m) < f(n). Seja

X = {p ∈ N; f(n+ p) > f(n), ∀n ∈ N}.

Temos que 1 ∈ N pois, dado n ∈ N,

f(n+ 1) = f(s(n)) = s′(f(n)) > f(n).

Supondo que p ∈ X, temos, para qualquer n ∈ N, que

f(n+ s(p)) = f(s(n) + p) > f(s(n)) = s′(f(n)) > f(n).

63

Logo, p ∈ X implica que s(p) ∈ X. Assim, concluımos, pelo PIF, X = N. Portanto, se m < n, temos, para p ∈ Ntal que n = m+ p, que

f(m) < f(m+ p) = f(n).

Agora, provaremos que f(m) < f(n) implica que m < n. Seja

X = {p ∈ N′; f(n) = f(m) + p implica m < n}.

Temos que 1′ ∈ X. De fato,f(n) = f(m) + 1′

implica quef(n) = s′(f(m)) = f(s(m))

e, consequentemente,n = s(m) > m.

Supondo que p ∈ X, temos quef(n) = f(m) + s′(p)

implica quef(n) = s′(f(m)) + p = f(s(m)) + p

e, consequentemente,n > s(m) > m.

Logo, p ∈ X implica que s′(p) ∈ X. Assim, concluımos que X = N′. Portanto, se f(m) < f(n), temos, para p ∈ N′tal que f(n) = f(m) + p, que

m < n.

b)Seja

X = {n ∈ N; f(n+m) = f(n) + f(m), ∀m ∈ N}.

Provaremos que X = N e teremos, assim, o resultado.Comecamos mostrando que 1 ∈ X. Temos que

f(1 + 1) = f(s(1)) = s′(f(1)) = f(1) + 1′ = f(1) + f(1).

E, sef(1 +m) = f(1) + f(m),

temos quef(1 + s(m)) = f(s(s(m))) = s′(f(s(m))) = 1′ + f(s(m)) = f(1) + f(s(m)).

Logo, pelo PIF, temos que f(1 +m) = f(1) + f(m) para todo m ∈ N. Ou seja, 1 ∈ X.Suponhamos que n ∈ X. Provaremos que s(n) ∈ X. Temos que

f(s(n) + 1) = f(s(s(n))) = s′(f(s(n))) = f(s(n)) + 1′ = f(s(n)) + f(1).

Adiante, sef(s(n) +m) = f(s(n)) + f(m),

temos quef(s(n) + s(m)) = f(s(s(n) +m))

= s′(f(s(n) +m))= f(s(n) +m) + 1′

= f(s(n)) + f(m) + 1′

= f(s(n)) + f(1) + f(m)= f(s(n)) + f(1 +m)= f(s(n)) + f(s(m)).

Assim, concluımos, pelo PIF, que f(s(n) + m) = f(s(n)) + f(m) para todo m ∈ N. Logo, n ∈ X implica ques(n) ∈ X.

Portanto, concluımos, pelo PIF, que X = N.

64

c)Seja

X = {n ∈ N; f(n ·m) = f(n) · f(m), ∀m ∈ N}.

Provaremos que X = N e teremos, assim, o resultado.Comecamos mostrando que 1 ∈ X. De fato, dado m ∈ N, temos que

f(1 ·m) = f(m) = 1′ · f(m) = f(1) · f(m).

Suponhamos que n ∈ X. Provaremos que s(n) ∈ X. Temos que

f(s(n) · 1) = f(s(n)) = f(s(n)) · 1′ = f(s(n)) · f(1)

Adiante, sef(s(n) ·m) = f(s(n)) · f(m),

temos quef(s(n) · s(m)) = f(s(n) ·m+ s(n) · 1)

= f(s(n) ·m) + f(s(n) · 1)= f(s(n) · f(m) + f(s(n)) · 1′= f(s(n)) · (f(m) + 1′)= f(s(n)) · s′(f(m))= f(s(n)) · f(s(m)).

Assim, concluımos, pelo PIF, que f(s(n)·m) = f(s(n))·f(m) para todo m ∈ N. Logo, n ∈ X implica que s(n) ∈ X.Portanto, concluımos, pelo PIF, que X = N.

65

Exercıcio 2.28:

Dada uma sequencia de conjuntosA1, A2, ...,An, ..., considere os conjuntos lim supAn = ∩∞n=1(∪∞i=nAi) e lim infAn =∪∞n=1(∩∞i=nAi).

a) Prove que lim supAn e o conjunto dos elementos que pertencem a An para uma infinidade de valores de n e quelim infAn e o conjunto dos elementos que que pertencem a todo An salvo para um numero finito de valoresde n.

b) Conclua que lim infAn ⊂ lim supAn;

c) Mostre que se An ⊂ An+1 para todo n entao lim infAn = lim supAn = ∪∞n=1An;

d) Por outro lado, se An ⊃ An+1 para todo n entao lim infAn = lim supAn = ∩∞n=1An;

e) De exemplo de uma sequencia (An) tal que lim supAn = lim infAn;

f) De exemplo de uma sequencia para a qual os dois limites coincidem mas Am 6⊂ An para todos m 6= n.

a)Sejam

X := {a ∈ ∪∞n=1An | a ∈ An para uma infinidade de valores de n}

eY := {a ∈ ∪∞n=1An | a ∈ An para todos exceto uma quantidade finita de valores de n}.

Dado a ∈ X, temos que{i ∈ N | a ∈ Ai}

e ilimitado. Sendo assim, dado n ∈ N, existe i ∈ N tal que n < i e a ∈ Ai. Isso implica que

a ∈ ∪∞i=nAi

para todo n ∈ N. Portanto,a ∈ ∩∞n=1(∪∞i=nAi) = lim supAn.

E, como a e um elemento arbitrario de X, temos que X ⊂ lim supAn.Seja a ∈ lim supAn. Temos que

a ∈ ∪∞i=nAipara todo n ∈ N. Segue daı que

I := {i ∈ N | a ∈ Ai}

e infinito, pois, caso contrario, existiria n ∈ N tal que n > i para todo i ∈ I e, consequentemente, terıamos quea 6∈ ∪∞i=nAi. Portanto, a ∈ X. E, como a e um elemento arbitrario de lim supAn, temos que X ⊃ lim supAn.

Dado a ∈ Y , temos queI := {i ∈ N | a ∈ Ai} = N− J,

para alum J ⊂ N finito. E, como J e finito e, consequentemente, limitado, existe k ∈ N tal que i > k implicai 6∈ J . Logo, para todo i > k, temos que i ∈ I e, consequentemente,

a ∈ ∩∞i=kAi.

Assim, como ∩∞i=kAi ⊂ ∪∞n=1(∩∞i=nAi), temos que

a ∈ ∪∞n=1(∩∞i=nAi) = lim infAn.

Como a e um elemento arbitrario de Y , concluımos daı que Y ⊂ lim infAn.Seja a ∈ lim infAn. Temos que existe k ∈ N tal que

a ∈ ∩∞i=kAi.

Isso implica que o conjuntoI := {i ∈ N | a ∈ Ai}

66

e tal queN− I ⊂ {1, ..., k − 1}.

Logo, a ∈ Y . Como a e um elemento arbitrario de lim infAn, temos que Y ⊃ lim infAn.b)Pelo ıtem anterior, temos que

lim supAn := {a ∈ ∪∞n=1An | a ∈ An para uma infinidade de valores de n}

elim infAn := {a ∈ ∪∞n=1An | a ∈ An para todos exceto uma quantidade finita de valores de n}.

Assim, segue quelim infAn ⊂ lim supAn.

c)Como

∪∞i=nAi ⊂ ∪∞i=1Ai

para todo n ∈ N, temos, imediatamente que

lim supAn = ∩∞n=1(∪∞i=nAi) ⊂ ∪∞n=1An.

Por outro lado, dado a ∈ ∪∞n=1An, temos que a ∈ Ak para algum k ∈ N. Por inducao em p ∈ N, prova-se que

Ak ⊂ Ak+p

para todo p ∈ N. Portanto,

a ∈ Ak ⊂ ∩∞p=0Ak+p = ∩∞i=kAi ⊂ ∪∞n=1(∩∞i=nAi) = lim infAn.

Assim, a ∈ lim infAn e, como a e um elemento arbitrario de ∪∞n=1An, concluimos que ∪∞n=1An ⊂ lim infAn.Concluımos, entao, do ıtem b) e do que foi discutido acima, que

∪∞n=1An ⊂ lim infAn ⊂ lim supAn ⊂ ∪∞n=1An.

Logo, temos quelim infAn = lim supAn = ∪∞n=1An.

d)Temos imediatamente que

∩∞n=1An = ∩∞i=1Ai ⊂ ∪∞n=1(∩∞i=nAi) = lim infAn.

Seja a ∈ lim supAn. Por inducao em p, temos que

Ak ⊂ Ak+p,

para todo k e p ∈ N. Assim,Ak ⊃ ∪∞p=0Ak+p = ∪∞i=kAi.

E, temos que a ∈ Ak, para todo k ∈ N, pois

a ∈ lim supAn = ∩∞n=1(∪∞i=nAi) ⊂ ∪∞i=kAi ⊂ Ak.

Ou seja, a ∈ ∩∞n=1An. Como a e um elemento arbitrario de lim supAn, temos que lim supAn ⊂ ∩∞n=1An.Concluımos, entao, do ıtem b) e do que foi discutido acima, que

∩∞n=1An ⊂ lim infAn ⊂ lim supAn ⊂ ∩∞n=1An.

Logo, temos quelim infAn = lim supAn = ∩∞n=1An.

e)

67

DefinindoA2k−1 := {1}

eA2k := ∅,

para todo k ∈ N, temos quelim supAn = ∩∞n=1(∪∞i=nAi) = ∩∞n=1{1} = {1}

elim infAn = ∪∞n=1(∩∞i=nAi) = ∪∞n=1∅ = ∅.

f)Definindo

An := {n},

temos queAn 6⊂ Am,

para todos n 6= m,lim supAn = ∩∞n=1(∪∞i=nAi) = ∩∞n=1{k ∈ N | k > n} = ∅

elim infAn = ∪∞n=1(∩∞i=nAi) = ∪∞n=1∅ = ∅.

68

Exercıcio 2.29:

(Teorema de Bernstein-Schroeder) Dados conjuntos A e B, suponha que existam funcoes injetivas f : A → B eg : B → A. Prove que existe uma bijecao h : A→ B.

Solucao 1:

Seja C := g(B) ⊂ A. Mostraremos que existe uma bijeccao H : A→ C. Assim, como g : B → C e uma bijecao,temos que h := g−1 ◦H : A→ B e uma bijecao.

Como f : A→ B e g : B → C sao funcoes injetivas, temos que F := g ◦ f : A→ C e uma funcao injetiva. Logo,F (X − Y ) = F (X)− F (Y ), para todos X e Y ⊂ A.

Definimos A1 := A e C1 := C. E, pelo princıpio da definicao recursiva, podemos definir

An := F (An−1)

eCn := F (Cn−1)

para todo n > 1.Desta forma, dado x ∈ A, temos que x ∈ An − Cn, para algum n ∈ Z+, ou x ∈ C (pois x /∈ A1 − C1 = A− C

implica que x ∈ C). Entao, a funcao H : A→ C dada por

H(x) =

{F (x) se x ∈ An − Cn para algum n ∈ Z+;x caso contrario,

para todo x ∈ A, e bem definida.Sejam x e y ∈ A tais que H(x) = H(y). Se x ∈ An − Cn, para algum n ∈ Z+, entao y ∈ Ak − Ck, para algum

k ∈ Z+. De fato, se y /∈ Ak −Bk, para todo k ∈ Z+, entao

y = H(y) = H(x) = F (x) ∈ F (An −Bn) = F (An)− F (Bn) = An+1 −Bn+1

(pois F e injetivo). Uma contradicao. Assim, se x ∈ An − Cn, para algum n ∈ Z+, entao

F (x) = H(x) = H(y) = F (y)

e, consequentemente, x = y. Alem disso, se x /∈ An − Cn, para todo n ∈ Z+, entao, como no caso anterior, temosque y /∈ An − Cn, para todo n ∈ Z+, e, consequentemente,

x = H(x) = H(y) = y.

Logo, em todo caso, x = y. Portanto, podemos concluir que H e injetivo.Seja x ∈ C. Se x ∈ An − Cn = F (An−1)− F (Cn−1) = F (An−1 − Cn−1), para algum n > 1, temos que

x = F (y) = H(y)

para algum y ∈ An−1 − Cn−1. Por outro lado, se x /∈ An − Cn, para todo n ∈ Z+, Temos que

x = h(x).

Logo, podemos concluir que H e sobrejetiva.Portanto, temos que H : A→ C e uma bijecao. E o resultado segue.

Solucao 2:

A grosso modo analizaremos as quantidades de elmentos das sequencias das formas

x, g−1(x), f−1 ◦ g−1(x), g−1 ◦ f−1 ◦ g−1(x), ...,

x ∈ A, ey, f−1(y), g−1 ◦ f−1(y), f−1 ◦ g−1 ◦ f−1(y), ...,

69

y ∈ B. Daı particionaremos A e B de modo a obter uma funcao entre A e B cuja bijetividade e herdada dainjetividade de f e g.

SejamAI = {x ∈ A; existe k ∈ Z>0 tal que (g ◦ f)−k(x) 6= ∅ e g−1 ◦ (g ◦ f)−k(x) = ∅},

AP = {x ∈ A; existe k ∈ Z>0 tal que g−1 ◦ (g ◦ f)−k(x) 6= ∅ e (g ◦ f)−(k+1)(x) = ∅},

A∞ = {x ∈ A; para todo k ∈ Z>0 vale (g ◦ f)−k(x) 6= ∅ e g−1 ◦ (g ◦ f)−k(x) 6= ∅},

BI = {y ∈ B; existe k ∈ Z>0 tal que (f ◦ g)−k(y) 6= ∅ e f−1 ◦ (f ◦ g)−k(y) = ∅},

BP = {y ∈ A; existe k ∈ Z>0 tal que f−1 ◦ (f ◦ g)−k(y) 6= ∅ e (f ◦ g)−(k+1)(x) = ∅}

eB∞ = {y ∈ A; para todo k ∈ Z>0 vale (f ◦ g)−k(y) 6= ∅ e f−1 ◦ (f ◦ g)−k(y) 6= ∅}.

Segue queA = AI ∪AP ∪A∞

eB = BI ∪BP ∪B∞.

Agora, provaremos tres detalhes tecnicos sobre f e g:

(I) f(AI) = BP

y ∈ BP ⇐⇒ ∃k ∈ Z>0 tal que f−1 ◦ (f ◦ g)−k(y) 6= ∅ e (f ◦ g)−(k+1)(x) = ∅⇐⇒ f−1 = {x} onde

∃k ∈ Z>0 tal que (g ◦ f)−k(x) = f−1 ◦ (f ◦ g)−k(y) 6= ∅ e g−1 ◦ (g ◦ f)−k(x) = (f ◦ g)−(k+1) = ∅⇐⇒ ∃x ∈ AI tal que f(x) = y⇐⇒ y ∈ f(AI).

(II) g(BI) = AP

Prova-se de modo analogo a prova de (I).

(III) f(A∞) = B∞

y ∈ B∞ ⇐⇒ ∀k ∈ Z>0 vale f−1 ◦ (f ◦ g)−k(y) 6= ∅ e (f ◦ g)k(x) 6= ∅⇐⇒ f−1 = {x} onde

∀k ∈ Z>0 vale (g ◦ f)−k(x) = f−1 ◦ (f ◦ g)−k(y) 6= ∅ e g−1 ◦ (g ◦ f)−k(x) = (f ◦ g)−k 6= ∅⇐⇒ ∃x ∈ A∞ tal que f(x) = y⇐⇒ y ∈ f(A∞).

Agora, segue de A = AI ∪AP ∪A∞ e de (II), que a funcao

H : A → Bx ∈ AI → f(x) ∈ BPx ∈ AP → y ∈ BI , tal que g(y) = xx ∈ A∞ → f(x) ∈ B∞.

e bem definida e, da injetividade de f e g e de (I)-(III), que esta e uma bijecao.

70

Capıtulo 3

Numeros Reais

71

Exercıcio 3.01:

Dados a, b, c, d num corpo K, sendo b e d diferentes de zero, prove:

(1)a

b+c

d=ad+ bc

bd;

(2)a

b· cd

=a · cb · d

.

(1)

Temos quea

b= b−1a = b−1d−1da = (bd)−1(ad) =

ad

bd

e, analogamente,c

d=bc

bd.

Assim,a

b+c

d=

ad

bd+bc

bd= (bd)−1(ad) + (bd)−1(bc)= (bd)−1(ad+ bc)

=ad+ bc

bd.

(2)

Temos quea

b· cd

= (b−1a) · (d−1c) = acb−1d−1 = (bd)−1(ac) =a · cb · d

.

72

Exercıcio 3.02:

Dado a 6= 0 em um corpo K, poe-se, por definicao, a0 = 1 e, se n ∈ Z+, a−n = 1an ou seja, a−n = (an)−1. Prove:

(1) am · an = am+n;

(2) (am)n = amn sejam quais forem m e n ∈ Z.

Lembremos que as potencias de a sao definidas indutivamente pelas igualdades

a1 := a

ean+1 := an · a.

(1)

Primeiramente, provaremos que, para todo m ∈ Z, vale a igualdade

am · a = am+1. (3.1)

Para m = 0,a0 · a = 1 · a = a1 = a0+1.

E, para m ∈ Z+, a igualdade am · a = am+1 segue da definicao.Logo, resta mostrarmos, por inducao em k ∈ Z+, que

a−k · a = a−k+1.

Para k = 1, temos quea−1 · a = 1 = a0 = a−1+1.

E, supondo que a−k · a = a−k+1 para algum k ∈ Z+, temos que

a−(k+1) · a = (ak+1)−1 · a = (ak · a)−1 = a−k · a−1 · a = a−k = a−(k+1)+1.

Logo, pelo PIF, a−k · a = a−k+1, para todo k ∈ Z+.Portanto, temos a equacao (3.1).Nosso proximo passo sera provar, por inducao em n ∈ Z+, que

am · an = am+n, (3.2)

para todo m ∈ Z.Para n = 1, a igualdade (3.2) e simililar a igualdade (3.1). E, supondo que am · an = am+n para algum n ∈ Z+,

temos, novamente por (3.1), queam · an+1 = am · an · a

= am+n · a= a(m+n)+1

= am+(n+1).

Portanto, temos a equacao (3.2).Para qualquer m ∈ Z,

am · a0 = am+0 (3.3)

ja que a0 = 1.Por fim, para todo n ∈ Z−,

(am · an)−1 = (am)−1(an)−1

= a−m · a−n= a−m−n

= (am+n)−1,

73

e, consequentemente,am · an = am+n. (3.4)

Portanto, por (3.2), (3.3) e (3.4), temos que

am · an = am+n,

para todos m e n ∈ Z.

(2)

Primeiramente, provaremos, por inducao em n ∈ Z+, que

(am)n = amn, (3.5)

para todo m ∈ Z.Para n = 1, a equacao (3.5) se verifica trivialmente. Suponhamos que, para algum n ∈ Z+, a igualdade

(am)n = amn

seja valida para todo m ∈ Z. Desta forma,

(am)n+1 = (am)n · am= amnam

= amn+m

= am(n+1).

Portanto, a igualdade (3.5) e valida.Com isso, para todo m ∈ Z e n ∈ Z−,

((am)n)−1 = (am)−n

= am(−n)

= a−mn

= (amn)−1

e, consequentemente,(am)n = amn.

Por fim,(am)0 = 1 = am0

e, desta forma, concluımos que(am)n = amn,

para todo m e n ∈ Z.

74

Exercıcio 3.03:

Sex1

y1=x2

y2= · · · = xn

yn

num corpo K, prove que, dados a1, a2, . . . , an ∈ K tais que

a1y1 + a2y2 + · · ·+ anyn 6= 0,

tem-sea1x1 + a2x2 + · · ·+ anxna1y1 + a2y2 + . . . anyn

=x1

y1.

Seja

p :=x1

y1=x2

y2= · · · = xn

yn.

Desta forma, devemos ter quexi = pyi,

para todo i = 1, . . . , n. Logo,

a1x1 + a2x2 + · · ·+ anxn = a1py1 + a2py2 + · · ·+ anpyn = p(a1y1 + a2y2 + · · ·+ anyn).

Portanto, como a1y1 + a2y2 + · · ·+ anyn 6= 0, devemos ter que

a1x1 + a2x2 + · · ·+ anxna1y1 + a2y2 + . . . anyn

= p =x1

y1.

75

Exercıcio 3.04:

Sejam K e L corpos. Uma funcao f : K → L chama-se um homomorfismo quando se tem

f(x+ y) = f(x) + f(y)

ef(x · y) = f(x) · f(y),

quaisquer que sejam x e y ∈ K.

i) Dado um homomorfismo f : K → L, prove que f(0) = 0.

ii) Prove tambem que, ou f(x) = 0 para todo x ∈ K, ou entao f(1) = 1 e f e injetivo.

(i)

Temos quef(0) = f(0 + 0) = f(0) + f(0).

Assim, pela Lei do Corte, devemos ter quef(0) = 0.

(ii)

Suponhamos que f : K → L nao seja um homomorfismo nao identicamente nulo. Provaremos que f(1) = 1 eque f e injetor.

Como f nao e uma funcao identicamente nula, existe x ∈ K tal que f(x) 6= 0. Desta forma,

f(1) · f(x) = f(1 · x) = f(x) = 1 · f(x)

implica, pela Lei do Corte, quef(1) = 1.

Mostraremos, agora, que f(x) 6= 0 para todo x ∈ K\{0}. Dado x ∈ K\{0},

f(x) · f(x−1) = f(x · x−1) = f(1) = 1.

Assim, se x ∈ K\{0}, devemos ter que f(x) possui inverso multiplicativo e, portanto, e diferente de zero.Por fim, dados x e y ∈ K com x 6= y, temos que

x− y 6= 0

e, consequentemenete,f(x)− f(y) = f(x− y) 6= 0.

Logo, se x e y ∈ K com x 6= y entaof(x) 6= f(y).

76

Exercıcio 3.05:

Seja f : Q→ Q um homomorfismo. Prove que, ou f(x) = 0 para todo x ∈ Q ou entao f(x) = x para todo x ∈ Q.

Suponhamos que o homomorfismo f : Q→ Q nao seja a funcao nula. Provaremos que f e a funcao identidade.Como f nao e a funcao nula, segue, do Exercıcio 3.4, temos que f e uma funcao injetiva tal que f(0) = 0,

f(1) = 1.Primeiramente, provaremos, por inducao em n ∈ Z+, que

f(n) = n.

Pelo que foi dito acima, a igualdade e valida para n = 1. Supondo que f(n) = n, para algum n ∈ Z+, temos que

f(n+ 1) = f(n) + f(1) = n+ 1.

Logo, pelo Princıpio da Inducao Finita, a identidade f(n) = n e valida para todo n ∈ Z+.Dado n ∈ Z+, temos que

n+ f(−n) = f(n) + f(−n) = f(n− n) = f(0) = 0

e, consequentemente,f(−n) = −n.

Com isso, concluimos quef(n) = n,

para todo n ∈ Z.Para m ∈ Z+, temos que

f

(1

m

)·m = f

(1

m

)· f(m) = f

(1

m·m)

= f(1) = 1

e, consequentemente,

f

(1

m

)=

1

m.

Por fim, para todo n/m ∈ Q, com n ∈ Z e m ∈ Z+, temos que

f( nm

)= f

(n · 1

m

)= f(n) · f

(1

m

)= n · 1

m=

n

m.

Portanto, f e a funcao identidade.

77

Exercıcio 3.08:

Seja K um conjunto onde sao validos todos os axiomas de corpo, salvo a existencia de inverso multiplicativo.

i) Dado a 6= 0 em K, prove que a funcao f : K → K, definida por f(x) = ax, e uma bijecao se, e somente se, apossui inverso.

ii) Mostre que f e injetiva se, e somente se, vale a lei do corte para a.

iii) Conclua que, se K e finito, a lei do corte e equivalente a existencia de inverso para cada elemento nao nulode K.

(i)

Seja a ∈ K\{0} e f : K → K a funcao dada por f(x) = ax, para cada x ∈ K.Suponhamos que f e bijetiva. Desta forma, existe x ∈ K tal que f(x) = 1. Ou seja, existe x ∈ X, tal que

ax = f(x) = 1.

Logo, a possui um inverso.Consideremos, agora, que existe a−1 ∈ K. Se x e y ∈ K sao tais que f(x) = f(y) entao

x = (a−1a)x = a−1(ax) = a−1f(x) = a−1f(y) = a−1(ay) = (a−1a)y = y.

E, dado x ∈ K,x = (aa−1)x = a(a−1x) = f(a−1x).

Portanto, f e bijetiva.

(ii)

Seja a ∈ K\{0} e f : K → K a funcao dada por f(x) = ax, para cada x ∈ K.A lei do corte para a e: se ax = ay para x e y ∈ K entao x = y.Suponhamos que valha a lei do corte para a. Se x e y ∈ K sao tais que f(x) = f(y) entao

ax = f(x) = f(y) = ay

e, pela lei do corte para a,x = y.

Logo, f e injetivo.Por outro lado, se f e injetivo, dados x e y ∈ K tais que ax = ay, devemos ter que

f(x) = ax = ay = f(y)

e, pela injetividade de f ,x = y.

Logo, vale a lei do corte para a se f for injetivo.

(iii)

Consideremos que K seja finito.Para cada a ∈ K, seja fa : K → K a funcao dada por fa(x) = ax, para cada x ∈ K.Suponhamos que valha a lei do corte em K e que a ∈ K\{0}. Pelo item (ii), temos que fa e injetiva. Sendo

fa : K → K injetiva e K um conjunto finito, a funcao fa tem de ser bijetiva (Exercıcio 2.10). Assim, pelo item (i),devemos ter que a possui um inverso em K.

Portanto, se vale a lei do corte em K, todo elemento de K\{0} possui inverso multiplicativo.Se, por outro lado, cada elemento de K\{0} possuir inverso multiplicativo devemos ter que dados a ∈ K\{0} e

x, y ∈ K tais que ax = ay entaox = a−1ax = a−1ay = y.

Ou seja, vale a lei do corte em K.

78

Exercıcio 3.09:

Explique por que as operacoes usuais nao tornam corpos o conjunto Z dos inteiros nem o conjunto Q[t] dospolinomios de coeficientes racionais.

Os conjuntos Z e Q[t], munidos das operacoes usuais, nao sao corpos pois nem todos os seus elementos possueminverso multiplicativo. Mostraremos que existem elementos a ∈ K = Z e Q[t] tais que a aplicacao f : K → K,dada por f(x) = ax, para x ∈ K, nao e uma sobrejecao. Logo, pelo item (i) do Exercıcio 3.8, devemos ter que taiselementos a ∈ K nao possuem inverso multiplicativo.

Sejam K = Q[t] e a = t. Temos que todo polinomio p ∈ f(Q[t]) tem zero como raız. Logo, f(Q[t]) 6= Q[t].Sejam K e a = 2. Todo numero n ∈ f(Z) e par. Logo, f(Z) 6= Z.

79

Exercıcio 3.10:

Propriedade 1. Dados x, y ∈ R, x2 + y2 = 0 ⇔ x = y = 0.

Demonstracao. ⇒).Suponha que x 6= 0, entao x2 > 0 e y2 ≥ 0 de onde segue que x2 + y2 > 0 , absurdo entao devevaler x2 = 0⇒ x = 0 logo temos tambem y2 = 0⇒ y = 0, portanto x = y = 0.⇐). Basta substituir x = y = 0 resultando em 0.

80

Exercıcio 3.11:

Exemplo 1. A funcao f : K+ → K+ com f(x) = xn, n ∈ N e crescente. Sejam x > y > 0 entao xn > yn poisxn =

∏nk=1 x >

∏nk=1 y = yn, por propriedade de multiplicacao de positivos. Se f : Q+ → Q+, Q+ o conjunto dos

racionais positivos, entao f nao e sobrejetiva para n = 2, pois nao existe x ∈ Q tal que x2 = 2 ∈ Q+.f(K+) nao e um conjunto limitado superiormente de K, isto e, dado qualquer x ∈ K existe y ∈ K+ tal que

yn > x. O limitante superior do conjunto, se existisse, nao poderia ser um numero negativou ou zero, pois paratodo y positivo tem-se yn positivo, que e maior que 0 ou qualquer numero negativo. Suponha que x positivo seja,tomando y = x+ 1 temos yn = (x+ 1)n ≥ 1 + nx > x, logo f(K+) nao e limitado superiormente.

81

Exercıcio 3.12:

Propriedade 2. Sejam X um conjunto qualquer e K um corpo, entao o conjunto F (X,K) munido de adicao emultiplicacao de funcoes e um anel comutativo com unidade, nao existindo inverso para todo elemento. Lembrandoque em um anel comutativo com unidade temos as propriedades, associativa, comutativa, elemento neutro e ex-istencia de inverso aditivo, para adicao. valendo tambem a comutatividade, associatividade, existencia de unidade1 para o produto e distributividade que relaciona as duas operacoes.

Demonstracao. • Vale a associatividade da adicao

((f + g) + h)(x) = (f(x) + g(x)) + h(x) = f(x) + (g(x) + h(x)) = (f + (g + h))(x)

• Existe elemento neutro da adicao 0 ∈ K e a funcao constante 0(x) = 0 ∀ x ∈ K, daı

(g + 0)(x) = g(x) + 0(x) = g(x).

• Comutatividade da adicao

(f + g)(x) = f(x) + g(x) = g(x) + f(x) = (g + f)(x)

• Existe a funcao simetrica, dado g(x), temos f com f(x) = −g(x) e daı

(g + f)(x) = g(x)− g(x) = 0.

• Vale a associatividade da multiplicacao

(f(x).g(x)).h(x) = f(x).(g(x).h(x))

• Existe elemento neutro da multiplicacao 1 ∈ K e a funcao constante I(x) = 1 ∀ x ∈ K, daı

(g.I)(x) = g(x).1 = g(x).

• Comutatividade da multiplicacao

(f.g)(x) = f(x)g(x) = g(x)f(x) = (g.f)(x)

Por ultimo vale a distributividade (f(g + h))(x) = f(x)(g(x) + h(x)) = f(x)g(x) + f(x)h(x) = (f.g + f.h)(x).Nao temos inverso multiplicativo para toda funcao, pois dada uma funcao, tal que f(1) = 0 e f(x) = 1 para

todo x 6= 1 em K, nao existe funcao g tal que g(1)f(1) = 1, pois f(1) = 0, assim o produto de f por nenhumaoutra funcao gera a identidade.

82

Exercıcio 3.13:

Propriedade 3. Sejam x, y > 0 . x < y ⇔ x−1 > y−1.

Demonstracao. ⇒). Como y > x e x−1 e y−1 sao positivos, multiplicamos a desigualdade por x−1y−1 em amboslados x−1y−1y > x−1y−1x implicando x−1 > y−1, entao se y > x temos 1

x >1y .

⇐). Se x−1 > y−1 . x, y sao positivos, multiplicamos a desigualdade por xy em ambos lados, de onde segue quey > x.

83

Exercıcio 3.14:

Propriedade 4. Sejam a > 0 em K e f : Z → K com f(n) = an. Nessas condicoes f e crescente se a > 1,decrescente se a < 1 e constante se a = 1.

Demonstracao. Para qualquer n ∈ Z vale f(n + 1) − f(n) = an+1 − an = an(a − 1), an e sempre positivo, entaoo sinal da diferenca depende do sinal de a − 1. Se a = 1 vale f(n + 1) = f(n) ∀ n ∈ Z logo f e constante, sea− 1 < 0, a < 1 entao f(n+ 1)− f(n) < 0, f(n+ 1) < f(n), f e decrescente e finalmente se a− 1 > 0, a > 1 entaof(n+ 1) > f(n) e a funcao e crescente.

Perceba que as propriedades citadas valem para todo n ∈ Z, por exemplo no caso de a > 1 temos

· · · < f(−4) < f(−3) < f(−2) < f(−1) < f(0) < f(1) < f(2) < f(3) < · · · < f(n) < f(n+ 1) < · · ·

analogamente para os outros casos.

84

Exercıcio 3.15:

Exemplo 2. Para todo x 6= 0 real, prove que (1 + x)2n > 1 + 2nx.Se x > −1 tomamos a desigualdade de bernoulli com 2n no expoente. Se x < −1 vale 1 +x < 0 porem elevando

a uma potencia par resulta num numero positivo, por outro lado 2nx < −2n logo 1 + 2nx < 1 − 2n < 0 entao(1 + x)2n e positivo e 1 + 2nx e negativo, logo nesse caso vale (1 + x)2n > 1 + 2nx .

85

Exercıcio 3.16:

Exemplo 3. Se n ∈ N e x < 1 entao (1 − x)n ≥ 1 − nx, pois de x < 1 segue que −x > −1 e daı aplicamos adesigualdade de Bernoulli (1 + y)n ≥ 1 + ny com y = −x.

86

Exercıcio 3.17:

Corolario 1. Se a e a+ x sao positivos, entao vale

(a+ x)n ≥ an + nan−1x.

Pois a+xa = (1+ x

a ) > 0 entao podemos aplicar a desigualdade de Bernoulli (1+y)n ≥ 1+ny com y = xa , resultando

em

(a+ x)n ≥ an + nan−1x.

Se a 6= 0, arbitrario em R, podendo agora ser negativo, substituımos y = xa em (1 + x)2n > 1 + 2nx. chegando

na desigualdade(a+ x)2n > a2n + a2n−12nx.

Se vale xa < 1 entao da desigualdade (1− y)n ≥ 1− ny, novamente tomamos y = x

a de onde segue

(a− x)n ≥ an − an−1nx.

87

Exercıcio 3.18:

Propriedade 5. Sejam sequencias (ak) , (bk) em um corpo ordenado K onde cada bk e positivo, sendo a1b1

omınimo e an

bno maximo dos termos da sequencia de termo ak

bkentao vale

a1

b1≤

n∑k=1

ak

n∑k=1

bk

≤ anbn.

Demonstracao. Para todo k vale a1b1≤ ak

bk≤ an

bn⇒ bk

a1b1≤ ak ≤ bk

anbn

pois bk > 0, aplicamos a soma∑nk=1 em

ambos lados, de onde seguen∑k=1

bka1

b1≤

n∑k=1

ak ≤n∑k=1

bkanbn

dividindo por∑nk=1 bk que e positivo, temos finalmente

a1

b1≤

n∑k=1

ak

n∑k=1

bk

≤ anbn.

88

Exercıcio 3.19:

Propriedade 6 (Multiplicatividade).|a||b| = |a.b|

para a e b reais quaisquer.

Demonstracao. Vale que |x.y|2 = (x.y)2 = x2y2 e (|x||y|)2 = |x|2|y|2 = x2.y2 os quadrados desses numeros saoiguais e eles sao nao negativos, entao segue que |x.y| = |x||y|.

2. |a.b| =√

(a.b)2 =√a2.b2 =

√a2.√b2 = |a||b|.

Propriedade 7. Se x 6= 0 entao | 1x | =1|x| .

Demonstracao. Vale |x|| 1x | = |xx | = 1 daı | 1x | e inverso de |x|, sendo 1

|x| .

Corolario 2 (Preserva divisao).

|xy| = |x||y|.

89

Exercıcio 3.20:

Propriedade 8.n∏k=1

|ak| = |n∏k=1

ak|

Demonstracao. Por inducao, para n = 1 vale, supondo para n numeros

n∏k=1

|ak| = |n∏k=1

ak|

vamos provar para n+ 1n+1∏k=1

|ak| = |n+1∏k=1

ak|

temosn+1∏k=1

|ak| =n∏k=1

|ak|.|an+1| = |n∏k=1

ak||an+1| = |n∏k=1

akan+1| = |n+1∏k=1

ak| .

Propriedade 9 (Desigualdade triangular generalizada). Sejam g(k) definida para k inteiro ,a, b ∈ Z, entao vale

|b∑

k=a

g(k)| ≤b∑

k=a

|g(k)|.

Demonstracao. Para cada k vale−|g(k)| ≤ g(k) ≤ |g(k)|

aplicando o somatorio em ambos lados segue

−b∑

k=a

|g(k)| ≤b∑

k=a

g(k) ≤b∑

k=a

|g(k)|

que implica

|b∑

k=a

g(k)| ≤ |b∑

k=a

|g(k)|| =b∑

k=a

|g(k)|

pois os termos |g(k)| somados sao nao negativos ,logo a soma desses termos e nao-negativa e o modulo da soma eigual a soma.

Propriedade 10. A identidade que provamos acima vale para numeros reais, vamos provar agora por inducao quese vale |z + w| ≤ |z|+ |w| para quaisquer z, w entao vale

|n∑k=1

zk| ≤n∑k=1

|zk|

de maneira que possa ser usada para numeros complexos , normas e outras estruturas que satisfazem a desigualdadetriangular.

2. Por inducao sobre n, para n = 1 tem-se

|1∑k=1

zk| = |z1| ≤1∑k=1

|zk| = |z1|

90

logo vale. Supondo a validade para n

|n∑k=1

zk| ≤n∑k=1

|zk|

vamos provar para n+ 1

|n+1∑k=1

zk| ≤n+1∑k=1

|zk|.

Da hipotese da inducao somamos |zn+1| em ambos lados, logo

|n+1∑k=1

zk| = |zn+1 +

n∑k=1

zk| ≤ |zn+1|+ |n∑k=1

zk| ≤n+1∑k=1

|zk|

Vejamos outras1 demonstracoes da desigualdade triangular

1Essas demonstracoes aprendi com Pedro Kenzo, obrigado por compartilhar as solucoes.

91

Exercıcio 3.22:

Vamos resolver um caso mais geral do problema.

Definicao 1 (Mediana). Dada uma sequencia finita (yk)n1 seus termos podem ser rearranjados para forma uma

sequencia nao-decrescente (xk)n1 . A mediana X e definida da seguinte maneira

• Se n e ımpar X = xn+12

.

• Se n e par X =xn

2+1+xn

2

2 .

Exemplo 4. Seja (xk)n1 uma sequencia crescente f : R→ R com f(x) =∑nk=1 |x− xk|. Se x < x1 entao

f(x) = −nx+

n∑k=1

xk

logo f e decrescente para x < x1. Tomando x > xn

f(x) = nx−n∑k=1

xk

logo f e crescente para x > xn.Seja agora x ∈ [xt, xt+1), t variando de 1 ate n− 1

f(x) =

t∑k=1

(x− xk)−n∑

k=t+1

(x− xk) = (2t− n)x+

t∑k=1

xk −n∑

k=t+1

xk

portanto a funcao e decrescente se t < n2 e crescente se t > n

2 , de t = 1 ate t = bn2 c em cada intervalo [xt, xt+1) afuncao e decrescente, sendo bn2 c segmentos decrescentes, de t = bn2 c + 1 ate n − 1, temos n − 1 − bn2 c segmentoscrescentes.

• Se n e ımpar f e decrescente em [xbn2 c, xbn2 c+1) e crescente em [xbn2 c+1, xbn2 c+2) logo o ponto xbn2 c+1 = xn+1

2

e o unico ponto de mınimo.

• Se n e par a funcao e constante em [xn2, xn

2 +1), todos os pontos desse intervalo sao pontos de mınimo. Em

especial o pontoxn

2+xn

2+1

2 e ponto de mınimo.

Concluımos que um ponto de mınimo acontece sempre na mediana da sequencia.

Exemplo 5. Achar o mınimo da funcao f(x) =∑nk=1 |x− k| para n ımpar e para n par.

Trocando n por 2n temos que o mınimo acontece no ponto x 2n2

= xn = n, substituımos entao tal valor na funcao

2n∑k=1

|n− k| =n∑k=1

|n− k|+2n∑

k=n+1

|n− k| =n∑k=1

(n− k) +

2n∑k=n+1

(−n+ k) =

=

n∑k=1

(n− k) +

n∑k=1

(k) =

n∑k=1

n = n.n = n2.

portanto o mınimo de∑2nk=1 |x− k| e n2.

• min{|x− 1|+ |x− 2|} = 1

• min{|x− 1|+ |x− 2|+ |x− 3|+ |x− 4|} = 4

• min{|x− 1|+ |x− 2|+ |x− 3|+ |x− 4|+ |x− 5|+ |x− 6|} = 9

• min{|x− 1|+ |x− 2|+ |x− 3|+ |x− 4|+ |x− 5|+ |x− 6|+ |x− 7|+ |x− 8|} = 16.

92

Agora para n ımpar, trocamos n por 2n + 1 o mınimo acontece no ponto x (2n+1)+12

= xn+1 = n + 1, aplicando na

funcao temos

2n+1∑k=1

|n+ 1− k| =n+1∑k=1

|n+ 1− k|+2n+1∑k=n+2

|n+ 1− k| =n+1∑k=1

(n+ 1− k) +

2n+1∑k=n+2

−(n+ 1) + k =

=

n∑k=1

(n+ 1− k) +

n∑k=1

k =

n∑k=1

(n+ 1) = n(n+ 1).

• min{|x− 1|+ |x− 2|+ |x− 3|} = 2

• min{|x− 1|+ |x− 2|+ |x− 3|+ |x− 4|+ |x− 5|} = 6

• min{|x− 1|+ |x− 2|+ |x− 3|+ |x− 4|+ |x− 5|+ |x− 6|+ |x− 7|} = 12

• min{|x− 1|+ |x− 2|+ |x− 3|+ |x− 4|+ |x− 5|+ |x− 6|+ |x− 7|+ |x− 8|+ |x− 9|} = 20.

93

Exercıcio 3.23:

Propriedade 11. |a− b| < ε⇒ |a| < |b|+ ε.

Demonstracao. Partindo da desigualdade |a− b| < ε, somamos |b| a ambos lados

|a− b|+ |b| < ε+ |b|

e usamos agora a desigualdade triangular

|a| ≤ |a− b|+ |b| < ε+ |b|

daı segue|a| ≤ ε+ |b|.

Da mesma forma vale se |a− b| < ε entao |b| ≤ ε+ |a| ⇒ |b| − ε ≤ |a| e com |a| ≤ ε+ |b|. temos

|b| − ε ≤ |a| ≤ ε+ |b|.

Vimos que |a− b| < ε implica |a| < |b|+ ε, mas como a ≤ |a| segue a < |b|+ ε.

94

Exercıcio 3.24:

Propriedade 12. Dado um corpo ordenado K , sao equivalentes

1. K e arquimediano.

2. Z e ilimitado superiormente e inferiormente.

3. Q e ilimitado superiormente e inferiormente.

Demonstracao. • 1⇒ 2. N ⊂ Z entao Z e ilimitado superiormente. Suponha por absurdo que Z seja limitadoinferiormente, entao existe a ∈ K tal que a < x ∀x ∈ Z, logo −a > −x, porem existe n natural tal quen > −a⇒ −n︸︷︷︸

∈Z

< a o que contraria a hipotese.

• 2⇒ 3 . Z ⊂ Q portanto Q e ilimitado superiormente e inferiormente.

• 3 ⇒ 1 . Para todo y ∈ K existe ab ∈ Q com a, b > 0 naturais tal que a

b > y, daı a > yb, podemos tomary = x

b , logo a > x, a ∈ N , portanto N e ilimitado superiormente e o corpo e arquimediano.

95

Exercıcio 3.25:

Propriedade 13. Seja K um corpo ordenado. K e arquimediado ⇔ ∀ε > 0 em K existe n ∈ N tal que 12n < ε.

Demonstracao. ⇒). Como K e arquimediano, entao ∀ε > 0 existe n ∈ N tal que n > 1ε ⇒ n + 1 > n > 1

ε pordesigualdade de Bernoulli temos 2n > n+ 1 > 1

ε ⇒1

2n < ε.⇐). Se ∀ε > 0 em K existe n ∈ N tal que 1

2n < ε, tomamos ε = 1x , x > 0 arbitrario entao x < 2n, com

2n = m ∈ N entao K e arquimediano, N nao e limitado superiormente.

96

Exercıcio 3.26:

Propriedade 14. Seja a > 1, K corpo arquimediano, f : Z → K com f(n) = an, entao

• f(Z) nao e limitado superiormente.

• inf(F (Z)) = 0.

Demonstracao. • Vale que a > 1 entao a = p+ 1 onde p > 0, por desigualdade de Bernoulli temos (p+ 1)n ≥1 + pn. ∀ x > 0 ∈ K existe n tal que n > x

p ⇒ pn > x ⇒ (p + 1)n ≥ 1 + pn > x, logo f(Z) nao e limitadosuperiormente.

• 0 e cota inferior de f(Z) pois vale 0 < an ∀n ∈ Z. Suponha que exista x tal que 0 < x < am ∀ m ∈ Z,sabemos que existe n ∈ N tal que an > 1

x daı x > 1an = a−n, absurdo, entao 0 deve ser o ınfimo.

97

Exercıcio 3.27:

Propriedade 15. Se s e irracional e u 6= 0 e racional entao u.s e irracional.

Demonstracao. Suponha que s e irracional e u.s seja racional, entao u.s = pq com p 6= 0 e q 6= 0 inteiros e como

u 6= 0 e racional ele e da forma u = jv , j 6= 0 e v 6= 0, inteiros, logo

j

vs =

p

q

multiplicando por vj ambos lados segue

s =p.v

j.q

que e um numero racional, logo chegamos a um absurdo.

Propriedade 16. Se s e irracional e t racional, entao s+ t e irracional.

Demonstracao. Suponha s + t racional, entao s + t = pq daı s = p

q − t que seria racional por ser diferenca de doisracionais, um absurdo entao segue que s+ t e irracional.

Exemplo 6. Existem irracionais a e b tais que a + b e a.b sejam racionais. Exemplos a = 1 +√

5 , b = 1 −√

5daı a+ b = 2 e a.b = 1− 5 = −4.

98

Exercıcio 3.28:

Propriedade 17. Sejam a, b, c, d racionais entao

a+ b√

2 = c+ d√

2⇔ a = c e b = d.

Demonstracao. ⇐). Se a = c e b = d a temos a+ b√

2 = c+ d√

2.⇒). Suponha a+ b

√2 = c+ d

√2 entao a− c =

√2(d− b), se d = b entao a = c e terminamos, se nao vale que

a− cd− b

=√

2

o que e absurdo pois√

2 e irracional.

99

Exercıcio 3.29:

Exemplo 7. O conjunto da forma {x+ y√p} onde x e y sao racionais e subcorpo dos numeros reais.

• O elemento neutro da adicao 0 pertence ao conjunto. Pois 0 = 0 + 0√p

• O elemento neutro da multiplicacao 1 pertence ao conjunto. Pois 1 = 1 + 0√p

• A adicao e fechada. Pois x+ y√p+ z + w

√p = x+ z + (y + w)

√p.

• O produto e fechado. Pois (x+ y√p)(z + w

√p) = xz + xw

√p+ yz

√p+ y.wp.

• Dado x ∈ A implica −x ∈ A. Pois dado x+ y√p temos o simetrico −x− y√p.

• Dado x 6= 0 ∈ A tem-se x−1 ∈ A. Pois dado x+ y√p temos inverso

x− y√px2 − y2p

como inverso multiplicativo.

Exemplo 8. O conjunto dos elementos da forma a+bα onde α = 3√

2 nao e um corpo pois o produto nao e fechado,vamos mostrar que α2 nao pertence ao conjunto.

Suponha que α2 = a+ bα entao α3 = aα+ bα2 = 2 substituindo a primeira na segunda temos que

aα+ b(a+ bα) = aα+ ab+ b2α = α(b2 + a) + ab = 2⇒ α(b2 + a) = 2− ab

se b2 + a 6= 0 entao α = 2−abb2+a o que e absurdo pois α e irracional, entao devemos ter a = −b2, multiplicamos a

expressao aα+ bα2 = 2 por α, de onde segue aα2 + 2b = 2α, substituindo α2 = a+ bα nessa ultima temos

a(a+ bα) + 2b = a2 + abα+ 2b = 2α⇒ α(2− ab) = 2b+ a2

se 2 6= ab chegamos num absurdo de α = 2b+a2

2−ab , temos que ter entao 2 = ab e a = −b2 de onde segue 2 = −b3,

porem nao existe racional que satisfaz essa identidade, daı nao podemos escrever α2 da forma a + bα com a e bracionais, portanto o produto de elementos nao e fechado e assim nao temos um corpo.

100

Exercıcio 3.30:

Propriedade 18. Sejam a, b ∈ Q+.√a+√b e racional ⇔

√a e√b sao racionais.

Demonstracao. ⇒).Se a = b entao 2

√a ∈ Q o que implica

√a =√b ∈ Q. Agora o caso de a 6= b.

Suponha que√a+√b e racional entao seu inverso tambem racional , que e

√a−√b

a−b , daı√a−√b ∈ Q , a soma

(√a+√b) + (

√a−√b) = 2

√a ∈ Q logo

√a ∈ Q, a diferenca de numeros racionais tambem e um numero racional

(√a+√b)−

√a =√b, portanto

√a e√b sao racionais.

⇐). A volta vale pois a soma de racionais e um racional.

101

Exercıcio 3.31:

Propriedade 19. Sejam A ⊂ R nao vazio limitado e c ∈ R, entao

1. c ≤ sup(A)⇔ ∀ ε > 0 ∃ x ∈ A tal que c− ε < x.

2. c ≥ inf(A)⇔ ∀ ε > 0 ∃ x ∈ A tal que c+ ε > x.

Demonstracao. 1. ⇒). Para todo ε > 0 vale que c− ε < sup(A). Dado ε > 0 fixo, se nao existisse x ∈ A tal quec− ε < x entao c− ε seria cota superior menor que o supremo, o que e absurdo, contraria o fato do supremoser a menor das cotas superiores.

⇐). Suponha por absurdo que fosse c > sup(A), poderıamos tomar c − sup(A) = ε daı c − c + sup(A) =sup(A) < x o que e absurdo.

2. ⇒). Para todo ε > 0 vale que c + ε < inf(A). Dado ε > 0 fixo, se nao existisse x ∈ A tal que c + ε > xentao c + ε seria cota superior menor que o ınfimo, o que e absurdo, contraria o fato do ınfimo ser a menordas cotas inferiores.

⇐). Suponha por absurdo que fosse c < inf(A), poderıamos tomar inf(A)− c = ε daı x < c+ inf(A)− c =inf(A) o que e absurdo.

102

Exercıcio 3.32:

Exemplo 9. Seja A = { 1n | n ∈ N} . Mostre que inf A = 0. Sabemos que 0 e uma cota inferior, agora vamos

mostrar que 0 e a menor delas. Dado 0 < x, x nao pode ser cota inferior, pois existe n natural tal que 1n < x, logo

0 e o ınfimo.

103

Exercıcio 3.33:

Propriedade 20. Se A e limitado inferiormente e B ⊂ A entao inf(A) ≤ inf(B).

Demonstracao. infA e cota inferior de A, logo tambem e cota inferior de B, sendo cota inferior de B vale infA ≤infB, pois inf B e a maior cota inferior de B.

Propriedade 21. Se A e limitado superiormente e B ⊂ A entao sup(A) ≥ sup(B).

Demonstracao. Toda cota superior de A e cota superior de B, logo o sup(A) e cota superior de B, como sup(B) ea menor das cotas superiores de B segue que sup(A) ≥ sup(B).

Corolario 3. Se A e B sao conjuntos limitados com B ⊂ A entao vale sup(A) ≥ sup(B) ≥ inf(B) ≥ inf(A) poistemos sup(A) ≥ sup(B) e inf(A) ≤ inf(B), tendo ainda que sup(B) ≥ inf(B).

104

Exercıcio 3.31:

Propriedade 22. Sejam A ⊂ R nao vazio limitado e c ∈ R, entao

1. c ≤ sup(A)⇔ ∀ ε > 0 ∃ x ∈ A tal que c− ε < x.

2. c ≥ inf(A)⇔ ∀ ε > 0 ∃ x ∈ A tal que c+ ε > x.

Demonstracao. 1. ⇒). Para todo ε > 0 vale que c− ε < sup(A). Dado ε > 0 fixo, se nao existisse x ∈ A tal quec− ε < x entao c− ε seria cota superior menor que o supremo, o que e absurdo, contraria o fato do supremoser a menor das cotas superiores.

⇐). Suponha por absurdo que fosse c > sup(A), poderıamos tomar c − sup(A) = ε daı c − c + sup(A) =sup(A) < x o que e absurdo.

2. ⇒). Para todo ε > 0 vale que c + ε < inf(A). Dado ε > 0 fixo, se nao existisse x ∈ A tal que c + ε > xentao c + ε seria cota superior menor que o ınfimo, o que e absurdo, contraria o fato do ınfimo ser a menordas cotas inferiores.

⇐). Suponha por absurdo que fosse c < inf(A), poderıamos tomar inf(A)− c = ε daı x < c+ inf(A)− c =inf(A) o que e absurdo.

105

Exercıcio 3.32:

Exemplo 10. Seja A = { 1n | n ∈ N} . Mostre que inf A = 0. Sabemos que 0 e uma cota inferior, agora vamos

mostrar que 0 e a menor delas. Dado 0 < x, x nao pode ser cota inferior, pois existe n natural tal que 1n < x, logo

0 e o ınfimo.

106

Exercıcio 3.33:

Propriedade 23. Se A e limitado inferiormente e B ⊂ A entao inf(A) ≤ inf(B).

Demonstracao. infA e cota inferior de A, logo tambem e cota inferior de B, sendo cota inferior de B vale infA ≤infB, pois inf B e a maior cota inferior de B.

Propriedade 24. Se A e limitado superiormente e B ⊂ A entao sup(A) ≥ sup(B).

Demonstracao. Toda cota superior de A e cota superior de B, logo o sup(A) e cota superior de B, como sup(B) ea menor das cotas superiores de B segue que sup(A) ≥ sup(B).

Corolario 4. Se A e B sao conjuntos limitados com B ⊂ A entao vale sup(A) ≥ sup(B) ≥ inf(B) ≥ inf(A) poistemos sup(A) ≥ sup(B) e inf(A) ≤ inf(B), tendo ainda que sup(B) ≥ inf(B).

107

Exercıcio 3.34:

Propriedade 25. Sejam A,B ⊂ R tais que para todo x ∈ A e todo y ∈ B se tenha x ≤ y. Entao supA ≤ inf B.

Demonstracao. Todo y ∈ B e cota superior de A, logo supA ≤ y para cada y pois supA e a menor das cotassuperiores, essa relacao implica que supA e cota inferior de B logo supA ≤ inf B, pois inf B e a maior cota inferior.

Propriedade 26. supA = inf B ⇔ para todo ε > 0 dado , existam x ∈ A e y ∈ B com y − x < ε.

Demonstracao. ⇐, usamos a contrapositiva. Nao podemos ter inf B < supA pela propriedade anterior, entaotemos forcosamente que inf B > supA, tomamos entao ε = inf B − supA > 0 e temos y − x ≥ ε para todo x ∈ Ae y ∈ B pois y ≥ inf B e supA ≥ x de onde segue −x ≥ − supA, somando esta desigualdade com a de y tem-sey − x ≥ inf B − supA = ε.⇒ , Se supA = inf B. Entao sendo para qualquer ε > 0, supA− ε

2 nao e cota superior de A, pois e menor queo supA (que e a menor cota superior), da mesma maneira inf A+ ε

2 nao e cota inferior de B, entao existem x ∈ Ae y ∈ B tais que

supA− ε

2< x ≤ supA = inf B ≤ y < inf B +

ε

2

inf B − ε

2< x ≤ y < inf B +

ε

2

de onde segue inf B − ε2 < x, −x < ε

2 − inf B e y < inf B + ε2 somando ambas tem-se y − x < ε.

108

Exercıcio 3.35:

Propriedade 27. Se c > 0 entao sup(c.A) = c. supA.

Demonstracao. Seja a = supA. Para todo x ∈ A tem-se x ≤ a, de onde segue cx ≤ ca, assim ca e cota superiorde cA. Seja d tal que d < ca entao d

c < a logo dc nao e cota superior de A, implicando a existencia de pelo menos

um x tal que dc < x, d < cx de onde segue que d nao e cota superior de cA, assim ca e a menor cota superior de

cA logo o supremo.

Propriedade 28. Se c > 0, inf cA = c inf A.

Demonstracao. Seja a = inf A, entao vale a ≤ x para todo x, multiplicando por c segue ca ≤ cx de onde concluımosque ca e cota inferior de cA. Seja d tal que ca < d, entao a < d

c , implicando que dc nao e cota inferior de A assim

existe x ∈ A tal que x < dc ⇒ cx < d, logo d nao e cota inferior de cA, implicando que c.a e a maior cota inferior,

logo o ınfimo do conjunto.

Propriedade 29. Se c < 0 entao inf(cA) = c supA.

Demonstracao. Seja a = supA . Tem-se x ≤ a para todo x ∈ A, multiplicando por c segue cx ≥ ca para todox ∈ A. Entao ca e uma cota inferior de cA. Se d > ca tem-se d

c < a como a e supremo, isso significa que existe

x ∈ A tal que dc < x logo d > cx, assim esse d nao e cota inferior, implicando que ca e a menor cota inferior, entao

ınfimo do conjunto.

A questao 35 segue da proxima propriedade com c = −1.

Propriedade 30. Se c < 0 entao sup(cA) = c inf A.

Demonstracao. Seja b = inf A entao vale b ≤ x para todo x ∈ A, multiplicando por c segue cb ≥ cx assim cb e cotasuperior de cA. Agora tome d tal que cb > d segue b < d

c , como b e ınfimo existe x ∈ A tal que x < dc , cx > d

assim esse d nao pode ser cota superior de cA, entao cb e a menor cota superior, logo o ınfimo.

109

Exercıcio 3.37:

Item I

Sejam A,B ⊂ R, conjuntos limitados .

Propriedade 31. O conjunto A+B = {x+ y | x ∈ A, y ∈ B} tambem e limitado.

Demonstracao. Se A e limitado , existe t tal que |x| < t para todo x ∈ A e se B e limitado existe u tal que |y| < u∀y ∈ B. Somando as desigualdades e usando desigualdade triangular segue |x|+|y| < u+t e |x+y| ≤ |x|+|y| < u+tlogo o conjunto A+B e limitado.

Item II

Propriedade 32 (Propriedade aditiva). Vale sup(A+B) = sup(A) + sup(B).

Demonstracao. Como A,B sao limitidados superiomente, temos supA := a e supB := b, como vale a ≥ x e b ≥ ypara todos x, y ∈ A,B respectivamente segue que a+ b ≥ x+ y logo o conjunto A+ B e limitado superiormente.Para todo e qualquer ε > 0 existem x, y tais que

a < x+ε

2, b < y +

ε

2

somando ambas desigualdades-segue-se quea+ b < x+ y + ε

que mostra que a+ b e a menor cota superior, logo o supremo, fica valendo entao

sup(A+B) = sup(A) + sup(B).

Item III

Propriedade 33. inf(A+B) = inf A+ inf B.

Demonstracao. Sejam a = infA e b = infB entao ∀x, y ∈ A,B tem-se a ≤ x, b ≤ y de onde segue por adicaoa+ b ≤ x+ y, assim a+ b e cota inferior de A+B. ∃x, y ∈ A,B tal que ∀ε > 0 vale x < a+ ε

2 e y < b+ ε2 pois a

e b sao as maiores cotas inferiores, somando os termos das desigualdades segue x+ y < a+ b+ ε, que implica quea+ b e a maior cota inferior logo o ınfimo.

110

Exercıcio 3.38:

Definicao 2 (Funcao limitada). Seja A ⊂ R, f : A → R e dita limitada quando o conjunto f(A) = {f(x) | x ∈A}, se f(A) e limitado superiormente entao dizemos que f e limitada superiormente e caso f(A) seja limitadoinferiormente dizemos que A e limitado inferiormente.

Seja uma funcao limitada f : V → R.

Definicao 3.sup f := sup f(V ) = sup{f(x) | x ∈ V }

Definicao 4.inf f := inf f(V ) = inf{f(x) | x ∈ V }

Propriedade 34. A funcao soma de duas funcoes limitadas e limitada.

Demonstracao. Vale |f(x)| ≤M1 e |g(x)| ≤M2 ∀x ∈ A entao

|f(x) + g(x)| ≤ |f(x)|+ |g(x)| ≤M1 +M2 = M

portando a funcao soma f + g de duas funcoes limitadas e tambem uma funcao limitada.

Sejam f, g : V → R funcoes limitadas e c ∈ R.

Propriedade 35.sup(f + g) ≤ sup f + sup g.

Demonstracao. Sejam

A = {f(x) | x ∈ V }, B = {g(y) | y ∈ V }, C = {g(x) + f(x) | x ∈ V }

temos que C ⊂ A+B, pois basta tomar x = y nos conjuntos, logo

sup(A+B) ≥ sup(f + g)

sup(A) + sup(B) = sup f + sup g ≥ sup(f + g)

Propriedade 36.inf(f + g) ≥ inf(f) + inf(g).

Demonstracao. De C ⊂ A+B segue tomando o ınfimo

inf(A+B) = inf(A) + inf(B) = inf(f) + inf(g) ≤ inf(C) = inf(f + g).

Exemplo 11. Sejam f, g : [0, 1]→ R dadas por f(x) = x e g(x) = −x

• Vale sup f = 1, sup g = 0, f + g = 0 logo sup(f + g) = 0 vale entao

sup f + sup g = 1 > sup(f + g) = 0.

• Temos ainda inf f = 0, inf g = −1, f + g = 0, inf(f + g) = 0 logo

inf f + inf g = −1 < inf(f + g) = 0.

As desigualdades estritas tambem valem se consideramos as funcoes definidas em [−1, 1], nesse caso sup f+sup g =2 e inf f + inf g = −2 e sup(f + g) = 0 = inf(f + g).

111

Exercıcio 3.39:

Definicao 5. Sejam A e B conjuntos nao vazios, definimos A.B = {x.y | x ∈ A, y ∈ B}.

Propriedade 37. Sejam A e B conjuntos limitados de numeros positivos, entao vale sup(A.B) = sup(A). sup(B).

Demonstracao. Sejam a = sup(A) e b = sup(B) entao valem x ≤ a e y ≤ b, ∀x ∈ A, y ∈ B daı x.y ≤ a.b, logoa.b e cota superior de A.B. Tomando t < a.b segue que t

a < b logo existe y ∈ B tal que ta < y daı t

y < a logo

existe x ∈ A tal que ty < x logo t < x.y entao t nao pode ser uma cota superior, implicando que a.b e o supremo

do conjunto.

Propriedade 38. Sejam A e B conjuntos limitados de numeros positivos, entao vale inf(A.B) = inf(A). inf(B).

Demonstracao. Sejam a = inf(A) e b = inf(B) entao valem x ≥ a e y ≥ b, ∀x ∈ A, y ∈ B daı x.y ≥ a.b, logo a.b ecota inferior de A.B. Tomando t > a.b segue que t

a > b logo existe y ∈ B tal que ta > y daı t

y > a logo existe x ∈ Atal que t

y > x logo t < x.y entao t nao pode ser uma cota inferior, implicando que a.b e o ınfimo do conjunto.

112

Exercıcio 3.40:

Propriedade 39. Sejam f, g : A→ R funcoes limitadas entao f.g : A→ R e limitada.

Demonstracao. Vale que |f(x)| < M1 e |g(x)| < M2 entao |f(x)g(x)| < M1M2 = M ∀ x ∈ A , portanto f.g : A→ Re limitada.

Propriedade 40. Sejam f, g : A→ R+ limitadas superiormente, entao

sup(f.g) ≤ sup(f) sup(g).

Demonstracao. Sejam C = {g(x).f(x) | x ∈ A} , B = {g(y). | y ∈ A} e A = {f(x) | x ∈ A} . Vale que C ⊂ A.Bpara ver isso basta tomar x = y nas definicoes acima, daı

sup(A.B) ≥ sup(C)

sup(A) sup(B) ≥ sup(C)

sup(f) sup(g) ≥ sup(f.g).

Propriedade 41. Sejam f, g : A→ R+ limitadas inferiormente, entao

inf(f.g) ≥ inf(f) inf(g).

Demonstracao. Sejam C = {g(x).f(x) | x ∈ A} , B = {g(y). | y ∈ A} e A = {f(x) | x ∈ A} . Vale que C ⊂ A.B,daı

inf(A.B) ≤ inf(C)

inf(A) inf(B) ≤ inf(C)

inf(f) inf(g) ≤ inf(f.g).

Exemplo 12. Sejam f, g : [1, 2]→ R dadas por f(x) = x e g(x) = 1x , vale sup f = 2, sup g = 1 sup f. sup g = 2 e

sup(f.g) = 1, pois f.g = 1 logosup f sup g > sup(f.g).

Da mesma maneira inf f = 1, inf g = 12 vale inf f. inf g = 1

2 e inf(f.g) = 1 portanto

inf f. inf g < inf(f.g).

Propriedade 42. Seja f : A→ R+ limitada superiormente entao sup(f2) = (sup f)2.

Demonstracao. Seja a = sup f tem-se f(x) ≤ a ∀x daı f(x)2 ≤ a2 entao a2 e cota superior de f2, e e a menorcota superior pois se 0 < c < a2 entao

√c < a logo existe x tal que

√c < f(x) < a e daı c < f(x)2 < a2 logo a2 e

a menor cota superior sup(f2) = sup(f)2.

Propriedade 43. Seja f : A→ R+ entao inf(f2) = (inf f)2.

Demonstracao. Seja a = inf f tem-se f(x) ≥ a ∀x daı f(x)2 ≥ a2 entao a2 e cota inferior de f2, e e a maior cotainferior pois se a2 < c entao a <

√c logo existe x tal que a < f(x) <

√c e daı a2 < f(x)2 < c logo a2 e a maior

cota inferior inf(f2) = inf(f)2.

113

Exercıcio 3.42:

Teorema 1 (Teorema das raızes racionais). Se o polinomio

f(x) =

n∑k=0

akxk

de coeficientes inteiros, tem uma raiz racional x = rs tal que mdc(r, s) = 1 entao s|an e r|a0.

Demonstracao. Se x = rs e raiz de f(x) =

n∑k=0

akxk, entao temos

f

(r

s

)=

n∑k=0

ak

(r

s

)k= 0

multiplicando por sn em ambos os lados temos

n∑k=0

akrk.sn−k = 0

como s|0 entao s|n∑k=0

akrk.sn−k , na soma s nao aparece como fator apenas quando n − k = 0, n = k, logo

abrindo o limite superior do somatorio temos

n−1∑k=0

akrk.sn−k + anr

n.sn−n =

n−1∑k=0

akrk.sn−k + anr

n = 0

daı s deve dividir anrn, como s e primo com r implica que tambem e primo com rn, portanto s deve dividir an.

Pelo mesmo argumento, temos que r|0 logo r deve dividirn∑k=0

akrk.sn−k, como o unico fator onde r nao aparece e

quando k = 0, abrimos o limite inferior do somatorio

a0r0.sn−0 +

n∑k=1

akrk.sn−k = a0.s

n +

n∑k=1

akrk.sn−k = 0

logo r deve dividir a0.sn, mas como r e primo com sn, ele deve dividir a0.

Corolario 5. Se o polinomio de coeficientes inteirosn∑k=0

akxk possui raızes racionais entao elas devem pertencer

ao conjunto

A = {pq| p|a0 q|an}.

Corolario 6. Se an = 1 em um polinomio de coeficientes inteiros P (x) =n∑k=0

akxk entao suas raızes racionais

devem ser inteiras, pois

A = {pq| p|a0 q|1}

entao q = 1 ou q = −1, e de qualquer forma implica que as solucoes sao da forma x = p para algum p ∈ Z. Entao, nessas condicoes, as raızes do polinomio P (x) sao inteiras ou irracionais.

Propriedade 44. Seja P (x) = xn − a, a > 0 ∈ Z, se a nao e n-esima potencia de um numero natural entao aunica raiz positiva de P , que e n

√a , e irracional.

Demonstracao. Como P possui coeficiente an = 1 entao ele possui raiz irracional ou inteira, se a raiz positiva mfosse inteira (logo natural) terıamos mn − a = 0 e daı a = mn e potencia de um numero natural, o que contraria ahipotese de a nao ser n-esima potencia de um numero natural, logo n

√a e irracional.

114

Exercıcio 3.43:

Propriedade 45. Sejam I um intervalo nao degenerado e k > 1 natural. O conjunto A = { mkn ∈ I | m,n ∈ Z} edenso em I.

Demonstracao. Dado ε > 0 existe n ∈ N tal que kn > 1ε , daı os intervalos [ mkn ,

m+1kn ] tem comprimento m+1

kn −mkn =

1kn < ε.

Existe um menor inteiro m+1 tal que x+ε ≤ m+1kn daı m

kn ∈ (x−ε, x+ε) pois se fosse x+ε < mkn iria contrariar

a minimalidade de m+ 1 e se fosse mkn < x− ε entao [ mkn ,

m+1kn ] teria comprimento maior do que de (x− ε, x+ ε),

que e ε, uma contradicao com a suposicao feita anteriormente.

115

Exercıcio 3.44:

Propriedade 46. O conjunto dos polinomios com coeficientes racionais e enumeravel.

Demonstracao. Seja Pn o conjunto dos polinomios com coeficientes racionais de grau ≤ n a funcao f : Pn → Qn+1

tal que

P (

n∑k=0

akxk) = (ak)n1

e uma bijecao. Como Qn+1 e enumeravel por ser produto cartesiano finito de conjuntos enumeraveis, segue que Pne enumeravel.

Sendo A o conjunto dos polinomios de coeficientes racionais, vale que

A =

∞⋃k=1

Pk

portanto A e uniao enumeravel de conjuntos enumeraveis , sendo assim A e enumeravel.

Definicao 6 (Numero algebrico). Um numero real (complexo) x e dito algebrico quando e raiz de um polinomiocom coeficientes inteiros.

Propriedade 47. O conjunto dos numeros algebricos e enumeravel.

1. Enumeramos A = {P1, P2, · · · , Pn, · · · }, o conjunto dos polinomios com coeficientes inteiros, definimos Bk comoconjunto das raızes reais de fk, entao vale que

B =

∞⋃k=1

Bk

como cada Bk e finito B fica sendo uniao enumeravel de conjuntos finitos, entao B e enumeravel.

2. Seja B o conjunto dos algebricos e A o conjunto dos polinomios com coeficientes inteiros. Para cada algebricox escolhemos um polinomio Px tal que Px(x) = 0.

Definimos a funcao f : B → A tal que F (x) = Px. Dado Px ∈ F (B), temos que o conjunto g−1(Px) dos valoresx ∈ B tal que f(x) = Px e finito pois Px︸︷︷︸

=y

possui um numero finito de raızes e daı tem-se

B =⋃

y∈f(B)

g−1(y)

logo B e uniao enumeravel de conjuntos enumeraveis ( no caso finitos), entao B e enumeravel.

Corolario 7. Existem numeros reais que nao sao algebricos, pois se todos fossem algebricos R seria enumeravel.

Definicao 7 (Numeros transcendentes). Os numeros reais que nao sao algebricos sao ditos transcendentais

Propriedade 48. O conjunto dos numeros algebricos e denso em R, pois todo racional e algebrico, o racional ba

e raiz do polinomio com coeficientes inteirosax− b = P (x)

ax− b = 0⇔ ax = b⇔ x = ba . E Q e denso em R.

116

Exercıcio 3.45:

Propriedade 49. Seja A enumeravel e B = R \A, entao para cada intervalo (a, b), (a, b) ∩B e nao enumeravel,em especial B e denso em R.

Com esse resultado garantimos que o complementar de um conjunto enumeravel e denso em R.

Demonstracao. Sabemos que (a, b) e nao enumeravel, escrevemos

(a, b) = [(a, b) ∩A] ∪ [(a, b) ∩ (R \A)] = [(a, b) ∩A] ∪ [(a, b) ∩B],

sabemos que (a, b)∩A e enumeravel se (a, b)∩B tambem o fosse, chegarıamos no absurdo de (a, b) ser enumeravel,por ser uniao finita de conjuntos enumeraveis , portanto (a, b) ∩B e nao enumeravel e B e denso em R.

Exemplo 13. Um conjunto pode nao ser enumeravel e tambem nao ser denso em R, como (a, b).

117

Exercıcio 3.46:

Corolario 8. O conjunto T dos numeros transcedentais e nao enumeravel e denso em R. Pois A o conjunto dosnumeros algebricos e enumeravel, T = R \ A, como complementar dos numeros algebricos T e nao enumeravel edenso em R.

118

Exercıcio 3.47:

Propriedade 50. Seja L|K uma extensao de corpo. Se α, β ∈ L sao algebricos sobre K, entao α ± β, α.β e αβ

com β 6= 0 sao algebricos sobre K, Desse modo

{α ∈ L|α e algebrico sobre K}

e um subcorpo de L que contem K.

Demonstracao. Seja δ ∈ {α ± β, α.β αβ β 6= 0} entao δ ∈ K(α, β) e K ⊂ K(δ) ⊂ K(α, β)). Vamos mostrar que

[K(α, β) : K] <∞.Sejam f , g ∈ K[x] os polinomios mınimos de α e β sobre K, com graus m e n respectivamente temos que

[K(α) : K] = m, [K(β) : K] = n.

f(x) ∈ k(x) ⊂ K(β)[x] e tal que f(α) = 0, logo α e algebrico sobre K(β), sendo P o polinomio mınimo de αsobre K(β) de grau s, ele divide f(x) em K(β)[x] logo s ≤ m, portanto [K(β)(α) : K(β)] = s ≤ m o grau e finitoe a extensao total [K(α, β) : K] = sn e finita por multiplicatividade dos graus. Como a extensao [K(α, β) : K] efinita ela e algebrica.

Definicao 8 (Fecho algebrico de Q). Consideremos a extensao de corpos C|Q. Chamamos de fecho algebrico deQ ao subcorpo Q de C definido por

Q = {α ∈ C,α e algebrico sobre Q}

Q e realmente corpo pela propriedade anterior. O conjunto dos numeros algebricos e um corpo.

119

Exercıcio 3.48:

Exemplo 14. Sendo Ak = [k,∞) temos uma sequencia de intervalos que sao conjuntos fechados porem a intersecao

∞⋂k=1

Ak = A

e vazia, pois suponha que exista t ∈ A, daı existe k > t e t /∈ [k,∞) = Ak logo nao pode pertencer a intersecao tetodos esses conjuntos.

Da mesma maneira existe uma sequencia decrescente de intervalos abertos limitados com intersecao vazia, sendoBk = (0, 1

k )∞⋂k=1

Bk = B

B e vazio, pois se houvesse um elemento nele x > 0, conseguimos k tal que 1k < x daı x nao pertence ao intervalo

(0, 1k ) = Bk portanto nao pode pertencer a intersecao.

120

Exercıcio 3.49:

Propriedade 51. Sejam B ⊂ A nao vazios, A limitado superiormente, se ∀x ∈ A existe y ∈ B tal que y ≥ xentao sup(B) = sup(A).

Demonstracao. B e limitado superiormente pois esta contido em um conjunto limitado e vale que sup(A) ≥ sup(B),pois B ⊂ A, suponha que fosse c = sup(A) > sup(B), entao tomando ε = sup(A) − sup(B) > 0, existe x ∈ A talque x > c − ε = sup(A) − sup(A) + sup(B) = sup(B), por hipotese existe y ≥ x > sup(B) com y ∈ B, o que eabsurdo, pois nao pode existir um elemento maior que o supremo.

Propriedade 52. Sejam B ⊂ A nao vazios, A limitado inferiormente, se ∀x ∈ A existe y ∈ B tal que y ≤ x entaoinf(B) = inf(A).

Demonstracao. B e limitado inferiormente pois esta contido em um conjunto limitado e vale que inf(A) ≤ inf(B),pois B ⊂ A, suponha que fosse c = inf(A) < inf(B), entao tomando ε = inf(B) − inf(A) > 0, existe x ∈ A talque x < c + ε = inf(A) − sup(A) + inf(B) = inf(B), por hipotese existe y ≤ x < inf(B) com y ∈ B, o que eabsurdo, pois nao pode existir um elemento menor que o ınfimo.

121

Exercıcio 3.50:

Definicao 9 (Corte de Dedekind). Um corte de Dedekind e um par ordenado (A,B) onde A,B ∈ Q nao vazios,tais que A nao possui maximo, A ∪B = Q e ∀ x ∈ A, y ∈ B vale x < y.

Seja C o conjunto dos cortes de Dedekind.

Propriedade 53. Em (A,B) vale sup(A) = inf(B).

Demonstracao. Ja sabemos que vale sup(A) ≤ inf(B), pois ∀ x ∈ A, y ∈ B vale x < y implica sup(A) < ye sup(A) ser cota inferior implica sup(A) ≤ inf(B), suponha por absurdo que fosse sup(A) < inf(B), entao ointervalo (sup(A), inf(B)) nao possui valores x ∈ A, pois se nao x > sup(A), nem y ∈ B pois daı y < inf(B), mascomo existem racionais em tal intervalo, pois Q e denso e A ∪B = Q, chegamos em um absurdo.

Propriedade 54. Existe bijecao entre R e C o conjunto dos cortes.

Demonstracao. Definimos f : C → R como f(A,B) = sup(A) = inf(B).

• f e injetora, suponha f(A,B) = f(A′, B′) entao sup(A) = inf(B) = sup(A′) = inf(B′).

Dado x ∈ A vamos mostrar que x ∈ A′.

x < sup(A′) = inf(B′) ≤ y′, ∀ y′ ∈ B′, daı x ∈ A′

a inclusao A′ ⊂ A e analoga. Entao vale A = A′.

• Dado y ∈ B, vamos mostrar que y ∈ B′.

x′ < sup(A) < inf(B′) ≤ y

com isso y ∈ B′. De maneira similar, B′ ⊂ B portanto B = B′. Como vale B = B′ e A = A′ entao a funcaoe injetiva.

• A funcao e sobrejetiva. Para qualquer y ∈ R, tomamos os conjuntos (−∞, y) ∩Q = A e B = [y,∞) ∩Q, Anao possui maximo, para todo x ∈ A e y ∈ B tem-se y > x e Q = [(−∞, y) ∩Q] ∪ [ [y,∞) ∩Q], alem dissovale sup(A) = y = inf(B), portanto f(A,B) = y e a funcao e sobrejetora, logo sendo tambem injetora f ebijecao.

122

Exercıcio 3.51:

Sejam X, Y conjuntos nao-vazios e f : X × Y → R uma funcao limitada. Para cada x0 ∈ X e cada y0 ∈ Y ,definimos s1(x0) = sup{f(x0, y); y ∈ Y } e s2(y0) = sup{f(x, y0);x ∈ X}. Isto define funcoes s1 : X → R es2 : Y → R. Prove que se tem supx∈Xs1(x) = supy∈Y s2(y). Em, outras palavras,

supx[supyf(x, y)] = supy[supxf(x, y)].

Primeiramente, verificaremos que supx∈Xs1(x) ∈ R. De fato, se {s1(x);x ∈ X} fosse ilimitado superiormente,deverımos ter que f e ilimitada superiormente. Pois, neste caso, dado A > 0 existiria x0 ∈ X tal que

A+ 1 < s1(x0),

tambem existiria, pois s1(x0) = sup{f(x0, y); y ∈ Y }, y0 ∈ Y tal que

s1(x0)− 1 < f(x0, y0)

e, consequentemente, terıamos queA < f(x0, y0).

Assim, concluirıamos que f e ilimitada superiormente. Um absurdo.De forma analoga, mostra-se que supy∈Y s2(y) ∈ R.Provaremos que

supx∈Xs1(x) = sup(x,y)∈X×Y f(x, y).

Seja ε > 0 arbitrario. Temos que existe x0 ∈ X tal que

supx∈Xs1(x)− ε

2< s1(x0).

Alem disso, como s1(x0) = sup{f(x0, y); y ∈ Y }, temos que existe y0 ∈ Y tal que

s1(x0)− ε

2< f(x0, y0).

Entao, segue das inequacoes acima, que

supx∈Xs1(x)− ε < f(x0, y0) 6 sup(x,y)∈X×Y f(x, y)

E, como ε > 0 e arbitrario, conclui-se que

supx∈Xs1(x) 6 sup(x,y)∈X×Y f(x, y).

Novamente, tomamos ε > 0 arbitrario. Existe (x0, y0) ∈ X × Y tal que

sup(x,y)∈X×Y f(x, y)− ε < f(x0, y0).

Como s1(x0) = sup{f(x0, y); y ∈ Y }, devemos ter que

sup(x,y)∈X×Y f(x, y)− ε < f(x0, y0) 6 s1(x0) 6 supx∈Xs1(x).

E, pela arbitrariedade de ε > 0, concluımos que

sup(x,y)∈X×Y f(x, y) 6 supx∈Xs1(x).

Portanto, temos quesupx∈Xs1(x) = sup(x,y)∈X×Y f(x, y).

De forma analoga, demonstra-se que

supy∈Y s2(y) = sup(x,y)∈X×Y f(x, y).

De onde concluımos quesupx∈Xs1(x) = supy∈Y s2(y).

123

Exercıcio 3.52:

Enuncie e demonstre um resultado analogo ao anterior com inf ao inves de sup. Considere, em seguida, o caso“misto” e prove que

supy[infxf(x, y)] = infx[supyf(x, y)].

Definimos funcoes i1 : X → R e i2 : Y → R por i1(x0) = inf{f(x0, y); y ∈ Y } e i2(y0) = inf{f(x, y0);x ∈ X}.Provaremos que

infx∈X i1(x) = infy∈Y i2(y),

ou, equivalentemente,infx∈X [infy∈Y f(x, y)] = infy∈Y [infx∈Xf(x, y)].

Primeiramente, verificaremos que infx∈X i1(x) ∈ R. De fato, se {i1(x);x ∈ X} fosse ilimitado inferiormente,deverımos ter que f e ilimitada inferiormente. Pois, neste caso, dado A > 0 existiria x0 ∈ X tal que

i1(x0) < −A− 1,

tambem existiria, pois i1(x0) = inf{f(x0, y); y ∈ Y }, y0 ∈ Y tal que

f(x0, y0) < i1(x0) + 1

e, consequentemente, terıamos quef(x0, y0) < −A.

Assim, concluirıamos que f e ilimitada inferiormente. Um absurdo.De forma analoga, mostra-se que infy∈Y i2(y) ∈ R.Provaremos que

infx∈X i1(x) = inf(x,y)∈X×Y f(x, y).

Seja ε > 0 arbitrario. Temos que existe x0 ∈ X tal que

i1(x0) < infx∈X i1(x) +ε

2.

Alem disso, como i1(x0) = inf{f(x0, y); y ∈ Y }, temos que existe y0 ∈ Y tal que

f(x0, y0) < i1(x0) +ε

2.

Entao, segue das inequacoes acima, que

inf(x,y)∈X×Y f(x, y) 6 f(x0, y0) < infx∈X i1(x) + ε

E, como ε > 0 e arbitrario, conclui-se que

inf(x,y)∈X×Y f(x, y) 6 infx∈X i1(x).

Novamente, tomamos ε > 0 arbitrario. Existe (x0, y0) ∈ X × Y tal que

f(x0, y0) < inf(x,y)∈X×Y f(x, y) + ε.

Como i1(x0) = inf{f(x0, y); y ∈ Y }, devemos ter que

infx∈X i1(x) 6 i1(x0) 6 f(x0, y0) < inf(x,y)∈X×Y f(x, y) + ε.

E, pela arbitrariedade de ε > 0, concluımos que

infx∈X i1(x) 6 inf(x,y)∈X×Y f(x, y).

Portanto, temos queinfx∈Xs1(x) = inf(x,y)∈X×Y f(x, y).

124

De forma analoga, demonstra-se que

infy∈Y i2(y) = inf(x,y)∈X×Y f(x, y).

De onde concluımos queinfx∈X i1(x) = supy∈Y i2(y).

Agora, provaremos a desigualdade

supy[infxf(x, y)] 6 infx[supyf(x, y)].

SejamI := {infx∈Xf(x, y) | y ∈ Y }

eS := {supy∈Y f(x, y) | x ∈ X}.

Dados elementos arbitrarios infx∈Xf(x, y0) em I e supy∈Y f(x0, y) de S, temos que

infx∈Xf(x, y0) 6 f(x0, y0) 6 supy∈Y f(x0, y).

Assim, pelo resultado do exercıcio 3.34, temos que

supy[infxf(x, y)] = sup(I) 6 inf(S) = infx[supyf(x, y)].

Por fim, um exemplo onde ocorre a desigualdade estrita. Sejam X = {x1, x2}, Y = {y1, y2} e f : X × Y → Rdefinido por f(x1, y1) = f(x2, y2) = 2 e f(x1, y2) = f(x2, y1) = 1. Assim, temos que

supy∈Y f(x1, y) = supy∈Y f(x2, y) = 2

einfx∈Xf(x, y1) = infx∈Xf(x, y2) = 1.

Portanto,supy[infxf(x, y)] = 1 < 2 = infx[infyf(x, y)].

125

Exercıcio 3.53:

Sejam x e y numeros reais positivos. Prove que se tem

√xy 6

x+ y

2.

Temos que(√x−√y)2 > 0.

Logo,x− 2

√xy + y > 0

e, portanto,x+ y

2>√xy.

126

Exercıcio 3.54:

A desigualdade entre a media aritmetica e a media geometrica, vista no exercıcio anterior, vale para n numerosreais positivos x1, x2, ..., xn. Sejam G = n

√x1x2 . . . xn e A = x1+x2+···+xn

n . Para provar a desigualdade no casogeral, considere a operacao que consiste em substituir o menor dos numeros dados, digamos xi e o maior deles,digamos xj , respectivamente por x′i =

xixj

G e x′j = G. Isto nao altera a media geometrica e, quanto a aritmetica,ela nao aumenta, pois, como e facil ver, x′i + x′j 6 xi + xj . Prove que, repetida a operacao no maximo n vezes,obtemos n numeros todos iguais a G e, portanto, sua media aritmetica e G. Como em cada operacao a mediaaritmetica nao aumentou, conclua que G 6 A, ou seja, n

√x1x2 . . . xn 6 x1+x2+···+xn

n .

Provaremos, por inducao em k = 0, 1, . . . , n, que se exatamente n − k termos da sequencia x1,...,xn sao iguaisa G, entao G 6 A.

Para k = 0, temos que

A =x1 + x2 + · · ·+ xn

n=nG

n= G,

como querıamos.Suponhamos que exatamente n − k termos da sequencia x1,...,xn sao iguais a G, para k > 0. Sejam xi e xj ,

respectivamente, elementos de maior e menor valor da sequencia em questao. Desta forma temos que

xi < G < xj .

De fato, temos quexni 6 x1x2 . . . xn 6 xnj ,

e, consequentemente,xi 6 G 6 xj .

Assim, como k > 0, devemos ter que xi < G 6 xj ou xi 6 G < xj . Se tivessemos xi < G = xj , entao

Gn = x1x2 . . . xn 6 (xj)n−1xi < Gn.

Absurdo. Da mesma forma, nao podemos ter xi = G < xj . Portanto, devemos ter, de fato xi < G < xj . Adiante,consideremos a sequencia dada por

x′p =

xp , p 6= i, jxixj

G , p = iG , p = j.

Temos que

x′i + x′j =xixjG

+G <xiG

G+ xj = xi + xj .

Assim,x′1 + x′2 + · · ·+ x′n

n<x1 + x2 + · · ·+ xn

n.

Tambem temos queG′ := n

√x′1x′2 . . . x

′n = n

√x1x2 . . . xn = G.

Concluımos daı que a sequencia x′1, x′2, ... ,x′n possui no maximo k − 1 elementos diferentes de G′ = G. Pelahipotese de inducao, temos que

G = G′ = n√x′1x′2 . . . x

′n 6

x′1 + x′2 + · · ·+ x′nn

<x1 + x2 + · · ·+ xn

n= A.

E o resultado segue por inducao.

127

Exercıcio 3.55:

Seja K um corpo ordenado e completo. Indique com 0′ e 1′ o zero e a unidade de K. Para cada n ∈ N, sejam

n′ = 1′ + ...+ 1′ (n vezes) e (−n)′ = −n′. Definamos uma funcao f : R→ K pondo f(pq ) = p′

q′ para todo pq ∈ Q e

para x irracional, seja f(x) = sup{p′

q′ ∈ K; pq < x}. Prove que f e um homomorfismo sobrejetivo e conclua que f euma bijecao, ou seja, um isomorfismo de R sobre K.

A solucao sera dada em 11 passos:

(I) f(m+ n) = f(m) + f(n) para todos n,m ∈ Z:

Provaremos a igualdade por inducao em n ∈ N que f(m+ n) = f(m) + f(n) e f(m− n) = f(m) + f(−n), paratodo m ∈ Z.

Temos que f e definido em Z indutivamente por f(m+ 1) = f(m) + f(1) e f(−m− 1) = −f(m)− f(1), param ∈ N. Daı segue o resultado para n = 1.

Adiante, temos que

f(m+ (n+ 1)) = f((m+ 1) + n) = f(m+ 1) + f(n) = f(m) + f(n) + f(1) = f(m) + f(n+ 1).

E de forma analoga mostra-se que f(m− (n+ 1)) = f(m) + f(−(n+ 1)). E o resultado segue pelo PIF.

(II) f(mn) = f(m)f(n) para todos n,m ∈ Z:

Demonstracao analoga a anterior.

(III) f(m) < f(n) ⇐⇒ m < n, para todos n,m ∈ Z:

Observemos que pela definicao indutiva de f em N (i.e. f(n + 1) := f(n) + 1) e pelo fato de f(1) := 1′ > 0′,temos que n ∈ Z com n > 0 implica f(n) > 0′. E por f(−n) = −f(n) temos que se n ∈ Z com n < 0 entaof(n) < 0′. Ou seja, n > 0 em Z se e somente se f(n) > 0′.

Assim, temos quef(m) < f(n) ⇐⇒ f(m)− f(n) < 0′

⇐⇒ f(m− n) < 0′

⇐⇒ m− n < 0′

⇐⇒ m < n.

(IV) f(s+ t) = f(s) + f(t) para todos s, t ∈ Q:

Sejam s = pq e t = m

n com p,m ∈ Z e q, n ∈ N . Entao, temos que

f(s+ t) = f(np+qmqn )

= f(np+qm)f(qn)

= f(n)f(p)+f(q)f(m)f(q)f(n)

= f(p)f(q) + f(m)

f(n)

= f(s) + f(t).

(V) f(st) = f(s)f(t) para todos s, t ∈ Q:

Analoga a anterior.

(VI) f(s) < f(t) ⇐⇒ s < t, para todos s, t ∈ Q:

Sejam s = pq e t = m

n com p,m ∈ Z e q, n ∈ N . Entao, temos que

f(s) < f(t) ⇐⇒ f(n)f(p) < f(q)f(m)⇐⇒ f(np) < f(qm)⇐⇒ np < qm⇐⇒ s < t.

128

(VII) f(x) = sup{f(r); r 6 x, r ∈ Q} para todo x ∈ R:

Pela definicao de f nos irracionais, basta provar a igualdade para x ∈ Q. De fato, por (IV) temos quef(x) > sup{f(r); r 6 x, r ∈ Q}. E por outro lado, f(x) ∈ sup{f(r); r 6 x, r ∈ Q}. E saı segue o resultado.

(VIII) f e um homomorfismo:

De fato, temos, para quaisquer x, y ∈ R, que

f(x) + f(y) = sup{f(r); r 6 x, r ∈ Q}+ sup{f(r); r 6 y, r ∈ Q}= sup{f(r1) + f(r2); r1 6 x, r2 6 y e r1, r2 ∈ Q} (pelo exercıcio 3.37)= sup{f(r); r 6 x+ y, r ∈ Q} (por (IV))= f(x+ y).

Analogamente, prova-se que f(xy) = f(x)f(y) utilizando-se do exercıcio 3.39 e do item (V).

(IX) f(x) < f(y) ⇐⇒ x < y, para todos x, y ∈ R:

Temos, por (VII), quef(x) < f(y) ⇐⇒ ∃r ∈ Q, f(x) < f(r) 6 f(y)

⇐⇒ ∃r ∈ Q, x < r 6 y⇐⇒ x < y.

(X) y = f(sup{r ∈ Q; f(r) 6 y}):

Seja x = sup{r ∈ Q; f(r) 6 y}. Observemos que x ∈ R pelo fato de K ser completo e ordenado (logo,arquimediano). Entao, pela escolha de x,

f(x) = sup{f(r); r 6 x, r ∈ Q} 6 y.

Por outro lado, se f(x) < y, haveria, novamente pela arquimedianeidade de K, r ∈ Q, tal que f(x) < f(r) < f(y).Contradizendo a escolha de x.

(XI) f e bijetiva:

O item (IX) implica que f e injetiva e o item (X) implica que f e sobrejetiva.

129

Exercıcio 3.56:

Seja f : R → R um isomorfismo de R em si mesmo. Prove que f = identidade. Conclua que se K e L sao corposordenados completos, existe um unico isomorfismo de K sobre L.

Seja f : R→ R um isomorfismo de corpos. Provaremos que f e igual a identidade IR : R→ R.Pelo exercıcio 3.4, temos que f(0) = 0 e f(1) = 1.Por inducao em n ∈ N, segue que

f(n) = n = −f(−n).

E, assim, para q ∈ Z+ temos que

1 = f

(q

q

)= f(q)f

(1

q

)= qf

(1

q

).

De onde concluımos que f(

1q

)= 1

q . Portanto, para p ∈ Z, temos que

f

(p

q

)= f(p)f

(1

q

)=p

q.

Ou seja, f(r) = r para todo r ∈ Q.Agora, observemos que se x > 0, entao f(x) > 0. De fato, temos que

f(x) = f(√x√x) = f(

√x)2 > 0.

Seja x ∈ R. Provaremos que f(x) = x mostrando que para quaisquer r, s ∈ Q tais que

s < x < r

tem-se ques < f(x) < r.

De fato, temos que0 < x− s

e0 < r − x

implicam que0 < f(x− s) = f(x)− f(s) = f(x)− s

e0 < f(r − x) = f(r)− f(x) = r − f(x)

pela observacao do paragrafo anterior. Portanto, temos que

s < f(x) < r

e concluımos quef(x) = x

para qualquer x ∈ R.Provaremos agora que se K e L sao corpos ordenados completos, existe um unico isomorfismo entre K e L.Pelo exercıcio 3.55, temos que existem isomorfismos fK : R → K e fL : R → L. Desta forma, existe um

isomorfismo ϕ = fL ◦ f−1K : K → L.

Suponhamos que ψ : K → L seja um isomorfismo de corpos. Provaremos que ψ = ϕ. De fato, como

f−1L ◦ ψ−1 ◦ ϕ ◦ fK : R→ R

e um isomorfismo, pelo que foi provado acima, temos que

f−1K ◦ ψ−1 ◦ ϕ ◦ fK = IR.

Consequentemente,ψ−1 ◦ ϕ = IK

eψ = ϕ.

130

Exercıcio 3.57:

Verifique que f : R→ (−1, 1), definida por f(x) = x√1+x2

, e uma bijecao de R no intervalo (−1, 1).

Seja g : (−1, 1)→ R definida por

g(x) =x√

1− x2.

Provaremos que g = f−1.Seja x ∈ (−1, 1). Entao,

f ◦ g(x) = f(

x√1−x2

)=

(x√

1−x2

)(

1+

(x√

1−x2

)2) 12

=

(x√

1−x2

)(

1+ x2

1−x2

) 12

=

(x√

1−x2

)(√

1−x2+x2√1−x2

)

=

(x√

1−x2

)(

1√1−x2

)= x.

Seja x ∈ R. Entao,

g ◦ f(x) = g(

x√1+x2

)=

(x√

1+x2

)(

1−(

x√1+x2

)2) 12

=

(x√

1+x2

)(

1+x2−x2

1+x2

) 12

=

(x√

1+x2

)(

1√1+x2

)= x.

Portanto, g = f−1.

131

Exercıcio 3.58:

Um conjunto G de numeros reais chama-se grupo aditivo quando 0 ∈ G e x, y ∈ G ⇒ x − y ∈ G. Entao,x ∈ G ⇒ −x ∈ G e x, y ∈ G ⇒ x + y ∈ G. Seja entao G ⊂ R um grupo aditivo de numeros reais. Indiquemoscom G+ o conjunto dos numeros reais pertencentes a G. Excetuando o caso trivial G = {0}, G+ e nao-vazio.Suponhamos pois G 6= {0}. Prove que:

(i) Se infG+ = 0, entao G e denso em R;

(ii) Se infG+ = a > 0, entao a ∈ G+ e G = {0,±a,±2a, ...};

(iii) Conclua que, se α ∈ R e irracional, os numeros reais da forma m + nα com m, n ∈ Z constituem umsubconjunto denso em R.

(i)Provaremos que dado um intervalo arbitrario (a, b) em R, existe g ∈ G e m ∈ Z tais que mg ∈ (a.b). Como

mg ∈ G (prova-se por inducao em m), concluımos daı que G intercepta todo intervalo aberto em R. Logo, G edenso em R.

Como infG+ = 0, temos que existe g ∈ G+ tal que

0 < g < b− a.

Assim, tomando-se m = min{n ∈ Z; a < ng}, teremos que

a < mg = (m− 1)g + g 6 a+ g < a+ (b− a) = b.

Logo, mg ∈ (a, b).(ii)Primeiramente, provaremos que a ∈ G. De fato, se a = infG+ /∈ G terıamos que existiriam h e g ∈ G+ tais que

a < h < g < 2a.

Daı0 < g − h < a

e, como g − h ∈ G, temos quea = infG+ 6 g − h < a.

Uma contradicao. Logo, a ∈ G.Agora, provaremos que todo g ∈ G e da forma na para algum n ∈ Z. Seja n = max{n ∈ Z;na 6 g} e r = g−na.

Pela escolha de n, temos que0 6 r < a.

Assim, como r = g − na ∈ G devemos ter que r = 0 pois, caso contrario, terıamos que

a = infG+ 6 r < a.

Portanto, g = na.(iii)Seja

G := {m+ nα;m,n ∈ Z}.Temos que G e um grupo aditivo. Como G 6= {0}, nos basta provar que a = infG+ = 0.

Suponhamos o contrario. Entao a > 0 e, pelo item (ii), temos que

G = {0,±a,±2a, ...}.

Assim, como α ∈ G, temos quea = kα ∈ R\Q.

Daı, temos queG = {0,±a,±2a, ...} ⊂ R\Q.

Mas, por outro lado,G := {m+ nα;m,n ∈ Z} ⊃ Z.

Uma contradicao.

132

Exercıcio 3.59:

Sejam f, g : R2 → R e φ, ψ : R3 → R as funcoes definidas por f(x, y) = 3x − y, g(x, y) = (x − 1)2 + (y + 1)2 − 9,ϕ(x, y, z) = 3z e ψ(x, y, z) = x2 + y2 − z. Interpretando (x, y) como as coordenadas cartesianas de um pontono plano R2 e (x, y, z) como as coordenadas de um ponto no espaco R3, descreva geometricamente os conjuntosf−1(0), g−1(0), ϕ−1(0) e ψ−1(0).

• f−1(0) = {(x, y) ∈ R2; y = 3x}: uma reta;

• g−1(0) = {(x, y) ∈ R2; (x− 1)2 + (y + 1)2 = 32}: uma circunferencia de raio 3 e centrada no ponto (1,−1);

• ϕ−1(0) = {(x, y, z) ∈ R3; z = 0}: um plano;

• ψ−1(0) = {(x, y, z) ∈ R3; z = x2 + y2}: dois cones.

133

Exercıcio 3.60:

Seja a um numero real positivo. Dado um numero racional p/q (onde p ∈ Z e q ∈ N), defina a potencia de base ae expoente racional p/q como ap/q = q

√ap. Prove:

(1) Para quaisquer r, s ∈ Q tem-se ar.as = ar+s e (ar)s = ars;

(2) Para todo r ∈ Q+, a funcao f : (0,+∞)→ (0,+∞), dada por f(x) = xr, e uma bijecao crescente;

(3) A funcao g : Q→ R definida por g(r) = ar (onde a e um numero real positivo fixado) e crescente se a > 1, edecrescente se 0 < a < 1.

(1)Sejam r = p/q e s = p′/q′, onde p e p′ ∈ Z e q e q′ ∈ N. Temos que

aras = apq a

p′q′

= apq′qq′ a

p′qq′q

=qq′√apq′

qq′√ap′q

=qq′√apq′ap′q

=qq′√apq′+p′q

= apq′+p′q

qq′

= apq + p′

q′

= ar+s

e

(ar)s = (apq )

p′q′

= q′√

( q√ap)p′

=qq′√app′

= app′qq′

= ars.

(2)

A funcao f tem inversa g : (0,+∞)→ (0,+∞) dada por g(x) = x1r . De fato, dado x ∈ (0,+∞), temos que

f ◦ g(x) = f(x1r )

= (x1r )r

= x1

= x.

eg ◦ f(x) = g(xr)

= (xr)1r

= x1

= x.

Portanto, f e uma bijecao.Dados x < y em (0,+∞), temos que

xp < yp

e, consequentemente,f(x) = xr

= q√xp

< q√yp

= yr

= f(y).

Portanto, f e crescente.

134

(3)

Dados r = p/q e s = p′/q′, onde p e p′ ∈ Z e q e q′ ∈ N, com r < s. Entao, r = pq′

qq′ <p′qqq′ = s e,

consequentemente,pq′ < p′q.

Caso a > 1, temos quepq′ < p′q ⇒ apq

′< ap

′q

⇒ qq′√apq′ <

qq′√ap′q

⇒ g(pq′

qq′ ) < g(p′qqq′ )

⇒ g(r) < g(s).

Concluımos daı que, g e crescente caso a > 1.Caso 0 < a < 1, temos que

pq′ < p′q ⇒ apq′> ap

′q

⇒ qq′√apq′ >

qq′√ap′q

⇒ g(pq′

qq′ ) > g(p′qqq′ )

⇒ g(r) > g(s).

Concluımos daı que, g e decrescente caso 0 < a < 1.

135

Capıtulo 4

Sequencias e Series de Numeros Reais

136

Exercıcio 4.1:

Se limxn = a, entao lim |xn| = |a|. De um contra-exemplo mostrando que a recıproca e falsa, salvo quando a = 0.

Como limxn = a, dado ε > 0 existe n0 ∈ N tal que

||xn| − |a|| ≤ |xn − a| < ε,

para todo n ≥ n0. Logo, temos quelim |xn| = |a|.

Temos que lim |xn| = |a| nao implica que limxn = a. Basta tomar xn = 1, para todo n ∈ N e a = −1.

137

Exercıcio 4.2:

Seja limxn = 0. Para cada n, ponha yn = min{|x1|, |x2|, ..., |xn|}. Prove que yn −→ 0.

Dado ε > 0, existe n0 ∈ N tal que se n ≥ n0, entao |xn| < ε. Como |yn| = yn = min{|x1|, ..., |xn|} ≤ |xn|, temosque para n ≥ n0, |yn| ≤ |xn| < ε. Logo,

lim yn = 0.

138

Exercıcio 4.3:

Se limx2n = a e limx2n−1 = a, prove que limxn = a.

Dado ε > 0, existem np, ni ∈ N tais que

n ≥ np ⇒ |x2n − a| < ε

en ≥ ni ⇒ |x2n−1 − a| < ε.

Tomemos n0 = max{2np, 2ni − 1}. Assim, para n ≥ n0, temos

• Se n = 2k, entao2k = n ≥ n0 ≥ 2np.

Logo, k ≥ np e consequentemente|xn − a| = |x2k − a| < ε.

• Se n = 2k − 1, entao2k − 1 = n ≥ n0 ≥ 2ni − 1.

Logo, k ≥ ni e consequentemente|xn − a| = |x2k−1 − a| < ε.

Em ambos os casos, se n ≥ n0, temos |xn − a| < ε. Logo, limxn = a.

139

Exercıcio 4.4:

Se N = N1 ∪ N2 ∪ ... ∪ Nk e limn∈N1

xn = limn∈N2

xn = ... = limn∈Nk

xn = a, entao limn∈N

xn = a.

Dado ε > 0, existem n1, n2, ..., nk ∈ N1,N2, ...,Nk tais que se n ∈ Ni com n ≥ ni, entao |xn − a| < ε. Tomandon0 = max{n1, n2, ..., nk}, temos que se n ≥ n0, entao n ≥ ni, para todo i = 1, .., k. Desta forma, |xn − a| < ε.Portanto,

limxn = a.

140

Exercıcio 4.5:

De exemplo de uma sequencia (xn) e uma decomposicao N = N1 ∪ N2 ∪ ... ∪ Nk ∪ ... de N como reuniao deuma infinidade de conjuntos infinitos tais que, para todo k, a sequencia (xn)n∈Nk

tenha limite a, mas nao se temlimxn = a.

Seja k ∈ N. DefinimosNk = {2k−1.n ∈ N;n ∈ N, n e ımpar}.

Temos dessa definicao que N =∞⋃k=1

Nk. Assim, podemos definir

x : N →

n 7→ x(n) =

{1, se n = 2k−1 para algum k

1/n, se n = 2k−1m para algum m > 1..

Provemos que limxn = 0. Dado ε > 0, existe p0 ∈ N tal que

1

3p0< ε2k−1.

Assim, se n = 2k−1m ∈ Nk, com n ≥ n0 = 2k−13p0 , entao

xn =1

2k−1m≤ 1

2k−13p0< ε.

Logo, limn∈Nk

= 0. Por outro lado, limx2k−1 = 1. Portanto, limn∈N

xn nao existe.

141

Exercıcio 4.6:

Se limxn = a e lim(xn − yn) = 0, entao lim yn e igual a a.

Dado ε > 0, existem n1, n2 ∈ N tais que

n ≥ n1 ⇒ |xn − a| < ε/2

en ≥ n2 ⇒ |xn − yn| < ε/2.

Tomando n0 = max{n1, n2}, teremos que se n ≥ n0, entao

|yn − a| ≤ |xn − yn|+ |xn − a| < ε.

Logo, lim yn = a.

142

Exercıcio 4.7:

Seja a 6= 0. Se limyna

= 1, entao lim yn e igual a a.

Sejam ε > 0 e a 6= 0. Neste caso ε/|a| > 0. Assim, existe n0 ∈ N tal que∣∣∣yna− 1∣∣∣ < ε

|a|,

ou seja,|yn − a| < ε.

Logo, lim yn = a.

143

Exercıcio 4.8:

Seja b 6= 0. Se limxn = a e limxnyn

= b, entao lim yn =a

b.

Pelo ıtem 3 do Teorema 6, temos que

limynxn

=1

b.

Pelo ıtem 2 do mesmo teorema,

lim yn = limxn.ynxn

= limxn. limynxn

= a.1

b=a

b.

144

Exercıcio 4.9:

Seja b 6= 0. Se limxn = a e limxnyn

= b, entao lim yn =a

b.

Pelo ıtem 3 do Teorema 6, temos que

limynxn

=1

b.

Pelo ıtem 2 do mesmo teorema,

lim yn = limxn.ynxn

= limxn. limynxn

= a.1

b=a

b.

145

Exercıcio 4.10:

Sejam k ∈ N e a > 0. Se a ≤ xn ≤ nk para todo n, entao lim n√xn = 1.

Temos quelim n√a = lim n

√n = 1.

Assim,

limn√nk = lim

(n√n)k

=(lim n√n)k

= 1k = 1.

Logo, limxn = 1.

146

Exercıcio 4.10:

Sejam k ∈ N e a > 0. Se a ≤ xn ≤ nk para todo n, entao lim n√xn = 1.

Temos quelim n√a = lim n

√n = 1.

Assim,

limn√nk = lim

(n√n)k

=(lim n√n)k

= 1k = 1.

Logo, limxn = 1.

147

Exercıcio 4.11:

Use a desigualdade entre as media aritmetica e geometrica dos n+ 1 numeros 1−1/n, 1−1/n, ..., 1−1/n, 1 e proveque a sequencia (1− 1/n)n e crescente. Conclua que (1− 1/n)n ≥ 1/4 para todo n > 1.

Pela desigualdade envolvendo a media aritmetica e a media geometrica, temos que

n+1

√(1− 1

n

)n.1 ≤

n

(1

1

n

)+ 1

n+ 1

n+1

√(1− 1

n

)n≤ n

n+ 1= 1− 1

n+ 1(1− 1

n

)n≤(

1− 1

n+ 1

)n+1

.

Logo, a sequencia (xn)n∈N dada por

xn =

(1− 1

n

)ne crescente. Daı, para n ≥ 2, temos

1

4=

(1− 1

2

)2

≤(

1− 1

n

)n.

148

Exercıcio 4.11a:

Sejam xn = (1 + 1/n)n e yn = (1− 1n+1 )n+1. Mostre que limxnyn = 1 e deduza daı que lim(1− 1/n)n = e−1.

Segue, pelas definicoes de xn e yn, que

xnyn =

(1 +

1

n

)n(1− 1

n+ 1

)n+1

=

(n

n+ 1

)n(n

n+ 1

)n(1− 1

n+ 1

)= 1− 1

n+ 1.

Logo,

limxnyn = lim 1− 1

n+ 1= 1.

Como limxn = e, temos que

lim

(1− 1

n

)n= lim yn1

= lim yn = e−1.

149

Exercıcio 4.12:

Fazendo yn = x1/k e b = a1/k na identidade yk−bk = (y−b)k−1∑i=0

yibk−i−1, obtenha x−a = (x1/k−a1/k)k−1∑i=0

xi/ka1−(i+1)/k

e use isto para provar que se limxn = a > 0, entao lim n√xn = n

√a. Conclua daı, que lim(xn)r = ar para todo

racional r.

Fazendo y = x1/k e b = a1/k na identidade

yk − bk = (y − b)k−1∑i=0

yibk−i−1,

obtemos

x− a = (x1/k − a1/k)

k−1∑i=0

xi/ka1−i+ 1

k .

como limxn = a > 0, existe c > 0 em e um nc ∈ N tal que para todo n ≥ nc, teremos

0 < c < xn.

Logo, para n ≥ nc temos que

xn − a = (x1/kn − a1/k)

k−1∑i=0

xi/kn a1−i+ 1

k

> (x1/kn − a1/k).S,

onde

S =

k−1∑i=0

ci/ka1−i+ 1

k > 0.

dado ε > 0, existe n1 ∈ N tal que para todo n ≥ n1, temos

εS > |xn − a|.

Daı, fazendo n0 = max{n1, n2}, temos que para todo n ≥ n0

εS > |xn − a| > |x1/kn − a1/k|.S.

Logo,ε > |x1/k

n − a1/k|

para n ≥ n0. Concluimos daı quelim n√xn = n

√a.

Sendo r =p

q, temos que

limxp/qn = lim( q√p)

= (lim q√xn)p

= ( q√a)p = ap/q = ar.

150

Exercıcio 4.14:

Propriedade 55. Seja a, b ≥ 0 e entao lim n√an + bn = max{a, b}.

Demonstracao. Seja c = max{a, b} entao vale Vale an ≤ cn, bn ≤ cn e daı an + bn ≤ 2cn da mesma maneiracn ≤ an + bn, pois c e a ou b, logo

cn ≤ an + bn ≤ 2cn

c ≤ n√an + bn ≤ n

√2 c

tomando limites, temos pelo teorema do sanduıche

limn√an + bn = c.

Propriedade 56. Sejam (ak ≥ 0)m1 e c = max{ak, k ∈ Im} entao

limn→∞

n

√√√√ m∑k=1

ank = c.

Demonstracao. Vale ank ≤ cn, tomando a soma, tem-se∑mk=1 a

nk ≤ m.cn, tem-se tambem cn ≤

∑mk=1 a

nk entao vale

cn ≤m∑k=1

ank ≤ m.cn

tomando a raiz

c ≤ n

√√√√ m∑k=1

ank ≤n√m.c

e novamente por teorema do sanduıche tem-se

lim n

√√√√ m∑k=1

ank = c.

151

Exercıcio 4.15:

Definicao 10 (Termo destacado). Dizemos que xn e um termo destacado quando xn ≥ xp para todo p > n. Isto equando xn e maior ou igual a todos seus sucessores.

Propriedade 57. Toda sequencia possui subsequencia monotona .

Demonstracao. Seja A ⊂ N o conjunto dos ındices s da sequencia (xn), tais que xs e destacado, existem dois casosa serem analisados

• Se A e infinito, entao podemos tomar uma subsequencia (xn1, xn2

, · · · ) de termos destacados formada peloselementos com ındices em A que e nao-crescente com n1 < n2 < n3 < · · · e com xn1

≥ xn2≥ · · · .

• Se A e finito, tomamos um n1 maior que todos elementos de A daı xn1 nao e destacado, existindo xn2 ≥ xn1

com n2 > n1, por sua vez xn2 nao e destacado logo existe n3 > n2 tal que xn3 ≥ xn2 , assim construımos umasubsequencia nao-decrescente .

152

Exercıcio 4.18:

Generalizamos o exercıcio em dois resultados.

Propriedade 58. Sejam (an) e (bn) sequencias limitada tais que an + bn = 1 ∀n ∈ N , (zn) e (tn) com o mesmolimite a, entao lim an.zn + bn.tn = a.

Demonstracao. Escrevemos

an.zn + bn.tn = an.zn − a.an + a. an︸︷︷︸=1−bn

+bn.tn = an(zn − a) + a(1− bn) + bn.tn =

= an(zn − a) + a− a.bn + bn.tn = an(zn − a) + a+ bn(tn − a)

daılim an(zn − a) + a+ bn(tn − a) = a = lim an.zn + bn.tn

pois an e bn sao limitadas e zn − a, tn − a tendem a zero.

Propriedade 59. Se limn→∞ zk(n) = a ∀ k e cada (xk(n)) e limitada com∑pk=1 xk(n) = vn → b entao

limn→∞∑pk=1 xk(n)zk(n) = a.b.

Demonstracao. Vale x1(n) = vn −∑pk=2 xk(n).

p∑k=1

xk(n)zk(n) = x1(n)z1(n) +

p∑k=2

xk(n)zk(n) =

= z1(n)vn −p∑k=2

xk(n)z1(n) +

p∑k=2

xk(n)zk(n) =

= z1(n)vn︸ ︷︷ ︸→a.b

+

p∑k=2

xk(n) (zk(n)− z1(n))︸ ︷︷ ︸→0

→ a.b.

153

Exercıcio 4.19:

Definicao 11 (Sequencia de variacao limitada). Uma sequencia (xn) tem variacao limitada quando a sequencia(vn) com

vn =

n∑k=1

|∆xk| e limitada.

Propriedade 60. Se (xn) tem variacao limitada entao (vn) converge.

Demonstracao. (vn) e limitada e nao-decrescente, pois ∆vn = |∆xn+1| ≥ 0, logo e convergente.

Propriedade 61. Se (xn) tem variacao limitada entao existe limxn.

Demonstracao. A serie∑∞k=1 |∆xk| converge portanto

∑∞k=1 ∆xk converge absolutamente e vale

xn − x1 =

n−1∑k=1

∆xk ⇒ xn =

n−1∑k=1

∆xk + x1

logo xn e convergente.

Exemplo 15. Se |∆xn+1| ≤ c|∆xn| ∀ n ∈ N com 0 ≤ c < 1 entao (xn) possui variacao limitada. Definimos

g(k) = |∆xk| logo a desigualdade pode ser escrita como g(k + 1) ≤ cg(k), Qg(k) ≤ c aplicamos∏n−1k=1 de ambos

lados, daı

g(n) = |∆xn| ≤ cn−1g(1)

somando em ambos lados temosn∑k=1

|∆xk| ≤n∑k=1

ck−1g(1)

como o segundo termo converge por ser serie geometrica segue que (xn) e de variacao limitada, logo converge.

Propriedade 62. (xn) tem variacao limitada ⇔ xn = yn − zn onde (yn) e (zn) sao sequencias nao-decrescenteslimitadas.

Demonstracao. ⇐).Seja xn = yn − zn onde (yn) e (zn) sao sequencias nao-decrescentes limitadas, entao xn tem variacao limitada.

vn =

n∑k=1

|∆xk| =n∑k=1

|∆yk −∆zk| ≤n∑k=1

|∆yk|+n∑k=1

|∆zk| ≤ |n∑k=1

∆yk|+ |n∑k=1

∆zk|

= |yn+1 − y1|+ |zn+1 − z1| < M

pois (yn) e (zn) sao limitadas, logo (vn) e limitada, isto e, (xn) tem variacao limitada.⇒). Dada (xn) com variacao limitada. (xn) tem variacao limitada ⇔ (xn + c) tem variacao limitada, pois ∆

aplicado as duas sequencias tem o mesmo valor. Escrevemos

xn − x1 =

n−1∑k=1

∆xk

Para cada n definimos Pn o conjunto dos k da soma∑n−1k=1 ∆xk tais que ∆xk ≥ 0 e Nn o conjunto dos k da mesma

soma tais que ∆xk < 0, com isso temos uma particao do conjunto dos ındices e vale

xn − x1 =

n−1∑k=1

∆xk =∑k∈Pn

∆xk︸ ︷︷ ︸yn

−∑k∈Nn

(−∆xk)︸ ︷︷ ︸zn

154

(yn) e nao decrescente, pois yn+1 = yn caso nao seja adicionado ındice a Pn+1 em relacao a Pn e yn+1 ≥ yn casoseja adicionado um ındice a Pn+1, pois adicionamos um termo da forma ∆xk ≥ 0 o mesmo para (zn).

(yn) e limitada pois

∑k∈Pn

∆xk ≤n−1∑k=1

|∆xk| =∑k∈Pn

|∆xk|+∑k∈Nn

|∆xk| =∑k∈Pn

∆xk +∑k∈Nn

(−∆xk) < M

da mesma maneira (zn) e limitada.

Exemplo 16. Existem sequencias convergentes que nao possuem variacao limitada, como por exemplo xn =∑n−1k=1

(−1)k

k , que e convergente porem ∆xn = (−1)n

n ⇒ |∆xn| = 1n e

∑n−1k=1

1k nao e limitada.

155

Exercıcio 4.20:

Exemplo 17. Seja (xn) definida como x1 = 1, xn+1 = 1 + 1xn

, entao vale que

|∆xn+1| ≤1

2|∆xn|.

• Primeiro vale que xn ≥ 1 para todo n pois vale para n = 1, supondo validade para n, entao vale para n+ 1,pois xn+1 = 1 + 1

xn.

• Vale que |xn+1xn| ≥ 2 para todo n, pois, substituindo xn+1 = 1 + 1xn

isso implica que xn+1xn ≥ xn + 1 ≥ 2.

• De |xn+1xn| ≥ 2 segue que | 1xn+1xn

| ≤ 12 , multiplicando por |xn+1 − xn| em ambos lados segue que

|xn − xn+1

xn+1xn| ≤ |xn+1 − xn|

2

| 1

xn+1− 1

xn| = | (1 +

1

xn+1)︸ ︷︷ ︸

xn+2

− (1 +1

xn)︸ ︷︷ ︸

xn+1

| ≤ |xn+1 − xn|2

portanto |∆xn+1| ≤ 12 |∆xn| portanto a sequencia e convergente. Calculamos seu limite limxn = a

a = 1 +1

a⇔ a2 − a− 1 = 0

cujas raızes sao 1±√

52 , ficamos com a raiz positiva pois a sequencia e de termos positivos, logo

limxn =1 +√

5

2.

156

Exercıcio 4.21:

Exemplo 18. Estudar a convergencia da sequencia xn+1 = 1 +√xn com x1 = 1.

A sequencia e crescente , pois x2 = 2 > x1, supondo xn+1 > xn temos

√xn+1 >

√xn ⇒ 1 +

√xn+1 > 1 +

√xn ⇒ xn+2 > xn+1.

A sequencia e limitada superiormente, por 3, por exemplo, pois x1 < 3, supondo xn < 3 < 4 tem-se

√xn < 2⇒ 1 +

√xn < 3⇒ xn+1 < 3.

Agora calculamos o limite da sequencia

a = 1 +√a⇒ (a− 1)2 = a⇒ a2 − 3a+ 1 = 0

cujas raızes sao 3±√

52 , nao podendo ser 3−

√5

2 que e menor que 1 logo o limite e 3+√

52 .

157

Exercıcio 4.22:

Propriedade 63. (xn) nao possui subsequencia convergente ⇔ lim |xn| =∞.

Demonstracao. ⇒).Se (xn) nao possui subsequencia convergente entao lim |xn| =∞.Se nao fosse lim |xn| =∞, existiria A > 0 tal que ∀ n0, existe n1 > n0 tal que |xn1 | < A, aplicando o resultado

com n1 no lugar de n0, existe n2 > n1 tal que |xn2 | < A e assim construımos uma subsequencia (xn1 , xn2 , · · · )limitada , que possui uma subsequencia convergente , o que e absurdo.⇐).Suponha por absurdo que lim |xn| = ∞ e (xn) possui subsequencia convergente, convergindo para a. Por

definicao de limite infinito, sabemos que existe n0 tal que n > n0 implica |xn| > |a|+ 10, por (xn) ter subsequenciaque converge para a, existe n1 tal que n > n1 e n ındice da subsequencia, implica |xn − a| < 10⇒ |xn| < |a|+ 10,podemos tomar ındice da subsequencia tal que n > n1 e n > n2, logo valeria |xn| < |a|+ 10 e |xn| > |a|+ 10 o quee absurdo, portanto (xn) nao pode possuir subsequencia convergente.

158

Exercıcio 4.25:

Propriedade 64 (Teste da razao para sequencias.). Se xn > 0 ∀n ∈ N e xn+1

xn≤ c < 1 para n suficientemente

grande entao limxn = 0.

Demonstracao. Existe n0 tal que para k > n0 vale 0 < xk+1

xk≤ c < 1, aplicamos o produtorio

∏nk=n0+1 em ambos

, de onde segue

0 <

n∏k=n0+1

xk+1

xk≤

n∏k=n0+1

c

0 < xn+1 < x(n0+1)cn−n0

como lim cn = 0, tem-se pelo teorema do sanduıche que limxn = 0.

Corolario 9. Dada uma sequencia de termos nao nulos (xn), entao (|xn|) e uma sequencia de termos positivos,se ela satisfaz a propriedade anterior entao lim |xn| = 0 o que implica limxn = 0.

Propriedade 65. Seja (xn) sequencia de termos positivos, se xn+1

xn≥ c > 1 para n suficientemente grande entao

limxn =∞.

Demonstracao. Existe n0 ∈ N tal que k > n0 implica xk+1

xk≥ c, onde c > 1. Aplicando o produtorio na desigualdade

tem-sen∏

k=n0+1

xk+1

xk> cn−n0

xn+1 >xn0+1

cn0cn

como lim cn =∞ segue que limxn =∞.

Corolario 10. Na propriedade anterior podemos trocar xn por |xn| onde xn nao se anula, pois (|xn|) e umasequencia de positivos.

Corolario 11. Se lim xn+1

xn= a < 1 entao para n suficientemente grande vale xn+1

xn≤ c < 1, logo tambem vale

limxn = 0.

Corolario 12. Se lim xn+1

xn= c > 1 a propriedade tambem se verifica pois existe n0 ∈ N tal que n > n0 implica

xn+1

xn> a > 1 para algum a.

Propriedade 66.

limn!

nn= 0.

Demonstracao. Definimos xn = n!nn e vale xn > 0, aplicamos a regra da razao

xn+1

xn=

(n+ 1)!

(n+ 1)n+1

nn

n!=

(n

n+ 1

)n=

1

(1 + 1n )n

o limite e lim xn+1

xn= 1

e < 1.nn cresce mais rapido que n!

Propriedade 67. Para todo a > 0 real temos lim an

n! = 0.

Demonstracao. Pelo teste da razao, definimos xn = an

n! temos xn > 0 segue xn+1

xn= an+1n!

(n+1).n!an = an+1 e temos

lim xn+1

xn= 0, logo limxn = 0.

A propriedade nos diz que n! cresce mais rapido que an.

159

Corolario 13. lim n!an = ∞, pois lim an

n! = 0, isso significa que ∀A > 0 ∃ n0 ∈ N tal que n > n0 ⇒ n!an > A, em

especial para A = 1, tem-se n! > an para n suficientemente grande.

Propriedade 68. Se a > 1 e p natural fixo vale

limnp

an= 0.

Demonstracao. Definimos xn = np

an , vale xn > 0 daı podemos aplicar o teste da razao

xn+1

xn=

(n+ 1)p

an+1

an

np=

(n+ 1

n

)p1

a⇒ limxn+1xn =

1

a︸︷︷︸0<

< 1

daı o limite e zero.

Corolario 14. Se a > 1, p ∈ N entao lim an

np =∞ pois lim np

an = 0.

Tal propriedade mostra que a exponencial an cresce muito mais rapido que np para n grande.

160

Exercıcio 4.31:

Exemplo 19. Mostrar que

lim

n∑k=1

kp

np+1=

1

p+ 1.

Iremos calcular o limite das diferencas do inverso da sequencia

lim(n+ 1)p+1 − np+1

(n+ 1)p= lim

[p−1∑k=0

(p+1k

)nk] + (p+ 1)np

(n+ 1)p= lim

p−1∑k=0

(p+1k

)nk

(n+ 1)p︸ ︷︷ ︸→0

+ lim(p+ 1)np

(n+ 1)p︸ ︷︷ ︸→p+1

= p+ 1

daı

lim

n∑k=1

kp

np+1=

1

p+ 1.

161

Exercıcio 4.33:

Questao digitada errada

Propriedade 69. Se limxn =∞ , com xn > 0 entao lim(∏nk=1 x

1n

k ) =∞

Demonstracao. Se limxn =∞ entao lim 1xn

= 0 daı lim (

n∏k=1

1

xk

1n

)︸ ︷︷ ︸=yn

= 0 que implica

lim1

yn=∞ = lim(

n∏k=1

x1n

k ).

Exemplo 20. Provar que lim n

√(2n)!n! = ∞. Tomamos xn = (2n)!

n! daı temos xn+1

xn= (2n+2)(2n+1)(2n)!

(n+1)n!n!

(2n)! =

(2n+2)(2n+1)(n+1) = 2(2n+ 1)→∞ logo lim n

√(2n)!n! =∞.

Exemplo 21. Mostrar que lim n

√(2n)!n!nn = 4

e .

Tomamos xn = (2n)!n!nn , daı xn+1

xn= 2(2n+1)

n+11

(1+ 1n )n→ 4

e .

162

Exercıcio 4.35:

Propriedade 70. Sejam∞∑n=u

an e∞∑n=s

bn series de termos positivos. Se∞∑n=s

bn = ∞ e existe n0 ∈ N tal que

an+1

an≥ bn+1

bnpara todo n > n0 entao

∞∑n=u

an =∞.

Demonstracao. an+1

an≥ bn+1

bn, Qak ≥ Qbk tomando o produtorio com k variando de k = n0 + 1 ate n − 1 na

desigualdade em ambos lados segue

n−1∏k=n0+1

Qak =an

an0+1≥

n−1∏k=n0+1

Qbk =bn

bn0+1, an ≥

an0+1

bn0+1bn

pois temos termos positivos, tomando a serie temos

∞∑n=n0+1

an ≥an0

bn0

∞∑n=n0+1

bn =∞

logo a serie tende ao infinito por comparacao.

163

Exercıcio 4.36:

Propriedade 71. 1. Sejam duas series∑ak e

∑bk de termos positivos, se existe lim ak

bk= a 6= 0 entao

∑ak

converge ⇔∑bk converge .

2. Se lim akbk

= 0 entao a convergencia de∑bk implica convergencia de

∑ak.

Demonstracao. 1. Existe n0 ∈ N tal que para k > n0 tem-se

0 < t1 < a− ε < akbk

< a+ ε < t2

como bk > 0 tem-set1bk < ak < t2bk

aplicamos a soma∑nk=n0+1, daı

t1

n∑k=n0+1

bk <

n∑k=n0+1

ak < t2

n∑k=n0+1

bk

usando essa desigualdade temos por comparacao que se∑bk converge entao

∑ak converge e se

∑ak converge

entao∑bk converge.

2. De maneira similar ao item anterior.

Existe n0 ∈ N tal que para k > n0 tem-se

0 ≤ akbk

< ε < t2

como bk > 0 tem-se0 ≤ ak < t2bk

aplicamos a soma∑nk=n0+1, daı

0 ≤n∑

k=n0+1

ak < t2

n∑k=n0+1

bk

usando essa desigualdade temos por comparacao que se∑bk converge entao

∑ak converge.

Exemplo 22. Pode valer que∑ak converge, valendo lim ak

bk= 0 e

∑bk nao converge, tome por exemplo ak = 1

k2 ,

bk = 1k ,∑bk nao converge, lim ak

bk= lim k

k2 = lim 1k = 0 e

∑ak converge, logo a recıproca do item 2 da propriedade

anterior nao vale.

164

Exercıcio 4.40:

Prove que para todo a ∈ R, a serie

a2 +a2

1 + a2+

a2

(1 + a2)2+ ...

e convergente e calcule a soma.

Se a = 0, a serie tende a 0 trivialmente.Suponhamos que a 6= 0. Entao, segue que

1

1 + a2< 1.

Assim, pelo exemplo 7 deste capıtulo, temos que

∞∑i=0

1

(1 + a2)i=

∞∑i=0

(1

1 + a2

)i=

1

1− 11+a2

=1 + a2

a2.

Logo,∞∑i=0

a2

(1 + a2)i= a2

( ∞∑i=0

1

(1 + a2)i

)= 1 + a2.

165

Exercıcio 4.41:

Para todo p ∈ N fixado, a serie ∑n

1

n(n+ 1)...(n+ p)

converge.

Para todos n e p ∈ Z+, temos que1

n(n+ 1)...(n+ p)<

1

np+1.

Como p+ 1 > 1, temos, pelo exemplo 29, que ∑n

1

np+1

e convergente. Logo, concluımos de

0 <i∑

n=1

1

n(n+ 1)...(n+ p)<

i∑n=1

1

np+1

que a serie ∑n

1

n(n+ 1)...(n+ p)

e limitada (e monotona). Portanto,∑n

1n(n+1)...(n+p) converge.

166

Exercıcio 4.42:

Se∑an converge e an > 0 entao

∑(an)2 e

∑1

1+anconvergem.

Como∑an e convergente, temos que limn→∞ an = 0. Assim, como tambem temos que an > 0 para todo

n ∈ Z+, existe n0 ∈ Z+ tal que0 < an < 1,

para todo n > n0. Logo, para n > n0, temos que

0 < a2n < an.

Assim,i∑

n=n0

a2n 6

i∑n=n0

an,

para todo i ∈ Z+, e, como existe∑∞n=n0

an, temos que existe∑∞n=n0

a2n. Logo, existe

∞∑n=1

a2n =

n0−1∑n=1

a2n +

∞∑n=n0

a2n.

Como an > 0, para todo n ∈ Z+, temos que

an1 + an

< an,

para todo n ∈ Z+. Assim,i∑

n=1

an1 + an

<

i∑n=1

an,

para todo i ∈ Z+, e, como∑∞n=1 an existe, temos que existe

∑∞n=1

an1+an

.

167

Exercıcio 4.43:

Se Σ(an)2 converge entao Σann converge.

SejamZ<+ := {n ∈ Z+; |an| < 1/n}

eZ>

+ := {n ∈ Z+; |an| > 1/n}.

Entao, Z+ = Z<+ ∪ Z>+. Alem disso, para cada n ∈ Z+, temos que

|an|n

<1

n2+ a2

n.

De fato, se n ∈ Z<+, temos que

|an| <1

n

implicando que|an|n

<1

n26

1

n2+ a2

n.

Se n ∈ Z>+ entao temos que

1

n6 |an|

e, consequentemente,|an|n

6 a2n <

1

n2+ a2

n.

Assim, temos quei∑

n=1

|an|n

<

i∑n=1

1

n2+

i∑n=1

a2n,

para todo i ∈ Z+. Pelo exemplo 29 e pela hiptotese, temos que∑∞n=1

1n2 e

∑∞n=1 a

2n existem. Assim, pela

desigualdadade acima, devemos ter que∑∞n=1

|an|n existe.

Como∞∑n=1

∣∣∣ann

∣∣∣ =

∞∑n=1

|an|n

existe, temos que∑∞n=1

ann tambem deve existir.

168

Exercıcio 4.44:

Se (an) e decrescente e Σan converge entao limnan = 0.

Primeiramente observemos que como (an) e decrescente e existe∑ak, entao an > 0 para todo n ∈ Z+. De

fato, se an0< 0 para algum n0 ∈ Z+, terıamos que

n0+i∑k=n0

ak 6n0+i∑k=n0

an0 = (i+ 1)an0 < 0

e, consequentemente, ∣∣∣∣∣n0+i∑k=n0

ak

∣∣∣∣∣ > (i+ 1)|an0|,

para todo i ∈ Z+. Contradizendo o fato de∑ak existir.

Assim, temos que (Σnk=1ak)n∈Z+ e uma sequencia crescente de Cauchy. Em especial,∣∣∣∣∣n+p∑k=1

ak −n∑k=1

ak

∣∣∣∣∣ =

n+p∑k=n+1

ak,

para todos n e p ∈ Z+.Seja ε > 0 arbitrario. Como (Σnk=1ak)n∈Z+ e de Cauchy, existe n0 ∈ Z+ tal que para todo n > n0 vale

ε

2>

∣∣∣∣∣n∑k=1

ak −n0∑k=1

ak

∣∣∣∣∣ =

n∑k=n0+1

ak > (n− n0)an.

Por outro lado, como∑an e convergente, devemos ter que limn→∞ an = 0 e, por conta disso, existe n1 ∈ Z+ tal

que para todo n > n1 temos queε

2n0> an

e, consequentemente,ε

2> n0an.

Assim, para n > n2 = max{n0, n1}, temos que

ε =ε

2+ε

2> (n− n0)an + n0an = nan = |nan|.

Portanto, concluimos que limn→∞ nan = 0.

169

Exercıcio 4.45:

Se (an) e decrescente e Σan = +∞, entao,

limn→∞

a1 + a3 + · · ·+ a2n−1

a2 + a4 + · · ·+ a2n= 1.

Como (an) e decrescente, temos quea2n−1 > a2n,

para todo n ∈ Z+. Assim, para todo n ∈ Z+, temos que

a1 + a3 + · · ·+ a2n−1 > a2 + a4 + · · ·+ a2n

e, consequentemente,

1 6a1 + a3 + · · ·+ a2n−1

a2 + a4 + · · ·+ a2n=: qn.

Provaremos agora que para todo c > 1 existe n0 tal que qn < c para todo n > n0. Daı conclui-se quelimn→∞ qn = 1. Como querıamos demonstrar.

Suponhamos, por contradicao, que exista c > 1 e uma subsequencia (qnk) de (qn) tais que

qnk> c

para todo k ∈ Z+.Entao temos, para todo k ∈ Z+, que

a1 + a3 + · · ·+ a2nk−1 > c(a2 + a4 + · · ·+ a2nk)

> c(a3 + a5 + · · ·+ a2nk+1)

e, consequentemente,a1 > a1 − ca2nk+1

> (c− 1)(a3 + a5 + · · ·+ a2nk−1).

E chegamos a um absurdo. De fato, como Σan =∞, existe k ∈ Z+ tal que

2a1

c− 1+ a1 < a1 + a2 + a3 + ·+ a2nk+1

e, consequentemente,

2a1c−1 + a1 < a1 + (a2 + a3) + (a4 + a5) + · · ·+ a2nk

+ a2nk+1

6 a1 + 2a3 + 2a5 + · · ·+ 2a2nk+1

6 a1 + 2(a3 + a5 + · · ·+ a2nk+1).

De onde temos que existe k ∈ Z+ tal que

a1 < (c− 1)(a3 + a5 + · · ·+ a2nk+1).

170

Exercıcio 4.46:

Seja (an) uma sequencia nao crescente, com lim an = 0. A serie∑an converge se e somente se

∑2na2n converge.

Como a sequencia (an) e nao cresente, devemos ter que an > 0, para todo n ∈ Z+, caso∑an ou

∑2nan sejam

convergentes. De fato, se ak0 < 0, temos que

limk→∞

k∑n=1

an =

k0−1∑n=1

an + limk→∞

k∑n=k0

an 6k0−1∑n=1

an + limn→∞

k∑n=k0

ak0 = −∞

e, analogamente,

limk→∞

k∑n=1

2na2n = −∞.

Em especial, temos, do fato de (an) ser uma sequencia de termos nao negativos, que∑an e

∑2nan sao

sequencias crescentes. Mostraremos, adiante, que se∑an e uma sequencia limitada (i.e. convergente) entao∑

2nan tambem e limitada e vice-versa.Suponhamos que

∑an seja convergente. Como (an) e nao crescente, temos que

2na2n = 2

2n−1∑k=2n−1

a2n 6 2

2n−1∑k=2n−1

ak,

para todo n ∈ Z+. Assim, para todo p ∈ Z+, temos que

p∑n=1

2na2n 6p∑

n=1

2

2n−1∑k=2n−1

ak 6 2

2p−1∑k=1

ak 6 limp→∞

2

2p−1∑k=1

ak <∞.

Assim, temos que∑

2na2n e uma sequencia crescente e limitada. Portanto,∑

2na2n e convergente.Suponhamos que

∑2na2n seja convergente. Como (an) e nao decrescente, temos, para todo k ∈ Z+, que

2k+1−1∑n=2k

an 62k+1−1∑n=2k

a2k = 2ka2k .

Assim, dado p ∈ Z+, temos, para q ∈ Z+ tal que p 6 2q, que

p∑n=1

an 6 a1 +

2q∑n=1

an = a1 +

q∑k=1

2k+1−1∑n=2k

an 6 a1 +

q∑k=1

2kak 6 a1 + limq→∞

q∑k=1

2kak <∞.

Logo,∑an e uma sequencia crescente e limitada. Portanto,

∑an e convergente.

171

Exercıcio 4.47:

Prove que o conjunto de valores de aderencia da sequencia xn = cos(n) e o intervalo fechado [0, 1].

Seja x ∈ [0, 1]. Como cos : R → [0, 1] e sobrejetivo1, existe θ ∈ R tal que cos θ = x. Encontraremos umasequencia crescente (nk)k∈Z+

em Z+ tal que

x = cos θ = limk∈Z+

cosnk = limk∈Z+

xnk.

Assim, como x e arbitrario, poderemos concluir que o conjunto de valores de aderencia da sequencia (xn) e ointervalo [0, 1].

SejaG := {n+m2π; n e m ∈ Z}.

Pelo Exercıcio 3.58, temos que G e denso em R.Para todo k ∈ Z+, temos que existe θk ∈ G tal que

θ − 1/k < θk < θ.

Desta forma, temos quelimk→∞

θk = θ.

Como 2π e irracional, a sequencia (θk)k∈Z+ define unicamente sequencias (nk)k∈Z+ e (mk)k∈Z+ em Z pelaigualdade

θk = nk +mk2π,

para todo kZ+. Por fim, definimos a sequencia (nk)k∈Z+por

nk = |nk|,

para todo kZ+. Assim, como limk→∞ θk = θ, temos que

limk→∞ cos(nk) = limk→∞ cos(nk) = limk→∞ cos(nk)= limk→∞ cos(θk −mk2π) = limk→∞ cos(θk)= cos(θ).

O conjunto {nk; k ∈ Z+} e infinito. Caso contrario, o conjunto {cos(nk); k ∈ Z+} e finito e, assim, existiriak ∈ Z+ tal que

cos θk = cos θ.

Consequentemente, θk = θ+m2π, para alguns k e m ∈ Z. Porem, como θ− 1/k < θk < θ, temos uma contradicao.Logo, (nk)k∈Z+

e uma sequencia ilimitada em Z+. Assim, (nk)k∈Z+admite uma subsequencia crescente. Em outros

termos, uma subsequencia que tambem e subsequencia de (n)n∈Z+. Denotaremos tal subsequencia tambem por

(nk)k∈Z+.

Portanto, temos que a subsequencia (xnk) e tal que

limk→∞

xnk= limk→∞

cos(nk) = cos(θ) = x.

1Isso pode ser verficado utilizando a continuidade da funcao cos e o teorema do valor intemediario. Mas, como o Elon assume quepodemos usar os nossos conhecimentos previos sobre as funcoes trigonometricas... Vamos tomar a afirmacao como fato.

172

Exercıcio 4.48:

Sejam a e b numeros reais positivos. Defina indutivamente as sequencias (xn) e (yn) pondo x1 =√ab, y1 = (a+b)/2,

xn+1 =√xnyn e yn+1 = (xn + yn)/2. Prove que xn e yn convergem para o mesmo limite, chamado de a media

aritmetico-geometrica entre a e b.

Pelo Princıpio da Definicao Indutiva, podemos as sequencias (xn)n∈Z>0e (yn)n∈Z>0

em R+ por

x0 := a, y0 := b, xn+1 :=√xnyn e yn+1 :=

xn + yn2

.

Se a = b temos que xn = yn = a, para todo n ∈ Z>0. De fato, se tomamos como hipotese de inducao que

xn = yn = a

entaoxn+1 =

√xnyn =

√a2 = a

e

yn+1 =xn + yn

2=

2a

2= a.

Assim, como x0 = a = b = y0, temos, pelo Princıpio da Inducao Finita, que xn = yn = a, para todo n ∈ Z>0.Desta forma,

limn→+∞

xn = a = limn→+∞

yn.

Portanto, nos resta mostrar o resultado para o caso em que a < b. Suponhamos, ate o fim desta demonstracao,que

b− a > 0.

Primeiramente, provaremos as desigualdades

xn < xn+1 < yn+1 < yn, (4.1)

para todo n ∈ Z>0, por inducao em n. Suponhamos que, para algum n fixo, D := yn − xn > 0. Entao,

x2n < xnyn = x2

n+1,

x2n+1 = xnyn

= xn(xn +D)

< x2n + xnD + D2

4

=(xn + D

2

)2=

(xn+yn

2

)2= y2

n+1

e

yn+1 =xn + yn

2= yn −

D

2= yn.

Logo, se xn < yn, temos (4.1). Como x0 = a < b = y0, temos, pelo Princıpio da Inducao Finita, que (4.1) valepara todo n ∈ Z>0.

Das desigualdades (4.1) segue que as sequencias (xn) e (yn) sao monotonas e limitadas. Logo, existemlimn→+∞ xn e limn→+∞ yn.

Provaremos, agora, quelim

n→+∞(yn − xn) = 0.

Usando (4.1), temos que

(yn − xn)2 = (yn + xn)2 − 4xnyn= 4y2

n+1 − 4x2n+1

= 4(y2n+1 − x2

n+1)= 4

((yn+1 − xn+1)2 + 2yn+1xn+1 − 2x2

n+1

)= 4(yn+1 − xn+1)2 + 8xn+1(yn+1 − xn+1)> 4(yn+1 − xn+1)2.

173

e, consequentemente,yn − xn

2> yn+1 − xn+1.

Em particular,b− a

2=y0 − x0

2> y1 − x1.

Supondo queb− a

2n> yn − xn,

como hipotese de inducao, temos que

b− a2n+1

>yn − xn

2> yn+1 − xn+1.

Assim, pelo Princıpio da Inducao Finita, temos que

b− a2n

> yn − xn = |yn − xn|,

para todo n ∈ Z+. Assim, pelo Teorema do Confronto (Teorema 8 do Capıtulo 4), temos que

limn→+∞

|yn − xn| = 0.

Logo, pelo exercıcio 4.1, temos que

limn→+∞

yn − limn→+∞

xn = limn→+∞

(yn − xn) = 0.

E o resultado segue.

174

Exercıcio 4.49:

Sejam a1 > a2 > · · · > 0 e sn = a1 − a2 + · · · + (−1)n−1an. Prove que a sequencia (sn) e limitada e quelim supsn − lim infsn = lim an.

Valem as desigualdadess2k 6 s2(k+1) 6 s2(k+1)+1 6 s2k+1, (4.2)

para todos k ∈ Z+. De fato, para todo k ∈ Z+, temos que

s2k = a1 − a2 + · · ·+ a2k−1 − a2k

6 a1 − a2 + · · ·+ a2k−1 − a2k + a2k+1 − a2(k+1)

= s2(k+1),

s2(k+1) = a1 − a2 + · · ·+ a2k+1 − a2(k+1)

6 a1 − a2 + · · ·+ a2k+1 − a2(k+1) + a2(k+1)+1

= s2(k+1)+1,

es2(k+1)+1 = a1 − a2 + · · ·+ a2k+1 − a2(k+1) + a2(k+1)+1

6 a1 − a2 + · · ·+ a2k+1

= s2k+1.

Segue, das desigualdades (4.2), que a sequencia (s2k)k∈Z+ e nao decrescente e limitada e que a sequencia(s2k+1)k∈Z+ e nao crescente e limitada. Em especial, temos que

limk→+∞

s2k = supk∈Z+

s2k e limk→+∞

s2k+1 = infk∈Z+

s2k+1.

Tambem valem as desigualdadess2k 6 sn 6 s2k+1, (4.3)

para todo k ∈ Z+ e n > 2k + 1 em Z+. De fato, se n > 2k + 1, temos que n = 2p ou 2p + 1 para algum p > k.Assim, como (s2k)k∈Z+

e nao decrescente, (s2k+1)k∈Z+e nao crescente e por (4.2), temos que

s2k 6 s2p 6 s2p+1 6 s2k+1.

Logo, obtemos (4.3). Em especial, concluimos que (sn)n∈Z+e limitada.

Como as sequencias (s2k)k∈Z+e (s2k+1)k∈Z+

sao monotonas e limitadas, temos que existem limk→∞ s2k elimk→∞ s2k+1. Provaremos que

lim infsn = limk→∞

s2k e lim supsn = limk→∞

s2k+1.

Como a sequencia (an)n∈Z+ e monotona e limitada, temos que esta e convergente. Daı, segue das igualdades acima,que

lim supsn − lim infsn = limk→+∞ s2k+1 − limk→+∞ s2k

= limk→+∞(s2k+1 − s2k)= limk→+∞ a2k+1

= limn→+∞ an.

Suponhamos que A seja um valor de aderencia de (sn)n∈Z+. Entao, devemos ter que

s2k 6 A 6 s2k+1,

para todo k ∈ Z+. De fato, se s2k+1 < A, para algum k ∈ Z+, temos que

sn 6 s2k+1 < A

para todo n > 2k + 1 em Z+, por (4.3). Isto e,

|A− sn| > |A− s2k+1|,

175

para um k ∈ Z+ fixo e todo n > 2k+ 1 em Z+. E isso contradiz o fato de A ser um valor de aderencia de (sn)n∈Z+ .Analogamente, prova-se que e impossıvel se ter A < s2k, para algum k ∈ Z+. Portanto, devemos ter que

limk→∞

s2k = supk∈Z+

s2k 6 A 6 infk∈Z+

s2k+1 = limk→+∞

s2k+1.

Em particular, os valores de aderencia lim inf sn e lim sup sn satisfazem

limk→∞

s2k 6 lim inf sn 6 lim sup sn 6 limk→+∞

s2k+1.

Logo, como lim inf sn e lim sup sn sao, respectivamente, o menor e o maior valor de aderencia de (sn)n∈Z+, temos

quelimk→∞

s2k = lim inf sn e limk→+∞

s2k+1 = lim sup sn.

176

Capıtulo 5

Topologia da Reta

177

Exercıcio 5.01:

Um conjunto A ⊂ e aberto se, e somente se, cumpre a seguinte condicao: “se uma sequencia (xn) converge paraum ponto a ∈ A entao xn ∈ A para todo n suficientemente grande”.

(⇒) Seja (xn)n∈N uma sequencia que tende a a ∈ (b, c) ⊂ A. Existe n0 ∈ N tal que n ≥ n0 implica que, paraε = min{a− b, c− a},

|xn − a| < ε,

ou seja,xn ∈ (a− ε, a+ ε) ⊂ (a, b) ⊂ A.

(⇐) Se a ∈ A e a ∈ (b, c) ⊂ A, para n suficientemente grande a ∈ (a− 1

n, a+

1

n) ⊂ (b, c) ⊂ A. Temos entao que

a ∈ intA se e somente se (a− 1

n, a+

1

n) ⊂ A para algum n ∈ N.

Por hipotese, existem n− e n+ ∈ N tais que

a− 1

n∈ A,∀n ≥ n−

e

a+1

n∈ A,∀n ≥ n+.

Assim, tomando-se n0 = max{n−, n+}, temos que

(a− 1

n0, a+

1

n0) ⊂ A.

Logo, a ∈ intA.

178

Exercıcio 5.02:

Tem-se que limxn = a se, e somente se, para todo aberto A contendo o ponto a, existe n0 ∈ N tal que n > n0

implica xn ∈ A.

(⇒) Repetir a ideia do exercıcio anterior.(⇐) Dado ε > 0, existe n0 ∈ N tal que, para todo n ≥ n0, temos que

xn ∈ (a− ε, a+ ε),

ou seja,|xn − a| < ε.

segue daı que limxn = a.

179

Exercıcio 5.03:

Seja B ⊂ aberto. Entao, para todo x ∈, o conjunto x + B = {x + y; y ∈ B} e aberto. Analogamente, se x 6= 0,entao o conjunto x.B = {x.y; y ∈ B} e aberto.

Seja B ⊂ aberto. Dado y ∈ x+ B, temos que y = x+ b para algum b ∈ B. Como B e aberto, existe ε > 0 talque (b− ε, b+ ε) ⊂ B. Desta forma, temos

(x+ b− ε, x+ b+ ε) = (y − ε, y + ε) ⊂ x+ b.

De fato, seja z ∈ (y − ε, y + ε) = (x+ b− ε, x+ b+ ε). Entao

x+ b− ε < z < x+ b+ ε

b− ε < z − x < b+ ε.

Logo, z − x ∈ B e consequentemente z ∈ x+B, como querıamos.De maneira analoga, temos que se x 6= 0, entao x.B e aberto. Seja y ∈ x.B. Entao y = x.b para algum b ∈ B.

Como B e aberto, existe ε > 0 tal que (b− ε, b+ ε) ⊂ B. Desta forma, temos que para δ = ε.|x| > 0, temos

(y − δ, y + δ) ⊂ x.B.

180

Exercıcio 5.04:

Sejam A e B abertos. Entao os conjuntos A+B = {x+y;x ∈ A, y ∈ B} e A.B = {x.y;x ∈ A, y ∈ B} sao abertos.

Como A + B = ∪a∈A(a + B) e A.B = ∪a∈A(a.B) e cada a + B e a.B sao abertos (pelo exercıcio anterior),temos que A+B e A.B sao unioes de abertos. Logo, tambem sao abertos.

181

Exercıcio 5.05:

Para quaisquer X,Y ⊂, tem-se int(X ∩Y ) = int(X)∩ int(Y ) e int(X ∪Y ) ⊃ int(X)∪ int(Y ). De um exemplo emque a inclusao nao se reduza a uma igualdade.

•int(X ∩ Y ) ⊂ int(X) ∩ int(Y ).

Dado w ∈ int(X ∩ Y ), existem a, b ∈ tais que w ∈ (a, b) ⊂ X ∩ Y. Daı, temos que w ∈ (a, b) ⊂ X ew ∈ (a, b) ⊂ Y. Assim, w ∈ int(X) ∩ int(Y ).

•int(X ∩ Y ) ⊃ int(X) ∩ int(Y ).

Dado w ∈ int(X) ∩ int(Y ), existem ax, ay, bx, by ∈ tais que w ∈ (ax, bx) ⊂ X e w ∈ (ay, by) ⊂ Y. Entaotomando a = max{ax, ay}eb = min{bx, by}, temos que w ∈ (a, b) ⊂ X ∩ Y. Assim, w ∈ int(X ∩ Y ).

•int(X ∪ Y ) ⊃ int(X) ∪ int(Y ).

Dado w ∈ int(X) ∪ int(Y ), existem ax, ay, bx, by ∈ tais que w ∈ (ax, bx) ⊂ X e w ∈ (ay, by) ⊂ Y. Entao,w ∈ (ax, bx) ⊂ X ∪ Y. Donde w ∈ int(X ∪ Y ).

• Sejam X = [−1, 0) e Y = [0, 1]. Entao,int(X) = (−1, 0)

int(Y ) = (0, 1)

int(X) ∪∫

(Y ) = (−1, 1) \ {0}

int(X ∪ Y ) = (−1, 1).

182

Exercıcio 5.06:

Se A ⊂ e aberto e a ∈ A entao A\{a} e aberto.

Seja x ∈ A \ {a}. Existe ε > 0 tal que x ∈ (x − ε, x + ε) ⊂ A. Tomando ε = min{|x − a|, ε} teremos quex ∈ (x− ε, x+ ε) ⊂ A \ {a}. Logo, x ∈ int(A \ {a}). Segue daı que A \ {a} e aberto.

183

Exercıcio 5.07:

Considere as funcoes f, g, h :→, dadas por f(x) = ax + b (a 6= 0), g(x) = x2 e h(x) = x3. Mostre que, para cadaA ⊂, f−1(A), g−1(A) e h−1(A) sao abertos.

Dado um intervalo (c1, c2) ⊂ . Temos que

f−1((c1, c2)) =

(c1 − ba

,c2 − ba

), a > 0

(c2 − ba

,c1 − ba

), a < 0

g−1((c1, c2)) =

(−√c2,−√c1) ∪ (

√c1,√c2), 0 ≤ c1 ≤ c2

(−√c2,√c2), c1 < 0 < c2

∅, c1 < c2 ≤ 0

h−1((c1, c2)) = ( 3√c1, 3√c2).

Assim, para cada A ⊂ aberto, temos que A = ∪Iλ, Iλ intervalo aberto, e

f−1(A)] ∪ f−1(Iλ), g−1(A)] ∪ g−1(Iλ) e h−1(A)] ∪ h−1(Iλ).

E como f−1(Iλ), g−1(Iλ) e h−1(Iλ) sao abertos, temos que f−1(A), g−1(A) e h−1(A) tambem o sao.

184

Exercıcio 5.08:

No exercıcio anterior, mostre que, para cada A ⊂ aberto, f(A) e h(A) sao abertos. De exemplo de A aberto talque g(A) nao seja aberto.

Seja A aberto em e a ∈ A. Entao, existem c1, c2 ∈ tais que a ∈ (c1, c2) ⊂ A. Daı,

f((c1, c2)) =

{(ac1 + b, ac2 + b), a > 0(ac2 + b, ac1 + b), a < 0

h((c1, c2)) = (c31, c32),

com f(a) ∈ f((c1, c2)) e h(a) ∈ h((c1, c2)). Segue daı que dado um b = f(a) ∈ f(A) qualquer, b ∈ intf(A). E omesmo vale para h. Logo, f(A) e h(A) sao abertos.

Exemplo: g((−1, 1)) = [0, 1) nao e aberto.

185

Exercıcio 5.09:

Toda colecao de abertos nao-vazios, dois a dois disjuntos e enumeravel.

Seja {Aλ}λ∈Λ uma colecao de abertos disjuntos. Para cada λ ∈ Λ escolhemos rλ ∈ Aλ ∩Q. Como Aλ ∩Aµ = ∅,para λ 6= µ, a funcao

Λ −→ Qλ 7→ rλ

e injetiva. Segue daı que Λ e finito ou enumeravel.

186

Exercıcio 5.10:

O conjunto dos valores de aderencia de uma sequencia e um conjunto fechado.

Seja A o conjunto dos valores de aderencia da sequencia (xn)n∈N e a ∈ um ponto de aderencia de A. Existe umasequencia (ak) em A que tende a a. E, para cada ak ∈ (ak), existe uma subsequencia (x(k,n)) de (xn) que tende aak.

Definimos uma sequencia (x(0,n)) de (xn). Para cada n ∈ N, existe k0 ∈ N tal que

|a− ak0 | <1

2n

e n0 ∈ N tal que

|ak0 − x(k0,n0)| <1

2n.

Assim, tomando-se x(0, n) = x(k0,n0) teremos que

|a− x(0,n)| <1

n.

Segue daı que a = limx(0,n) e, consequentemente, a ∈ A. Portanto, A e fechado.

187

Exercıcio 5.11:

Se X ⊂ F e F e fechado entao X ⊂ F.

Para cada x ∈ X, existe uma sequencia em X que tende a x. Logo, x ∈ F = F ja que (xn) e uma sequencia emF. Segue daı que X ⊂ F.

188

Exercıcio 5.12:

Se limxn = a e X = {x1, x2, ..., xn, ...} entao X = X ∪ {a}.

Segue diretamente da definicao que X ⊃ X ∪ {a}. Seja b ∈ X e (yn) uma sequencia em X que tende a b.Consideremos o conjunto {n ∈ N;xn ∈ {y1, y2, ..., yk, ...}} = I. Se I e finito , entao b = xn para algum n ∈ I. CasoI seja infinito, a subsequencia yki = nn tal que n = min{n ∈ N;xn ∈ {yki+1, ..., yk}} de (yk) e uma sequencia de(xn). E como limxn = a, temos que b = lim yki = a. Segue daı que X ⊂ ∪{a}.

189

Exercıcio 5.13:

O numero 14 pertence ao conjunto de Cantor.

O conjunto K de Cantor e dado porK = ∩∞n=0Kn,

com Kn+1 sendo obtido de Kn retirando-se de cada intervalo de Kn um subintervalo de comprimento1

3n+1de seu

centro.

Temos que2

32e o ınfimo de um intervalo de K2. Supondo que

p∑n=1

2

32ne o ınfimo de um intervalo de K2p,

temos quep∑

n=1

2

32n+

2

32(p+1)e o ınfimo de um intervalo em K2(p+1). Segue daı que cada somatorio reduzido da

serie∑ 2

32npertence a K.

E como

∞∑n=1

= 2

(1

1− 19

− 1

)= 2

(9

8− 1

)=

1

4,

temos pela compacidade de K que1

4∈ K.

190

Exercıcio 5.14:

Sejam F e G conjuntos fechados disjuntos tais que F ∪G seja um intervalo fechado (limitado ou nao). Entao F = ∅ou G = ∅.

Sejam F e G conjuntos fechados tais que F ∪G seja um intervalo fechado.Suponhamos que existam x ∈ F e y ∈ G. Suponhamos, sem perda de generalidade, que x < y. Assim,

[x, y] ⊂ F ∪G.Seja m = inf G ∩ [x, y]. Como G ∩ [x, y] e fechado, m ∈ G ∩ [x, y] e, consequentemente, m ∈ G. Por outro lado,

F ⊃ [x,m) e, consequentemente, F = F ⊃ [x,m) = [x,m]. Segue daı que m ∈ G ∩ F.Portanto, se tivessemos F ∩G = ∅, deverıamos ter tambem que F = ∅ ou G = ∅.

191

Exercıcio 5.15:

Se E ⊂ enumeravel. Consiga uma sequencia cujo conjunto dos valores de aderencia e E. Use este fato para mostrarque todo conjunto fechado F ⊂ e o conjunto dos valores de aderencia de alguma sequencia.

Seja E = {ak}k∈N. Definimos Nk = {2k−1.m ∈ N;m e ımpar } e, para cada n ∈ N, xn = ak onde n ∈ Nk.Assim, ak = lim

n∈Nk

xn e, consequentemente, o conjunto dos valores de aderencia da sequencia (xn), A, contem

E. Dado a ∈ E, existe uma sequencia (aki)i∈N em E tendendo a a. Por sua vez, a subsequencia (x2ki−1)i∈N e iguala sequencia (aki)i∈N. Logo, a ∈ A. Concluımos que A ⊃ E. Por outro lado, A ⊂ E ja que a sequencia (xn) ( econsequentemente todas as suas subsequencias) pertence a E. Portanto, A = E.

Dado um conjunto fechadoX ⊂, pelo Teorema 6, existe um conjunto enumeravel E denso emX. Por conseguinte,existe uma sequencia (xn) cujo conjunto dos valores de aderencia e E = X (pois X e fechado).

192

Exercıcio 5.16:

Com a notacao do Exercıcio 4, se α e irracional, os conjuntos FZ e G = αZ sao fechados porem F + G nao efechados. Tambem H = {0, 1, 1/2, ..., 1/n, ...} e fechados mas F.H nao e fechado.

Os conjuntos F e G sao, respectivamente, complementares dos conjuntos abertos⋃n∈Z

(n, n+ 1) e⋃n∈Z

(αn, α(n+ 1)).

Logo, F e G sao fechados.Temos que F +G = Z + αZ e um grupo aditivo em R. Assim,

(i) inf([G+ F ] ∩ (0,∞)) = a > 0.

Neste caso, pelo exercıcio 3.58, terıamos que F + G = {0,±a,±2a, ...} = aZ. Daı, F + G = aZ 3 ak = α ∈Z+αZ = F +G e, consequentemente, a =

1

k∈ Q. Por outro lado, F +G = aZ 3 ap = α ∈ Z+αZ = F +G,

para algum p ∈ Z, e consequentemente, α = ap ∈ Q. Contradicao.

Portanto, inf([F +G] ∩ (0,∞)) = 0.

(ii) inf([F +G] ∩ (0,∞)) = 0.

Novamente pelo exercıcio 3.58, temos que F+G e denso em R. Por outro lado,1

2/∈ Z+αZ, pois α e irracional.

Logo, F +G + F +G = R. Ou seja, F +G nao e fechado.

O conjunto H e fechado, pois e o complementar do conjunto (1,∞) ∪ (⋃n∈N

( 1n ,

1n+1 )). Por outro lado, F.H =

Q + Q = R. Ou seja, F.H nao e um fechado.

193

Exercıcio 5.17:

Seja K o conjunto de Cantor. Mostre que {|x− y|;x ∈ K, y ∈ K} = [0, 1].

Nao e difıcil provar que toda extremidade de subintervalo em cada passo da construcao do conjunto de Cantore do tipo

p∑n=0

an3n∈ [0, 1]

com an = 0 ou 2, 0 ≤ n < p, ap = 0, 1 ou 2.Dado

p∑n=0

cn3n∈ [0, 1], (5.1)

com cn = 0, 1 ou 2, definiremos a, b ∈ R tais que a, b ∈ K e c = b− a. Sejam

a =

p∑n=0

an3n

e b =

p+1∑n=0

bn3n

:

• c0 = 0 ou 1a0 = c0 e b0 = 0.

• c1c13

+s1

32=a1

3− b1

3− b2

9(5.2)

c1 = 0a1 = b1 = b2 = 0, s1 = 0

c1 = 1a1 = 2, b1 = 0, b2, s1 = 1

c1 = 2a1 = 2, b1 = 0, b2 = 0, s1 = 0

• cn, 1 ≤ n < pcn3n

+sn

3n+1=an3n− bn+1

3n+1+sn−1

3n(5.3)

cn = 0sn−1 = 0

an = 0, bn+1 = 0, sn = 0

sn−1 = 1an = 0, bn+1 = 2, sn = 1

cn = 1sn−1 = 0

an = 2, bn+1 = 2, sn = 1

sn−1 = 1an = 0, bn+1 = 0, sn = 0

cn = 2sn−1 = 0

an = 2, bn+1 = 0, sn = 0

sn−1 = 1an = 2, bn+1 = 2, sn = 1

194

• cpcp3p

=ap

3p− bp+1

3p+1+sp−1

3p(5.4)

cp = 0sp−1 = 0

ap = 0, bp+1 = 0

sp−1 = 1ap = 1, bp+1 = 2

cp = 1sp−1 = 0

ap = 1, bp+1 = 0

sp−1 = 1ap = 0, bp+1 = 0

cp = 2sp−1 = 0

ap = 2, bp+1 = 0

sp−1 = 1ap = 1, bp+1 = 0

Assim, somando-se as equacoes (5.2), (5.3) e (5.4) obtemos que c = b− a e a, b ∈ K.O conjunto D = {|x − y|;x, y ∈ K} contem todos os elementos do tipo (5.1). E como o conjunto de todos os

elementos do tipo (5.1) e denso em [0, 1] teremos o resultado assim que provarmos que D e fechado.

Seja Kn =2n⋃i=1

[ai, bi] o conjunto resultante da n−esima etapa da construcao do conjunto de Cantor. O conjunto

Dn = {|x− y|;x, y ∈ Kn} e dado por

Dn =⋃

1≤i,j≤2n

Dnij, onde Dnij

= {|x− y|;x ∈ [ai, bi], y ∈ [aj , bj ]},

e fechado, pois cada Dnije fechado. Assim, D =

∞⋂n=1

Dn tambem e fechado.

195

Exercıcio 5.18:

Dado qualquer numero real a > 0, existe x1, x2, ..., xn no conjunto de Cantor tais que x1 + x2 + ...+ xn = a.

Dado a > 0 em R podemos escrever a na forma

k +

∞∑n=1

an3n

com k ∈ N e an = 0, 1 ou 2. Se provarmos que∞∑n=1

an3n

pode ser escrito como soma de dois elementos α, β ∈ K,

temos o resultado definindo x2 = x2+k = 1, x1 = α e x2 = β.E posıvel demonstrar que

K = {∑ cn

3n; cn = 0 ou 2}.

Assim, definiremos αn e βn tais que

α =∑ αn

3ne β =

∑ βn3n.

Definimos

• a1 tal quea1

3=α1

3+β1

3+s1

3(5.5)

a1 = 0α1 = β1 = 0, s1 = 0

a1 = 1α1 = β1 = 0, s1 = 1

a1 = 2α1 = 2, β1 = 0, s1 = 0

• an, n > 1, tal quean3n

+sn−1

3n−1=αn3n

+βn3n

+sn3n

(5.6)

an = 0 sn−1 = 0 αn = βn = 0, sn = 0 sn−1 = 1 αn = 2, βn = 0, sn = 1

an = 1 sn−1 = 0 αn = βn = 0, sn = 1 sn−1 = 1 αn = 2, βn = 2, sn = 0

an = 2 sn−1 = 0 αn = 2, βn = 0, sn = 0 sn−1 = 1 αn = 2, βn = 2, sn = 1

Somando as equacoes (5.5) e (5.6), temos para todo k ∈ N que(k∑

n=1

an3n

)=

(k∑

n=1

αn3n

)+

(k∑

n=1

βn3n

)+sk3k.

Assim,∞∑n=1

an3n

=

∞∑n=1

αn3n

+

3n∑βn

βn3n

e, consequentemente, a− k = α+ β.

196

Exercıcio 5.19:

Seja K o conjunto de Cantor. Dado ε > 0 arbitrario, existem um intervalos abertos J1 = (a1, b1), ..., Jn = (an, bn)

tais que que K ⊂ J1 ∪ J2 ∪ ... ∪ Jn en∑i=1

(ai − bi) < ε.

Seja Kp o conjunto resultante do p-esimo passo da contracao do conjunto K de Cantor. Sabemos que

Kp =⋃2p

i=1[ci, ci +

1

3p].

Assim, tomando-se ai = ci − 13p e bi = 2

3p temos que

J =

2p⋃i=1

(ai, bi) ⊃ Kp ⊃ K

e2p∑i=1

(bi − ai) = 2p1

3p−1= 3(

2

3)p.

Assim, dado ε > 0, basta tomar p ∈ N tal que

3(2

3)p < ε

e teremos J que satisfaz as condicoes do enunciado.

197

Exercıcio 5.20:

Para X, Y ⊂ R qualquer, tem-se X ∪ Y = X ∪ Y e X ∩ Y ⊂ X ∩ Y . De um exemplo no qual a inclusao nao sereduz a uma igualdade.

• X ∪ Y ⊃ X ∪ Y .Seja a ∈ X ∪ Y . Entao, existe uma sequencia (xn) em X ou (yn) em Y que tende a a. Em ambos os casosexiste uma sequencia em X ∪ Y que tende a a.

• X ∪ Y ⊂ X ∪ Y .Seja a ∈ X ∪ Y . Entao, existe uma sequencia (zn) em X ∪ Y que tende a a. E, assim, esta sequencia possuiuma subsequencia em X (no caso em que possui infinitos termos em X) ou uma subsequencia em Y (no casoem que possui infinitos termos em Y ). No primeiro caso a ∈ X e no segundo caso a ∈ Y . Em todo caso,a ∈ X ∪ Y .

• X ∩ Y ⊂ X ∩ Y .Seja a ∈ X ∩ Y . Entao, existe uma sequencia (zn) em X ∩ Y que tende a a. Assim, (zn) e uma sequencia emX que tende a a, e, consequentemente, a ∈ X. Analogamente, a ∈ Y . Portanto, a ∈ X ∩ Y .

•[−1, 0) ∩ (0, 1] = ∅ = ∅.

[−1, 0) ∩ (0, 1] = [−1, 0] ∩ [0, 1] = {0}.

198

Exercıcio 5.21:

Um conjunto A ⊂ R e aberto se, e somente se, A ∩X ⊂ A ∩X para todo X ⊂ R.

(⇒) Seja A um conjunto aberto. Dado X ⊂ R e a ∈ A∩X. Entao, existe uma sequencia (xn) em X que tendea a. Assim, como A e aberto, existe n0 ∈ N tal que para todo n ≥ n0, xn ∈ A. Daı, existe uma sequencia em Aque tende a a. Ou seja, a ∈ A. Logo, a ∈ A ∩X ⊂ A ∩X.

(⇐) Seja A ⊂ R tal que A ∩X ⊂ A ∩X para todo X ⊂ R.Dado a ∈ A, suponhamos que para todo n ∈ N, existe xn ∈ (a − 1

n , a + 1n ) − A. Seja X = {xn}n∈N. Entao,

{a} = A∩X ⊂ A ∩X = ∅ = ∅. Contradicao. Logo, existe n ∈ N tal que (a− 1n , a+ 1

n ) ⊂ A. E, assim, a ∈ int(A).

199

Exercıcio 5.22:

Sejam F1 ⊃ F2 ⊃ ... ⊃ Fn ⊃ ... nao vazio se os Fn sao apenas fechados ou apenas limitados.

• Fn apenas fechado

Fn = [n,∞)⇒⋂n∈N

Fn = ∅.

• Fn apenas limitado

Fn =

(0,

1

n

]⇒⋂n∈N

Fn = ∅.

200

Exercıcio 5.23:

Um conjunto nao vazio X ⊂ R e um intervalo se, e somente se, satisfaz a condicao seguinte: “a, b ∈ X, a < x <b⇒ x ∈ X”.

(⇒) Seja X um intervalo nao-vazio com α = inf X e β = supX.Dados a e b ∈ X, a < b, e x ∈ R tal que a < x < b. Assim,

X ⊃ (α, β) ⊃ (a, b) ∈ X.

(⇐) Seja X um conjunto que satisfaz a condicao

“a, b ∈ X, a < x < b⇒ x ∈ X ”.

Sejam α = inf X e β = supX. Temos entao que X ⊃ (α, β). De fato, dado x ∈ (α, β), existem a e b ∈ X taisque

α ≤ a < x < b ≤ β.

Assim, pela propriedade, x ∈ X. Por outro lado, X − {α, β} ⊂ (α, β) pela definicao de α e β. Portanto,

X = [α, β] ou X = [α, β) ou X = (α, β] ou X = (α, β).

201

Exercıcio 5.24:

Mostre que a intersecao de uma sequencia decrescente I1 ⊃ I2 ⊃ ... ⊃ In ⊃ ... de intervalos e um intervalo ou oconjunto vazio.

Sejam an = inf In e bn = sup In, entao In ⊃ (an, bn) e [an, bn] ⊃ In. Assim,

[an, bn] ⊃ [an+1, bn+1]

e(an, bn) ⊃ (an+1, bn+1).

Daı,

[sup{an}, inf{bn}] =⋂

[an, bn]

⊃⋂In

⊃⋂

(an, bn)

= (sup{an}, inf{bn}).

Assim, como −∞ ≤ sup{an} ≤ inf{bn} ≤ ∞,⋂In = (sup{an}, inf{bn}) ou

= (sup{an}, inf{bn}] ou

= [sup{an}, inf{bn}) ou

= [sup{an}, inf{bn}]

e temos o resultado.

202

Exercıcio 5.25:

Um conjunto e denso em R se, e somente se, seu complementar tem interior vazio.

Seja D um conjunto denso em R. Dado x ∈ R−D e ε > 0, temos que (x−ε, x+ε)∩D 6= ∅. Assim, x /∈ int(R−D).E, como int(R−D) ⊂ R−D, temos que int(R−D) 6= ∅.

203

Exercıcio 5.26:

Se F e fechado e A e aberto entao F −A e fechado.

Como A e aberto, Ac e fechado. Entao, F ∩ Ac = F − A e uma intersecao de dois fechados. Logo, F − A efecado.

204

Exercıcio 5.27:

De exemplo de um aberto A tal que A ⊃ Q mas R−A seja nao-enumeravel.

Seja (xn) uma enumeracao de Q. Definimos A =∞⋃n=1

In, onde

In = (xn −1

2n, xn +

1

2n).

Temos que A e aberto, pois e uma uniao de abertos e tambem A ⊃ Q.Provemos que R − A 6= ∅. Consideremos o intervalo compacto [0, 10] e suponhamos que R − A = ∅. Entao,

existem n1, n2, ..., nk ∈ N tais quek⋃i=1

Ini ⊃ [0, 10].

Sgue daı, entao, que

10 <

k∑i=1

(sup Ini− inf Ini

)

<

k∑i=1

(sup In − inf In)

=

∞∑i=1

1

2n−1= 2.

Absurdo. Assim, R−A 6= ∅.Se R−A e finito ou enumerael, podemos adicionar todos os elementos de R−A em uma sequencia (yn) ( de termos

repetidosse necessario ) que enumera os elementos de R−A. Daı, A = A ∪ (∞⋃n=1

In), onde In = (yn −1

2n, yn +

1

2n)

e tal que A ⊃ A e A ⊃ R − A. Mas, por argumento analogo ao acima, podemos mostrar que R − A 6= ∅. Umacontradicao. Portanto, R−A e nao enumeravel.

205

Exercıcio 5.28:

De exemplo de um conjunto fechado, nao-enumeravel, formado apenas por numeros transcendentes.

Seja (xn) uma enumeracao do conjunto dos numeros algebricos (exercıcio 3.44). Seja

A =

∞⋃i=1

In,

onde In = (xn −1

2n, xn +

1

2n). Temos que A e um aberto que contem todos os numeros naturais. Assim, pelo

mesmo argumento usado no execıcio anterior, F = R−A e nao enumeravel. E F e um fechado que contem somentenumeros algebricos.

206

Exercıcio 5.29:

Defina a distancia de um ponto a ∈ R a um conjunto nao-vazio X ⊂ R como d(a,X) inf{|x− a|;x ∈ X}. Prove:

1) d(a,X) = 0⇔ a ∈ X.

2) Se F ⊂ R e fechado, entao para todo a ∈ R existe b ∈ F tal que d(a, F ) = |b− a|.

(1) (⇒) Para cada n ∈ N, existe xn ∈ N tal que |xn − a| <1

n. Assim, (xn) e uma sequencia em X que tende a

a. Logo, a ∈ X.(⇐) Seja (xn) uma sequencia em X que tende a a. Entao, para todo ε > 0, existe n0 ∈ N tal que |xn0

−a| < ε.Assim,

0 = inf{|xn − a|;xn ∈ (xn)}≥ inf{|x− a|;x ∈ X} ≥ 0,

ou seja, d(a,X) = 0.

(2) Consideremos o compacto C = F ∩ B[a; 2d(a, F )]. Temos, pela definicao de d(a, F ), que C 6= ∅. Seja d =

d(a, F ) e xn ∈ C tal que |xn − a| < d +1

n. Como C e compacto, existe uma subsequencia (xnk

) de (xn)

tal que xnk→ b ∈ C. Assim, dado ε > 0, existe n + k ∈ N tal que |xnk

− a| < d +ε

2e |b − xnk

| > ε

2.

Consequentemente,|b− a| ≤ |b− xnk

|+ |xnk− a| < d+ ε.

Portanto, |b − a| ≤ d. Mas, como b ∈ C ⊂ F e pela definicao de d = d(a, F ), temos que |b − a| ≥ d e,consequentemente, |b− a| = d.

207

Exercıcio 5.30:

Se X e limitado superiormente, seu fecho X tambem o e. Alem disso, supX = supX. Enuncie e prove um resultadoanalogo para inf .

Temos que X e limitado somente se X e limitado. De fato, se A > 0 em R e tal que existe a ∈ X com a > A,tomando-se ε = |A− a|, existe x ∈ X tal que |x− a| < ε. Consequentemente, x > A.

Como X ⊂ X, temos imediatamente que (pelo exercıcio 3.33) que supX ≥ supX. Assim, para mostrar quesupX ≥ supX, basta provar que para qualquer c ∈ R tal que c < supX, existe x ∈ X tal que c < x. Entao, dadoc tal que c < supX, existe x ∈ X tal que c < x. Assim, tomando algum x ∈ X tal que |x− x| < |x− c|, temos quex > c. Portanto, supX = supX.

O resultado analogo seria inf A = inf A. E a demonstracao desse resultado e tambem analoga.

208

Exercıcio 5.31:

Para todo X ⊂ R limitado superiormente, supX e aderente a X. Resultado analogo para inf .

Pela definicao de supremo, para todo ε > 0 existe x ∈ X tal que supX − ε < x ≤ supX.

Assim, para n ≥ 1 em N, tomando xn ∈ X tal que supX − 1

n> xn ≤ supX. Temos, assim, que para todo

n ∈ N, |xn − supX| < 1n . Logo, a sequencia (xn) em X tende a supX e supX ∈ X.

209

Exercıcio 5.32:

Para todo X ⊂ R, X ′ e fechado.

Seja a ∈ X ′. Dado ε > 0, existe x ∈ X ′ tal que x ∈ (a−ε, a+ε). E como x ∈ X ′, existem infinitos elementos de Xem (x−δ, x+δ), onde δ = min{|x−(a±ε)|}. Assim, infinitos elementos de X pertencem a (a−ε, a+ε) ⊃ (x−δ, x+δ).Isso implica que a ∈ X ′. Concluımos que X ′ ⊃ X ′.

210

Exercıcio 5.33:

Um numero a e ponto de acumulacao de X se, e somente se, e ponto de acumulacao de X.

(⇒) Seja a um ponto de acumulacao de X. Entao, para todo ε > 0 existem infinitos elementos de X em(a− ε, a+ ε). E como X ⊂ X, existem infinitos elementos de X em (a− ε, a+ ε), ou seja, a e ponto de acumulacaode X.

(⇐) Seja a um ponto de acumulacao de X. Dado ε > 0, existe x ∈ X tal que x ∈ (a−ε, a+ε)−{a}. Tomando-seδ = min{|x− a|, |x− (a± ε)|}, temos que existe x ∈ X tal que |x− x| < δ.

Assim, x ∈ (x− δ, x+ δ) ⊂ (a− ε, a+ ε)− {a}. Segue daı que a e ponto de acumulacao de X.

211

Exercıcio 5.34:

(X ∪ Y )′ = X ′ ∪ Y ′.

• (X ∪ Y )′ ⊂ X ′ ∪ Y ′.Seja a ∈ (X ∪ Y ) e (an) uma sequencia em X ∪ Y − {a} que tende a a. Entao, existem infinitos termos de(an) em Y. Assim, existe uma subsequencia de (an) em X − {a} ou em Y − {a}. E como esta subsequenciatende a a, temos que a ∈ X ′ ∪ Y ′.

• (X ∪ Y )′ ⊃ X ′ ∪ Y ′.Seja a ∈ X ′ ∪ Y ′. Entao existe uma sequencia (an) em X −{a} ou em Y −{a} que tende a a. Em todo caso,existe uma subsequencia em X ∪ Y − {a} que tende a a, ou seja, a ∈ (X ∪ Y )′.

212

Exercıcio 5.35:

Todo ponto de um conjunto aberto A e ponto de acumulacao de A.

Seja a ∈ A. Entao existe δ > 0 tal que (a− δ, a+ δ) ⊂ A.Assim, dado ε, para δ1 = min{δ, ε} temos que

(a− ε, a+ ε) ⊃ (a− δ1, a+ δ1) ⊂ A.

Logo, (a− ε, a+ ε) contem infinitos elementos de A ja que (a− δ1, a+ δ1) tem infinitos elementos. Portanto, a eponto de acumulacao.

213

Exercıcio 5.36:

Sejam F fechado e x ∈ F. Entao x e um ponto isolado de F se, somente se, F − {x} e ainda fechado.

(⇒) Se F = {x}, temos diretamente que F − {x} = ∅ e fechado. Suponhamos que F − {x} 6= ∅. Seja (an) umasequencia em F − {x} com lim an = a. Como F e fechado e (an) e uma sequencia em F, entao a ∈ F. Temos quea 6= x, pois para algum ε > 0, (x − ε, x + ε) ∩ F = {x}. Portanto, a ∈ F − {x}. Concluımo daı que F − {x} efechado.

(⇐) Suponhamos que x nao seja um ponto isolado de F. Entao, para todo n ∈ N, existe xn ∈ ((xn −1

n, x +

1

n) ∩ F )− {x}. Assim, a sequencia (xn) em F − {x} tende a x. O que implica que F − {a} nao e fechado.

214

Exercıcio 5.37:

Seja X ⊂ R tal que X ′ ∩X = ∅. Mostre que existe, para cada x ∈ X, um intervalo aberto Ix, de centro em x, talque x 6= y ⇒ Ix ∩ Iy = ∅.

Para cada x ∈ X definimosδx = inf{|x− x|; x ∈ X − {x}}.

Temos que δx = 0 somente se x ∈ X ′. Entao, δx > 0 para cada x ∈ X ja que X ′ ∩X = ∅.Definimos para cada x ∈ X,

Ix = (x− δx2, x+

δx2

).

Assim, para x 6= y em X, temos que se z ∈ Ix ∩ Iy

|x− y| ≤ |x− z|+ |z − y|

≤ δx2

+δx2

≤ |x− y|2

+|x− y|

2= |x− y|.

Portanto, Ix ∩ Iy = ∅. E o resultado segue.

215

Exercıcio 5.38:

Seja F ⊂ R fechado, infinito enumeravel. Mostre que F possui uma infinidade de pontos isolados.

Pelo corolario 1 do Teorema 9, temos que F possui algum ponto isolado. Suponhamos que x1, x2, ..., xn saopontos isolados de F. Por inducao no resultado do exercıcio 3.36, temos que F − {x1, x2, ..., xn} e fechado. Alemdisso, F −{x1, x2, ..., xn} e um conjunto sem pontos isolados e infinito enumeravel, contradizendo o Corolario 1 doTeorema 9.

216

Exercıcio 5.39:

Mostre que todo numero real x e limite de uma sequencia de numeros transcendentes dois a dois distintos.

Sejam A o conjunto dos numeros algebricos em Q (complementar do conjunto dos trancendentes) e x ∈ R.Dado ε > 0, temos que (x−ε, x+ε)−(A∪{x}) e infinito. De fato, pelo exercıcio 3.44, A e enumeravel enquanto

(x− ε, x+ ε) e nao enumeravel. Logo, (x− ε, x+ ε)− (A ∪ {x}) e infinito.Seja ε1 = 1. Podemos escolher

x1 ∈ (x− ε1, x+ ε1)− (A ∪ {x}).

E, indutivamente, escolherxn ∈ (x− εn, x+ εn)− (A ∪ {x}),

onde εn = |x− xn|. Assim (xn) e uma sequencia de termos trancendentes, dois a dois distintos e que tende a x.

217

Exercıcio 5.40:

Mostre que se X ⊂ R nao e enumeravel, entao X ∩X ′ 6= ∅.

Se X ∩X ′ = ∅, entao todo ponto de X e isolado. Mas, pelo Corolario 2 do Teorema 8, temos que isso implicaque X e enumeravel.

218

Exercıcio 5.41:

Se A e A ∪ {a} sao abertos entao a e ponto de acumulacao de A a direita e a esquerda.

Como A ∪ {a} e aberto, existe ε > 0 tal que (a− ε, a+ ε) ⊂ A ∪ {a}.Assim,

[a, a+ ε) ⊂ A ∪ {a}

e, consequentemente,(a, a+ ε) ⊂ A ∪ {a}.

Daı segue que A contem infinitos pontos de [a, a+ ε). Logo, a e ponto de acumulacao a direita de A.Analogamente, mostra-se que a e ponto de acumulacao a esquerda de A.

219

Exercıcio 5.42:

De explicitamente o significado de cada uma das seguintes afirmacoes. Em suas explicacoes, voce esta proibido deusar qualquer das palavras grifadas abaixo:

1) a ∈ X nao e ponto interior de X;

2) a ∈ R nao e aderente a X;

3) X ⊂ R nao e um conjunto aberto;

4) O conjunto Y ⊂ R nao e fechado;

5) a ∈ R nao e ponto de acumulacao do conjunto X ⊂ R;

6) X ′ = ∅;

7) X ⊂ Y mas X nao e denso em Y ;

8) int(X) = ∅;

9) X ∩X ′ = ∅;

10) X nao e compacto.

(1) Nao existe ε ∈ R+ tal quea ∈ (a− ε, a+ ε) ⊂ X.

(2) Existe ε ∈ R+ tal que(a− ε,A+ ε) ⊂ R−X.

(3) Existe x ∈ X tal que para todo ε ∈ R+

(x− ε, x+ ε) * X.

(4) Existe x ∈ R−X tal que para todo ε ∈ R+

(x− ε, x+ ε) ∩X 6= ∅.

(5) Existe ε ∈ R+ tal que(a− ε, a+ ε) ∩X ⊂ {a}.

(6) Para todo a ∈ R, tal que para todo ε ∈ R+

(a− ε, a+ ε) ∩X 6= ∅ entao a ∈ X.

(7) Existe y ∈ Y e ε ∈ R+ tais que(y − ε, y + ε) ∩X = ∅

mas X ⊂ Y.

(8) Para todo x ∈ X e ε ∈ R+, existe a ∈ R e δ ∈ R+ tais que

(x− ε, x+ ε) ⊃ (a− δ, a+ δ) ⊂ R−X.

(9) Para todo x ∈ X, existe ε ∈ R+ tal que

(x− ε, x+ ε) ∩X = {x}.

(10) Existe uma sequencia (xn) em R e (εn) em R+ tais que

X ⊂⋃n∈N

(xn − εn, x+ εn),

mas para nenhum N ⊂ N finito temos que

X ⊂⋃n∈N

(xn − εn, x+ εn).

220

Exercıcio 5.43:

Se todo ponto de acumulacao de X e unilateral, X e enumeravel.

Seja D o conjunto dos elementos de X que sao pontos de acumulacao a direita de X.Dado n ∈ Z seja

{aλ}λ∈Λ = [n, n+ 1] ∩D.

Para cada λ ∈ Λ, existe Iλ = (aλ−ελ, aλ), ελ > 0, tal que Iλ∩X = ∅. Observemos que para quaisquer λ1 e λ2 ∈ λ,λ1 6= λ2, teremos que Iλ1

∩ Iλ2= ∅. Se Λ e nao enumeravel, existe ε > 0 tal que para cada n ∈ N existe λn tal que

ελn > ε. Assim,∞∑n=1

ελn =∞

e como os I ′λs sao disjuntos e estao contidos em um intervalo de comprimento 1, obtemos uma contradicao. Logo,Λ e finito ou enumeravel. E como

D =⋃n∈Z

[n, n+ 1] ∩D,

temos que D e enumeravel.De forma analoga, mostra-se que o conjunto E dos pontos de acumulacao a esquerda de X e finito ou enumeravel.E como X −X ′ e finito ou enumeravel pelo Corolario 2 do Teorema 8, temos que

X = (X −X ′) ∪ E ∪D

e finito ou enumeravel.

221

Exercıcio 5.44:

SejaX ⊂ R um conjunto arbitrario. Toda cobertura deX por meio de abertos possui uma subcobertura enumeravel.(Teorema de Lindelof).

Seja {Aλ}λ∈Λ uma cobertura aberta de X. Tomemos E = {xn}n∈N um subconjuntto denso em X. Para cadan ∈ N fixemos

Rn = {ε ∈ R+; (xn − ε, xn + ε) ⊂ Aλ, λ ∈ Λ}.

Sabemos que Rn 6= ∅. Logo, existe supRn = 2εn.Alem disso, existe ε ∈ Rn tal que εn < ε. Ou seja, existe ε ∈ R+ tal que

(x− ε, x+ ε) ⊂ Aλ

para algum λ ∈ Λ. Existe, tambem n ∈ N tal que

xn ∈ (x− ε/4, x+ ε/4).

Para qualqueryn ∈ (xn − 3ε/4, xn + 3ε/4)

temos quey ∈ (x− ε, x+ ε).

Daı, 3ε/4 ∈ Rn. E, assim,

εn ≥3ε

8>ε

4.

Logo,x ∈ (xn − εn, xn + εn) ⊂ Aλn

.

Concluımos daı que {Aλn}n∈N ⊂ {Aλ}λ∈Λ e uma cobertura de X.

222

Exercıcio 5.45:

Com a notacao do Exercıcio 4, prove:

a) Se A e compacto e B e fechado entao A+B e fechado;

b) se A e B sao compactos, entao A+B e A.B sao compactos;

c) se A e fechado e B e compacto, A.B pode nao ser fechado.

(a) Seja x ∈ A+B e (cn = an + bn)n∈N uma sequencia em A+B tendendo a x. A sequencia (an)n∈N em A possuiuma subsequencia (ank

)k∈N tendendo a algum a ∈ A. Dado ε > 0, existe n0 ∈ N tal que, para todo n ≥ n0,

|an + bn − x| < ε/2.

E tambem, existe k0 ∈ N tal que, para todo k ≥ k0,

|ank− a| < ε/2.

Assim, parak1 = max{k0,min{k ∈ N;nk ≥ n0}}

e todo k ≥ k1, temos que

|bnk− (x− a)| = |(ank

+ bnk− x)− (a− ank

)|≤ |ank

+ bnk− x|+ |a− ank

|< ε/2 + ε/2 = ε.

Logo, limn→∞

bnk= x− a. Segue daı que x− a ∈ B e, consequentemente, x ∈ A+B. Concluımos daı que A+B

e fechado.

(b) A+B e fecado pelo item (a). E, tambem, temos que

sup(A+B) = supA+ supB <∞

einf(A+B) = inf A+ inf B > −∞.

Seja x ∈ A.B e (cn = an.bn)n∈N uma sequencia em A.B tendendo a x. A sequencia (an)n∈N em A possui umasubsequ?ncia (an)n∈N1

tendendo a algum a ∈ A. Por sua vez a sequencia (bn)n∈N1possui uma subsequencia

(bn)n∈N2tendendo a algum b ∈ B. Assim,

x = limn∈N2

anbn

= limn∈N2

an limn∈N2

bn

= a.b ∈ A.B

Segue daı que A.B e fechado. E, como

sup(A.B) = supA. supB <∞

einf(A.B) = inf A. inf B > −∞,

temos que A.B e limitado. Portanto, A.B e compacto.

(c) Tome B = {0, 1, 1/2, ..., 1/n, ...} e A = Z. Temos que A.B nao e fechado. (Vide exercıcio 5.16)

223

Exercıcio 5.46:

Obtenha coberturas abertas de Q e de [0,∞) que nao admitam subcoberturas finitas.

Temos que {(−n, n)}n∈N e uma cobertura aberta tanto de Q quanto de [0,+∞) que nao admite subcoberturafinita.

224

Exercıcio 5.47:

Considere as funcoes f, g, h do Exercıcio 7. Mostre que paraK e L compactos arbitrarios, f(K), g(K), h(K), f−1(L), g−1(L)e h−1(L) sao compactos.

Seja (xn) uma sequencia em f−1(L) tendendo a x ∈ R. Temos que

lim f(xn) = lim(axn + b)

= a limxn + b

= ax+ b = f(x)

Como f(xn) e uma sequencia convergente no fechado L, temos que f(x) ∈ L e, consequentemente, x ∈ f−1(L).Segue daı que f−1(L). Segue daı que f−1(L) e fechado.

Seja (yn = f(xn))n∈N uma sequencia em f(K). Como K e compacto, existe uma subsequencia (xnk) de (xn)

que converge a algum x ∈ K. Assim,

y = lim f(xn) = lim f(xnk) = f(x).

Ou seja, y ∈ f(K). Concluımos daı que f(K) e fechado.Analogamente, prova-se que g−1(L), g(K), h−1 e h−1(K) sao fechados.Por fim, temos que

f(K) = aK + b

g(K) ⊂ K.K

eK.K.K.

E como os conjuntos aK + b, K2 e K3 sao limitados(ver exercıcio 5.45(b)), temos que f(K), g(K) e h(K) saocompactos. E, tambem,

f(f−1(L)) ⊂ L,

g(g−1(L)) ⊂ L

eh(h−1(L)) ⊂ L.

Assim, pelas definicoes de f, g e h temos que se f−1(L), g−1 ou h−1(L) fosse limitado, terıamos que f(f−1(L)),g(g−1(L)) ou h(h−1(L)) seria limitado contradizendo o fato de L ser limitado.

225

Exercıcio 5.48:

As seguintes afirmacoes a respeito de um conjunto X ⊂ R sao equivalentes:

(1) X e limitado;

(2) Todo subconjunto infinito de X possui ponto de acumulacao (que pode nao pertencer a X);

(3) Toda sequencia de pontos de X possui uma subsequencia convergente.

(1) ⇒ (2) Como X e limitado, temos que X e compacto. Entao, todo conjunto infinito em X ⊂ X possui umponto de acumulacao por ser subconjunto de um compacto.

(2)⇒ (3) Seja (xn)n∈N uma sequencia em X. Se o conjunto P = {xn}n∈N for finito, entao para algum p ∈ P einfinitos n ∈ N, temos que xn = p. Logo, existe uma subsequencia de xn tendendo a p. Se P for infinito, entao pelahipotese, P possui um ponto de acumulacao x. Logo, e possıvel encontrar uma subsequencia de (xn) tendendo a x.

(3) ⇒ (1) Se X nao fosse limitado, seria possıvel encontrar uma sequencia crescente e ilimitada em X. Talsequencia nao teria subsequencia convergente. O que contradiz a hipotese.

226

Exercıcio 5.49:

Seja X ⊂ R um conjunto compacto cujos pontos, com excecao de a = inf X e b = supX, sao pontos de acumulacaoa direita e a esquerda. Entao X = [a, b] ou X = {a, b}.

Como X e compacto, temos que a = inf X e b = supX ∈ X. Suponhamos que X 6= {a, b}. Entao, existe c ∈ Xtal que a < c < b. Seja x ∈ (a, b). Se c ≤ x temos que s = sup([a, x] ∩X) e tal que a < c ≤ s ≤ x. Assim, se s 6= xtemos que s ∈ X − {a, b} e [s, x) ∩ X = {s}. Ou seja, s nao seria um ponto de acumulacao a direita. Absurdo.Logo, c = s ∈ X. Do mesmo modo, se x ≤ c teremos que x ∈ X. Concluımos que X = [a, b].

227

Exercıcio 5.50:

Se (Kλ)λ∈L e uma famılia qualquer de compactos, entao⋂Kλ e compacto. Se K1, ...,Kn sao compactos entao

K1 ∪K2 ∪ ... ∪Kn e compacto. Se K e compacto e F e fechado, entao K ∩ F e compacto.

• Kλ, λ ∈ Λ, compactos ⇒⋂λ∈ΛKλ e compacto.

Como cada Kλ e fechado, temos que ∩Kλ e fechado. Alem disso, temos que, dado algum λ0 ∈ Λ qualquer

inf(∩Aλ) ≥ inf Aλ0> −∞

esup(∩Aλ) ≤ inf Aλ0

<∞.

Daı, ∩Aλ e limitado. Portanto, ∩Aλ e compacto.

• Ki, i = 1, 2, ..., n, compacto ⇒⋃ni=1Ki e compacto.

Como cada Ki e fechado, temos que⋃ni=1Ki e fechado. Alem disso,

sup(

n⋃i=1

Ki) = sup{supKi}ni=1 <∞

e

inf(

n⋃i=1

Ki) = inf{inf Ki}ni=1 > −∞.

Daı,⋃ni=1Ki e limitado. Portanto,

⋃ni=1Ki e compacto.

• K e compacto e F e fechado ⇒ K ∩ F e compacto.

Como K e F sao fechados, K ∩ F e fechado. Alem disso,

sup(K ∩ F ) ≤ supK <∞

einf(K ∩ F ) ≥ inf K > −∞.

Daı, K ∩ F e limitado. Portanto, K ∩ F e compacto.

228

Exercıcio 5.51:

Seja X ⊂ R. Uma funcao f : X → R diz-se nao-decrescente no ponto a ∈ X quando existe δ > 0 tal quea− δ < x ≤ a ≤ y < a+ δ ⇒ f(x) ≤ f(a) ≤ f(y). (Bem entendido: x, y ∈ X.) Mostre que se f e nao-decrescenteem [a, b] (isto e, x, y ∈ [a, b], x ≤ y ⇒ f(x) ≤ f(y)).

Sejam x, y ∈ [a, b] com x ≤ y. Provaremos que f(x) ≤ f(y). Para cada α ∈ [x, y] existe δx ∈ R+ tal queα− δα < z ≤ α ≤ w < α+ δα implica f(z) ≤ f(α) ≤ f(z)(para z, w ∈ [a, b]).

Temos entao que [x, y] ⊂⋃α∈[x,y](α − δα, α + δα). Asssim, existem α1, α2, ..., αn ∈ [x, y] tais que [x, y] ⊂⋃n

i=1(αi − δα1, αi + δα1

). Podemos mostrar, por inducao em n, que podemos decompor [x, y] como

[x, β1 − δβ1) ⊂ (β1 − δβ1

, β1 + δβ1),

[βi−1 − δβi−1) ⊂ (βi − δβi , βi + δβi), i = 1, 2, ..., p− 1,

e[βp−1 − δβp−1 , y] ⊂ (βp − δβp , βp + δβp)

para {βi}pi=1 ⊂ {αi}ni=1 e βi < βi+1, i = 1, 2, ..., p− 1. Daı escolhemos γi ∈ R, i = 1, 2, ..., p− 1, tais que

βi+1 − δi+1 < γi < βi + δβie βi < γi < βi+1.

Assim,f(x) ≤ f(γ1) ≤ f(γ2) ≤ ... ≤ f(γp−1) ≤ f(y).

229

Exercıcio 5.52:

Seja [a, b] ⊂ ∪Aλ onde cada Aλ e aberto. Mostre que e possıvel decompor [a, b] em um numero finito de intervalosjustapostos de modo que cada um deles esteja contido em algum Aλ.

Pelo Teorema 2, cada Aλ pode ser decomposto como⋃j∈NA(λ,j),Nλ ⊂ N, onde os A′(λ,j)s sao intervalos abertos

disjuntos.Existe uma subcobertura {Bi}ni=1 ⊂ {A(λ.j)} de [a, b]. Provaremos que e possıvel decompor [a, b] como

[a, x1) ⊂ C1, [) ⊂ Ci, i = 2, ..., p, e [xp, b) ⊂ Cp+1 (5.7)

para {Ci}p+1i=1 ⊂ {Bi}ni=1.

Se n = 1, temos o resultado diretamente. Suponhamos que o resultado seja valido para n ≤ k e que estejamosno caso em que n = k + 1. Como a ∈ Bi, para algum i = 1, 2, ...n, tomemos C1 = Bi e assim

[supC1] ⊂⋃

({Bj}nj=1 − {Bi}).

Pela hipotese indutiva, existem y1, y2, ..., yp e C1, C2, ..., Cp que decompoe [supC1, b] como em (5.7). Assim,tomando-se

x1 = supC1

xi = yi, i = 2, ..., p+ 1

Ci = Ci−1, i = 2, ..., p+ 2

temos o que querıamos.Entao, como cada intervalo da decomposicao de [a, b] esta contido em algum Ci e este por sua vez contido em

algum Aλ, temos o nosso resultado.

230

Exercıcio 5.53:

No exercıcio anterior, mostre que os intervalos nos quais se decompos [a, b] podem ser tomados com o mesmocomprimento.

No exercıcio anterior obtivemosC1 = [a, x1)

Ci = [xi−1, xi), i = 2, ..., n,

eCn+1 = [xn, b]

tais que {Ci}ni=1 e uma particao de [a, b] sendo que Ci ⊂ Aλ1 .Temos que xi ∈ Aλi+1. Assim, para cada i = 1, ..., n, existe εi > 0 tal que

(xi − εi, xi + εi) ⊂ Aλi+1.

Tomemos q ∈ Z+ tal queb− aq

< min{εi}ni=1.

Consideremos, agora, os intervalos

Ii = [a+i− 1

q(b− a), a+

i

q(b− a)), i = 1, ..., q.

Assim, temos que, para cada p = 1, ..., q,

(i) a ou xi ≤ a+p− 1

q(b− a) < a+

p

q(b− a) < xi+1 ou b

Ip ⊂ Ci+1 ⊂ Aλi+1

(ii) a+p− 1

q(b− a) < xi < a+

p

q(b− a)

Ip ⊂ (xi − εi, xi + εi) ⊂ Aλi+1.

Concluimos daı que, para p = 1, ..., q, Ip ⊂ Aλ, para algum λ. E temos o nosso resultado.

231

Exercıcio 5.54:

(Teorema de Baire) Se F1, F2, ..., Fn, ... sao fechados com interior vazio entao S = F1∪F2∪ ...∪Fn∪ ... tem interiorvazio. (E possıvel mostrar que, dado arbitrariamente um intervalo aberto I, existe algum x ∈ I ∩ (R− S). Imite ademonstracao do Teorema 6, Capıtulo III, onde se tem pontos em vez dos fechados Fn.)

Seja I um intervalo aberto. Como F tem interior vazio, devemos ter x1 ∈ I − F1. E como F1 e fechado e Iaberto, existe ε1 > 0 tal que (x1 − ε, x1 + ε) ⊂ I ∩ (R − F ). Adiante, J1 = [x1 − ε/2, x1 + ε/2] ⊂ I ∩ (R − F1).Definimos I1 = (x− 1− ε/2, x1 + ε/2).

Da mesma forma podemos obter J2 compacto contido em I1 ∩ (R− F2) e que desta forma J2 ⊂ J1.Por inducao, podemos tomar intervalos compactos Jn ⊂ Jn−1, n ∈ N ∩ [2,∞), tais que Fn ∩ Jn = ∅.Logo, existe, pelo Teorema 12 do Capıtulo V, x ∈

⋂j∈N Jn. E como Jn ∩ Fn = ∅, x ∈ Fn e, consequentemente

x /∈ S.Asim, I * S. E como I e arbitrario, segue daı que S tem interior vazio.

232

Exercıcio 5.55:

O conjunto R−Q dos numeros irracionais nao pode ser expresso como reuniao enumeravel de fechados. Analoga-mente, Q nao e interseccao de uma famılia enumeravel de abertos.

Suponhamos, por absurdo, que existam conjuntos fechados Fn, n ∈ Z+, em R tais que R−Q = ∪n∈Z+Fn.

Como cada Fn esta contido em R−Q e R−Q possui interior vazio, concluımos que Fn tem interior vazio, paracada n ∈ Z+.

Seja (qn)n∈Z+ uma enumeracao do conjunto Q. Como o conjunto {qn}, para todo n ∈ Z+, e fechado e teminterior vazio, temos que R e uma uniao enumeravel de conjuntos fechados com interior vazio pois

R = (R−Q) ∪Q = (∪n∈Z+Fn) ∪ (∪n∈Z+{qn}).

Assim, pelo Teorema de Baire, R e um conjunto de interior vazio (em R). Uma contradicao. Portanto, nao podemexistir fechados Fn, n ∈ Z+, em R tais que R−Q = ∪n∈Z+Fn.

Suponhamos, por absurdo, que existam abertos An, n ∈ Z+, em R tais que Q = ∩n∈Z+An.

Consideremos os fechados Fn := R−An, para todo n ∈ Z+. Desta forma, terıamos que

R−Q = R− ∪n∈Z+An = ∪n∈Z+

(R−An) = ∪n∈Z+Fn.

Ou seja, desta forma R − Q seria uma uniao enumeravel de conjuntos fechados. Uma contradicao. Portanto, naopodem existir abertos An, n ∈ Z+, em R tais que Q = ∩n∈Z+

An.

233

Exercıcio 5.56:

Se [a, b] ⊂ ∪ni=1[ai, bi], entao b− a 6∑ni=1(bi − ai). Tambem [a, b] ⊂ ∪∞n=1[an, bn] implica b− a 6

∑∞n=1(bn − an).

Finalmente, resultados analogos valem para (a, b) em vez de [a, b].

(I) [a, b] ⊂n⋃i=1

[ai, bi]⇒ b− a 6n∑i=1

(bi − ai)

Para todo k ∈ Z+ e i = 1, . . . , n, temos que

[ai, bi] ⊂(ai −

1

2k, bi +

1

2k

).

Logo,

[a, b] ⊂n⋃i=1

[ai, bi] ⊂n⋃i=1

(ai −

1

2k, bi +

1

2k

).

Assim, pela Proposicao 1 deste capıtulo, segue que

b− a <n∑i=1

((bi +

1

2k

)−(ai −

1

2k

))e, consequentemente,

b− a <n∑i=1

(bi − ai +

1

k

)=

n∑i=1

(bi − ai) +n

k.

Portanto,

b− a 6 limk→∞

(n∑i=1

(bi − ai) +n

k

)=

n∑i=1

(bi − ai).

(II) [a, b] ⊂∞⋃i=1

[ai, bi]⇒ b− a 6∞∑i=1

(bi − ai)

Para todo k e i ∈ Z+, temos que

[ai, bi] ⊂(ai −

1

2i+1k, bi +

1

2i+1k

).

Logo,

[a, b] ⊂∞⋃i=1

[ai, bi] ⊂∞⋃i=1

(ai −

1

2i+1k, bi +

1

2i+1k

).

Assim, pela Proposicao 2 deste capıtulo, segue que

b− a <∞∑i=1

((bi +

1

2i+1k

)−(ai −

1

2i+1k

))e, consequentemente,

b− a <∞∑i=1

(bi − ai +

1

2ik

)=

∞∑i=1

(bi − ai) +1

k

∞∑i=1

1

2i=

∞∑i=1

(bi − ai) +1

k.

Portanto,

b− a 6 limk→∞

( ∞∑i=1

(bi − ai) +1

k

)=

∞∑i=1

(bi − ai).

234

(III) (a, b) ⊂n⋃i=1

[ai, bi]⇒ b− a 6n∑i=1

(bi − ai)

Temos que

[a, b] = [a, a] ∪ [b, b] ∪n⋃i=1

[ai, bi].

Assim, pelo item (I), temos que

b− a 6 (a− a) + (b− b) +

n∑i=1

(bi − ai) =

n∑i=1

(bi − ai).

(IV) (a, b) ⊂∞⋃i=1

[ai, bi]⇒ b− a 6n∑i=1

(bi − ai)

Temos que

[a, b] = [a, a] ∪ [b, b] ∪∞⋃i=1

[ai, bi].

Assim, pelo item (I), temos que

b− a 6 (a− a) + (b− b) +

∞∑i=1

(bi − ai) =

∞∑i=1

(bi − ai).

235

Exercıcio 5.57:

Seja X ⊂ R. Uma funcao f : X → R chama-se localmente limitada quando para cada x ∈ X existe um intervaloaberto Ix, contendo x, tal que f |Ix∩X e limitada. Mostre que se X e compacto, toda funcao f : X → R localmentelimitada e limitada.

Sejam X um subconjunto compacto de R, f : X → R uma funcao localmente limitada, para cada x ∈ X, Ixum intervalo contendo x e Ax > 0 tais que

|f(y)| < Ax,

para cada y ∈ X ∩ Ix.Como x ∈ Ix, para todo x ∈ X, temos que

X =⋃x∈X

Ix.

Sendo X um conjunto compacto, existem x1,. . . ,xn em X tais que

X = Ix1 ∪ · · · ∪ Ixn .

Para A ∈ R definido porA := max{Ax1 , . . . , Axn}

temos, para y ∈ Ixi, que

|f(y)| < Axi 6 A.

Ou seja,|f(y)| < A

para todo y ∈ X = ∪ni=1Ixi. Portanto, f e limitada.

236

Exercıcio 5.58:

Dado X ⊂ R nao-compacto, defina uma funcao f : X → R que seja localmente limitada mas nao seja limitada.

Sendo X nao-compacto, basta provarmos que existe uma funcao f : X → R que seja localmente mas nao sejalimitada nos casos:

• X nao e limitado;

Seja f : X → R dada porf(x) = x,

para todo x ∈ X. Desta forma, para cada x ∈ X, o intervalo

Ix = (x− 1, x+ 1)

e tal que f |X∩Ix e limitada poisx− 1 6 f(y) 6 x+ 1,

para todo y ∈ Ix ∩X.Por outro lado, f ainda nao e limitada ja que se existe A > 0 tal que

|f(x)| < A,

para todo x ∈ X, terıamos que|x| < A,

para todo x ∈ X, contradizendo o fato de X ser ilimitado.

• X nao e fechado;

Como X nao e fechado, existe a ∈ X ′ −X. Definimos f : X → R por

f(x) =1

a− x,

para todo x ∈ X.Desta forma, para cada x ∈ X, o intervalo

Ix =

(x− |x− a|

2, x+

|x− a|2

)e tal que f |X∩Ix e limitada. De fato, para y ∈ Ix, temos que

|y − a| = |(x− a) + (y − x)|> |x− a| − |y − x|> |x− a| − |x−a|2

> |x−a|2

e, consequentemente,

|f(y)| = 1

|y − a|6

2

|x− a|.

E f nao e limitada ja que, dado A > 0 existe x ∈ X tal que

|f(x)| > A.

De fato, como a ∈ X ′ −X, existe x ∈ X tal que

|x− a| < 1

A

e, desta forma,

|f(x)| = 1

|x− a|> A.

237

Exercıcio 5.59:

Sejam C compacto, A aberto e C ⊂ A. Mostre que existe ε > 0 tal que x ∈ C, |y − x| < ε⇒ y ∈ A.

Como A e aberto e A ⊃ C, temos que, para todo x ∈ C, existe εx > 0 tal que

(x− 2εx, x+ 2εx) ⊂ A.

A famılia{(x− εx, x+ εx) : x ∈ C}

e uma cobertura aberta para o conjunto C. Como C e compacto, existem x1,. . . ,xn ∈ C tais que

C ⊂n⋃i=1

(xi − εxi, xi + εxi

)

Tomemosε := min{εx1 , . . . , εxn} > 0.

Supondo que x ∈ C e |x− y| < ε temos que y ∈ A. De fato, como x ∈ C, temos que

x ∈ (xk − εxk, xk + εxk

),

para k = 1,. . . ,n− 1 ou n. Assim,

|xk − y| 6 |xk − x|+ |x− y| < εxk+ ε 6 2εxk

e, consequentemente,y ∈ (xk − 2εxk

, xk + 2εxk) ⊂ A.

238

Exercıcio 5.60:

Dada uma sequencia (xn), seja Xn = {xn, xn+1, . . . } para todo n ∈ Z+. Mostre que ∩∞n=1Xn e o conjunto dosvalores de aderencia de (xn).

Seja A o conjunto dos valores de aderencia de (xn).Provaremos separadamente que A ⊂ ∩∞n=1Xn e que ∩∞n=1Xn ⊂ A.Seja a ∈ A. Entao, existe uma subsequencia (xnk

)k∈Z+ de (xn) tal que limk→∞ xnk= a. Dado n ∈ Z+, temos

que existe k0 ∈ Z+ tal quen 6 nk0 < nk0+p,

para todo p ∈ Z+. Assim, (xnk0+p)p∈Z+ e uma sequencia em Xn tal que

limp→∞

xk0+p = limk→∞

xk = a.

Desta forma, a ∈ Xn, para n ∈ Z+ arbitrario. Ou seja, a ∈ ∩∞n=1Xn.Seja a ∈ ∩∞n=1Xn. Provaremos que existe uma subsequencia (xnk

)k∈Z+ de (xn)n∈Z+ tal que limk→∞ xnk= a.

Ou seja, que a ∈ A. Comecemos observando que para todo m e k ∈ Z+ existe nk ∈ Z+ tal que

nk > m e |xnk− a| < 1

k. (5.8)

De fato, como a ∈ Xm+1 e Xm+1 = {xn : n > m + 1}, temos que existe xnk∈ Xm+1 tal que |xnk

− a| < 1k e,

desta forma, nk satisfaz (5.8). Assim, pelo princıpio da definicao recursiva, fica bem definida a sequecia de ındices(nk)k∈Z+ tal que

n1 := 1

e

nk := min

{n ∈ Z+ : n > nk−1, |xn − a| <

1

k

},

para k > 1 em Z+. Logo, a subsequencia (xnk)k∈Z+

e tal que

|xnk− a| 6 1

k,

para todo k ∈ Z+, e, consequentemente,lim

k→+∞xnk

= a.

239

Exercıcio 5.61:

Uma famılia de conjuntos (Kλ)λ∈L chama-se uma cadeia quando, para quaisquer λ e µ ∈ L tem-se Kλ ⊂ Kµ ouKµ ⊂ Kλ. Prove que se (Kλ)λ∈L e uma cadeia nao vazia de compactos nao-vazios entao a intersecao K = ∩λ∈LKλ

e nao vazia (e compacta).

Primeiramente, mostraremos que, dada um conjunto finito L′ contido em L, o conjunto⋂λ∈L′

nao e vazio. Procederemos por inducao no numero de elementos de L′. Para L′ = {µ}, temos a afirmacaotrivialmente ja que ⋂

λ∈L′Kλ = Kµ 6= ∅.

Suponhamos, como hipoteses de inducao, que, para cada conjunto finito L′′ de cardinalidade menor ou igual quen ∈ Z+, seja verdade que ⋂

λ∈L′′Kλ 6= ∅.

Sejam L′ um subconjunto de L com cardinalidade n+ 1 e λ0 ∈ L′. Definimos s conjuntos

L′− := {λ ∈ L′\{λ0} : Kλ ⊂ Kλ0}

eL′+ := {λ ∈ L′\{λ0} : Kλ0

⊂ Kλ}.Temos que

Kλ0∩(⋂

λ∈L′+Kλ

)=

⋂λ∈L′+

(Kλ ∩Kλ0) =

⋂λ∈L′+

Kλ0

= Kλ0

e, como L′− tem cardinalidade menor ou igual que n,

Kλ0∩(⋂

λ∈L′−Kλ

)=

⋂λ∈L′−

(Kλ ∩Kλ0)

=⋂λ∈L′−

6= ∅.

Assim, ⋂λ∈L′ Kλ =

(Kλ0∩(⋂

λ∈L′+Kλ

))∩(⋂

λ∈L′−Kλ

)= Kλ0

∩(⋂

λ∈L′−Kλ

)6= ∅.

Portanto, provamos a afirmacao do inıcio do paragrafo.Fixemos λ0 ∈ L.Suponhamos, por absurdo, que

∅ =⋂λ∈L

Kµ = Kλ0 ∩

(⋂λ∈L

).

Desta forma terıamos que

Kλ0 ⊂⋃λ∈L

(R\Kλ),

ou seja, {R\Kλ}λ∈L seria uma cobertura de Kλ0por conjuntos abertos em R. Como Kλ0

e compacto, existiria umsubconjunto finito L′ de L tal que

Kλ0⊂⋃λ∈L′

(R\Kλ).

Porem, isso implicaria que ⋂λ∈L′∪{λ0}

Kλ = Kλ0∩

( ⋂λ∈L′

)= ∅.

Contradizendo, ja que L′ ∪ {λ0} e finito, o que foi provado no primeiro paragrafo desta demonstracao.

240

Exercıcio 5.62:

Se X ⊂ R e nao-enumeravel, entao X ′ tambem o e.

Todos os pontos de X\X ′ sao isolados. De fato, dado x ∈ X\X ′, temos que x nao e um ponto de acumulacaode X e, consequentemente, existe ε > 0 tal que X ∩ (x− ε, x+ ε) = {x}.

Como todos os pontos de X\X ′ sao isolados, temos, pelo corolario 2 do Teorema 8, que este conjunto eenumeravel.

O conjunto X ∩X ′ nao e enumeravel. De fato, como X = (X\X ′) ∪ (X ∩X ′), X nao e enumeravel e X\X ′ eenumeravel, devemos ter que X ∩X ′ e nao enumeravel.

Por fim, como X ′ contem o conjunto nao enumeravel X ∩X ′, devemos ter que X ′ e nao enumeravel.

241

Exercıcio 5.63:

Para todo X ⊂ R, X −X ′ e enumeravel.

Todos os pontos de X−X ′ sao isolados. De fato, dado x ∈ X−X ′, temos que x nao e um ponto de acumulacaode X e, consequentemente, existe ε > 0 tal que X ∩ (x− ε, x+ ε) = {x}.

Como todos os pontos de X − X ′ sao isolados, temos, pelo corolario 2 do Teorema 8, que este conjunto eenumeravel.

Por fim, como X = X ∪X ′, temos que X −X ′ = X −X ′. Logo, X −X ′ e enumeravel.

242

Exercıcio 5.64:

Um numero real a chama-se ponto de condensacao de um conjunto X ⊂ R quando todo intervalo aberto de centroa contem uma infinidade nao-enumeravel de pontos de X. Seja F0 o conjunto dos pontos de condensacao de umconjunto F ⊂ R. Prove que F0 e um conjunto perfeito (isto e, fechado, sem pontos isolados) e que F − F0 eenumeravel. Conclua daı o Teorema de Bendixon: todo fechado da reta e reuniao de um conjunto perfeito com umconjunto enumeravel.

(I) F0 e fechado:

Mostraremos que todo a ∈ F0 e um ponto de condensacao de F . Isto e, que, para um ε > 0 arbitrario,(a− ε, a+ ε) ∩ F nao e enumeravel.

De fato, sendo a um elemento do feixo de F0, devemos ter que existe x ∈ (a− ε, a+ ε)∩ F0. Assim, para ε > 0tal que

(a− ε, a+ ε) ⊃ (x− ε, x+ ε),

temos que(a− ε, a+ ε) ∩ F ⊃ (x− ε, x+ ε) ∩ F.

Como x e um ponto de condensacao de F , (x − ε, x + ε) ∩ F nao e enumeravel. Logo, (a − ε, a + ε) ∩ F tambemnao e enumeravel.

(II) Se I e um intervalo finito tal que F0 ∩ I = ∅ entao F ∩ I e finito ou infinito enumeravel

Suponhamos que F ∩ I infinito e nao e enumeravel. Provaremos que F0 ∩ I 6= ∅.Definiremos uma sequencia de intervalos compactos ([an, bn])n∈Z+

contidos em I tais que, para todo n ∈ Z+,

[an, bn] ∩ F e infinito e nao e enumeravel, (5.9)

[an, bn] ⊃ [an+1, bn+1] (5.10)

e

bn − an =b− a2n−1

. (5.11)

Verificaremos que a sequencia ([an, bn])n∈Z+fica bem definida por

[a1, b1] := I

e

[an, bn] :=

[an−1,

bn−1−an−1

2

], se

[an−1,

bn−1−an−1

2

]∩ F

e infinito e nao e enumeravel;[bn−1−an−1

2 , bn−1

], caso contrario,

(5.12)

para n > 1, e possui as propriedades (5.9), (5.10) e (5.11). O intervalo [a1, b1] = I, satisfaz (5.9) e (5.11) pelashipoteses sobre I. Suponhamos que os intervalos [a1, b1], . . . , [an−2, bn−2] e [an−1, bn−1] estejam bem definidos por(5.12) e satisfazem as condicoes (5.9), (5.10) e (5.11). Como [an−1, bn−1] ∩ F e infinito e nao e enumeravel, temosque [an−1, (bn−1 − an−1)/2] ou [(bn−1 − an−1)/2, bn−1] sao infinitos e nao sao enumeraveis. Desta forma, (5.12)define [an, bn] de forma que (5.9) e (5.10) sao prontamente satisfeitos. Tambem temos que [an, bn] satisfaz (5.11)pois

bn − an =bn−1 − an−1

2=b− a2n−1

.

Assim, temos, pelo Principio da Definicao Indutiva, que existe uma sequencia de intervalos compactos ([an, bn])n∈Z+

contidos em I que satisfaz (5.9), (5.10) e (5.11).Pelo Teorema 12 do Capıtulo 5, a propriedade (5.10) da sequencia de compactos ([an, bn])n∈Z+

implica queexiste x0 ∈ ∩∞n=1[an, bn]. Mostraremos que x0 ∈ F0.

Seja ε > 0. Para algum n ∈ Z+, temos que

ε >b− a2n−1

.

243

Como x0 ∈ [an, bn], segue que(x0 − ε, x0 + ε) ⊃ [an, bn].

Logo, (x0 − ε, x0 + ε) ∩ F e infinito e nao enumeravel ja que

(x0 − ε, x0 + ε) ∩ F ⊃ [an, bn] ∩ F

e [an, bn] ∩ F e infinito e nao e enumeravel por (5.9). Com isso, devemos ter que x0 ∈ F0.Portanto, x0 ∈ I ∩ F0 = [a1, b1] ∩ F0.

(III) F0 nao possui pontos isolados

Suponhamos que exista x0 em F0 e ε > 0 tais que (x0−2ε, x0+2ε)∩F0 = {x0}. Provaremos que (x0−ε, x0+ε)∩Fe finito ou infinito enumeravel.

Para todo n ∈ Z+, a inclusao(x0 − ε, x0 −

ε

n+ 1

)=

[x0 − ε, x0 −

ε

n+ 1

]⊂ (x0 − 2ε, x0)

implica que (x0 − ε, x0 −

ε

n+ 1

)∩ F0 = ∅.

Assim, pelo item (II), temos que (x0 − ε, x0 −

ε

n+ 1

)∩ F

e finito ou infinito enumeravel.Analogamente, (

x0 +ε

n+ 1, x0 + ε

)∩ F

e finito ou infinito enumeravel, para todo n ∈ Z+.Segue que

(x0 − ε, x0 + ε) ∩ F ==((x0 − ε, x0) ∩ F

)∪ ({x0} ∩ F ) ∪

((x0, x0 + ε) ∩ F

)=(∪n∈Z+

(x0 − ε, x0 − ε

n+1

)∩ F

)∪ ({x0} ∩ F )

∪(∪n∈Z+

(x0 + ε

n+1 , x0 + ε)∩ F

)e finito ou infinito enumeravel ja que

∪n∈Z+

((x0 − ε, x0 −

ε

n+ 1

)∩ F

)e

∪n∈Z+

((x0 +

ε

n+ 1, x0 + ε

)∩ F

)sao unioes enumeraveis de conjuntos finitos ou infinitos enumeraveis.

(IV) F − F0 e enumeravel

Para cada x ∈ F − F0, o conjunto

{ε ∈ R; 0 < ε < 1 e (x− ε, x+ ε) ∩ F e finito ou infinito enumeravel}

nao e vazio e e limitado. Logo, podemos definir, para cada x ∈ F − F0,

εx := sup{ε ∈ R; 0 < ε < 1 e (x− ε, x+ ε) ∩ F e finito ou infinito enumeravel}.

Desta forma,(x− εx, x+ εx) ∩ F

244

e finito ou infinito enumeravel.Seja E um subconjunto denso e finito ou infinito enumeravel de F−F0 (cuja existencia e garantida pelo Teorema

6 do Capıtulo 5). Provaremos queF − F0 ⊂ ∪e∈E(e− εe, e+ εe) ∩ F.

E, deste fato, concluiremos que F − F0 e finito ou infinito enumeravel ja que cada (e− εe, e+ εe)∩ F , para e ∈ E,e finito ou infinito enumeravel e E e finito ou infinito enumeravel.

Seja x ∈ F − F0. Como E e denso em F − F0, existe e ∈ (x− εx/2, x+ εx/2). Desta forma

(e− εx/2, e+ εx/2) ∩ F ⊂ (x− εx, x+ εx) ∩ F

e finito ou infinito enumeravel. Logo, pela definicao de εe, temos que εe > εx/2. Assim,

x ∈ (e− εx/2, e+ εx/2) ⊂ (e− εe, e+ εe).

Portanto, concluimos que F − F0 ⊂ ∪e∈E(e− εe, e+ εe) ∩ F .

(V) Teorema da Bendixon: Todo fechado de R e uma uniao de um conjunto perfeito e um conjunto enumeravel.

Seja F um conjunto fechado. Denotemos por F0 o conjunto dos seus pontos de condensacao.Todo ponto de condensacao de F e um ponto de acumulacao de F . De fato, para todo x ∈ F0, temos que

(x− ε, x+ ε)∩F e infinito e enumeravel, para todo ε > 0. Logo, para todo x ∈ F0, temos que (x− varepsilon, x+ε) ∩ (F − {x}) 6= ∅, para todo ε > 0. Assim, todo ponto de F0 e um ponto de acumulacao de F . Isto e, F0 ⊂ F ′.

ComoF0 ⊂ F ′ ⊂ F,

Temos que F = F0 ∪ (F −F0). Assim, pelos intens (I), (III) e (IV), temos que F e a uniao do conjunto perfeito F0

e o conjunto finito ou infinito enumeravel F − F0.

245

Capıtulo 6

Limites de Funcoes

246

Exercıcio 6.01:

Na definicao do limx→a

f(x), retire a exigencia de ser x 6= a. Mostre que esta nova definicao coincide com a anterior

no caso a /∈ X mas, para a ∈ X, o novo limite existe se, e somente se, o antigo existe e e igual a f(a).

Seja L = limx→a

f(x) pela definicao antiga.

• a /∈ XDado ε > 0, existe δ > 0 tal que 0 < |x − a| < δ, x ∈ X, implica |f(x) − L| < ε. Entao, como a ∈ X, se|x− a| < δ, x ∈ X, entao |f(x)− L| < ε. Portanto, ainda temos L = lim

x→af(x).

• a ∈ X e f(a) 6= L.

Tomando ε = |L− f(a)| > 0 temos que para todo δ > 0 existe x ∈ X tal que |x− a| < δ e |f(x)− L| ≥ ε ( asaber x = a). Portanto, lim f(x) nao mais existe.

• a ∈ X e f(a) = L

Temos que dado ε > 0 existe δ > 0 tal que 0 < |x− a| < δ, x ∈ X, implica |f(x)− L| < ε. Mas, alem disso,|f(a) − L| = 0 < ε. Assim, para todo x ∈ X tal que |x − a| < δ temos que |f(x) − L| < ε. Portanto, aindatemos lim f(x) = L.

Por fim, se limx→a

f(x) = L pela definicao nova, entao f e L satisfazem tambem as condicoes da definicao antiga.

Logo, limx→a

f(x) = L tambem pela definicao antiga.

247

Exercıcio 6.02:

Considere o seguinte erro tipografico na definicao de limite:

∀ε > 0∃δ > 0;x ∈ X, |x− a| < ε⇒ |f(x)− L| < δ.

Mostre que f cumpre esta condicao se, e somente se, e limitada em qualquer intervalo limitado de centro a. Nocaso afirmativo, L pode ser qualquer numero real.

(⇒) Seja I um intervalo de comprimento ε e centrado em a. Entao,

(a− ε, a+ ε) ⊃ I

e, pela hipotese, existe δ > 0 tal que|f(x)| − |L| ≤ |f(x)− L| < δ,

para todo x ∈ (a− ε, a+ ε). Logo, para todo x ∈ I temos que

|f(x)| < δ + |L|

e, consequentemente, f e limitada em I.(⇐) Seja ε > 0. Existe A ∈ R tal que para todo x ∈ (a − ε, a + ε) temos que |f(x)| < A. Daı temos que para

todo x ∈ R tal que |x− a| < ε temos

|f(x)− L| ≤ |f(x)|+ |L| < A+ |L|.

Assim, tomando δ = A+ |L| teremos a condicao que querıamos. E como ε e arbitrario a afirmacao esta provada.

248

Exercıcio 6.03:

Seja X = Y ∪Z, com a ∈ Y ′ ∩Z ′. Dada f : X → R, tomemos g = f |Y e h = f |Z. Se limx→a

g(x) = L e limx→a

h(x) = L

entao limx→a

f(x) = L.

Seja ε > 0 dado. Entao existem δ1 e δ2 positivos tais que se

x ∈ (a− δ1, a+ δ1) ∩ Y

oux ∈ (a− δ2, a+ δ2) ∩ Z

implicam|f(x)− L| < ε.

Fixemos δ = min{δ1, δ2}.Seja

x ∈ (a− δ, a+ δ) ∩X.

Temos que x ∈ Y ou x ∈ Z. No primeiro caso

x ∈ (a− δ, a+ δ) ∩ Y ⊂ (a− δ1, a+ δ1) ∩ Y.

Isso implica que|f(x)− L| < ε.

Analogamente, no segundo caso, tambem temos que

|f(x)− L| < ε.

Concluımos, assim, que em todo caso

|x− a| < δ, x ∈ X ⇒ |f(x)− L| < ε.

Como ε e arbitrario, entao temos o resultado.

249

Exercıcio 6.04:

Seja f : R\ → R definida por f(x) =1

1 + e1/x. Entao lim

x→0+f(x) = 0 e lim

x→0−f(x) = 1.

Seja f : R \ {0} → R \ {0} dada por f1(x) =1

x. Temos entao que

limx→0+

f1(x) = +∞ e limx→0−

f1(x) = −∞.

Seja f2 : R→ R \ {0} dada por f2(y) = 1 + ey. Entao,

limy→+∞

f2(y) = +∞ e limy→−∞

f2(y) = 1.

Por fim, seja f3 : R\ → R dada por f3(z) =1

z. Entao

limz→+∞

f3(z) = 0 e limz→1

f3(z) = 1.

Pelo Teorema 9, temos quelimx→0+

f(x) = limx→0+

(f3 ◦ f2 ◦ f1)(x) = 0

elimx→0−

f(x) = limx→0−

(f3 ◦ f2 ◦ f1)(x) = 1.

250

Exercıcio 6.05:

Seja f(x) = x + 10 sinx para todo x ∈ R. Entao limx→+∞

f(x) = +∞ e limx→−∞

f(x) = −∞. Prove o mesmo para a

funcao g(x) = x+x

2sinx.

Da relacao−1 ≤ sinx ≤ 1,

para todo x ∈ R, segue a relacaox

2= x− x

2≤ g(x) ≤ x+

x

2=

3x

2,

para todo x ∈ R.Seja A ∈ R arbitrario. Tomemos B = 2A e C =

2

3A. Se x ∈ R e x > B = 2A, temos que

g(x) ≥ x

2> A.

E como B so depende de A, segue quelim

x→+∞g(x) = +∞.

Se x ∈ R e x < C =2

3A, temos que

g(x) ≤ 3

2x < A.

E como C so depende de A, segue quelim

x→−∞g(x) = −∞.

251

Exercıcio 6.06:

Seja f : X → R monotona, com f(X) ⊂ [a, b]. Se f(X) e denso no intervalo [a, b] entao, para cada c ∈ X ′+ ∩X ′−,tem-se lim

x→c−f(x) = lim

x→c+f(x). Se c ∈ X entao este limite e igual a f(c).

Como c ∈ X ′+ ∩X ′− e f : X → R e monotona, pelo Teorema 12, existem os limites

L = limx→c+

f(x) e M = limx→c−

f(x).

Alem disso, como f(X) ⊂ [a, b] e [a, b] e compacto temos que L = limn→∞

f(xn)( para alguma sequencia em (c, b]) e

M = limn→∞

f(yn)( para alguma sequencia em [a, c)) estao em [a, b].

Suponhamos que f e nao-decrescente. Os casos em que f e crescente, decrescente ou nao-crescente sao demon-strados de forma analoga.

Seja x ∈ X. Se x < c entaof(x) ≤M = sup{f(y); y ∈ X e y < c}.

Tambem, f(x) ≤ f(y) para todo y > c e, consequentemente,

f(x) ≤ L = inf{f(y); y ∈ X e y > c}.

Assim, f(x) /∈ (min{L,M},max{L,M}). Se x > c, obtemos de forma analoga que f(x) /∈ (min{L,M},max{L,M}).Se x = c, entao

M ≤ f(x) ≤ L

pois f(y) ≤ f(x) para todo y < c = x e f(x) = f(y) para todo y > c = x ja que f e nao-decrescente.Se c /∈ X, entao

∅ = (min{L,M},max{L,M}) ∩ f(X),

o que implica que L = M pois caso contrario haveria um aberto em [a, b] sem nenhum ponto de f(X).Se c ∈ X, devemos ter

M = f(c) = L

pois caso contrario haveria um aberto(M,f(c)) ou (f(c), L)

nao vazio em [a, b] sem nenhum elemento de f(X) pelo que foi dito acima. Neste caso, pelo Teorema 11, temosque lim

x→cf(x) = f(c).

252

Exercıcio 6.07:

Demonstre o Teorema 2.

Suponhamos que limx→a

f(x) = L. Dado ε > 0, existe δ > 0 tal que se x ∈ X e 0 < |x−a| < δ, entao |f(x)−L| < ε.

Daı segue que se y ∈ Y ⊂ X e 0 < |y − a| < δ, entao |g(y)− L| = |f(x)− L| < ε. Como ε e arbitrario, temos quelimx→a

g(x) = L.

Suponhamos que Y = I ∩X, com I aberto contendo a. Entao, existe δ0 > 0 tal que (a − δ0, a + δ0) ⊂ I. Sejaε > 0 arbitrario. Existe δ1 > 0 tal que y ∈ Y e 0 < |y − a| < δ1 temos que |g(y) − L| < ε. Entao, tomando-seδ = min{δ0, δ1}, temos que se x ∈ X e 0 < |x− a| < δ entao

x ∈ (X \ {a}) ∩ (a− δ, a+ δ) ⊂ Y.

Assim, |f(x)− L| = |g(x)− L| < ε. Concluımos que limx→a

f(x) = L.

253

Exercıcio 6.08:

Sejam f : X → R monotona e a ∈ X ′+. Se existir uma sequencia de pontos xn ∈ X com xn > a, limxn = a elim f(xn) = L, entao lim

x→a+f(x) = L.

Consideremos o caso em que f e nao-decrescente e (xn)n∈N e tal que xn > a para todo n ∈ N, limxn = a elim f(xn) = L.

Dado x ∈ X tal que x > a, devemos terL < f(x).

De fato, se existisse x ∈ X ∩ (a,+∞) tal que f(x) < L existiria uma subsequencia de (f(xn)) que nao tende a L.Por exemplo, seja (xnk

) dada por{xn0 ∈ (a, x) ∩ {xn}n∈Nxnk

∈ (a, xnk−1) ∩ {xn}n>nk−1.

Entao,f(xnk

) ≤ ... ≤ f(xn0) ≤ f(x) < L.

Logo, L nao e limite da sequencia (f(xnk)). Assim, dado ε > 0, tomemos n0 ∈ N tal que |f(xn0

) − L| < ε eδ = xxn0

− a. Daı, se 0 < x− a < δ teremos que a < x < xn0 e, pelo que foi dito acima, L < f(x) ≤ f(xn0). Logo,

|f(x)− L| ≤ |f(xn0)− L| < ε.

Portanto, limx→a+

f(x) = L.

Os casos em que f e crescente, nao-crescente e decrescente sao analogos.

254

Exercıcio 6.09:

Se f : X → R e monotona entao o conjunto dos pontos a ∈ X ′ para os quais nao se tem limx→a−

f(x) = limx→a+

f(x) e

enumeravel.

Seja D o conjunto dos a ∈ X ′− ∩X ′+ tais que

limx→a+

f(x) 6= limx→a−

f(x).

Tais limites existem pelo Teorema 12. Suponhamos que f seja nao-crescente. Pela demonstracao do Teorema 12,para cada a ∈ D, temos que

limx→a−

= sup{f(x);x ∈ X,x < a} (6.1)

elimx→a+

= inf{f(x);x ∈ X,x > a}. (6.2)

Para cada a ∈ D, sejaIa = ( lim

x→a−f(x), lim

x→a+f(x)).

Dados a, b ∈ D tais que a < b temos que existe x ∈ (a, b) ∩X. Assim, por (6.1) e (6.2) temos que

limx→a+

f(x) ≤ f(x) ≤ limx→b−

f(x).

Daı, segue que Ia ∩ Ib = ∅.Do paragrafo anterior concluımos que se a e b ∈ D e a 6= b entao Ia∩ Ib = ∅. Assim, qualquer funcao h : D → Q

tal que h(a) ∈ Ia ∩ Q (tais funcoes existem gracas ao axioma da escolha) e injetiva. Concluımos daı que D eenumeravel.

Os casos em que f e crescente, nao-crescente e decrescente sao analogos.

255

Exercıcio 6.10:

Enuncie e demonstre para funcoes o analogo do Teorema 14 do Capıtulo IV.

(1) Sejam f, g : X → R e a ∈ X ′. Se g e limitada inferiormente e limx→a

f(x) = +∞, entao limx→a

f(x) + g(x) = +∞.

De fato, seja c ∈ R um limitante inferior de g. Seja dado A ∈ R arbitrario. Existe δ > 0 tal que se x ∈(a− δ, a+ δ) ∩ (X \ {a}), entao f(x) > A− c.Assim, se x ∈ (a− δ, a+ δ) ∩ (X \ {a})

f(x) + g(x) > (A− c) = (c) = A.

Segue daı que limx→a

f(x) + g(x) = +∞.

(2) Sejam f, g : X → R e a ∈ X ′. Se existe um limitante inferior c > 0 de g e limx→a

f(x) = +∞, entao limx→a

f(x).g(x) =

+∞.De fato, seja A ∈ R arbitrario. existe δ > 0 tal que se x ∈ (a− δ, a+ δ) ∩ (X \ {a}), entao f(x) > A

c .

Assim, se x ∈ (a− δ, a+ δ) ∩ (X \ {a}), entao

f(x).g(x) > (A

c).(c) = A.

Segue daı que limx→a

f(x).g(x) = +∞.

(3) Sejam f : X → R tal que f(x) > 0 para todo X e a ∈ X ′. Temos neste caso que

limx→a

f(x) = 0⇔ limx→a

1

f(x)= +∞.

De fato,

(⇒) Seja A ∈ R∩ (0,+∞) arbitrario. Existe δ > 0 tal que se x ∈ (a− δ, a+ δ)∩ (X \{a}), entao 0 < f(x) < 1A .

Assim, dado x ∈ (a− δ, a+ δ) ∩ (X \ {a}), entao

A <1

f(x).

Segue daı que limx→a

f(x) = +∞.

(⇐) Seja ε > 0 arbitrario. Existe δ > 0 tal que x ∈ (a − δ, a + δ) ∩ (X \ {a}) implica1

f(x)>

1

ε. Assim, para

todo x ∈ (a− δ, a+ δ) ∩ (X \ {a}), temos que 0 < f(x) < ε. Segue daı que limx→a

f(x) = 0.

(4) Sejam f, g : X → R, tais que f e g s ao funcoes positivas e a ∈ X ′.

(a) Se existe c > 0 tal que f(x) > c para todo x ∈ X e limx→a

g(x) = 0, entao limx→a

f(x)

g(x)= +∞.

(b) Se f e limitada e limx→a

g(x) = +∞, entao limx→a

f(x)

g(x)= 0.

De fato,

(a) Temos que lim1

g(x)= +∞ pelo item (3). Daı o resultado segue aplicando-se o item (2) as funcoes f e

1

g.

256

(b) Seja k > 0 tal que 0 < f(x) < k. Entao,1

f(x)>

1

kpara todo x ∈ X.

Como limx→a

= +∞, temos que limx→a

1

g(x)= 0. Assim,

limx→a

g(x)

f(x)= limx→a

1f(x)

1g(x)

= +∞,

pelo item (a).

Portanto, temos pelo item (3) que

limx→a

f(x)

g(x)= limx→a

1g(x)f(x)

= 0.

257

Exercıcio 6.11:

Dado a > 1, defina f : Q→ R pondo, para cadap

q∈ Q, f

(p

q

)= ap/q. Prove que lim

x→0f(x) = 1. conclua que para

cada b ∈ R existe limx→b

f(x), sendo este limite igual a f(b) se b ∈ Q. Chame este limite de ab. Prove que ab.ab′

= ab+b′

e que b < b′ ⇒ ab < ab′.

Como a > 1, temos pelas propriedades basicas da multiplicacao que f e crescente.Seja n0 ∈ N tal que se n0 > a. Entao, para todo n > n0

n√n > n

√a > 1.

Logo, como limn→ +∞ n√n = 1, temos que lim

n→+∞f(1/n) = 1. Segue do exercıcio 8 que lim

x→0+f(x) = 1. Tambem

temos que

limn→+∞

f(−1/n) = limn→+∞

1

f(1/n)=

1

limn→+∞

f(1/n)=

1

1= 1.

E, novamente pelo resultado do exercıcio 8, temos que limx→0−

f(x) = 1. Concluımos, entao, que limx→0

f(x) = 1.

Seja b ∈ R.

1. b ∈ Q.lim

n→+∞f(b+ 1/n) = lim

n→+∞f(b)f(1/n) = f(b)

limn→+∞

f(b− 1/n) = limn→+∞

f(b)f(−1/n) = f(b).

E do exercıcio 6.8 temos que limx→b

f(x) = f(b).

2. b ∈ R \Q.

Sejam xn =m

ntal que m = max{u ∈ Z;

u

n< b} e yn =

m

ntal que m = min{u ∈ Z;

u

n> b}. Segue que se

xn =m

nentao yn =

m+ 1

n(pois b /∈ Q). As sequencias (xn) e (yn) s ao, respectivamente, n ao-decrescente e

n ao-crescente. Isso implica que as sequencias (f(xn)) e (f(yn)) tambem o sao. Daı segue que limn→∞

f(xn) e

limn→∞

f(yn) existem pois f(xn) < f(yn) para todo n ∈ N, isto e, tais sequencias sao limitadas. Por fim, temosque

1 = lim f

(1

n

)= lim

f(yn)

f(xn)=

lim f(yn)

lim f(xn).

Daı, pelo exercıcio 6.8, temos que

limx→b−

f(x) = limn→∞

f(xn) = limn→∞

f(yn) = limx→b+

f(x).

Portanto, o limite limx→b

f(x) existe e denotamos tal limite por ab.

Passaremos agora a verificacao das duas propriedades.

• ab.ab′ = ab+b′

Sejam (xn) e (yn) sequencias que tendem a b e b′, respectivamente. Entao (xn + yn) tende a b+ b′ e

ab.ab′

= limx→b

f(x). limx→b′

f(x)

= limn→∞

f(xn). limn→∞

f(yn)

= limn→∞

f(xn).f(yn)

= limn→∞

f(xn + yn)

= limx→b+b′

f(x) = ab+b′.

258

• b < b′ ⇒ ab < ab′.

Sejam r1, r2 ∈ Q ∩ (b, b′) tais que r1 < r2. Entao, como f e crescente

ab = inf{f(r); r ∈ Q \ {0} tal que r < b}

eab′

= sup{f(r); r ∈ Q \ {0} tal que r < b}.

Assim,ab ≤ f(r1) < f(r2) ≤ ab

e o resultado segue.

259

Exercıcio 6.12:

Dado a > 1, defina g : R → R, pondo g(x) = ax(veja o exercıcio anterior). Prove que limx→+∞

g(x) = +∞ e

limx→−∞

g(x) = 0.

Seja a = 1 + ε. Dado A > 0 arbitrario, tomemos n0 ∈ N tal que n0ε > A. Assim, dado x > n0 temos, peladesigualdade de Bernoulli, que

A < 1 + n0ε ≤ (1 + ε)n0 = g(n0) < g(x).

Segue daı que limx→+∞

g(x) = +∞.

Dado, novamente, A > 0, tomemos n0 ∈ N tal que n0ε >1

A. Entao, dado x < −n0 temos que

A >1

n0ε+ 1≥ 1

(1 + ε)n0= g(−n0) > g(x) > 0.

Daı segue que limx→−∞

g(x) = 0.

260

Exercıcio 6.13:

Seja p : R → R, um polinomio real. Se o coeficiente do termo de grau mais elevado de p e positivo entaolim

x→+∞p(x) = +∞ e lim

x→−∞p(x) e igual a +∞ ou −∞, conforme o grau de p seja par ou ımpar.

Seja p(x) = anxn + an−1x

n−1 + ... + a1x + a0. Provaremos por inducao em n que limn→∞

p(x) = +∞. O caso

n = 1 e trivial. Suponhamos que o resultado seja verdadeiro para o polinomio de grau n− 1. Seja p(x) o polinomiotomado inicialmente. Temos que

p(x) = x(anxn−1 + ...+ a1) + a0.

Pela hipotese de inducao, temos que limn→+∞

anxn−1 + ...+a1 = +∞, o que nos garante que lim

n→+∞p(x) = +∞ pelos

resultados do exercıcio 6.10.Suponhamos agora n = 2k− 1. Provaremos por inducao sobre k que lim

x→−∞p(x) = −∞. Para k = 1, o resultado

e trivial. Temos que

p(x) = x2[(a2k−1x2(k−1)−1 + ...+ a2) +

a1

x] + a0.

Assim, sabendo que limx→−∞

a1x = 0 e supondo que lim

x→−∞(a2k−1x

2(k−1)−1 + ...+ a2) = −∞, temos novamente pelos

resultados do exercıcio 6.10 que limx→−∞

p(x) = −∞.Suponhamos agora que n = 2k. Entao

p(x) = x(a2kx2k−1 + ...+ a1) + a0.

E, pelo resultado do paragrafo anterior, temos que

limx→−∞

(a2kx2k−1 + ...+ a1) = −∞.

Assim, pelos resultados do exercıcio 6.10 temos que limx→−∞

p(x) = +∞.

261

Exercıcio 6.14:

Determine o conjunto dos valores de aderencia da funcao f : R \ {0} → R, f(x) =sin(1/x)

1 + e1/x, no ponto x = 0.

Temos que0 ≤ e1/x e − 1 ≤ sin(1/x) ≤ 1

para todo x ∈ R \ {0}. Entao,

−1 ≤ sin(1/x)

1 + e1/x≤ 1.

Logo, o conjunto A dos pontos de aderencia de f no ponto 0 e tal que A ⊂ [−1, 1].

Seja λ ∈ [−1, 1]. Tomemos θ = arcsinλ. Entao, definindo xn =1

θ − 2πntemos que lim

n→+∞xn = 0 e lim e1/n = 0,

pois limn→+∞

1

xn= −∞. Assim,

f(xn) =sin(θ − 2πn)

1 + eθ−2πn=

λ

1 + eθ+2πn→ λ

quando n→ +∞. Logo, λ ∈ Λ. E segue que A ⊃ [−1, 1]. Concluımos que A = [−1, 1].

262

Exercıcio 6.15:

Se limx→a f(x) = L entao limx→a |f(x)| = |L|. Se limx→a |f(x)| = |L| entao o conjunto dos valores de aderenciade f no ponto a e {L}, {−L} e {−L,L}.

Consideremos a funcao f : X → R e a ∈ X.Suponhamos que limx→a f(x) = L. Dado ε > 0, existe δ > 0 tal que

|f(x)− L| < ε,

para todo x ∈ (X\{a}) ∩ (a− δ, a+ δ). Desta forma, dado ε > 0, existe δ > 0 tal que

||f(x)| − |L|| 6 |f(x)− L| < ε,

para todo x ∈ (X\{a}) ∩ (a− δ, a+ δ). Logo, limx→a |f(x)| = |L|.Suponhamos que limx→a |f(x)| = |L|. Provaremos que o conjunto A dos pontos de aderencia de f e {−L,L},

{L} ou {−L}. O faremos mostrando que {L,−L} ⊃ A 6= ∅.Seja L′ ∈ A. Entao, existe uma sequencia (xn)n∈Z+

em X tal que limn→∞ f(xn) = L′. Segue que

|L′| = | limn→∞

f(xn)| = limn→∞

|f(xn)| = |L|

ja que limx→a |f(x)| = |L|. Logo, L′ ∈ {L,−L}. Concluimos daı que {L,−L} ⊃ A.Pelo Teorema 13 do Capıtulo VI, temos que L′ e um ponto de aderencia da funcao g : X → R no ponto a se e

somente se para todos ε e δ > 0 vale

(L′ − ε, L′ + ε) ∩ g((X\{a}) ∩ (a− δ, a+ δ)

)6= ∅.

Suponhamos, por absurdo, que L e −L nao sao valores de aderencia de f no ponto a. Segue que existem ε+,ε−, δ+ e δ− > 0 tais que

(L− ε+, L+ ε+) ∩ f((X\{a}) ∩ (a− δ+, a+ δ+)

)= ∅

e(L− ε−, L+ ε−) ∩ f

((X\{a}) ∩ (a− δ−, a+ δ−)

)= ∅.

Como limx→a |f(x)| = |L|, existe

f(x) ∈ (L− ε, L+ ε) ∩ |f |((X\{a}) ∩ (a− δ, a+ δ)

),

com x ∈ (X\{a}) ∩ (a− δ, a+ δ). Desta forma, terıamos que

|f(x)− L| = ||f(x)| − |L|| < ε 6 ε+,

caso |f(x)| = ±f(x) e |L| = ±L, e

|f(x)− (−L)| = ||f(x)| − |L|| < ε 6 ε−,

caso |f(x)| = ±f(x) e |L| = ∓L. Consequentemente, seguiria que

f(x) ∈ (L− ε+, L+ ε+) ∩ f((X\{a}) ∩ (a− δ+, a+ δ+)

),

pois x ∈ (X\{a}) ∩ (a− δ, a+ δ) ⊂ (X\{a}) ∩ (a− δ+, a+ δ+), ou

f(x) ∈ (−L− ε−,−L+ ε−) ∩ f((X\{a}) ∩ (a− δ−, a+ δ−)

),

pois x ∈ (X\{a}) ∩ (a− δ, a + δ) ⊂ (X\{a}) ∩ (a− δ−, a + δ−). Em todos os casos, chegamos a uma contradicaocom as hipoteses de que L e −L nao sao valores de aderencia de f no ponto a. Portanto, L ou −L sao valores deaderencia de f .

263

Exercıcio 6.16:

Dados um numero real a e um conjunto compacto nao-vazio K, obtenha uma funcao f : R→ R tal que o conjuntodos valores de aderencia de f no ponto a seja K.

Consideremos o ponto a ∈ R. Provaremos que para todo conjunto F fechado em R existe uma funcao f : R→ Rtal que o conjunto dos pontos de aderencia de f no ponto a e igual a F .

Seja E = {en}n∈Z+ um subconjunto enumeravel de F tal que E = F (veja o Teorema 6 do Capıtulo V).Para cada numero inteiro positivo n, definimos o conjunto

Cn :=

{x ∈ R : |x− a| = 3n

2k, para algum k ∈ Z+

}.

Se n e m sao inteiros distintos devemos ter que Cn ∩Cm = ∅. De fato, se x ∈ Cn ∩Cm, existiriam k e k′ ∈ Z+ taisque

3n

2k= |x− a| = 3m

2k′

Como 2 e 3 sao numeros inteiros coprimos, temos uma contradicao. Logo, Cn ∩ Cm 6= ∅.Assim, podemos definir uma funcao f : R→ R por

f(x) =

{en, se x ∈ Cn,e1, se x ∈ R\ ∪n∈Z+

Cn.

Para todo δ > 0 e n ∈ Z+ existe k ∈ Z+ tal que 3n/2k < δ e, consequentemente,

en = f

(a+

3n

2k

)∈ f((a− δ, a+ δ)\{a}

).

Desta forma, temos que f(a− δ, a+ δ) = E. Logo, para todo δ > 0,

f((a− δ, a+ δ)\{a}

)= E = F.

Pelo Colorario 3 do Teorema 13 do Capıtulo VI, o conjunto dos valores de aderencia de f no ponto a e⋂δ>0

f((a− δ, a+ δ)\{a}

)=⋂δ>0

E = E = F.

264

Exercıcio 6.17:

Seja f : R→ R definida por f(x) = x se x e irracional, f(pq ) = q se pq e uma fracao irredutıvel com p > 0, f(0) = 0.

Mostre que f e ilimitada em qualquer intervalo nao-degenerado.

Suponhamos que I seja um intervalo nao degenerado com a = inf(I) e b = sup(I).Mostraremos que, dado A > 0, arbitrario existe x ∈ (a, b) ⊂ I tal que f(x) > A.Sejam n ∈ Z+, tal que

2n > A e2

b− a,

ep := inf

{m ∈ Z; a <

m

2n

}.

Desta forma temos quep− 1

2n6 a <

p

2n

ep+ 1

2n=p− 1

2n+

2

2n< a+ (b− a) = b.

Logo,

a <p

2n<

2p+ 1

2n+1<p+ 1

2n< b.

Assim, x = 2p+12n+1 pertence a I e

f(x) = 2n+1 > 2n > A.

265

Exercıcio 6.18:

Sejam X, Y ⊂ R, f : X → R, g : Y → R com f(X) ⊂ Y . Se, para a ∈ X ′ e b ∈ Y ′ tem-se limx→a

f(x) = b, limy→b

g(y) = c

e, alem disso, f(x) 6= b para todo x ∈ X\{a}, entao limx→a

g(f(x)) = c. Mostre que a condicao b ∈ Y ′ decorre de ser

f(x) 6= b para x 6= a.

Primeiramente, mostraremos que se f(x) 6= b para x 6= a entao b ∈ Y ′. De fato, se ε > 0, temos que existe,pois limx→af(x) = b, δ > 0 tal que |f(x)− b| < ε para todo x ∈ (a− δ, a+ δ)∩ (X\{a}). Como a ∈ X ′, segue que(a− δ, a+ δ) ∩ (X\{a}) 6= ∅. Logo, existe x ∈ (a− δ, a+ δ) ∩ (X\{a}), tal que

f(x) ∈ (b− ε, b+ ε) ∩ (Y \{b}).

Portanto, devemos ter que b ∈ Y ′.Provaremos, agora, que lim

x→ag ◦ f(x) = c. Seja ε > 0. Como lim

y→bg(y) = c, existe ν > 0 tal que

|g(y)− c| < ε,

para todo y ∈ Y \{b} que satifaz a desigualdade

|y − b| < ν.

Tambem existe δ > 0, pois limx→a

f(x) = b, tal que

|f(x)− b| < ν,

para todo x ∈ X\{a} que satisfaz a desigualdade

|x− a| < δ.

Assim, para todo x ∈ X\{a} tal que|x− a| < δ,

temos que|g ◦ f(x)− c| < ε.

Portanto, segue que limx→a

g ◦ f(x) = c.

266

Exercıcio 6.19:

Para todo numero real x indiquemos, com [x] o maior inteiro 6 x. Mostre que se a e b sao positivos entao

limx→0+

x

a

[b

x

]=b

ae lim

x→0+

b

x

[xa

]= 0.

Prove tambem que, no primeiro caso o limite a esquerda seria o mesmo mas no segundo caso o limite e +∞ quandox→ 0 por valores negativos.

• limx→0+

x

a

[b

x

]= limx→0−

x

a

[b

x

]=b

a

Pela definicao de [·], temos, para todo x ∈ R, que

b

x− 1 <

[b

x

]6b

x.

Assim, segue queb

a− x

a=x

a

(b

x− 1

)<x

a

[b

x

]6x

a

b

x=b

a,

para x > 0, eb

a=x

a

b

x6x

a

[b

x

]<x

a

(b

x− 1

)=b

a− x

a,

para x < 0. Logo, ∣∣∣∣xa[b

x

]− b

a

∣∣∣∣ < ∣∣∣xa ∣∣∣ ,para todo x ∈ R\{0}.

Dado ε > 0 e tomando-se δ = εa, para todo x ∈ R\{0} tal que

|x| < δ,

temos que ∣∣∣∣xa[b

x

]− b

a

∣∣∣∣ < ∣∣∣xa ∣∣∣ < εa

a= ε.

Portanto,

limx→0

x

a

[b

x

]=b

a

e, consequentemente,

limx→0+

x

a

[b

x

]= limx→0−

x

a

[b

x

]=b

a.

• limx→0+

b

x

[xa

]= 0 e lim

x→0−

b

x

[xa

]= +∞

Consideremos a funcao f : R\{0} → R dada, no ponto x ∈ R\{0}, por

f(x) =b

x

[xa

].

Se x ∈ (0, a), temos que 0 < x/a < 1 e, consequentemente, [x/a] = 0. Desta forma, para todo x ∈ (0, a),

f(x) =b

x

[xa

]=b

x0 = 0.

Por outro lado, se x ∈ (−a, 0), temos que −1 < x/a < 0 e, consequentemente, [x/a] = −1. Assim, para todox ∈ (−a, 0),

f(x) =b

x

[xa

]= − b

x.

267

Portanto, temos que

limx→0+

b

x

[xa

]= limx→0+

f |(0,a)(x) = limx→0+

0 = 0.

e

limx→0−

b

x

[xa

]= limx→0−

f |(−a,0)(x) = limx→0+

− bx

= +∞.

268

Exercıcio 6.20:

Dadas funcoes f e g : X → R, defina h = max{f, g} : X → R pondo

h(x) =

{f(x), se f(x) > g(x),g(x), se g(x) > f(x).

Seja a ∈ X ′. Prove que se limx→a

f(x) = L e limx→a

g(x) = M entao limx→a

h(x) = N , onde N e o maior dos dois numeros

L e M .

Suponhamos que L > M . Como limx→a

f(x) = L e limx→a

g(x) = M , tomando-se ε := (L −M)/2 existe δ > 0 tal

que para todox ∈ (a− δ, a+ δ) ∩ (X\{a})

vale

f(x) ∈ (L− ε, L+ ε) =

(L+M

2,

3L−M2

)e

g(x) ∈ (M − ε,M + ε) =

(3M − L

2,L+M

2

).

Assim, para todo x ∈ (a− δ, a+ δ) ∩ (X\{a}),

g(x) < f(x)

e, consequentemente,h(x) = f(x).

Desta forma, temos que h|(a−δ,a+δ)∩(X\{a}) = f |(a−δ,a+δ)∩(X\{a}). Portanto, quando L > M ,

limx→a

h(x) = limx→a

h|(a−δ,a+δ)∩(X\{a})(x) = limx→a

f |(a−δ,a+δ)∩(X\{a})(x) = L = N.

De modo analogo, mostra-se que se M > L entao limx→a

h(x) = N .

Por fim, consideremos o caso em que L = M . Seja ε > 0. Como limx→a

f(x) = L = N e limx→a

g(x) = M = N , existe

δ > 0 tal que para todox ∈ (a− δ, a+ δ) ∩ (X\{a})

valef(x) ∈ (N − ε,N + ε)

eg(x) ∈ (N − ε,N + ε).

Assim, para todo x ∈ (a− δ, a+ δ) ∩ (X\{a}), tem-se que

h(x) ∈ {f(x), g(x)} ⊂ (L− ε, L+ ε).

Desta forma, concluimos quelimx→a

h(x) = N.

269

Exercıcio 6.21:

Sejam f e g : X → R funcoes limitadas numa vizinhanca do ponto a ∈ X ′. Mostre que

limx→a

sup(f + g)(x) 6 limx→a

sup f(x) + limx→a

sup g(x)

e quelimx→a

sup(− f(x)

)= − lim

x→ainf f(x).

Enuncie e prove resultados analogos para lim inf(f + g) e para o produto de duas funcoes.

Pelos exercıcios 3.38 e 3.40, temos que a soma e o produto de funcoes limitadas sao tambem funcoes limitadas.Isto e, f + g e f · g : X → R sao funcoes limitadas.

Para funcoes limitadas f e g : X → R e a ∈ X ′, provaremos que

limx→a

sup(f + g)(x) 6 limx→a

sup f(x) + limx→a

sup g(x),

limx→a

inf(f + g)(x) > limx→a

inf f(x) + limx→a

inf g(x)

elimx→a

sup(− f(x)

)= − lim

x→ainf f(x).

Alem disso, mostraremos que se f e g forem funcoes nao negativas (i.e. f(X) e g(X) ⊂ R+) entao

limx→a

sup(f · g)(x) 6(

limx→a

sup f(x))(

limx→a

sup g(x))

elimx→a

inf(f · g)(x) >(

limx→a

inf f(x))(

limx→a

inf g(x)).

Se f ou g : X → R nao forem nao negativas, podemos ter que

limx→a

sup(f · g)(x) >(

limx→a

sup f(x))(

limx→a

sup g(x)).

De fato, definindo f e g : R→ R por

g(x) = f(x) =

{0, se x ∈ Q,−1, se x /∈ Q,

temos quelimx→0

sup(f · g)(x) = 1 > 0 = 0 · 0 =(

limx→0

sup f(x))(

limx→0

sup g(x)).

Se (xn)n∈Z+e uma sequencia em X entao (f(xn))n∈Z+

e (g(xn))n∈Z+sao sequencias em f(X) e g(X), respec-

tivamente. Como f(X) e g(X) sao compactos (pois sao fechos de conjuntos limitados, veja exercicio 5.30), existemsubsequencias de (f(xn))n∈Z+ e (g(xn))n∈Z+ que sao convergentes. Em particular, existe1 uma subsequencia(xnk

)n∈Z+ de (xn)n∈Z+ tal que (f(xnk))k∈Z+ e (g(xnk

))k∈Z+ sao convergentes.

• limx→a

sup(f + g)(x) 6 limx→a

sup f(x) + limx→a

sup g(x) e limx→a

inf(f + g)(x) > limx→a

inf f(x) + limx→a

inf g(x)

Sejam S ∈ R um valor de aderencia da funcao f + g no ponto a e (xn)n∈Z+ uma sequencia em X tal que

limn→+∞

xn = a e limn→+∞

(f + g)(xn) = S.

Mostraremos quelimx→a

inf f(x) + limx→a

inf g(x) 6 S 6 limx→a

sup f(x) + limx→a

sup g(x).

1Tome uma subsequencia convergente (f(xmp ))p∈Z+de (f(xm))m∈Z+

e extraia dela uma subsequencia convergente (g(xmpk))k∈Z+

de (g(xmp ))p∈Z+. Assim, (xnk )k∈Z+

, para nk := mpk , e como desejado.

270

E, como S ∈ R um valor de aderencia arbitrario da funcao f + g no ponto a, concluiremos que

limx→a

sup(f + g)(x) 6 limx→a

sup f(x) + limx→a

sup g(x)

elimx→a

inf(f + g)(x) > limx→a

inf f(x) + limx→a

inf g(x).

Como (xn)n∈Z+ e uma sequencia em X, segue que existe uma subsequencia (xnk)k∈Z+ de (xn)n∈Z+ tal que

(f(xnk))k∈Z+ e (g(xnk

))k∈Z+ sao convergentes. Segue que limk→+∞ f(xnk) e limk→+∞ g(xnk

) sao pontos deaderencia de f e g, respectivamente, no ponto a = limk→+∞ xnk

. Logo,

S = limk→+∞

(f + g)(xnk)

= limk→+∞

f(xnk) + lim

k→+∞g(xnk

)

6 limx→a

sup f(x) + limx→a

sup g(x)

e, analogamente,S > lim

x→ainf f(x) + lim

x→ainf g(x).

• limx→a

sup(− f(x)

)= − lim

x→ainf f(x)

Sejam VA(f ; a) e VA(−f ; a) os conjuntos dos valores de aderencia no ponto a das funcoes f e −f .Dado L ∈ VA(f ; a), existe uma sequencia (xn)n∈Z+

tal que a = limn→+∞ xn e L = limn→+∞ f(xn). Assim,

−L = − limn→+∞

f(xn) = limn→+∞

(−f)(xn) ∈ VA(−f ; a).

Portanto, como L ∈ VA(f ; a) e arbitrario, concluimos que −VA(f ; a) ⊂ VA(−f ; a).Analogamente, prova-se que −VA(f ; a) ⊃ VA(−f ; a). E, com isso, concluimos que −VA(f ; a) = VA(−f ; a).Portanto,

limx→a

sup(− f(x)

)= supVA(−f ; a)

= sup(− VA(f ; a)

)= − inf VA(f ; a)= − lim

x→ainf f(x).

• limx→0

sup(f · g)(x) 6(

limx→0

sup f(x))(

limx→0

sup g(x))

e limx→a

inf(f · g)(x) >(

limx→a

inf f(x))(

limx→a

inf g(x))

se f e

g > 0

Sejam P ∈ R um valor de aderencia da funcao f · g no ponto a e (xn)n∈Z+ uma sequencia em X tal que

limn→+∞

xn = a e limn→+∞

(f · g)(xn) = S.

Mostraremos que (limx→a

inf f(x))(

limx→a

inf g(x))6 P 6

(limx→a

sup f(x))(

limx→a

sup g(x)).

E, como P ∈ R um valor de aderencia arbitrario da funcao f · g no ponto a, concluiremos que

limx→a

sup(f · g)(x) 6(

limx→a

sup f(x))(

limx→a

sup g(x))

elimx→a

inf(f · g)(x) >(

limx→a

inf f(x))(

limx→a

inf g(x)).

Como (xn)n∈Z+ e uma sequencia em X, segue que existe uma subsequencia (xnk)k∈Z+ de (xn)n∈Z+ tal que

(f(xnk))k∈Z+

e (g(xnk))k∈Z+

sao convergentes. Segue que limk→+∞ f(xnk) e limk→+∞ g(xnk

) sao pontos deaderencia de f e g, respectivamente, no ponto a = limk→+∞ xnk

. Logo,

P = limk→+∞

(f · g)(xnk)

=(

limk→+∞

f(xnk))(

limk→+∞

g(xnk))

6(

limx→a

sup f(x))(

limx→a

sup g(x))

271

e, analogamente,P >

(limx→a

inf f(x))(

limx→a

inf g(x)).

272

Exercıcio 6.22:

Seja f : [0,+∞)→ R uma funcao limitada em cada intervalo limitado. Se

limx→+∞

[f(x+ 1)− f(x)] = L

entao

limx→+∞

f(x)

x= L.

Como f e limitada em todos os intervalos limitados de [0,+∞), para todo C ∈ Z+ existe MC > 0 tal que|f(λ)| < MC , para todo λ ∈ [C − 1, C).

Para C ∈ Z+, λ ∈ [C − 1, C) e k ∈ Z+, temos que∣∣∣ f(λ+k)λ+k − L

∣∣∣ 6∣∣∣ f(λ+k)λ+k −

f(λ+k)k + f(λ)

k

∣∣∣+∣∣∣ f(λ+k)

k − f(λ)k − L

∣∣∣6

∣∣∣ f(λ+k)λ+k −

f(λ+k)k

∣∣∣+∣∣∣ f(λ)k

∣∣∣++∣∣∣ f(λ+k)

k − f(λ)k − L

∣∣∣6

∣∣∣ λλ+k

∣∣∣ ∣∣∣ f(λ+k)k

∣∣∣+∣∣∣ f(λ)k

∣∣∣+∣∣∣ f(λ+k)

k − f(λ)k − L

∣∣∣6

∣∣∣ λλ+k

∣∣∣ (∣∣∣ f(λ+k)k − f(λ)

k − L∣∣∣+∣∣∣ f(λ)k

∣∣∣+ |L|)

+

+∣∣∣ f(λ)k

∣∣∣+∣∣∣ f(λ+k)

k − f(λ)k − L

∣∣∣6 C

k

(∣∣∣ f(λ+k)k − f(λ)

k − L∣∣∣+∣∣∣ f(λ)k

∣∣∣+ |L|)

+

+∣∣∣ f(λ)k

∣∣∣+∣∣∣ f(λ+k)

k − f(λ)k − L

∣∣∣6 C

k |L|+ (1 + Ck ) |f(λ)|

k + (1 + Ck )∣∣∣ f(λ+k)

k − f(λ)k − L

∣∣∣< 1

k

(C|L|+ (1 + C

k )MC

)+ (1 + C

k )∣∣∣ f(λ+k)

k − f(λ)k − L

∣∣∣Seja ε > 0. Provaremos que existem C e n0 ∈ Z tal que para todo k > n0 e λ ∈ [C−1, C) valem as desigualdades

1

k

(C|L|+

(1 +

C

k

)M

)<ε

2e

(1 +

C

k

) ∣∣∣∣f(λ+ k)

k− f(λ)

k− L

∣∣∣∣ < ε

2.

Assim, dado x > (C − 1) + n0, existem λ ∈ [C − 1, C) e k ∈ Z+ tais que x = λ+ k, k > n0 e, consequentemente,∣∣∣ f(x)x − L

∣∣∣ =∣∣∣ f(λ+k)λ+k − L

∣∣∣< 1

k

(C|L|+ (1 + C

k )MC

)+(1 + C

k

) ∣∣∣ f(λ+k)k − f(λ)

k − L∣∣∣

< ε2 + ε

2= ε.

Com isso, conclui-se que limx→+∞f(x)x = L.

Por hipotese, limx→+∞(f(x+ 1)− f(x)

)= L. Fixemos C ∈ Z+ tal que para todo numero real x > C − 1 vale

a desigualdade

|f(x+ 1)− f(x)− L| < ε

3.

Assim, para todo λ ∈ [C − 1, C) e k ∈ Z+, temos que∣∣∣∣f(λ+ k)

k− f(λ)

k− L

∣∣∣∣ =

∣∣∣∣∣k−1∑i=0

(f(λ+ i+ 1)

k− f(λ+ i)

k− L

k

)∣∣∣∣∣6

k−1∑i=0

|f(λ+ i+ 1)− f(λ+ i)− L|k

<

k−1∑i=0

ε/3

k

= ε3 .

273

Como

limk→+∞

1

k= 0

e

limk→+∞

(C|L|+

(1 +

C

k

)MC

)= C|L|+M,

segue que

limk→+∞

1

k

(C|L|+

(1 +

C

k

)MC

)= 0.

Alem disso,

limk→+∞

(1 +

C

k

)= 1.

Fixemos n0 ∈ Z+ tal que para todo k > n0 valem as desigualdades∣∣∣∣1k(C|L|+

(1 +

C

k

)MC

)∣∣∣∣ < ε

2

e

1 +C

k<

3

2.

Por fim, para λ ∈ [C − 1, C) e k > n0, temos que∣∣∣∣1k(C|L|+

(1 +

C

k

)MC

)∣∣∣∣ < ε

2

e (1 +

C

k

) ∣∣∣∣f(λ+ k)

k− f(λ)

k− L

∣∣∣∣ < 3

2

ε

3=ε

2.

Como querıamos demonstrar.

274

Exercıcio 6.23:

Seja f : R→ R definida porf(x) = x+ ax · sen(x).

Mostre que se |a| < 1 entaolim

x→+∞f(x) = +∞ e lim

x→−∞f(x) = −∞.

Suponhamos que 1− |a| > 0.Seja A > 0. Provaremos que

x >A

1− |a|> 0⇒ f(x) > A

e

x < − A

1− |a|< 0⇒ f(x) < −A.

Assim, como A > 0 e arbitrario, podemos concluir que

limx→+∞

f(x) = +∞ e limx→−∞

f(x) = −∞.

Para todo x ∈ R, temos as desigualdades

−|a| = −|a| · 16 −|a|| sen(x)|= −|a · sen(x)|6 a · sen(x).

Assim, para x > A1−|a| > 0,

f(x) = x+ xa · sen(x)> x− x|a|= x(1− |a|)> A

e, para x < − A1−|a| < 0,

f(x) = x+ xa · sen(x)6 x− x|a|= x(1− |a|)< −A.

275

Exercıcio 6.24:

Seja p : R→ R um polinomio nao constante. Dado b ∈ R, suponha que exista uma sequencia (xn), tal que

limn→+∞

p(xn) = b ∈ R.

Prove que (xn) e limitada e o conjunto dos seus pontos de aderencia nao e vazio e esta contido em p−1(b). Emparticular, se existe uma sequencia (xn), tal que lim p(xn) = 0, entao p tem alguma raiz real.

Suponhamos que lim p(xn) = b ∈ R.Pelo exercıcio 6.13, temos que

limx→+∞

p(x) = +∞ ou −∞

elim

x→−∞p(x) = +∞ ou −∞.

Assim, se (xn)n∈Z+ possuisse uma subsequencia (xnk)k∈Z+ tal que

limk→+∞

xnk= +∞ ou −∞

terıamos quelim

k→+∞p(xnk

) = +∞ ou −∞.

Contradizendo o fato de que toda subsequencia de (xn)n∈Z+ converge para b ∈ R. Portanto, a sequencia (xn) elimitada.

Como a sequencia (xn) e limitada, segue que o seu conjunto de pontos de aderencia nao e vazio.Sejam (xnk

)k∈Z+uma subsequencia convergente de (xn)n∈Z+

e a = limk→+∞ xnk. Suponhamos que c0, c1, . . . ,

cm ∈ R sao tais quep(x) = cmx

m + · · ·+ c1x+ c0,

para todo x ∈ R. Assim,

p(a) = cmam + · · ·+ c1a+ c0

= cm

(lim

k→+∞xnk

)m+ · · ·+ c1

(lim

k→+∞xnk

)+ c0

= limk→+∞

(cmx

mnk

+ · · ·+ c1xnk+ c0

)= lim

k→+∞p(xnk

)

= b.

Logo, a ∈ p−1(b). Com isso, concluimos que o conjunto dos pontos de aderencia de (xn) esta contido em p−1(b).Em especial, se b = 0, o conjunto dos pontos de aderencia de (xn) esta contido no conjunto de raızes de p.

Logo, neste caso, conclui-se que p possui raizes reais.

276

Capıtulo 7

Funcoes Contınuas

277

Exercıcio 7.38:

A funcao R f : [0,+∞) → [0,+∞), definida por f(x) = n√x, n > 1, nao e Lipschitziana num intervalo da forma

[0, a], a > 0, embora seja uniformemente continua nestes intervalos. Por outro lado, f e Lipschitziana, comconstante

c :=1

nn√an−1

,

no intervalo [a,+∞). Concluir que f e uniformemente continua em [0,+∞).

Mostraremos, primeiramente, que f : [0, a]→ R nao e Lipschitziana, para todo a > 0. Faremos isso verificandoque o conjunto

L :=

{|f(x)− f(y)||x− y|

: x e y ∈ [0, a]

}e ilimitado superiormente e, consequentemente, nao existe c ∈ R tal que

|f(x)− f(y)| 6 c|x− y|,

para todo x e y ∈ [0, a].Seja A > 0. Para todo z ∈ R tal que

max

{1

n−1√A,

1n√a

}< z

temos que

zn−1 > A e1

zn∈ [0, a].

Assim,

S :=

{1

zn: z > max{1/ n−1

√A, 1/ n

√a}}⊂ [0, a]

e, para todo x = 1zn ∈ S,

|f(x)− f(0)||x− 0|

=|f(1/zn)− f(0)||1/zn − 0|

= zn−1 > A.

Desta forma, temos que L e ilimitado superiormente.Verificaremos, a seguir, que f : [a,+∞)→ R e Lipschitziana com

c :=1

nn√an−1

como constante de Lipschitz.Sejam x e y = x+ λ ∈ [a, 0], λ > 0. Pela Desigualdade de Bernulli

1 + n

(1

nx

)6

(1 +

1

nx

)n,

temos que

n

√1 +

λ

x6 1 +

λ

nx.

Assim,

n

√1 +

λ

x− 1 6

λ

nxe, consequentemente,

n√x+ λ− n

√x 6

λ

nn√xn−1

.

Logo,|f(x)− f(y)| = |f(x+ λ)− f(x)|

= n√x+ λ− n

√x

6 λ

nn√xn−1

6 λ

nn√an−1

6 c|x− y|.

278

Com isso, concluimos que f : [a,+∞)→ R e Lipschitziana com c como constante de Lipschitz.Por fim, provaremos que f : [0,∞)→ R e uniformemente contınua.Seja ε > 0. Para todos x e y ∈ [0,+∞), com y = x+ λ e λ ∈ [0, εn), temos que

|f(x)− f(y)| = |f(x+ λ)− f(x)|= n√x+ λ− n

√x

6 ( n√x+ n√λ)− n

√x

= n√λ

< n√εn

= ε.

Logo, se x e y ∈ [0,+∞) e |x− y| < εn temos que |f(x)− f(y)| < ε. Portanto, f e uniformemente continua.

279

Exercıcio 7.39:

Sejam

Z∗+ :=

{n+

1

n: n ∈ Z+

}e F := Z+ ∪ Z∗+. Defina a funcao f : F → R por

f(x) =

{2, se x ∈ Z+;x, se x ∈ Z∗+.

Mostre que os conjuntos Z+ e Z∗+ sao fechados, que f |Z+ e f |Z∗+ sao funcoes uniformemente contınuas, mas f : F → Rnao e uniformemente contınua.

Primeiramente, verifiquemos que z+ e fechado. Pela construcao de R, temos que

(n− 1, n+ 1) ∩ Z+ = {n},

para todo n ∈ Z+. Desta forma, se (xn)n∈Z+ e uma sequencia convergente (de Cauchy) em Z+ existe n0 ∈ Z+ talque para todos p > n0 vale a desigualdade

|xn0− xp| < 1.

Logo, como(xn0

− 1, xn0+ 1) ∩ Z+ = {xn0

},

devemos ter que xn0 = xp, para todo p > n0. Assim, limn→∞

xn = xn0 ∈ Z+. Portanto, concluimos que o limite de

toda sequencia convergente de elmentos de Z+ concverge para um elemento de Z+. Desta forma, Z+ e fechado.Pela equivalencia

n < m ⇐⇒ n+1

n< m+

1

m,

para todos n e m ∈ Z+, obtemos que os intervalos

Cn :=

(

3

2,

5

2

), se n = 1;(

n+1

n− 1

2, n+

1

n+

1

2

), se n 6= 1.

(

0, 2 +1

2

), se n = 1;(

n− 1 +1

n− 1, n+ 1 +

1

n+ 1

), se n 6= 1.

sao tais que

Cn ∩ Z∗+ =

{n+

1

n

}.

Assim, de modo analogo ao caso de Z+, verifica-se que toda sequencia de elementos de Z∗+ converge para umelemento de Z∗+ e, por isso, Z∗+ e fechado.

Como f |Z+e constante e f |Z∗+ e a funcao identidade, temos que estas funcoes sao uniformemente contınuas.

Provaremos, agora, que f nao e uniformemente contınua verificando que para todo δ > 0 existem x e y ∈ Ftais que |x− y| < δ e |f(x)− f(y)| < 1.

Seja δ > 0. Podemos escolher n ∈ Z+ tal que

1

n< δ e n > 3.

Assim, para x = n+ 1/n e y = n, temos que

|x− y| = 1

ne |f(x)− f(y)| = n− 2 +

1

n> 1.

Portanto, f nao e uniformemente contınua.

280

Exercıcio 7.40:

De um exemplo de dois abertos A e B e uma funcao contınua f : A∪B → R tal que f |A e f |B sejam uniformementecontınuas, mas f nao seja.

Sejam A e B os subconjuntos abertos (−∞, 0) e (0,+∞), respectivamente, em R. Podemos definir uma funcaof : A ∪B → R por

f(x) =

{−1, se x ∈ (−∞, 0) = A;1, se x ∈ (0,+∞) = B.

As restricoes f |A e f |B sao constantes e, por isso, sao uniformemente contınuas.Mostraremos, agora, que f nao e uniformemente contınua verificando que para todo δ > 0 existem x e y ∈ A∪B

tais que|x− y| < δ e |f(x)− f(y)| > 1.

Seja δ > 0. Para x = δ/4 e y = −δ/4, temos que

|x− y| = δ

2< δ e |f(x)− f(y)| = 2 > 1.

Portanto, concluimos que f nao e uniformemente contınua.

281

Exercıcio 7.41:

Toda funcao contınua monotona limitada f : I → R, definida num intervalo I, e uniformemente contınua.

Seja I um intervalo em R.Suponhamos que f : I → R seja crescente, contınua e limitada.Seja ε > 0. Provaremos que existe δ > 0 tal que se x e y ∈ I satisfazem |x− y| < δ entao

|f(x)− f(y)| < ε. (7.1)

Daı, obtemos diretamente que f e uniformemente contınua.Sejam

A := inf f(I) e B := sup f(I).

Se B −A < ε a desigualdade (7.1) se verifica automaticamente pois, neste caso

|f(x)− f(y)| 6 B −A < ε,

para todos x e y ∈ I. Desta forma, basta varificarmos o caso em que B −A > ε.Como

A = inf f(I) < A+ε

36 B − ε

3< sup f(I) = B

e I e um intervalo, temos, pelo Teorema do Valor Intermediario, que f−1(A+ ε3 ) e f−1(B − ε

3 ) sao conjuntos naovazios.

Tomemosc ∈ inf f−1

(A+

ε

3

)e d ∈ sup f−1

(B − ε

3

).

Sejama := inf I e b := sup I.

Como f−1(A+ ε3 ) e f−1(B − ε

3 ) ⊂ I, devemos ter que a =6 c e d 6 b. Por fim, como f e crescente, todo elementode f−1

(A+ ε

3

)e menor que qualquer elemento de f−1

(B − ε

3

). Logo,

a 6 c < d 6 b.

Seja x ∈ [a, c) ∩ I. Como A = inf f(I), devemos ter que f(x) > A. Por outro lado, como f e crescente e x < c,devemos ter que f(x) 6 f(c) = A+ ε

3 . Portanto, f(x) ∈ [A,A+ ε3 ].

De modo analogo, prova-se que se x ∈ (d, b] entao f(x) ∈ [B − ε3 , B].

Como c e d ∈ I, temos que [c, d] e um intervalo compacto contido em I. Sendo f contınua, devemos ter quef |[c,d] tambem e contınua. Assim, como toda funcao contınua com domınio compacto e uniformemente contınua,devemos ter que f[c,d] e uniformemente contınua. Logo, existe δ0 > 0 tal que para quaisquer x e y ∈ [c, d], com|x− y| < δ0, a desigualdade

|f(x)− f(y)| < ε

2

e valida.Por fim, tomemos

δ := min{δ0, d− c}.

Se x e y ∈ I, com x < y, sao tais que |x− y| < δ, pela escolha de δ 6 d− c, devemos ter um dos casos:

• x e y ∈ [a, c) ∩ I;

• x ∈ [a, c) ∩ I e y ∈ [c, d];

• x e y ∈ [c, d];

• x ∈ [c, d] e y ∈ (d, b] ∩ I;

• x e y ∈ (d, b] ∩ I.

Verificaremos, em cada um destes casos, que a desigualdade (7.1) e valida:

282

• Se x e y ∈ [a, c) ∩ I temos que f(x) e f(y) ∈ [A,A+ ε3 ]. Logo, a desigualdade (7.1) e valida;

• Se x ∈ [a, c) ∩ I e y ∈ [c, d] temos que f(x) e f(c) ∈ [A,A+ ε3 ] e |c− y| < δ 6 δ0. Logo,

|f(x)− f(y)| 6 |f(x)− f(c)|+ |f(c)− f(y)| < ε

3+ε

2< ε;

• Se x e y ∈ [c, d] temo imediatamente que |f(x)− f(y)| < ε2 < ε;

• Se x ∈ [c, d] e y ∈ (d, b] ∩ I temos que |d− x| < δ 6 δ0 e f(y) e f(d) ∈ [B − ε3 , B]. Logo,

|f(x)− f(y)| 6 |f(x)− f(d)|+ |f(d)− f(y)| < ε

2+ε

3< ε;

• Se x e y ∈ (d, b] ∩ I temos que f(x) e f(y) ∈ [B − ε3 , B]. Logo, a desigualdade (7.1) e valida.

Suponhamos agora que f : I → R seja decrescente, contınua e limitada. Provaremos que f e uniformementecontınua.

Como f : I → R e decrescente, contınua e limitada, temos que −f : I → R e crescente, contınua e limitada.Pelo que foi demonstrado acima, f e uniformemente contınua.

Seja ε > 0. Como −f e uniformemente contınua, existe δ > 0 tal que, para todos x e y ∈ I com |x− y| < δ, adesigualdade

|(−f)(x)− (−f)(y)| < ε

e valida. Logo, para todos x e y ∈ I com |x− y| < δ, a desigualdade

|f(x)− f(y)| = |(−f)(x)− (−f)(y)| < ε

e valida. Desta forma, concluimos que f e uniformemente contınua.Portanto, toda funcao f : I → R monotona, contınua e limitada e uniformemente contınua.

283

Exercıcio 7.42:

Seja f : X → R uma funcao contınua. Para que f se estenda continuamente a uma funcao ϕ : X → R e necessarioe suficiente que exista lim

x→af(x) para todo a ∈ X ′.

Suponhamos que haja uma funcao contınua ϕ : X → R tal que ϕ|X = f . Entao, para todo a ∈ X ′, existelimx→a

f(x) pois, pelo Teorema 2 do Capıtulo VI,

limx→a

f(x) = limx→a

ϕ|X(x) = limx→a

ϕ(x).

Suponhamos que, para todo a ∈ X ′, exista limx→a

f(x).

Seja ϕ : X → R definida pela igualdadeϕ(a) := lim

x→af(x),

para todo a ∈ X ′. Assim, pelo fato de f ser contınua,

ϕ(a) = limx→a

f(x) = f(a),

para todo a ∈ X. Mostraremos que, para todo a ∈ X ′,

limy→a

ϕ(y) = ϕ(a),

e concluiremos daı que ϕ e uma extensao contınua de f .Fixemos a ∈ X ′ e ε > 0. Como ϕ(a) := lim

x→af(x), existe δ > 0 tal que

|ϕ(a)− f(x)| < ε

2,

para todo x ∈ X tal que |a − x| < δ. Seja y ∈ X ′ tal que |a − y| < δ. Novamente pela definicao de ϕ,ϕ(y) := lim

x→yf(x), existe δ0 > 0 tal que

δ0 < δ − |a− y|

e|ϕ(y)− f(x)| < ε

2,

para todo x ∈ X tal que |y − x| < δ0. Assim, como y ∈ X ′, existe

x0 ∈ (y − δ0, y + δ0) ∩X ⊂ (a− δ, a+ δ) ∩X.

Logo,

|ϕ(a)− ϕ(y)| 6 |ϕ(a)− f(x0)|+ |f(x0)− ϕ(y)| < ε

2+ε

2= ε.

Desta forma, concluimos que|ϕ(a)− ϕ(y)| < ε,

para todo y ∈ X ′ tal que |a− y| < δ.Portanto, para todo a ∈ X ′,

limy→a

ϕ(y) = ϕ(a).

284

Exercıcio 7.43:

Seja f : [a, b]→ R contınua. Dado ε > 0, existem

a = a0 < a1 < · · · < an−1 < an = b

tais que, para cada i = 1,2,. . . ,n,x, y ∈ [ai−1, ai]⇒ |f(x)− f(y)| < ε.

Seja ε > 0. Mostraremos que existem

a = a0 < a1 < · · · < an−1 < an = b

tais que, para cada i = 1,2,. . . ,n,x, y ∈ [ai−1, ai]⇒ |f(x)− f(y)| < ε.

Como o intervalo [a, b] e compacto e f : [a, b]→ R e contınua, temos que f : [a, b]→ R e uniformemente contınua.Assim, podemos escolher um δ > 0 tal que

|f(x)− f(y)| < ε,

para todos x e y ∈ [a, b] tais que |x− y| < δ.A famılia de intervalos abertos {(

p− δ

2, p+

δ

2

)}p∈[a,b]

e uma cobertura aberta para o intervalo compacto [a, b]. Pelo exercıcio 5.52, existe uma particao

a = a0 < a1 < · · · < an−1 < an = b

do intervalo [a, b] tal que para cada subintervalo [ak, ak+1] existe pk ∈ [a, b] tal que

[ak, ak+1] ⊂(pk −

δ

2, pk +

δ

2

).

Por fim, utilizando-se uma particao de [a, b] como acima, dados x e y ∈ [ak, bk] ⊂(pk − δ

2 , pk + δ2

), e valida a

desigualdade|x− y| < δ

e, por isso e pela escolha de δ, temos que|f(x)− f(y)| < ε.

Logo, temos uma particao de [a, b] com a propriedade desejada.

285

Exercıcio 7.44:

Uma funcao contınua ϕ : [a, b]→ R chama-se poligonal quando existe uma particao

a = a0 < a1 < · · · < an = b

tais que ϕ|[ai−1,ai] e um polinomio de grau 6 1, para cada i = 1, . . . , n. Prove que, se f : [a, b] → R e contınua,entao, dado ε > 0, existe uma funcao poligonal ϕ : [a, b]→ R, tal que |f(x)− ϕ(x)| < ε para todo x ∈ [a, b].

Seja ε > 0.Pelo exercıcio 7.43, existe uma particao

a = a0 < a1 < · · · < an = b

tal que

|f(x)− f(y)| < ε

2,

para x e y ∈ [ak, ak+1].Podemos definir uma funcao poligonal ϕ : [a, b]→ R pela equacao

ϕ(x) :=(f(ak+1)− f(ak)

)( x− akak+1 − ak

)+ f(ak),

para cada x ∈ [ak, ak+1]. Por esta definicao e pela escolha da particao de [a, b], temos que

|ϕ(x)− f(ak)| = |f(ak+1)− f(ak)|∣∣∣∣ x− akak+1 − ak

∣∣∣∣ 6 |f(ak+1)− f(ak)| < ε

2,

para cada x ∈ [ak, ak+1].Por fim, dado x ∈ [a, b],

|f(x)− ϕ(x)| 6 |f(x)− f(ak)|+ |ϕ(x)− f(ak)| < ε

2+ε

2= ε,

onde x ∈ [ak, ak+1]. Portanto, ϕ e uma funcao poligonal que satisfaz a condicao do enunciado.

286

Exercıcio 7.45:

Dado ξ : [a, b]→ R, se existem a = a0 < a1 < · · · < an = b tais que ξ|(ai−1,ai) e constante para cada i = 1, 2, . . . ,n,ξ e chamada de funcao escada. Mostre que se f : [a, b]→ R e contınua, entao, para cada ε > 0, existe uma funcaoescada ξ : [a, b]→ R, tal que |f(x)− ξ(x)| < ε qualquer que seja x ∈ [a, b].

Seja ε > 0.Como f e contınua, existe, pelo exercıcio 7.43, uma particao

a = a0 < a1 < · · · < an = b

de [a, b] tal que a desigualdade|f(x)− f(y)| < ε,

e valida sempre que existe k = 1, . . . , n, tal que x e y ∈ [ak, ak+1].Podemos definir uma funcao escada ξ : [a, b]→ R pela igualdade

ξ(x) =

{f(ak), x ∈ [ak, ak+1),f(an), x = an.

Logo, se x ∈ [ak, ak+1) entao|f(x)− ξ(x)| = |f(x)− f(ak)| < ε

pela escolha da particao de [a, b]. Portanto, como |f(an)− ξ(an)| = 0, segue que a desigualdade

|f(x)− ξ(x)| < ε

e valida para todo x ∈ [a, b].

287

Exercıcio 7.46:

Dada uma funcao f : X → R, suponha que para cada ε > 0 se possa obter uma funcao contınua g : X → R, tal que|f(x)− g(x)| < ε qualquer que seja x ∈ X. Entao f e contınua.

Concluiremos que f e contınua mostrando que f e contınua em um ponto arbitrario a ∈ X.Seja ε > 0.Pela hipotese sobre f , existe uma funcao contınua g : X → R tal que

|f(x)− g(x)| < ε

3,

para todo x ∈ X.Como g e contınua, existe δ > 0 tal que

|g(x)− g(a)| < ε

3,

para todo x ∈ X tal que |x− a| < δ.Assim,

|f(x)− f(a)| 6 |f(x)− g(x)|+ |g(x)− g(a)|+ |g(a)− f(a)|< ε

3 + ε3 + ε

3= ε.

para todo x ∈ X tal que |x− a| < δ. Desta forma, concluimos que f e contınua em a.

288

Exercıcio 7.47:

Seja X ⊂ R. Uma funcao f : X → R diz-se semicontınua superiormente no ponto a ∈ X quando, para cada ε > 0dado, pode-se obter δ > 0, tal que se

x ∈ X e |x− a| < δ ⇒ f(x) < f(a) + ε.

Diz-se que f e semicintınua superiormente quando ela o e em todos os pontos de X.

(a) Defina funcao semicontınua inferiormente e mostre que f e contınua num ponto se, e somente se, e semi-contınua superior e inferiormente naquele ponto.

(b) Prove que um subconjunto A ⊂ R e aberto se, e somente se, sua funcao caracterıstica ξA : R → R (definidapor ξA(x) = 1 se x ∈ A e ξA(x) = 0 se x /∈ A) e semicontınua inferiormente.

(c) Enuncie e prove um resultado analogo ao anterior para conjuntos fechados.

(d) Mostre, mais geralmente, que para todo subconjunto X ⊂ R, sua funcao caracterıstica ξX : R → R e de-scontınua precisamente nos pontos da fronteira de X. Dado a ∈ frX, mostre que ξX e semicontınua superior-mente no ponto a se a ∈ X e inferiormente se a /∈ X. Conclua que a funcao f : R→ R, definida por f(x) = 1para x ∈ Q e f(x) = 0 para x irracional, e semicontınua superiormente nos numeros racionais e inferiormentenos numeros irracionais.

(e) Seja f : R→ R definida por f(x) = sen(1/x) se x 6= 0 e f(0) = c. Mostre que f e semicontınua superiormenteno ponto 0 se, e somente se, c > 1. (E inferiormente se, e somente se, c 6 −1.) Tomando −1 < c < 1, mostreque f nao e semicontınua inferiormente ou superiormente no ponto 0.

(f) As funcoes f e g : R → R, onde f(0) = g(0) = 0 e, para x 6= 0, f(x) = x sen(1/x), g(x) = 1/|x|, saosemicontınuas inferiormente, mas seu produto f · g nao e uma funcao semicontınua no ponto 0.

(g) Para que f : X → R seja semicontınua superiormente no ponto a ∈ X ∩ X ′ e necessario e suficiente quelimx→a sup f(x) 6 f(a) (Errata: e necessario mas nao suficinte se f for ilimitada. Contra exemplo: a = 0e f : R → R definida por f(x) = 1/x, em x 6= 0, e f(0) = 0). Equivalentemente: para toda sequencia depontos xn ∈ X com limn→∞ xn = a, que seja limn→∞ sup f(xn) 6 f(a). Vale o resultado analogo parasemicontinuidade inferior.

(h) A soma de duas funcoes semicontınuas superiormente num ponto ainda goza da mesma propriedade. Useo item (e) com c = 1 e c = −1 para dar exemplo de duas funcoes semicontınuas (uma superiormente eoutra inferiormente) cuja soma nao e semicontınua. Mostre que se f e semicontınua superiormente, −f einferiormente.

(i) Sejam f e g : X → R semicontınuas superiormente num ponto. Se f(x) > 0 e g(x) > 0 para todo x ∈ X,entao o produto f · g e uma funcao semicontınua superiormente no mesmo ponto.

(j) Quando X ⊂ R e compacto, toda funcao semicontınua superiormente f : X → R e limitada superiormente eatinge seu valor maximo num ponto de X. Enuncie e prove um fato analogo para semicontinuidade inferior.

(a)

Uma funcao f : X → R diz-se semicontınua inferiormente no ponto a ∈ X quando, para cada ε > 0 dado,pode-se obter δ > 0, tal que se

x ∈ X e |x− a| < δ ⇒ f(a)− ε < f(x).

Diz-se que f e semicintınua inferiormente quando ela o e em todos os pontos de X.Suponhamos que f : X → R seja uma funcao semicontınua inferiormente e superiormente. Provaremos que f e

contınua em um ponto arbitrario a ∈ X e concluiremos daı que f e contınua.Seja ε > 0. Como f e semicontınua inferiormente e superiormente em a, existem δ− e δ+ > 0 tais que

x ∈ X e |x− a| < δ− ⇒ f(a)− ε < f(x)

289

ex ∈ X e |x− a| < δ+ ⇒ f(x) < f(a) + ε.

Assim, para δ := min{δ−, δ+},

x ∈ X e |x− a| < δ ⇒ f(a)− ε < f(x) < f(a) + ε.

Logo, podemos concluir que f e contınua em a ∈ X.Suponhamos, por outro lado, que f e contınua. Provaremos que f e semicontınua inferiormente e superiormente

em um ponto arbitrario a ∈ X e concluiremos daı que f e semicontınua inferiormente e superiormente.Seja ε > 0. Como f e contınua em a, existe δ > 0 tal que

x ∈ X e |x− a| < δ ⇒ f(a)− ε < f(x) < f(a) + ε.

Assim,x ∈ X e |x− a| < δ ⇒ f(a)− ε < f(x)

ex ∈ X e |x− a| < δ ⇒ f(x) < f(a) + ε.

Logo, podemos concluir que f e semicontınua inferiormente e superiormente em a ∈ X.

(b)

Suponhamos que A ⊂ R e aberto. Provaremos que ξA : R → R e semicontınua inferiormente em um pontop ∈ R arbitrario. E conluiremos daı que ξA e semicontınua inferiormente.

Seja ε > 0.Se p ∈ A entao existe δ > 0 tal que

(p− δ, p+ δ) ⊂ A.

Assim, neste caso, se x ∈ R e |x− p| < δ entao

ξA(p)− ε = 1− ε < 1 = ξA(x).

Logo, se p ∈ A entao ξA e semicontınua inferiormente em p.Se p ∈ R\A entao, para todo x ∈ R, temos que

ξA(p)− ε = 0− ε < 0 6 ξA(x).

Logo, se p ∈ R\A entao ξA e semicontınua inferiormente em p.Portanto, ξA e semicontınua inferiormente em p ∈ R.Suponhamos, por outro lado, que ξA seja semicontınua inferiormente. Provaremos que, para um ponto arbitrario

a ∈ A, existe δ > 0 tal que (a− δ, a+ δ) ⊂ A. Desta forma concluiremos que A e aberto.Como ξA e semicontınua inferiormente, tomando-se ε = 1/2, existe δ > 0 tal que, para todo x ∈ (a− δ, a+ δ),

vale1

2= ξA(a)− ε < ξA(x).

Desta forma, ξA(x) 6= 0, para todo x ∈ (a− δ, a+ δ). Logo, (a− δ, a+ δ) ⊂ A.

(c)

Provaremos que F ⊂ R e fechado se, e somente se, sua funcao caracterıstica ξF : R → R e semicontınuasuperiormente.

Suponhamos que F ⊂ R seja fechado. Provaremos que ξF e semicontınua superiormente em um ponto arbitrariop ∈ R e concluiremos daı que ξF e semicontınua superiormente.

Seja ε > 0.Se p ∈ F , para todo x ∈ R, temos que

ξF (x) 6 1 < ξF (p) + ε.

Logo, se p ∈ F entao ξF e semicontınua superiormente em p.

290

Se p ∈ R\F entao existe δ > 0 tal que(p− δ, p+ δ) ⊂ R\F.

Assim, neste caso, se x ∈ R e |x− p| < δ entao

ξF (x) = 0 < 0 + ε = ξA(p) + ε.

Logo, se p ∈ R\F entao ξF e semicontınua inferiormente em p.Portanto, ξF e semicontınua superiormente em p ∈ R.Suponhamos, por outro lado, que ξF seja semicontınua superiormente. Provaremos que, para um ponto ar-

bitrario p ∈ R\F , existe δ > 0 tal que (p− δ, p+ δ) ⊂ R\F . Desta forma, concluiremos que F e fechado.Como ξF e semicontınua inferiormente, tomando-se ε = 1/2, existe δ > 0 tal que, para todo x ∈ (p− δ, p+ δ),

vale

ξF (x) < ξF (p) + ε = 0 +1

2=

1

2.

Desta forma, ξF (x) 6= 1, para todo x ∈ (p− δ, p+ δ). Logo, (p− δ, p+ δ) ⊂ R\F .

(d)

Seja X ⊂ R.Para concluirmos que ξX : R→ R e descontınuo exatamente nos pontos de fronteira de X, verificaremos que ξX

e contınua em int(X) e int(R\X), semicontınua superiormente, mas nao inferiormente, em fr(X)∩X e semicontınuainferiormente, mas nao superiormente, em fr(X) ∩ (R\X).

Seja a ∈ int(X). Como int(X) e aberto, existe δ > 0 tal que (a − δ, a + δ) ⊂ int(X). Assim, para todo x ∈ Rtal que |x− a| < δ, temos que x ∈ int(X) e, consequentemente,

|ξX(x)− ξX(a)| = |1− 1| = 0.

Logo, ξX e contınua em a ∈ int(X) pois, para todo ε > 0,

|ξX(x)− ξX(a)| = 0 < ε

sempre que |x− a| < δ.De modo analogo, verifica-se que ξX : R→ R e contınua em int(R\X).Seja a ∈ (frX) ∩X. Para quaisquer ε e δ positivos, vale, para todo x ∈ (a− δ, a+ δ), a desigualdade

ξX(x) 6 1 < 1 + ε = ξX(a) + ε.

Logo, ξX e semicontınua superiormente em a. Por outro lado, para qualquer δ > 0, existe x ∈ (a−δ, a+δ)∩(R\X).Assim, para ε = 1/2 e qualquer δ > 0, temos que

ξX(a)− ε = 1/2 > 0 = ξX(x),

para todo x ∈ (a− δ, a+ δ) ∩ (R\X). Logo, ξX nao e semicontınua inferiormente.Seja a ∈ (frX) ∩ (R\X). Para quaisquer ε e δ positivos, vale, para todo x ∈ (a− δ, a+ δ), a desigualdade

ξX(a)− ε = 0− ε < 0 6 ξX(x).

Logo, ξX e semicontınua inferiormente em a. Por outro lado, para qualquer δ > 0, existe x ∈ (a − δ, a + δ) ∩X.Assim, para ε = 1/2 e qualquer δ > 0, temos que

ξX(x) = 1 > 1/2 = 0 + 1/2 = ξX(a) + ε.

para todo x ∈ (a− δ, a+ δ) ∩X. Logo, ξX nao e semicontınua superiormente.Por fim, f = ξQ e semicontınua superiormente em

Q = R ∩Q = (frQ) ∩Q

e semicontınua inferiormente emR\Q = R ∩ (R\Q) = (frQ) ∩ (R\Q)

pelo que foi provado anteriormente.

291

(e)

Seja f : R→ R dada por

f(x) =

{sen 1

x , se x 6= 0c, se x = 0.

Como f e uma composicao de funcoes contınuas no aberto R\{0}, temos que f e contınua em R\{0}.Desta forma, podemos concluir que f e semicontınua superiormente (inferiormente) se, e somente se, c > 1

(c 6 −1) mostrando que f e f e semicontınua superiormente (inferiormente) em 0 se, e somente se, c > 1 (c 6 −1).Assim, podemos tambem concluir que f nao e semicontınua superiormente ou inferiormente se −1 < c < 1.

Suponhamos que f seja semicontınua superiormente em 0. Mostraremos, para ε > 0 arbitrario, que 1− ε < c econcluiremos daı que 1 6 c. Existe δ > 0 tal que

f(x) < f(0) + ε = c+ ε,

para todo x ∈ (−δ, δ). Assim, para n ∈ Z+ suficientemente grande, temos que

1π2 + 2nπ

∈ (−δ, δ)

e, consequentemente,

1 = sen(π

2+ 2nπ

)= f

(1

π2 + 2nπ

)< c+ ε.

Consideremos, por outro lado, o caso em que 1 6 c. Seja ε > 0. Para todo x ∈ R\{0}, temos que

f(x) = sen

(1

x

)6 1 < 1 + ε = c+ ε = f(0) + ε.

Logo, para todo x ∈ R, temos quef(x) < f(0) + ε.

Portanto, f e semicontınua superiormente em 0.Suponhamos que f seja semicontınua inferiormente em 0. Mostraremos, para ε > 0 arbitrario, que c < −1 + ε

e concluiremos daı que c 6 −1. Existe δ > 0 tal que

c− ε = f(0)− ε < f(x),

para todo x ∈ (−δ, δ). Assim, para n ∈ Z+ suficientemente grande, temos que

13π2 + 2nπ

∈ (−δ, δ)

e, consequentemente,

c− ε < f

(1

3π2 + 2nπ

)= sen

(3π

2+ 2nπ

)= −1.

Consideremos, por outro lado, o caso em que c 6 −1. Seja ε > 0. Para todo x ∈ R\{0}, temos que

f(0)− ε = c− ε 6 −1− ε < −1 6 sen

(1

x

)= f(x).

Logo, para todo x ∈ R, temos quef(0)− ε < f(x).

Portanto, f e semicontınua inferiormente em 0.

(f)

Sejam f e g : R→ R definidas por

f(x) =

{0, se x = 0,x sen

(1x

), se x 6= 0,

292

e

g(x) =

{0, se x = 0,1|x| , se x 6= 0,

Como f e g sao produtos, composicoes e quocientes de funcoes contınuas em R\{0}, temos que f e g saocontınuas em R\{0}. Logo, f e g sao semicontınuas inferiormente em R\{0}. Desta forma, provando que f e g saosemicontınuas inferiormente em 0 podemos, pelo item (a), concluir que f e g sao semicontınuas inferiormente.

Seja ε > 0. Tomando δ := ε, temos, para todo x ∈ (−δ, δ)\{0}, que∣∣∣∣x sen

(1

x

)∣∣∣∣ = |x|∣∣∣∣sen

(1

x

)∣∣∣∣ 6 |x| < δ = ε

e

−ε < 0 <1

|x|.

Assim,

f(0)− ε = −ε < x sen

(1

x

)= f(x)

e

g(0)− ε = −ε < 1

|x|= g(x),

para todo x ∈ R tal que 0 < |x| < δ. Logo, podemos concluir que f e g sao semicontınuas inferiormente em 0.Provaremos, agora, que f · g nao e semicontınua em 0. Seja δ > 0. Existe n ∈ Z+ tal que

x :=1

(2n+ 1)π< δ.

Assim, x ∈ (−δ, δ)\{0} e

f · g(x) =x

|x|sen

(1

x

)= sen

((2n+ 1)π

)= −1.

Logo, podemos concluir que para todo δ > 0 existe x ∈ R\{0} tal que |x| < δ e

f · g(x) = −1 = 0− 1 = f · g(0)− 1.

Portanto, f · g nao e semicontınua em 0.

(g)

Seja f : X → R uma funcao e a ∈ X ∩X ′.Suponhamos que f seja semicontınua superiormente em a. Provaremos que limx→a sup f(x) 6 f(a).Sejam c ∈ R um valor de aderencia de f em a e (xn)n∈Z+ uma sequencia em X\{a} tal que limn→∞ xn = a e

limn→∞ f(xn) = c. Provaremos, para ε > 0 arbitrario, que

c < f(a) + ε.

Com isso, poderemos concluir que c 6 f(a). Existe δ > 0 tal que

f(x) < f(a) +ε

2

sempre que x ∈ X e tal que |x− a| < δ. Tambem existe n0 ∈ Z+ tal que

|xn0− a| < δ e |f(xn0

)− c| < ε

2.

Assim,

c− ε

2< f(xn0

) < f(a) +ε

2

e, consequentemente,c < f(a) + ε.

293

Concluimos que f(a) e maior ou igual que qualquer valor de aderencia de f em a. Portanto, limx→a sup f(x) 6f(a).

Consideremos, agora, que limx→a sup f(x) 6 f(a). Para provarmos que provaremos que f e semicontınuasuperiormente em a precisamos de mais hipoteses. Por exemplo, se f : R→ R e dada por

f(x) =

{0, se x = 0,1x , se x 6= 0.

entaolimx→0

sup f(x) = −∞ < f(0).

Desta forma, iremos assumir tambem que f e limitada em uma vizinhanca de a.Seja ε > 0. Suponhamos, por absurdo, que nao exista δ > 0 tal que

f(x) < f(a) + ε

sempre que x ∈ X e tal que |x− a| < δ. Entao, para cada n ∈ Z+, exite xn ∈ X\{a} tal que

|xn − a| <1

n

ef(a) + ε 6 f(xn).

Assim, temos uma sequencia (xn)n∈Z+ em X\{a} tal que limn→∞ xn = a. Como f e limitada em uma vizinhancade a, existe uma subsequencia (f(xnk

))k∈Z+de (f(xn))n∈Z+

que converge para um ponto c ∈ R. Logo,

f(a) + ε 6 limk→∞

f(xnk) = c

ja quef(a) + ε 6 f(xnk

),

para todo k ∈ Z+. Por outro lado, como c e um ponto de acumulacao de f em a,

c 6 limx→a

sup f(x) 6 f(a).

Uma contradicao.Portanto, f e semicontınua superiormente no ponto a.De modo analogo, admitindo que f e limitada em uma vizinhanca de a, prova-se que f e semicontınua inferi-

ormente em a se, e somente se,f(a) 6 lim

x→ainf f(x).

(h)

Suponhamos que f e g : X → R sejam duas funcoes semicontınuas superiormente em um ponto a ∈ X.Mostraremos que f + g e semicontınua superiormente em a.

Seja ε > 0. Existem δf e δg > 0 tais que

f(x) < f(a) +ε

2,

sempre que x ∈ X e tal que |x− a| < δf , e

g(x) < g(a) +ε

2,

sempre que x ∈ X e tal que |x − a| < δg. Denotemos min{δf , δg} por δ. Assim, para x ∈ X tal que |x − a| < δ,temos que

(f + g)(x) = f(x) + g(x) <(f(a) +

ε

2

)+(g(a) +

ε

2

)= (f + g)(a) + ε.

Logo, podemos concluir que f + g e semicontınua superiormente em a.Sejam f e g : R→ R dadas por

f(x) =

{sen 1

x , se x 6= 0,−1, se x = 0,

294

e

g(x) =

{sen 1

x , se x 6= 0,1, se x = 0.

Pelo item (e), f e semicontınua inferiormente e g e semicontınua superiormente. Provaremos que f + g nao esemicontınua superiormente nem inferiormente em 0.

Seja δ > 0. Existe n ∈ Z+ tal que ∣∣∣∣ 1

2πn+ π2

∣∣∣∣ < δ.

Assim, para

x :=1

2πn+ π2

temos que|x| < δ

e(f + g)(x) = f(x) + g(x)

= sen 1x + sen 1

x= 2> 0 + 1= f(0) + g(0) + 1.

Desta forma, como δ > 0 e arbitrario, concluımos que f + g nao e semicontınua superiormente em 0.De forma analoga, podemos mostrar que f + g nao e semicontınua inferiormente em 0.Suponhamos que f : X → R sejam semicontınua superiormente. Provaremos que −f e semicontınua inferior-

mente em um ponto arbitrario a ∈ X. E concluiremos daı que −f e semicontınua inferiormente.Seja ε > 0. Existe δ > 0 tal que

f(x) < f(a) + ε,

para todo x ∈ X tal que |x− a| < δ. Assim,

−f(a)− ε < −f(x),

para todo x ∈ X tal que |x− a| < δ. Portanto, como ε > 0 e arbitrario, −f e semicontınua inferiormente em a.

(i)

Suponhamos que f e g : X → R sejam duas funcoes semicontınuas superiormente em um ponto a ∈ X e quef(x) e g(x) > 0, para todo x ∈ X. Mostraremos que f · g e semicontınua superiormente em a.

Seja ε > 0. Como f(a) + g(a) > 0, podemos escolher λ ∈ R tal que

0 < λ < min

{1,

1

f(a) + g(a) + 1

}.

Desta forma,(f(a) + g(a))λ+ λ2 =

((f(a) + g(a)) + λ

<((f(a) + g(a)) + 1

< ε.

Seja δ > 0 tal quef(x) < f(a) + λ

eg(x) < g(a) + λ,

para todo x ∈ X tal que |x| < δ. Logo,

(f · g)(x) = f(x) · g(x)=

(f(a) + λ

)(g(a) + λ

)= (f · g)(a) + (f(a) + g(a))λ+ λ2

= (f · g)(a) + ε,

295

para todo x ∈ X tal que |x| < δ. Portanto, f · g e semicontınua superiormente em a.

(j)

Suponhamos que f : X → R seja uma funcao semicontınua superiormente em um conjunto compacto X ⊂ R.Para cada a ∈ X, existe δ(a) > 0 tal que

f(x) < f(a) + 1,

para todo x ∈ X tal que |x| < δ(a).Como

X ⊂ ∪a∈X(a− δ(a), a+ δ(a)

)e X e compacto, existem a1, . . . , an ∈ X tais que

X ⊂ ∪nk=1

(ak − δ(ak), ak + δ(ak)

).

SejaM := max{f(ak) + 1: k = 1, . . . , n}.

Provaremos que M e uma cota superior para f(X) e concluiremos daı que f e limitada superiormente.Seja x ∈ X. Para algum k ∈ {1, . . . , n}, x ∈

(ak − δ(ak), ak + δ(ak)

)ja que X ⊂ ∪nk=1

(ak − δ(ak), ak + δ(ak)

).

Logo, pela escolha de δ(ak), temos quef(x) < f(ak) + ε 6M.

Portanto, M e uma cota superior para f(X).Seja (xn)n∈Z+ uma sequencia em X tal que

limn→+∞

f(xn) = sup f(X).

Como X e compacto, existe uma subsequencia convergente (xnk)k∈Z+

de (xn)n∈Z+tal que a = limk→+∞ xnk

∈ X.Assim, pelo item (g),

sup f(X) = limn→+∞ f(xn)= limn→+∞ f(xnk

)6 limx→a sup f(x)6 f(a).

Logo, f(a) = sup f(X) e, portanto, f assume seu valor maximo em um ponto de X.De modo analogo, podemos provar que se f : X → R for uma funcao semicontınua inferiormente em um conjunto

compacto X ⊂ R entao f e limitada inferiormente e assume seu valor mınimo em um ponto de X.

296

Capıtulo 8

Derivadas

297

Exercıcio 8.46:

Dadas f e g analıticas no intervalo aberto I, seja X ⊂ I um conjunto que possui um ponto de acumulacao a ∈ I.Se f(x) = g(x), para todo x ∈ I. Em particular, se f(x) = 0, para todo x ∈ X, entao f(x) = 0, para todo x ∈ I.

Sejam f e g : I → R funcoes analıticas definidas no intervalo aberto I e X ⊂ I. Suponhamos que X e tal quef |X = g|X e exista a ∈ X ′ ∩ I.

Pelo exercıcio 8.45, basta provarmos que f (n)(a) = g(n)(a), n ∈ Z+, para concluirmos que f = g. Assim,mostrando, por inducao em n ∈ Z+, que a funcao ϕ := f − g e tal que

0 = ϕ(n)(a) = f (n)(a)− g(n)(a),

para todo n ∈ Z+, teremos o resultado do enunciado.Seja (xk)k∈Z+ uma sequencia em X tal que limk→+∞ xk = a e xk 6= a, para todo k ∈ Z+. Denotemos por

(hk)k∈Z+ definida por hk = xk − a. Segue que

limk→+∞

hk = 0.

Para n = 0, temos queϕ(0)(a) = f(a)− g(a)

= limx→a f(x)− limx→a g(x)= limk→+∞ f(xk)− limk→+∞ g(xk)= limk→+∞

(f(xk)− g(xk)

)= limk→+∞ 0= 0.

Suponhamos que ϕ(m)(a) = 0 para todo 0 6 m < n. Pela Formula da Taylor com Resto de Lagrange (Teorema8.10), temos que existe θk ∈ (0, 1) tal que

0 = ϕ(xk) = ϕ(a+ hk) =

n−1∑i=0

ϕ(i)(a)

i!hik +

ϕ(n)(a+ θkhk)

n!hnk =

ϕ(n)(a+ θkhk)

n!hnk .

Logo,lim

k→+∞θkhk = 0

e, como hk = a− xk 6= 0,ϕ(n)(a+ θkhk) = 0.

Desta forma,ϕ(n)(a) = lim

x→aϕ(n)(x) = lim

k→+∞ϕ(n)(a+ θkhk) = 0.

Portanto, pelo princıpio da inducao finita, ϕ(n)(a) = 0, para todo n ∈ Z+.Em particular, provamos que se f : I → R e uma funcao analıtica definida no intervalo aberto I, X ⊂ I, f |X = 0

e a ∈ X ∩ I entao f = 0 pois 0 e analıtica em I.

298

Exercıcio 8.47:

Seja I = (a − δ, a + δ). Dada f : I → R, de classe C∞, suponha que existam constantes a0, a1, . . . , an, . . . taisque, para todo x ∈ I, se tenha

f(x) =

∞∑n=0

an(x− a)n.

Prove que∑an(x− a)n e a serie de Taylor de f em torno de a, isto e, que

an =f (n)(a)

n!,

para todo n = 0, 1, 2, . . . .

Seja

f(x) = f(a) + f ′(a) · (x− a) +f ′′(a)

2!· (x− a)2 + · · ·+ f (n)(a)

n!· (x− a)n + r(x− a),

para x ∈ I, a n-esima expansao de Taylor de f em torno de a.Provaremos, por inducao em n ∈ Z>0, que

an =f (n)(a)

n!.

Para n = 0, temos da igualdade

f(x) =

∞∑n=0

an(x− a)n

que

f(a) =

∞∑k=0

ak(a− a)k =

∞∑k=0

ak(0)k = a0.

Suponhamos, como hipotese de inducao, que

ak =f (k)(a)

k!,

para todo k ∈ Z>0 ∩ [0, n). Assim,

f(a) + · · ·+ f(n−1)(a)(n−1)! (x− a)n−1 + f(n)(a)

n! (x− a)n + r(x− a)

= f(x)= a0 + · · ·+ an−1(x− a)n−1 + an(x− a)n +

∑∞k=n+1 ak(x− a)k

= f(a) + · · ·+ f(n−1)(a)(n−1)! (x− a)n−1 + an(x− a)n +

∑∞k=n+1 ak(x− a)k

e, consequentemente,

f (n)(a)

n!(x− a)n + r(x− a) = an(x− a)n +

∞∑k=n+1

ak(x− a)k.

Logo, para x ∈ I\{a},f (n)(a)

n!− an = (x− a)−n

∞∑k=n+1

ak(x− a)k − r(x− a)

(x− a)n.

Por fim,f(n)(a)n! − an = limx→a

((x− a)−n

∑∞k=n+1 ak(x− a)k − r(x−a)

(x−a)n

)= limx→a

((x− a)−n

∑∞k=n+1 ak(x− a)k

)− limx→a

(r(x−a)(x−a)n

)= 0 + 0= 0.

Portanto, o resultado segue pelo PIF.

299

Exercıcio 8.48:

Seja f(x) = x5

1+x6 . Calcule as derivadas de ordem 2001 e 2003 da funcao f : R→ R no ponto 0.

Para y ∈ (−1, 1) temos, pelo Exemplo 24 do Capıtulo IV, temos que

1

1− y=

∞∑k=0

yk.

Assim, para todo x ∈ (−1, 1), temos que

f(x) = x5

1+x6

= x5 11−(−x6)

= x5∑∞k=0(−x6)k

= x5∑∞k=0(−1)kx6k

=∑∞k=0(−1)kx6k+5

=∑∞n=0 anx

n,

onde

an :=

{(−1)k, se n = 6k + 5 para algum k ∈ Z>0,0, caso contrario.

Pelo exercıcio 8.47, temos que

f (n)(0) = ann! :=

{(−1)kn!, se n = 6k + 5 para algum k ∈ Z>0,0, caso contrario.

Assim,f (2001)(0) = f (333·6+3)(0) = 0

ef (2003)(0) = f (333·6+5)(0) = (−1)3332003! = −2003!.

300

Exercıcio 8.49:

Seja f : I → R definida num intervalo. Prove que f e convexa se, e somente se, para quaisquer a e b em I e0 6 t 6 1, vale

f((1− t)a+ tb) 6 (1− t)f(a) + tf(b).

Suponhamos que f : I → R seja convexa. Mostraremos que

f((1− t)a+ tb) 6 (1− t)f(a) + tf(b), (8.1)

para todos a e b ∈ I e t ∈ [0, 1]. Para a = b, temos que

f((1− t)a+ tb) = f((1− t)a+ ta) = f(a) = (1− t)f(a) + tf(a) = (1− t)f(a) + tf(b).

Adiante, mostraremos para o caso em que a < b. Deste caso segue o caso em que b < a. De fato, para a′ = b,b′ = a e t′ = 1− t,

f((1− t)a+ tb) = f((1− t′)a′ + t′b′) 6 (1− t′)f(a′) + t′f(b′) = (1− t)f(a) + tf(b).

Portanto, se f e convexa, entao a desigualdade (8.1) e valida para todos a e b ∈ I e t ∈ [0, 1].Sejam a e b ∈ I, com a < b, e t ∈ [0, 1]. Desta forma x := (1− t)a+ tb e tal que

x− a = t(b− a) > 0

eb− x = (1− t)(b− a) > 0.

Ou seja, x ∈ [a, b] ⊂ I. Alem disso, segue da definicao de x que

t =x− ab− a

e

1− t =b− xb− a

.

Como f e convexa, temos quef(x)− f(a)

x− a6f(b)− f(a)

b− ae, consequentemente,

f(x) 6

(b− xb− a

)f(a) +

(x− ab− a

)f(b).

Logo,f((1− t)a+ tb) = f(x)

6(b−xb−a

)f(a) +

(x−ab−a

)f(b)

= (1− t)f(a) + tf(b).

Consideremos, agora, que f satisfaz a desigualdade (8.1), para todos a e b ∈ I e t ∈ [0, 1]. Mostraremos que fe convexa.

Sejam a e b ∈ I, com a < b, e x ∈ [a, b]. Definindo

t :=x− ab− a

temos quex = (1− t)a+ tb

e, da desigualdade a 6 x 6 b, que t ∈ [0, 1]. Daı,

f(x) = f((1− t)a+ tb)6 (1− t)f(a) + tf(b)

=(b−xb−a

)f(a) +

(x−ab−a

)f(b)

e, consequentemente,f(x)− f(a)

x− a6f(b)− f(a)

b− a.

Com isso, concluimos que f e convexa.

301

Exercıcio 8.50:

Verifique que f : R→ R, dada por f(x) = ex, e convexa e conclua que, para 0 6 t 6 1 e x e y ∈ R quaisquer vale

e(1−t)x+ty 6 (1− t)ex + tey.

Deduza daı a desigualdadeaαbβ 6 αa+ βb,

para α, β, a, b nao-negativos, com α+ β = 1.

Seja exp: R → R a aplicacao exponencial. Como exp′′ = exp > 0, temos, pelo Teorema 11 do Capıtulo 8, queexp e uma funcao convexa. Assim, pelo Excercıcio 8.50,

e(1−t)x+ty = exp((1− t)x+ ty

)6 (1− t) exp(x) + t exp(y) = (1− t)ex + tey,

para todos x e y ∈ R e t ∈ [0, 1].Dados a, b, α e β ∈ R+, com α+ β = 1, provaremos que vale a desigualdade

aαbβ 6 αa+ βb.

Sejam x := ln(a) e y := ln(b) ∈ R e t = β ∈ [0, 1]. Temos, pela desigualdade provada acima, que

aαbβ = eα ln(a)eβ ln(b)

= e(1−t)xety

= e(1−t)x+ty

6 (1− t)ex + tey

= αeln(a) + βeln(b)

= αa+ βb.

302

Exercıcio 8.51:

Seja F : I → R convexa no intervalo I. Se a1, . . . , an pertencem ao intervalo I, t1, . . . , tn pentencem a [0, 1] e∑ni=1 ti = 1, prove que

f

(n∑i=1

tiai

)6

n∑i=1

tif(ai).

Primeiramente, verifiquemos que se a1, . . . , ak pertencem ao intervalo I, t1, . . . , tk pentencentem a [0, 1] e∑ki=1 ti = 1 entao

∑ni=1 tiai ∈ I. Sejam

a := min{a1, . . . , an} ∈ I

eb := max{a1, . . . , an} ∈ I.

Segue que

a =

(n∑i=1

ti

)a =

n∑i=1

tia 6n∑i=1

tiai

en∑i=1

tiai >n∑i=1

tib =

(n∑i=1

ti

)b = b.

Logo,n∑i=1

tiai ∈ [a, b] ⊂ I.

Provaremos o resultado por inducao em n.No caso n = 2, temos que

t1 = 1− t2e, pelo Exercıcio 8.49, temos que

f(t1a1 + t2a2) = f((1− t2)a1 + t2a2)= (1− t2)f(a1) + t2f(a2)= t1f(a1) + t2f(a2).

Suponhamos, como hipotese de inducao, que o caso n = k − 1 seja verdadeiro.Sejam a1, . . . , ak pertencentes ao intervalo I, t1, . . . , tk pentencentes a [0, 1] e

∑ki=1 ti = 1.

Se tk = 1 segue que t1 = · · · = tk−1 = 0 e, consequentemente,

f

(k∑i=1

tiai

)= f(ak) =

k∑i=1

tif(ai).

Consideremos agora o caso em que tk 6= 1. Desta forma,

t′i :=ti

1− tk

sao tais quek−1∑i=1

t′i =

k−1∑i=1

ti1− tk

=

∑k−1i=1 ti

1− tk= 1

et′i ∈ [0, 1].

Seja

a :=

k−1∑i=1

t′iai ∈ I.

303

Assim, pela hipotese de inducao,

f

(k∑i=1

tiai

)= f

((1− tk)

k−1∑i=1

ti1− tk

ai + tkak

)= f

((1− tk)a+ tkak

)6 (1− tk)f(a) + tkf(ak)

= (1− tk)f(∑k−1

i=1 t′iai

)+ tkf(ak)

6 (1− tk)∑k−1i=1 t

′if(ai) + tkf(ak)

=∑k−1i=1 tif(ai) + tkf(ak)

=∑ki=1 tif(ai).

Portanto, o resultado segue pelo PIF.

304

Exercıcio 8.52:

Sejam x1, x2, . . . , xn e t1, . . . , tn numeros nao-negativos, com t1 + · · ·+ tn = 1. Prove que

xt11 · xt22 · · · · · xtnn 6 t1x1 + t2x2 + · · ·+ tnxn.

Conclua, em particular a desigualdade entre as medias aritmetica e geometrica.

Denotemos por exp: R→ R a aplicacao exponecial. Como exp′′ = exp > 0, temos, pelo Teorema 11 do Capıtulo8, que exp e uma funcao convexa.

Sejam x1, . . . , xn, t1, . . . , tn ∈ R>0 tais que t1 + · · ·+ tn = 1. Se algum xi e nulo, temos imediatamente que

xt11 · · · xtnn = 0 6 t1x1 + . . . tnxn.

Se x1, . . . , xn ∈ (0,+∞) entao

xt11 · xt22 · · · · · xtnn = et1 ln x1 · et2 ln x2 · · · · · etn ln xn

= et1 ln x1+t2 ln x2+···+tn ln xn

= exp(t1 lnx1 + t2 lnx2 + · · ·+ tn lnxn).

Assim, como exp e convexa, temos, pelo Exercıcio 8.51, que

xt11 · xt22 · · · · · xtnn = exp(t1 lnx1 + t2 lnx2 + · · ·+ tn lnxn)

= t1 exp(lnx1) + t2 exp(lnx2) + · · ·+ tn exp(lnxn)= t1x1 + t2x2 + · · ·+ tnxn.

Sejam x1, . . . xn numeros reais nao negativos,

G := n√x1 · x2 · · · · · xn

sua media geometrica e

A :=x1 + · · ·+ xn

n

sua media aritmatica. Pelo que foi demonstrado acima, temos que

G = x1n1 · x

1n2 · · · · · x

1nn

6 x1

n + x2

n + · · ·+ xn

n= A.

305

Exercıcio 8.53:

Seja ϕ : [a, b]→ R duas vezes derivavel, com ϕ(a) = ϕ(b) = 0 e ϕ′′(x) < 0 para todo x ∈ [a, b]. Prove que ϕ(x) > 0,para todo x ∈ (a, b). Conclua que, se f : I → R e duas vezes derivavel e f ′′(x) > 0, para todo x ∈ I, entao f eestritamente convexa no intervalo I.

Seja ϕ : [a, b]→ R, a < b, duas vezes derivavel, com ϕ(a) = ϕ(b) = 0 e ϕ′′(x) < 0 para todo x ∈ [a, b].Como ϕ′′(x) < 0 para todo x ∈ [a, b], temos que ϕ′ e estritamente decrescente (Corolario 6 do Teorema 7 do

Capıtulo VIII). Alem disso, como ϕ(a) = 0 = ϕ(b), pelo Toerema de Rolle, existe c ∈ (a, b) tal que ϕ′(c) = 0.Desta forma, como ϕ′ e estritamente decrescente em [a, b] e ϕ′(c) = 0, ϕ′ > 0 em [a, c) e ϕ′ < 0 em (c, b]. Com isso,concluimos que ϕ e estritamente crescente em [a, c] e estritamente decrescente em [c, b] (novamente pelo Corolario6 do Teorema 7 do Capıtulo VIII).

Sendo ϕ(a) = 0 e ϕ estritamente crescente em [a, c] segue que ϕ > 0 em (a, c]. Em particular, temos queϕ(c) > ϕ(a) = 0. Por outro lado, como ϕ(c) > 0 = ϕ(b) e ϕ e estritamente decrescente em [c, b), ϕ > 0 em [c, b].

Portanto, ϕ > 0 em (a, b).Seja f : I → R e duas vezes derivavel e f ′′(x) > 0, para todo x ∈ I, entao f e estritamente convexa no intervalo

I. Provaremos, para a e b em I, com a < b, arbitrarios, que

f(x)− f(a)

x− a<f(b)− f(a)

b− a,

para todo x ∈ (a, b). Com isso, concluiremos que f e estritamente convexa.Definimos ϕ : [a, b]→ R por

ϕ(x) =f(b)− f(a)

b− a(x− a) + f(a)− f(x),

para todo x ∈ [a, b]. Desta forma,ϕ(a) = ϕ(b) = 0,

ϕ′(x) =f(b)− f(a)

b− a− f ′(x)

eϕ′′(x) = −f ′′(x) < 0,

para todo x ∈ I. Logo, pelo que foi provado acima, ϕ > 0 em (a, b).Assim, para todo x ∈ (a, b),

0 < ϕ(x) =f(b)− f(a)

b− a(x− a) + f(a)− f(x)

e, consequentemente,f(x)− f(a)

x− a<f(b)− f(a)

b− a.

306

Exercıcio 8.54:

Seja f contınua num ponto. Prove que se f e derivavel nesse ponto entao existe na maximo uma reta que coincidecom o grafico de f uma infinidade de vezes em qualquer vizinhanca do ponto.

Seja f : X → R derivavel em a ∈ X ′.Suponhamos que c e d ∈ R sao tais que, para qualquer vizinhanca U de a em X,

f(x) = cx+ d,

para infinitos x ∈ U . Em particular, existe uma sequencia (xn)n∈Z+em X tal que

limn→∞

xn = a

ef(xn) = cxn + d,

para todo n ∈ Z+.Como f e derivavel em a, temos que

c = limn→∞

(cxn + d)− (ca+ d)

xn − a= limn→∞

f(xn)− f(a)

xn − a= limx→a

f(x)− f(a)

x− a= f ′(a).

Alem disso, como f e contınua em a,

ca+ d = limn→∞

(cxn + d) = limn→∞

f(xn) = f(a)

e, consequentemente,d = f(a)− ca = f(a)− f ′(a)a.

Portanto, concluimos que se f coincide em infinitos pontos, em cada vizinhanca de a, com uma reta

{(x, ca+ d) : x ∈ R}

entao c e d sao determinados unicamente pelos valores de f(a) e f ′(a). Em particular, existe no maximo uma retacom esta propriedade.

307

Exercıcio 8.55:

Seja f : [a,+∞)→ R duas vezes derivavel. Se limx→+∞

f(x) = f(a) entao existe x ∈ (a,+∞) tal que f ′′(x) = 0.

Usaremos o seguinte resultado: Se g : [c,+∞)→ R e uma funcao derivavel que adimite uma sequencia (xn)n∈Z+

em [c,+∞) tal que limn→+∞ xn = +∞ e limn→+∞ g(xn) = g(c) entao existe x ∈ (c,+∞) tal que g′(x) = 0.Se g(y) = g(c), para todo y ∈ (c,+∞), qualquer x ∈ (c,+∞) e tal que g′(x) = 0. Suponhamos que exista

y ∈ (c,+∞) tal que g(y) 6= g(c). Provaremos que exsite x como desejado para o caso em que

g(y) > g(c).

O caso em que g(y) < g(c) e analogo. Como limn→+∞ xn = +∞ e limn→+∞ g(xn) = g(c), deve existir algumxn ∈ (y,+∞) tal que

g(xn) ∈ (g(c)− 1, g(y)).

Seg(xn) ∈ (g(c)− 1, g(c)]

temos queg(xn) 6 g(c) < g(y)

e, pelo Teorema do Valor Intermediario, existe um z ∈ (y, xn] tal que

g(z) = g(c).

Logo, pelo Teorema de Rolle, existe x ∈ (a, z) tal que g′(x) = 0. Se

g(xn) ∈ (g(c), g(y)),

temos, pelo Teorema do Valor Intermadiario, que existe z ∈ (c, y) tal que

g(z) = g(xn).

Logo, pelo Teorema de Rolle, existe x ∈ (z, xn) tal que g′(x) = 0.Consideremos, agora, f : [a,+∞)→ R como no enunciado.Como limx→+∞ f(x) = f(a), deve existir uma sequencia (xn)n∈Z+

em [a,+∞) tal que limn→+∞ xn = +∞ elimn→+∞ f(xn) = f(a). Logo, existe um ponto b ∈ (a,+∞) tal que f ′(b) = 0.

Provaremos que existe uma sequencia (xn)n∈Z+ em [b,+∞) tal que limn→+∞ xn = +∞ e limn→+∞ f ′(xn) =0 = f ′(b). Assim, concluımos que a funcao derivavel f ′ : [b,+∞) → R admite um ponto x ∈ (b,+∞) tal quef ′′(x) = 0.

Pelo Teorema do Valor Medio, podemos escolher, para cada n ∈ Z+, xn ∈ (a+ n, a+ n+ 1) tal que

f ′(xn) = f(a+ n+ 1)− f(a+ n).

Segue daı que limn→+∞ xn = +∞ e

limn→+∞

f ′(xn) = limn→+∞

(f(a+ n+ 1)− f(a+ n)

)= lim

n→+∞f(a+ n+ 1)− lim

n→+∞f(a+ n)

= f(a)− f(a)= 0.

308

Capıtulo 9

Integral de Riemann

309

Exercıcio 9.36:

Seja f : R → R contınua tal que f(x + y) = f(x) · f(y). Prove que, para algum a ∈ R+, f(x) = ax ou f(x) = 0para todo x ∈ R.

Comof(0) = f(0 + 0) = f(0) · f(0),

devemos ter que f(0) = 0 ou 1. Provaremos que f e nula se f(0) = 0 e que f e igual a funcao x → ax,a := f(1) ∈ R+, se f(0) = 1.

Suponhamos que f(0) = 0. Neste caso, temos que

f(x) = f(x+ 0) = f(x) · f(0) = f(x) · 0 = 0,

para todo x ∈ R. Desta forma, a funcao f e constante igual a zero.Consideremos, de agora em diante, o caso em que f(0) = 1.Seja a := f(1). Temos que

a = f(1) = f

(1

2+

1

2

)= f

(1

2

)· f(

1

2

)> 0.

Alem disso,a · f(−1) = f(1) · f(−1) = f(1− 1) = f(0) = 1

e, consequentemente, f(1) 6= 0. Com isso, concluımos que a ∈ R+.Provaremos, por inducao em n ∈ Z+, que f(nx) =

(f(x)

)n, para todo x ∈ R. Para n = 1 a afirmacao e trivial.

Se f(nx) =(f(x)

)nentao

f((n+ 1)x

)= f(nx+ x) = f(nx) · f(x) =

(f(x)

)nf(x) =

(f(x)

)n+1.

Logo, pelo Princıpio da Inducao Finita, f(nx) =(f(x)

)npara todo n ∈ Z+ e x ∈ R.

Com isso, temos, para todo n ∈ Z+ e x ∈ R, que

f(−nx) ·(f(x)

)n= f(−nx) · f(nx) = f(−nx+ nx) = f(0) = 1

e, consequentemente,

f(−nx) =1(

f(x))n =

(f(x)

)−n.

Logo, como

f(0x) = f(0) = 1 =(f(x)

)0,

concluımos quef(nx) =

(f(x)

)n,

para todo n ∈ Z e x ∈ R.Para m ∈ Z+, (

f

(1

m

))m= f

(m · 1

m

)= f(1) = a

e, consequentemente,

f

(1

m

)= a

1m .

Portanto, para todo n ∈ Z e m ∈ Z+,

f( nm

)=

(f

(1

m

))n= (a

1m )n = a

nm .

A funcao g : R→ R definida porg(x) = ax = ex·log a

e uma composicao de funcoes contınuas. Logo, g e contınua.Desta forma, concluımos que as funcoes f e x → ax coincidem em Q. Portanto, como ambas funcoes sao

contınuas, devemos ter que f = g.

310

Exercıcio 9.37:

Prove que limn→∞

n n√a− 1 = log a para todo a > 0.

Este exercıcio esta errado.A sequencia ( n

√a− 1)n∈Z+

e limitada. Logo,

limn→∞

n n√a− 1 = +∞ 6= log a.

311

Exercıcio 9.38:

Mostre que

limn→+∞

(nn+1 + (n+ 1)n

nn+1

)n= ee.

Seja (bn)n∈Z+a sequencia dada por

bn =

(1 +

1

n

)n,

para todo n ∈ Z+. Temos que (1 +

bnn

)n=

(1 +

1

n·(

1 +1

n

)n)n=

(1 +

1

n· (n+ 1)n

nn

)n=

(nn+1 + (n+ 1)n

nn+1

)nPortanto, devemos provar que

limn→+∞

(1 +

bnn

)n= ee. (9.1)

Para verificarmos a igualdade acima, provaremos tres desigualdades que serao utilizadas.A primeira desigualdade e(

1 +a

m

)m<(

1 +a

n

)n, para todo a ∈ R+ e m < n em Z+. (9.2)

Supondo que a ∈ R+ e m < n em Z+, segue que

(1 +

a

m

)m=

m∑k=0

m!

k!(m− k)!· a

k

mk

= 1 + a+

m∑k=2

ak

k!

m(m− 1) . . . (m− (k − 1))

mk

= 1 + a+

m∑k=2

ak

k!

(1− 1

m

)·(

1− 2

m

)· · · · ·

(1− k − 1

m

)< 1 + a+

m∑k=2

ak

k!

(1− 1

n

)·(

1− 2

n

)· · · · ·

(1− k − 1

n

)< 1 + a+

n∑k=2

ak

k!

(1− 1

n

)·(

1− 2

n

)· · · · ·

(1− k − 1

n

)= 1 + a+

n∑k=2

ak

k!

n(n− 1) . . . (n− (k − 1))

nk

=

n∑k=0

n!

k!(n− k)!· a

k

nk

=(

1 +a

n

)n.

Com isso, e pelo fato de que

limn→∞

(1 +

a

n

)n= ea

(veja o Exercıcio 9.34), temos que a sequencia ((1 +

a

n

)n)n∈Z+

312

e estritamente crescente e convergente com(1 +

a

n

)n< ea, para todo n ∈ Z+ (9.3)

A segunda desigualdade e(1 +

a

n

)n<

(1 +

b

n

)n, para a < b em R+ e todo n ∈ Z+. (9.4)

Se a < b em R+ entao

1 +a

n< 1 +

b

n,

para qualquer n ∈ Z+. Segue, por inducao em k ∈ Z+ que,

(1 +

a

n

)k<

(1 +

b

n

)k.

Em particular, (9.4) e valida.A terceira desigualdade e

ea < eb, para a < b em R+. (9.5)

Pelo Teorema 9.22, a funcao exponencial possui derivada estritamente positiva. Por isso, a desigualdade (9.5) evalida.

Voltemos ao limite (9.1).Seja ε > 0. Verificaremos que existe n0 ∈ Z+ tal que para todo n > n0 vale

ee − ε <(

1 +bnn

)n< ee.

Daı concluiremos que a igualdade (9.1) e valida.Como a aplicacao exponencial e contınua (Teorema 9.22) e limn→+∞ bn = e (Exercıcio 9.34) existe k ∈ Z+ tal

que

ebk ∈(ee − ε

2, ee +

ε

2

).

Assim, por (9.5), devemos ter que

ee − ε

2< ebk < ee. (9.6)

Novamente pelo Exercıcio 9.34, devemos ter que

limm→+infty

(1 +

bkm

)m= ebk .

Assim, existe m ∈ Z+ tal que (1 +

bkm

)m∈(ebk − ε

2, ebk +

ε

2

).

Logo, pela desigualdade (9.3),

ebk − ε

2<

(1 +

bkm

)m< ebk . (9.7)

Pelas desigualdades (9.6) e (9.3), devemos ter que

ee − ε <(

1 +bkm

)m< ee.

Segue daı que, para todon > no := max{m, k},

313

valem as desigualdades

ee − ε <

(1 +

bkm

)m<

(1 +

bkn

)n(pela desigualdade (9.2))

<

(1 +

bnn

)n(pela desigualdade (9.4))

e (1 +

bnn

)n< ebn (pela desigualdade (9.3))

< ee (pela desigualdade (9.5)).

Ou seja,

ee − ε <(

1 +bnn

)n< ee,

para todo n > n0.

314

Capıtulo 10

Sequencias e Series de Funcoes

315

Exercıcio 10.44:

A sequencia de funcoes fn(x) = nx2 possui derivadas equilimitadas no ponto 0 mas nao e equicontınua nesteponto.

Como f ′n(0) = 0 para todo n ∈ Z+ temos que a sequencia (fn) possui derivadas limitadas em 0.Por outro lado, para todo δ > 0, existe x ∈ (−δ, δ) e n ∈ Z+ tais que

|fn(0)− fn(x)| > 1.

De fato, dado δ > 0, existe k ∈ Z+ tal que 1/k < δ. Assim, para n = k2 e x = 1/k, temos que

|fn(0)− fn(x)| = |0− 1| = 1.

Portanto, a sequencia nao e equicontınua em 0.

316

Exercıcio 10.45:

Um conjunto de polinomios de grau 6 k, uniformemente limitado em um intervalo compacto, e equicontınuo nesteintervalo.

Seja E tal conjunto de polinomios definidos no intervalo compacto I.Podemos considerar, sem perda de generalidade, que I = [0, b] para algum b > 0. De fato, se I = [a, b], podemos

definirE := {p : [0, b− a]→ R | ∃p ∈ E, p(x) = p(x+ a)∀x ∈ [0, b− a]}

e teremos, assim, que E e uniformemente limitado se e somente se E e uniformemente limitado e que E eequicontınuo se e somente se E e equicontıno.

Provaremos, por inducao em k, que existe uma constante C > 0 tal que, para todo p ∈ E dado por p(x) =∑ki=0 aix

i, vale|ai| < C, i = 0, ..., k. (10.1)

Donde conclui-se que o conjunto das derivadas dos polinomios de E e equilimitado e, consequentemente, que E eequicontınuo.

Para k = 0, temos que (10.1) segue do fato de E ser equilimitado.Suponhamos que exista C > 0 satifazendo (10.1) para todo k 6 n− 1. E suponhamos que k = n.Como E e equilimitado, segue que o conjunto

E′ := {p− p(0) | p ∈ E}

e tambem equilimitado. Consideremos

E′′ := {p | ∃p ∈ E′, p(x) = p(x)x, ∀x ∈ I}

Adiante, dada uma constante A > 0 tal que

|p(x)| < A,∀p ∈ E′, x ∈ I,

temos que A′ = A/b e tal que|p(x)| < A′,∀p ∈ E′′, x ∈ I.

Ou seja, E′′ e equilimitada. Alem disso, todos os polinomios em E′′ sao de grau 6 n − 1. Pelo passo indutivo,existe C ′ > 0 tal que, para todo p ∈ E′′ dado por p(x) =

∑ni=1 aix

i−1, vale

|ai| < C ′, i = 1, ..., n.

E, como E e equilimitado, existe C ′′ > 0 tal que, para todo p ∈ E, vale

|a0| = |p(0)| < C ′′.

Assim, para C = max{C ′, C ′′} temos (10.1). E o resultado segue pelo PIF.

317

Exercıcio 10.46:

Diz-se que uma sequencia de funcoes fn : X → R converge fracamente para uma funcao f : X → R quandolimn→∞ fn(x) = f(x) para cada ponto x ∈ X na qual f e contınua. Seja D ⊂ R denso. Prove que se umasequencia de funcoes monotonas fn : R→ R converge simplesmente em D para uma funcao f : R→ R entao (fn)converge fracamente para f em R.

Seja x0 ∈ R um ponto de continuidade de f . Provaremos que

limn→∞

fn(x0) = f(x0). (10.2)

Para tanto, basta mostrar, para (fnk) e (fnp

) sendo as subsequencia nao-decrescentes e nao-crescentes de (fn), que

limk→∞

fnk(x0) = lim

p→∞fnp

(x0) = f(x0)

Com igual razao, basta provar a afirmacao (10.2) para o caso em que todas as fn’s sao decrescentes e para o casoem que todas as fn’s sao nao-crescentes.

Suponhamos que todas as fn’s sao nao-decrescentes.Comecaremos provando que f e nao-decrescente em D. Suponhamos, por absurdo, que existam

x− < x+

em D tais quef(x−) > f(x+).

Entao, comolimn→∞

fn(x−) = f(x−),

limn→∞

fn(x+) = f(x+)

e

f(x−) >f(x−) + f(x+)

2> f(x+),

temos, para n suficientemente grande, que

fn(x−) >f(x−) + f(x+)

2> fn(x+).

Contradizendo o fato de fn ser nao-decrescente. Portanto, f e nao-decrescente em D.O proximo passo desta demonstracao e provar que se

x−0 < x0 < x+0 ,

onde x−0 e x+0 ∈ D, entao

f(x−0 ) 6 f(x0) 6 f(x+0 ).

De fato, sef(x−0 ) > f(x0),

entao, como f e contınua em x0 e D e denso em R, existe x ∈ D tal que

x−0 < x < x0

ef(x−0 ) > f(x).

Contradizendo o fato de f ser nao-decrescente em D. Portanto, devemos ter que

f(x−0 ) 6 f(x0).

E, de forma analoga, mostra-se que devemos ter que

f(x0) 6 f(x+0 ).

318

Por fim, provaremos que, dado ε > 0, existe n0 ∈ Z+ tal que para todo natural n > n0 vale

|f(x0)− fn(x0)| < ε.

Com efeito, existem (pois f e contınua em x0) x−0 e x+0 ∈ D tais que

x−0 < x0 < x+0

e|f(x−0 )− f(x+

0 )| < ε/2.

E, como (fn) converge simplesmente para f em D, existe n0 ∈ Z+, tal que, para todo n > n0, vale

|f(x−0 )− fn(x−0 )| < ε/2

e|f(x+

0 )− fn(x+0 )| < ε/2.

Pelo que foi dito nos paragrafos acima, temos que

f(x−0 ) 6 f(x0) 6 f(x+0 )

efn(x−0 ) 6 fn(x0) 6 fn(x+

0 ).

Logo,f(x−0 )− fn(x+

0 ) 6 f(x0)− fn(x0) 6 f(x+0 )− fn(x−0 ).

E, como|f(x−0 )− fn(x+

0 )| 6 |f(x−0 )− f(x+0 )|+ |f(x+

0 )− fn(x+0 )| 6 ε/2 + ε/2 = ε

e|f(x+

0 )− fn(x−0 )| 6 |f(x+0 )− f(x−0 )|+ |f(x−0 )− fn(x−0 )| 6 ε/2 + ε/2 = ε,

para todo n > n0, segue que|f(x0)− fn(x0)| < ε,

para todo n > n0. E temos o resultado.O caso em que todas as fn’s sao nao-crescentes e analogo.

319

Exercıcio 10.47:

Seja f(x) = x+ x|x| se x 6= 0 e f(0) = 0. Obtenha uma sequencia de funcoes contınuas crescentes fn : R→ R que

convirjam para f em R− {0}, mas (fn(0)) nao converge.

Considere

fk(x) =

{f(x), x < −1/k e 0 < x(2 + 1

k

) (x+1/k1/k

)− 1

k − 1, −1/k 6 x 6 0

para k par e

fk(x) =

{f(x), x < −1/k e 0 < x(2 + 1

k

) (x

1/k

)− 1, 0 6 x 6 1/k

para k ımpar. Temos, entao, que (fk) e uma sequencia de funcoes contınuas e crescentes que converge para f emR− {0}. E, como

fk(0) = (−1)k,

temos que (fk(0)) nao e convergente.

320

Exercıcio 10.48:

Uma sequencia de funcoes monotonas fn : R → R possui uma subsequencia que converge fracamente para umafuncao monotona f : R→ R, a qual podemos tomar contınua a direita.

Provaremos o resultado para o caso em que (fn) e uma sequencia de funcoes nao decrescentes. A demonstracaopara o caso em que (fn) e uma sequencia de funcoes nao-crescentes e analoga. No caso geral, (fn) possui umasubsequencia de pelo menos um destes dois tipos. E esta, por sua vez, possuira um subsequencia com a propriedadedo enunciado.

Seja (fn) uma sequencia simplesmente limitada de funcoes nao decrescentes.Pelo Teorema de Cantor-Tychonov existe uma subsequencia (fnk

) de (fn) que converge em Q para uma funcaof : Q→ R. Como (fnk

), devemos ter que f e nao-decrescente.Como f e monotona, existe limx→x+

0f(x). Assim, podemos definir

f : R → Rx0 → limx→x+

0f(x).

Provaremos que f e contınua a direita e (fnk) converge fracamente para f .

Seja x0 ∈ R. Provaremos quef(x0) = lim

x→x+0

f(x).

Seja (xn) uma sequencia em R que converge a direita para x0. Pela definicao de f , para cada n ∈ Z+ existeyn ∈ Q ∩ (xn,+∞) tal que

|xn − yn| <1

ne

|f(xn)− f(yn)| < 1

n.

Assim,limn→∞

yn = limn→∞

xn = x0

e, consequentemente,limn→∞

f(xn) = limn→∞

f(yn) = limx→x+

0

f(x) = f(x0).

Como (xn) e uma sequencia que converge para x0 a direita arbitraria, segue que

limx→x+

0

f(x) = f(x0).

Por sua vez, como x0 e arbitrario, concluımos que f e contınua a direita.Para todos x0 ∈ R, x−0 e x+

0 ∈ Q tais quex−0 < x0 < x+

0 ,

temos quef(x−0 ) 6 f(x0) 6 f(x+

0 ) (10.3)

poisf(x0) = limx→x+

0f(x)

e f e nao-decrescente.Seja x0 um ponto de continuidade de f e ε > 0. Provaremos que existe k0 ∈ Z+ tal que para todo k > k0 em

Z+ temos que|f(x0)− fnk

(x0)| < ε.

Como x0 e ponto de continuidade de f , existem y−0 e y+0 ∈ R tais que

y−0 < x0 < y+0

e|f(y+

0 )− f(y−0 )| < ε

2.

321

Tomando-se x−0 e x+0 ∈ Q tais que

y−0 < x−0 < x0 < x+0 < y+

0

temos, por (10.3), quef(y−0 ) 6 f(x−0 ) 6 f(x0) 6 f(x+

0 ) 6 f(y+0 ).

Assim,

|f(x+0 )− f(x−0 )| < ε

2.

Como (fnk) converge simplesmente para f em Q, existe k0 ∈ Z+ tal que, para todo k > k0 vale

|f(x−0 )− fnk(x−0 )| < ε

2

e|f(x+

0 )− fnk(x+

0 )| < ε

2.

Do que foi dito acima, e pelo fato de cada fnkser nao-decrescente, temos que

f(x−0 ) 6 f(x0) 6 f(x+0 ),

fnk(x−0 ) 6 fnk

(x0) 6 f(x+0 )

e, consequentemente,f(x−0 )− fnk

(x+0 ) 6 f(x0)− fnk

(x0) 6 f(x+0 )− fnk

(x−0 ).

Por fim, como

|f(x−0 )− fnk(x+

0 )| 6 |f(x−0 )− f(x+0 )|+ |f(x+

0 )− fnk(x+

0 )| < ε

2+ε

2= ε

e|f(x+

0 )− fnk(x−0 )| 6 |f(x+

0 )− f(x−0 )|+ |f(x−0 )− fnk(x−0 )| < ε

2+ε

2= ε

para todo k > k0 em Z+, temos que|f(x0)− fnk

(x0)| < ε

para todo k > k0 em Z+.

322

Exercıcio 10.49:

Seja (fn) uma sequencia equicontınua e simplesmente limitada num compacto X ⊂ R. Se toda subsequenciauniformemente convergente em X tem o mesmo limite f : X → R, entao fn → f uniformemente.

Suponhamos, por absurdo, que a sequencia (fn) nao convirja uniformemente para f . Entao, existe uma sub-sequencia (fnk

) de (fn) e uma sequencia|f(xnk

)− fnk(xnk

)| > ε (10.4)

para algum ε > 0 fixo. Porem, como X e compacto, (fnk) e equicontınua e simplesmente limitada, temos, pelo

Teorema 10.23, que alguma subsequencia de (fnk) converge uniformemente em X para uma funcao f . E, como

toda subsequencia uniformemente convergente de (fn) converge para f , devemos ter que f = f . O que contradiz(10.4).

323

Exercıcio 10.50:

De exemplo de uma sequencia equicontınua de funcoes fn : (0, 1)→ (0, 1) que nao possua subsequencia uniforme-mente convergente em (0, 1).

Seja (fn) a sequencia dada por

fn : (0, 1) → (0, 1)x ∈

(0, 1

2πn

]→ 1

4 sin(

1x

)+ 1

2x ∈

[1

2πn , 1)→ 1

2 .

Esta sequencia converge simplesmente para a funcao

f : (0, 1) → (0, 1)x → 1

2 .

Porem nenhuma subsequencia desta sequencia converge uniformemente.

324

Exercıcio 10.51:

Dada uma sequencia de funcoes duas vezes derivaveis (fn) definidas no intervalo compacto I, suponha que fn → fsimplesmente em I, que (f ′n(a)) e limitada para um certo a ∈ I e que (f ′′n ) e uniformemente limitada em I. Proveque f ∈ C1.

Temos que a sequencia (f ′n) e equicontınua e uniformemente limitada. De fato, como (f ′′n ) e uma sequenciauniformemente limitada, segue que (fn) e equicontınua. Adiante, como (f ′′n ) e uniformemente limitada e I ecompacto, existe, pelo teorema do valor medio, C ′ > 0 tal que

|f ′n(x)− f ′n(a)| < C ′,∀n ∈ Z+, x ∈ I.

E, como (f ′n(a)) e uma sequencia limitada, existe C > 0 tal que

|f ′n(a)| < C,∀n ∈ Z+.

Entao,|f ′n(x)| 6 |f ′n(x)− f ′n(a)|+ |f ′n(a)| < C ′ + C,∀n ∈ Z+, x ∈ I.

Ou seja, (f ′n) e uniformemente limitada.Adiante, temos que toda subsequencia uniformemente convergente de (f ′n) converge para f ′. De fato, dada uma

subsequencia uniformemente convergente (f ′nk) temos, pelo Teorema 10.7, que f ′nk

→ f ′ ja que fnk→ f .

Como (fn) e uma sequencia equincontınua, uniformemente limitada tal que todas as suas subsequencias uni-formemente convergentes convergem para f ′, temos, pelo exercıcio 10.49, que f ′n → f ′ uniformemente em I.

Portanto, f ′ e contınua, pois e o limite uniforme de uma sequencia de funcoes contınuas.

325

Exercıcio 10.52:

Dada uma sequencia de funcoes k+1 vezes derivaveis (fn) definidas no intervalo I, suponha que existam a0, ..., ak ∈ Ie c > 0, tais que |fn(a0)| 6 c, |f ′n(a1)| 6 c,...,|f (k)

n (ak)| 6 c para todo n ∈ Z+ e que a sequencia (f(k+1)n ) seja

uniformemente limitada em I. Prove que existe uma subsequencia (fni) que converge, juntamente com suas k

primeiras derivadas, uniformemente em cada parte compacta I.

E suficiente provar que existe uma subsequencia (fni) que converge, juntamente com suas k primeiras derivadas,uniformemente em I para o caso em que I e um intervalo compacto. Entao, assumiremos sem perda de generalidade,que I e um intervalo compacto.

Temos que a sequencia (f(k)n ) e equicontınua e uniformemente limitada. De fato, como (f

(k+1)n ) e uma sequencia

uniformemente limitada, segue que (f(k)n ) e equicontınua. Adiante, como (f

(k+1)n ) e uniformemente limitada e I e

compacto, existe, pelo teorema do valor medio, C ′ > 0 tal que

|f (k)n (x)− f (k)

n (ak)| < C ′,∀n ∈ Z+, x ∈ I.

E, como (f(k)n (ak)) e uma sequencia limitada, existe C > 0 tal que

|f (k)n (ak)| < C,∀n ∈ Z+.

Entao,|f (k)n (x)| 6 |f (k)

n (x)− f (k)n (ak)|+ |f (k)

n (ak)| < C ′ + C,∀n ∈ Z+, x ∈ I.

Ou seja, (f(k)n ) e uniformemente limitada.

Agora, provaremos o resultado por inducao em k.Consideremos o caso k = 0. Pelo que foi dito acima, temos que (fn) e equicontınua e uniformemente limitada.

Segue daı e do fato de I ser compacto, pelo Teorema 10.23, que (fn) possui uma subsequencia uniformementeconvergente.

Faremos agora o passo indutivo. Como (f(k)n ) e uniformemente limitada, temos que existe uma subsequencia

(fni) que converge, juntamente com suas k − 1 primeiras derivadas, uniformente. Adiante, temos que (f(k)ni ) e

equicontınua e uniformemente limitada. Isto implica que existe uma subsequencia (f(k)nj ) de (f

(k)ni ) que converge

uniformemente. Portanto, devemos ter que (fnj) e suas primeiras k derivadas convergem uniformemente.

326

Exercıcio 10.53:

Demonstre o corolario de Teorema 22 para intervalos arbitrarios (abertos ou nao) I ⊂ R.

Se int(I) = ∅, os unicos compactos em I sao ∅ e I. Assim, o Teorema de Arzela-Ascoli se aplica imediatamentee o resultado segue. Adiante assumiremos que existe c ∈ int(I).

Consideremos o intervalo J = [a, b] ⊂ I dado por

a =

{c, caso inf(I) /∈ Iinf(I), caso contrario

e

b =

{c, caso sup(I) /∈ Isup(I), caso contrario.

Se J = I, temos que I e compacto, e, pelo Teorema de Arzela-Ascoli, temos que existe uma subsequencia de (fn)que converge uniformemente em I. E, logo, em cada parte compacta de I. Assumiremos daqui em diante queJ 6= I.

Entao existe uma sequencia de compactos Ki ⊂ I, i ∈ Z+ tais que

K1 = J,

Ki ⊂ Ki+1

eint(I) = ∪i∈Z+ int(Ki).

Seja K ⊂ I um compacto. Pela escolha de J , temos que K\J ⊂ int(I) e que K\J e um compacto. Daı segueque

K\J ⊂ ∪i∈Z+int(Ki)

e, consequentemente,K\J ⊂ Ki

para algum i ∈ Z+. Logo,K ⊂ J ∪Ki = Ki

para algum i ∈ Z+.Agora, como na demonstracao dada no texto, existe uma subsequencia de (fn) que converge uniformemente em

cada Ki, i ∈ Z+. Portanto, como cada parte compacta de I esta contida em algum Ki, temos que esta subsequenciaconverge uniformemente em toda parte compacta de K.

327

top related